You are on page 1of 289

मं�ज़ल उ�� को �मलती है , �जनके सपन� म� जान होती है !!

पंख से कुछ नह� होता , हौसल� से उड़ान होती है !

AVP
CONTENT
INDEX
1. Simplifications

2. Approximations

3. Missing Number Series

4. Wrong Number Series

5. Quadratic Equations

6. Percentage

7. Profit and Loss

8. Simple Interest

9. Compound Interest

10. Ratio and Mixture

11. Time and Work Wages

12. Pipe and Cistern

13. Problems on Age

14. Problems on Average

15. Partnership

16. Speed time and Distance

17. Problems on Train

18. Boat and Stream

19. Mensuration 2D

20. Mensuration 3D

21. Permutation & Combinations

22. Probability

23. Tabular Graph

24. Bar Graph

25. Line Graph

26. Pie Chart

27. Combination DI

28. Caselet DI - Tabular

29. Caselet DI - Venn Diagram

30. Radar Graph


1 2
= 6 % = 6.66 %
15 3
1=100%
1 1
½ = 50% = 6 % = 6.25 %
16 4
1 =331 %=33.33% 1 15
3 3 = 5 % = 5.88 %
17 17
¼ = 25%
1 5
= 5 % = 5.55 %
1/5 = 20% 18 9
1 2 1 5
= 16 % = 16.66% = 5 % = 5.26 %
6 3 19 19
1 2 1/20 = 5%
= 14 % = 14.28%
7 7 3/8 = 37.5%
1 1 5/8 = 62.5%
= 12 % = 12.5%
8 2
7/8 = 87.5%
1 1
= 11 % = 11.11% 2/3=66.66%
9 9
7/16=43.75%
1
= 10%
10 9/16 = 56.25%
1 1 5 3
=9 % = 9.09% = 71 % = 71.42%
11 11 7 7
1 1 2 4
= 8 % = 8.33% = 28 % = 28.57%
12 3 7 7
1
=7
9
% = 7.69% 4 1
= 57 % = 57.14%
13 13 7 7
1 1 5 1
= 7 % = 7.14%
14 7 = 83 % = 83.33%
6 3
58.33%=581 %= 7
3 12

45.45%=45 5 %= 5
11 11
1 5
31.25%=31 %=
4 16

18.18%=18 2 %= 2
11 11
4 4
44.44%=44 %=
9 9

18.75%=18 3 %= 3
4 16
2 4
26.66%=26 %=
3 15

Discount Code =
Chapter - 1 | Simplifications
नींव Booklet
For All Banking & Insurance Exam
Ch - 1 | Simplifications | Maths by Arun sir
1. 5% of 3100+ 41% of 1100 – 50% of 790 = ?
(1) 230 (2) 221 9. 36460 + 53750 = ? + 85450
(3) 211 (4) 255 (1) 4190 (2) 4590
(5) 250 (3) 4760 (4) 4390
(5) 4250
2. 1107 ÷ 9 × 6 + 236 – 874 = ?
(1) 110 (2) 105 10. 50% of 650 + ?% of 350 = 500
(3) 100 (4) 120 (1) 40 (2) 55
200 )5( (3) 45 (4) 50
(5) 60
3 4 18
3. 4
𝑜𝑓 27 𝑜𝑓 11 𝑜𝑓 2673 = ?
11. (46351–36418–4505) ÷ ? = 1357
(1) 470 (2) 450
(1) 4 (2) 6
(3) 465 (4) 486
(3) 3 (4) 2
(5) 490
(5) 5
4 3 7 3 12. 8888 + 848 + 88 – ? = 7337 + 737
4. 17
of √4913 + 15 𝑜𝑓 √3375 = ? (1) 1750 (2) 1650
(1) 13 (2) 11 (3) 1550 (4) 1450
(3) 9 (4) 15 (5) 1250
(5) 17
13. 618 + 62 × 0.50 – 29 = ?
5. √? = 40% 𝑜𝑓 70 + 32% 𝑜𝑓100 (1) 625 (2) 660
(1) 3844 (2) 3424 (3) 640 (4) 655
(3) 3600 (4) 4024 (5) 620
(5) 3640
5 4 1
14 2 9 × 3 5 + ? = 12 5
4 4 1
6. 25 − 37 𝑜𝑓 2 10 ÷5 =? 13 4
(1) 2 45 (2) 2 5
7 3
(1) 3 10 (2) 3 10 22 5
(3) 3 45 (4) 3 9
3 9
(3) 2 10 (4) 1 10 22
(5) 2 45
3
(5) 1 10
15. 3.2% of 500 × 2.4% of ? = 288
7. [(60)2 ÷ 30 × 18] ÷ 24 = 9 × ? (1) 650 (2) 700
(1) 15 (2) 10 (3) 600 (4) 750
(3) 20 (4) 25 (5) 550
(5) 40

8.
6 3
2 7 + (3 14 − 2 14) =?
3 16. ? % of 800 = 293 – 22% of 750
1 3 (1) 14 (2) 18
(1) 4 7 ((2) 4 7 (3) 12 (4) 16
(5) 20
4 6
(3) 4 (4) 3
7 7
1 3 1 22
(5)
2
63 17. 33 ÷ 67 × 12 × 7
=?
Chapter - 1 | Simplifications
नींव Booklet
For All Banking & Insurance Exam
Ch - 1 | Simplifications | Maths by Arun sir
(1) 21/7 (2) 22/9 (5) 5/23
(3) 9/22 (4) 81/2
(5) 22/5 26. 346.64 ÷ 4 + 440 ÷ 2.2 + 46% of 550 =?
(1) 544.66 (2) 539.66
18. 25.6% of 250 + √? = 119 (3) 439.66 (4) 343.66
(1) 4225 (2) 3025 (5) 493.66
(3) 2025 (4) 5625
4 3 12+? 5
(5) None of these 27. 3 7 of 5 5 of = 55 9
6
2 2
5 5 1 11 (1) 4 3 (2) 6 3
19. 46 − 59 =? −2 3 + 18 2 2
3 1 (3) 8 3 (4) 7 3
(1) 4
(2) 2 18 1
7 11 (5) 4
3
(3) 19 (4) 1 18 3
28. 49 × 546 ÷ 6 + 243 – 534 =? – of 465
(5) 1 5
(1) 4474 (2) 4847
20. 555.05 + 55.50 + 5.55 + 5 + 0.55 = ? (3) 4347 (4) 4664
(1) 621.65 (2) 655.75 (5) 4447
(3) 634.85 (4) 647.35
3 2 1
(5) None of these 29. 14 + 23 + 22 = ?
7 11
(1) 9 12 (2) 6 12
.21 39% of 450 + 51% of 340 – 40% of 255 =? 1 5
(1) 264.9 (2) 246.9 (3) 3 (4) 4
12 12
(3) 236.9 (4) 242.9 5
(5) 7
12
(5) 251.7
6
2 5 7 2 30. 35% of 42% of 7 th of 500 =?
22. 3 +4 + 5 =? +7
5 7 10 7 (1) 36 (2) 44
47 7
(1) 9 20 (2) 7 70 (3) 52 (4) 60
37 43 (5) 63
(3) 6 70 (4) 11 69
41
(5) 5 70 31. 8888 + 848 + 88 - ? = 7337 + 737
?
23 67% of 700 + 73% of 510 – 52% of 220 = 10
(1) 1750 (2) 1650
(1) 6369 (2) 6269 (3) 1550 (4) 1450
(3) 7629 (4) 7269 (5) 1250
(5) 7379
32. 414 × ? × 7 = 127512
24 3564 + 3486 ÷ 10 + 57434 ÷ 100 – 67385 ÷ 100 (1) 36 (2) 40
=? (3) 44 (4) 48
(1) 3813.09 (2) 3138.09 (5) 52
(3) 3613.09 (4) 3713.09
(5) 3513.09
? 3 15 13 3
25. of 4 of 17 of 6 of 544 = 7 7
5 33 28% of 550 + 55.5% of 600 =?
(1) 2/21 (2) 8/21 (1) 487 (2) 1014
(3) 1/21 (4) 1/23
Chapter - 1 | Simplifications
नींव Booklet
For All Banking & Insurance Exam
Ch - 1 | Simplifications | Maths by Arun sir
(3) 1011 (4) 817 (5) 850
(5) 917
42. (– 251 × 21 × (– 12)) ÷? = 158.13
4 7 121 (1) 250 (2) 400
34 of of of 81 = ?
9 11 12
(3) 300 (4) 15
(1) 120 (2) 119
(5) 18
(3) 231 (4) 108
(5) 321
.43 [(130)2 ÷ 25 × 15] ÷ 30 = ?
2 3 2 3 (1) 352 (2) 314
35 23 + 34 × 25 ÷65 = ? (3) 326 (4) 338
1 2
(1) 433 (2) 433 (5) 426
(3) 42 (4) 18
23 44. (6.5% of 375) – (0.85% of 230) =?
(5) 1 36 (1) 23.42 (2) 24.24
(3) 21.64 (4) 25.76
1 2 11 1
36 5 6 + 3 5 − 4 15 =? +1 3 (5) 22.42
1 2
(1) 1 7 (2) 1 7 45. 13.141 + 31.417 – 27.118 =?
1 1
(3) 2 7 (4) 2 2 (1) 16.441 (2) 17.543
2 (3) 17.490 (4) 17.440
(5) 3 3
(5) 17.590

37 46% of 450 + 84% of 525 – 55% of 880=? 46. ?2 + (14)2 × 18 ÷ 6 − 1029 = 80 × (12 − 7)
(1) 164 (2) 166 (1) 25 (2) 841
(3) 170 (4) 162 (3) 729 (4) 27
(5) 160 (5) 29

38
4
√256 + 5 𝑜𝑓 400 + 4 𝑜𝑓 ? = 13 𝑜𝑓 208
3 3 47. [(84)2 ÷ 28 × 12] ÷ 24 = 7 ×?
(1) 15 (2) 17
(1) 36 (2) 32 (3) 18 (4) 21
(3) 34 (4) 30 (5) 24
(5) 28
48. (7.9% of 134) – (3.4% of 79) =?
39. ? % of 250 – 48% of 250 =? (1) 8.1 (2) 7.9
(1) 70 (2) 75 (3) 8.6 (4) 7.3
(3) 85 (4) 72 (5) 6.8
(5) 80
2
1 49. (2√392 – 21) + (√8– 7) = (? )2
40. √46225 ÷ (3125) =? −215 5
(1) 6 (2) 4
(1) 258 (2) 255 (3) 12 (4) 2
(3) 218 (4) 235 (5) 8
(5) 248
50. 46% of 450 +? % of 19.07 = 359.56
41. 3.2% of 500 × 2. 4% of ? = 288 (1) 795 (2) 805
(1) 650 (2) 700 (3) 815 (4) 800
(3) 600 (4) 750 (5) 820
Chapter - 1 | Simplifications
नींव Booklet
For All Banking & Insurance Exam
Ch - 1 | Simplifications | Maths by Arun sir
Answer Key
1. (3) 11. (1) 21. (2) 31. (1) 41. (4)
2. (3) 12. (1) 22. (3) 3. (3) 42. (2)
3. (4) 13. (5) 23. (4) 33. (1) 43. (4)
4. (2) 14. (5) 24. (1) 34. (2) 44. (5)
5. (3) 15. (4) 25. (3) 35. (1) 45. (4)
6. (5) 16. (4) 26. (2) 36. (4) 46. (5)
7. (2) 17. (2) 27. (1) 37. (1) 47. (3)
8. (4) 18. (2) 28. (5) 38. (2) 48. (2)
9. (3) 19. () 29. (2) 39. (5) 49. (1)
10. (4) 20. () 30. (5) 40. (1)
Chapter - 1 | Simplifications
नींव Booklet
For All Banking & Insurance Exam
Ch - 1 | Simplifications | Maths by Arun sir
1. Solution (3) 10. Solution (4)
? = 31 × 5 + 41 × 11 – 5 × 79 35
?× = 500 − 325
= 606 – 395 = 211 10
⇒ ? = 50
2. Solution (3) 11. Solution (1)
5428
? = 738 + 236 – 874 ? = 1357 = 4
= 100
12. Solution (1)
3. Solution (4) ? = 9824 – 8074 = 1750
3 4 18
?= × × × 2673
4 27 11 13. Solution (5)
= 2 × 243
? = 618 + 31 – 29 = 620
= 486
14. Solution (5)
4. Solution (2) 61 23 19 112 22
4 7 ?= 5
− 9 × 5
= 45
= 2 45
?= × 17 + × 15
17 15
= 11 15. Solution (4)
500 × 3.2 ?× 2.4
× = 288
5. Solution (3) 100 100
√? = 4 × 7 + 32 ⇒ 16 × 2.4 × ? = 288 × 100
= 60 288 × 100
⇒ ?= = 750
⇒ ? = 3600 16 × 2.4

16. Solution (4)


6. Solution (5)
800 ×? 750 × 22
14 25 21 1 = 293 −
?= − × × 100 100
5 7 10 5
14 3 ⇒ 8 × ? = 293 − 165 = 128
= − 128
5 2 ⇒ ?= = 16
13 3 8
= =1
10 10
17. Solution (2)
7. Solution ((2) 10 45 3 22
?= ÷ × ×
90 3 7 2 7
?= 10 7 3 22 22
9 = × × × =
= 10 3 45 2 7 9

8. Solution (4) 18. Solution (2)


20 45 31 25.6
?= + − 250 × + √? = 119
7 14 14 100
27 6 ⇒ 64 + √? = 119
= =3
7 7 ⇒ √? = 119 − 64 = 55
⇒ ? = 3025
9. Solution (3)
? = 4,760
Chapter - 1 | Simplifications
नींव Booklet
For All Banking & Insurance Exam
Ch - 1 | Simplifications | Maths by Arun sir
19. Solution 28. Solution (5)
5 5 1 11
?= 4 − 5 +2 +( − + − )
6 9 3 18
15 − 10 + 6 − 11 ? = 4168 + 279 = 4447
⇒? = 1 + ( )=1+0=1
18 29. Solution (2)
3 2 1
20. Solution ?= 1 + + 2 + + 2 +
4 3 2
? = 621.65 3 2 1
=5+( + + )
4 3 2
21. Solution (2) 23 11 11
=5+ =5+1 =6
39 51 40 12 12 12
?= × 450 + × 340 – × 255
100 100 100
= 246.9 30. Solution (5)
6 42 35
= 500 × × ×
22. Solution (3) 7 100 100
2 5 7 2 = 63
? = (3 + 4 + 5 − 7) + ( + + − )
5 7 10 7
37
? =6 31. Solution (1)
70
8888 + 848 + 88 – ? = 7337 + 737
23. Solution (4) ⇒ 9824 – ? = 8074
? ⇒ ? = 9824 – 8074
= 67 × 7 + 73 × 5.1 – 52 × 2.2
10 = 1750
? = 7269
32. Solution (3)
24. Solution (1) 127512
?= 414×7
= 44
? = 3564 + 348.6 + 574.34 – 673.85
= 3813.09
33. Solution (1)
55 60
25. Solution (3) ?= × 280 + × 555 = 487
? 3 15 13 52 100 100
× × × × 544 =
5 4 17 6 7
52 34. Solution (3)
⇒ ?× 52 × 3 = 4 7 121
7 ?= × × × 81 = 231
1 9 11 12
⇒ ?=
21
35. Solution (1)
26. Solution (2) 8 15 12 5 133 1
?= + × × = =4
? = 86.66 + 200 + 253 3 4 5 33 33 33
= 539.66
36. Solution (4)
1 2 11 1
27. Solution (1) ? = (5 + 3 − 4 − 1) + ( + − − )
6 5 15 3
25 28 12+? 500 1 5 1
× × = = 3− = = 2
7 5 6 9 2 2 2
50
⇒ ?= –1
3
Chapter - 1 | Simplifications
नींव Booklet
For All Banking & Insurance Exam
Ch - 1 | Simplifications | Maths by Arun sir
37. Solution (1) 46. Solution (5)
?=
46×450+84×525−55×880
= 164 ?2 − 441 = 80 × 5
100
⇒ ? = √841
38. Solution (2) ⇒ ? = 29

3 47. Solution (3)


√256 + 320 + 4 × ? = 3×16 84×84 1
7 ×? = × 12 × 24
? = 32 28
? = 18
39. Solution (5)
? (2.5 − 1) = 48 × 2.5 48. Solution (2)
120 7.9 3.4
Or , ? = = 80 ? = 100 × 134 − 100 × 79 = 7.9
1.5

40. Solution (1) 49. Solution (1)


215 ÷ 5 =? −215 (?)² = 2 × 7√8 – 21 + 8 + 49 – 14√8
? = 43 + 215 = 258 = 57 – 21
= 36
41. Solution (4) ∴ ? = ±6
2.4
16 × ×? = 288
100 50. Solution (4)
? = 750 ?
× 19.07 = 359.56 – 207
100
42. Solution (2) 152.56 × 100
?=
(−251 × 21 × (−12)) 15813 19.07
= = 400 ? = 800
? 100

43. Solution (4)


130 × 130 1
?= [ × 15] = 338
25 30

44. Solution (5)


? = 24.375 − 1.955 = 22.420

45. Solution (4)


? = 13.141 + 31.417 – 27 .118
= 17.44
नींव Booklet
For All Banking & Insurance Exam
Ch - 2 | Approximations | Maths by Arun sir

What approximate value should come in place (5) 2019


of question mark (?) in the following questions?
10. ? % of (5224 ÷ 5.001 × 3 ÷ 11) = 375.05
SBI PO PRELIMS 2022 (1) 80 (2) 32
1. (√625.021 + √1599.78) ÷ (3) 98 (4) 58
(560.31% of 30 – 250.23% of 62) = ? (5) 132
(1) 2 (2) 3
(3) 4(4) 5 IBPS PO PRELIMS 2022
(5) 6
11. 499.99 + 1999 ÷ 39.99 × 50.01 = ?
2. ? = 729.11 ÷ 26.97 × 81.11 ÷ 35.91 × 108.18 (1) 3200 (2) 2700
(1) 2187 (2) 6561 (3) 3000 (4) 2500
(3) 729 (4) 243 (5) 2400
(5) 81
12. 73.99% of 1299 + 9.98% of 1899 = ?
3. 2
? + (32.08)2 – √144.44 × (7.99)2 = 72.9% of 401.1 (1) 1250 (2) 1230 (3) 1150
(1) 3(2) 12 (4) 1180 (5) 1200
(3) 8(4) 4
(5) 6 13. 67% of 801 – 231.17 = ? – 23% of 789
(1) 490 (2) 440 (3) 540
4. 47.87% of 350 + 60.11% of 280 = 96.98% of (4) 520 (5) 590
299.78 +? % of 150 1
(1) 30 (2) 40 14. (15.95)4 + (3.01)3 − 111.99 × 2.02 +
(3) 50 (4) 80 (9.98)2 = ?
(5) 45 (1) 95 (2) –95
(3) 105 (4) –105
5. 2? = 128.18 × 511.77 ÷ 2047.59 ÷ 31.89 × 15.91 (5) –115
(1) 3(2) 6
(3) 8(4) 5 (5) 4 15. 126.99% of 1539.98 + 5.5% of 149.99 + 103.98%
of 7 = ?
RRB PO PRELIMS 2022 (1) 1860 (2) 1970
(3) 2080 (4) 2150
6. 16.02 + 144 ÷ 23.96+ ? = 83.567 (5) 1055
(1) 16 (2) 62
(3) 58 (4) 30 SBI PO PRELIMS 2020
(5) 44 16. 1559.95 −7.99 × 24.96 − ?2 = 1154
(1) 14 (2) 24
7. 2430 ÷ 16 − 16.97 + √(? ) = 164 (3) 32 (4) 18
(1) 1089 (2) 841 (5) 8
4
(3) 1369 (4) 289 17. 1599 ÷ 39.99 + 5 × 2449 − 120.05 = ?
(5) 529 (1) 1680 (2) 1940
(3) 1640 (4) 1880
8. 9601÷ 11.98 × √(529.01) + 95.88 = ? (5) 1780
(1) 17470 (2) 17496 18. 576 ÷ 45.02 + 23.99 × √255 =?
(3) 18496 (4) 18086 (1) 340 (2) 420
(5) 18156 (3) 380 (4) 460
9. 156.25 × 12.4 + 1.8 × 52.05 =? −175.85 (5) 360
(1) 2025 (2) 2000
(3) 2128 (4) 2152 19 . ? +30.01% of 651 ÷ 25.05% of 59.98 = 135
नींव Booklet
For All Banking & Insurance Exam
Ch - 2 | Approximations | Maths by Arun sir

(1) 68 (2) 140


(3) 122 (4) 78 29. √?+ √1229 × 14 − 12.06 = 511
(5) 128.5 (1) 729 (2) 1849
(3) 2209 (4) 1369
20. 3899 ÷ 11.99 – 2379 ÷ 13.97 =? (5) 1089
(1) 125 (2) 250
(3) 155 (4) 135 IBPS PO PRELIMS 2019
(5) 225
30. 1019.95 × 5.04 + 237 − 302.11 = ?
RRB PO PRELIMS 2020 (1) 5125 (2) 5000
(3) 5035 (4) 5005
21. √575 ÷? × (14.98)2 = 450 (5) 5085
(1) 15 (2) 10
(3) 7(4) 4 31. 23% of 6783 + 57% of 8431 = ?% of 12720
(5) 12 (1) 40 (2) 60
(3) 55 (4) 50
22. (30.01)2 − (19.98)2 −? = (21.81)2 (5) 45
(1) 49 (2) 50
(3) 30 (4) 39 32. 180% of 25501 + 50% of 28999 =?
(5) 16 (1) 62400 (2) 64000
(3) 60400 (4) 64200
23. 820.15 + 2379.85 + 140.01 × 4.99 = ? (5) 60600
(1) 4400 (2) 3900
(3) 3000 (4) 4000 33. 171.995 × 14.995 ÷ 25 = ?
(5) 4300 (1) 103 (2) 115
(3) 110 (4) 125
24. 39.97% of 649.8 ÷ 13.05 = 45.12− ? (5) 118
(1) 40 (2) 15
(3) 25 (4) 10 34. 175 × 28 + 275 × 27.98 = ?
(5) 30 (1) 11800 (2) 12600
(3) 12800 (4) 11600
25. (674.87 +59.98) ÷ 35.02 = ? (5) 16200
(1) 29 (2) 27 (3) 19
(4) 21 (5) 11 35. 324.995 × 15.98 ÷ 4.002 + 36.88 = ?
(1) 1300 (2) 1230
RRB PO PRELIMS 2019 (3) 1340 (4) 1380
(5) 1390
26. 30.06% of 560.14 + 53.02% of 1100) ÷ 8 = ?
(1) 78 (2) 94 IBPS PO PRELIMS 2018
𝑥
(3) 99 (4) 81 36. 294.0× 8.01 − 19.99% of 119.99x = 254.9 ÷ 1.9
(5) 85 (1) 10 (2) 16
(3) 8 (4) 5
27. ? × 5 × 4.92 − 13.13 × 4.02 × 4 = 117 (5) 12
(1) 18 (2) 13 (3) 7
(4) 21 (5) 9 410
37. 55.2 × 69 + 499.9 − 127.9 =? % of 5600
28. (9117.88 − 8021.85 + 903.92) × 12 = 1500 ×? (1) 17.5 (2) 12.5
(1) 12 (2) 8 (3) 10 (4) 15
(3) 15 (4) 22 (5) 8
(5) 4
नींव Booklet
For All Banking & Insurance Exam
Ch - 2 | Approximations | Maths by Arun sir

1
38. 630 ×? +599.85 − 55% of 1200037 2 % of 9200 Q.45 (23.89)2 + √3598.97 − 171.93 + (? )2
(1) 12 (2) 18 = (27.98)2 + 4
(3) 15 (4) 20 (1) 18 (2) 24
(5) 8 (3) 28 (4) 22
(5) 26
11999.87 1
39. + 54.9% of 1800 − 389.9 = 11 % of 9900
? 9 SBI PO PRELIMS 2018
(1) 24 (2) 28
(3) 20 (4) 18 46 (13.97)2− 39.87% of 239.97 + (29.87)2 +
(5) 32 330.97 = (? )3
(1) 18 (2) 19
31 5 1 (3) 17 (4) 11
40. 8099.9 × + 2699.8 × − 1799.8 = 62 %of ? (5) 15
27 3 2
(1) 22500 (2) 15200
(3) 16200 (4) 18200 ?+134.5
47 + 209.87 + (69.87)2 − 1999.83 =
(5) 19200 24
(54.87)2 + 99.85
SBI PO PRELIMS 2019 (1) 235.5 (2) 245.5
(3) 225.5 (4) 215.5
(5) 205.5
41. 119.89% of 2000.23 + (35.96)2 −
59.49% of 2999.89 = (49.89)2 −? 48. ? × 12.97 + (24.97)2 − (19.89)2 + 363.83 =
(1) 559 (2) 569 (27.93)2
(3) 589 (4) 549 (1) 10 (2) 20
(5) 539 (3) 18 (4) 12
(5) 15
42 (11.87)2 + 12.493 × 15.89 − √13224.98 −
(? )2 = (14.96)2 593.89 1259.81
49. ?
+ 14.87 × 35.88 + 17.93 = (25.89)2
(1) 9(2) 2
(3) 8(4) 5 (1) 12 (2) 15
(5) 10 (3) 13 (4) 9
(5) 11
359.93
Q.43 = (8.89)3 − 14.5 × 39.89 + (1.95)2 − 34
? 50. 48.01% of 249.99 + x% 499.99 = (11.01)2 +
(1) 8(2) 2 (2.11)2
(3) 3 (4) 9 (1) 2(2) 3
(5) 5 (3) 5(4) 1
(5) 6
Q.44 √429.87 + 520.23 + √120.97 = (? )2 + √35.98
(1) 2(2) 3
(3) 8(4) 5
(5) 9
नींव Booklet
For All Banking & Insurance Exam
Ch - 2 | Approximations | Maths by Arun sir

Answer Key
1. (4) 11. (3) 21. (5) 31. (4) 41. (3)
2. (2) 12. (3) 22. (5) 32. (3) 42. (2)
3. (5) 13. (1) 23. (2) 33. (1) 43. (5)
4. (1) 14. (2) 24. (3) 34. (2) 44. (4)
5. (5) 15. (2) 25. (4) 35. (3) 45. (1)
6. (2) 16. (1) 26. (2) 36. (1) 46. (4)
7. (2) 17. (4) 27. (2) 37. (2) 47. (3)
8. (3) 18. (2) 28. (3) 38. (3) 48. (5)
9. (4) 19. (3) 29. (5) 39. (1) 49. (4)
10. (5) 20. (3) 30. (3) 40. (5) 50. (4)
नींव Booklet
For All Banking & Insurance Exam
Ch - 2 | Approximations | Maths by Arun sir

Hints & Solutions


1. Solution (4) = 800 × 23 + 96 = 18496
560 × 30 250 × 62 9. Solution (4)
? ≃ (√625 + √1600) ÷ ( – )
100 100 156.25 × 12.4 + 1.8 × 52.05 =? −175.85
(25 + 40) 65
?≃ ≃ =5 ⇒ 156 × 12 + 2 × 52 =? −176
(168 – 155) 13
⇒ 1872 + 104 = ? −176
2. Solution (2)
729 × 81 × 108 ⇒ 1976 + 176 = ?
?≃ ∴ ? = 2152
27 × 36
= 6561 10. Solution (5)
3. Solution (5) 5224 3
? % of ( × ) = 375.05
?2 + (32)2 – √144 × 82 ≃ 73% of 400 5.001 11
? 5224 3
?² + 1024 – 12 × 64 = 292 ⇒ × × = 375
100 5 11
?² ≃ 292 – 1024 + 768 ∴ ?=
375×55×100 171875
= 1306
2×5224
?² ≃ 36
= 131.60
?≃6
≈ 132
4. Solution (1)
11. olution (3)
48% of 350 + 60% of 280 ≃ 97% of 300 +? % of
? ≈ 500 + 50 × 50
150
≈ 3000
⇒ ? × 1.5 ≃ 168 + 168 – 291
12. Solution (3)
45 74×1300 10×1900
?≃ = 30 ?≈ + 100
1.5 100
5. Solution (5) ≈ 960 + 190
128 × 512 × 16 ≈1150
2? ≃ ≃ 16
2048 × 32 13. Solution (1)
2? ≃ 24 ⇒ ? = 4 67 × 800 23 × 790
6. Solution (2) ?≈ − 231 +
100 100
16.02 + 144 ÷ 23.96+ ? = 83.567 ≈ 536 − 231 + 181.7
⇒ 16 +
144
+ ? = 84 ≈ 490
24
14. Solution (2)
⇒ 16 + 6 + ? = 84
129 − 224 = −95
⇒ 22 + ? = 84
15. Solution (2)
∴ ? = 84 − 22 = 62
? ≈ 1956 + 8.25 + 7.28 ≈ 1970
7. Solution (2)
2430 16. Solution (1)
− 16.97 + √? = 164
16 1560 − 8 × 25 −?2 = 1154
2430
⇒ 16 − 17 + √? = 164 ⇒ 1560 − 200 −?2 = 1154
⇒ 135 + √? = 164 ⇒ 1360 −?2 = 1154
⇒ ?2 = 1360 − 1154 = 206
⇒ √? = 29
∴ ? = 29 × 29 = 841 ∴ ? = √206 = 14
8. Solution (3) 17. Solution (4)
4
9601
× √529.01 + 95.88 =? ? = 1600 ÷ 40 + 5 × 2450 − 120
11.98
9601 = 1600 ÷ 40 + 1960 − 120
∴ ?= × 23 + 96
12 = 40 + 1960 − 120 = 1880
नींव Booklet
For All Banking & Insurance Exam
Ch - 2 | Approximations | Maths by Arun sir

18. Solution (2) 2000×12


∴ ?= 1500
= 16 ≈ 15
? = 1575 ÷ 45 + 24 × √256 29. Solution (5)
= 35 + 24 × 16 = 35 + 384 = 419 ≈ 420
√? + 35 × 14 − 12.06 = 511
19. Solution (3)
30×650 25×60 ⇒ √? + 490 − 12 = 511
?+ ÷ 100 = 135
100 ⇒ √? = 511 − 478 = 33
⇒ ? +195 ÷ 15 = 135 ∴ ? = (33)2 = 1089
⇒ ? +13 = 135 30. Solution (3)
∴ ? = 135 − 13 = 122 ? = 1020 × 5 + 237 − 302 = 5100 + 237 − 302
20. Solution (3) = 5337 − 302 = 5035
? = 3900 ÷ 12 − 2380 ÷ 14 31. Solution (4)
= 325 − 170 = 155
21. Solution (5)
√575 24×152
× (15)2 = 450 ⇒ = 450
? ?
2
⇒ 450 ×? = 24 × 15
24×225
∴ ?= = 12
450
22. Solution (5)
302 − 202 −? = 222 ⇒?
⇒ 900 − 400− ? = 484
⇒ 500− ? = 484
∴ ? = 500 − 484 = 16
23. Solution (2) ⇒ ? ≃ 50
? = 820.15 + 2379.85 + 140 × 5 32. Solution (3)
= 3200 + 700 = 3900 ?≃ 1.8×25500+50/100
24. Solution (3) ×29000 ≃ 60,400
40% × 650 ÷ 13 = 45− ? 33. Solution (1)

40×650 1
× 13 = 45−? ? ≃ 172 × 15 ÷ 25 ≃ 103
100
34. Solution (2)
⇒ 20 = 45−?
? ≃ 175 × 28 + 275 × 28 ≃ 12,600
∴ ? = 45 − 20 = 25
35. Solution (3)
25. Solution (4)
735
? ≃ 325 × 16 ÷ 4 + 37 ≃ 1337 ≃ 1340
? = (675 + 60) ÷ 35 = 35
= 21 36. Solution (1)
26. Solution (2) x 1 255
≈ 294 × − × 120x =
560.14×30.06 53.02×1100 8 5 2
?= ( + )÷8 ≈ 36.75x − 24x = 127.5
100 100
(168.38+583.22) 751.6 ≈ 12.75x = 127.5
= 8
= 8 = 93.95 ≈ 94
≈ x = 10
27. Solution (2)
37. Solution (2)
? × 5 × 5 − 13 × 4 × 4 = 117 X
⇒ 25 ×? = 117 + 208 = 325 100
× 5600 = 8 × 41 + 500 − 128
325 700
∴ ?= = 13 X= 56
25
28. Solution (3) X = 12.5
(9118 − 8022 + 904) × 12 = 1500 × ? 38. Solution (3)
⇒ 1500 ×? = 2000 × 12
नींव Booklet
For All Banking & Insurance Exam
Ch - 2 | Approximations | Maths by Arun sir

630X + 600 = 37.5 ×


9200
+
55
× 12000 31 − 6 = (? )2
100 100
?= 5
630X + 600 = 3450 + 6600
45. Solution (1)
630X = 10050 − 60
9450 (23.89)2 + √3600 − 172 + (? )2 = (28)2 + 4
X= 360 576 + 60 − 172 + (? )2 = 784 + 4
X = 15 464 + (? )2 = 788
(? )2 = 788 − 464
? = 18
39. Solution (1) 46. Solution (4)
12000 55 100 1
= X
+ 100 × 1800 = 9 × 100 × 9900 + 390 (14)2 − 40% of 240 + (30)2 + 331 = (? )3
12000 40×240
= X = 1490 − 990 196 − + 900 + 331 = (? )3
100
12000
X = 500 1000 + 331 = (? )3
? = 11
X = 24
47. Solution (3)
40. Solution (5) ? +134.5
62.5X 31 5 + 210 + (70)2 − 2000 = (55)2 + 100
= 8100 × + 2700 × − 1800 24
100 27 3 ? +134.5
5X + 3110 = 3125
= 9300 + 4500 − 1800 24
8
12000×8 ? +134.5 = 15 × 24
X=
5 ? = 360 − 134.5
X = 19200 ? = 225.5
41. Solution (3) 48. Solution (5)
120 59.5
100
× 2000 + (36)2 − 100 × 3000 = (50)2 −? ? × 13 + (25)2 − (20)2 + 364 = (28)2
2400 + 1296 − 1785 = 2500−? ? × 13 + 225 + 364 = 784
2500 − 1911 = ? ? = 589 ? × 13 = 784 − 589
195
42. Solution (2) ?= 13
(12)2 + 12.5 × 16 − √13225 − (? )2 = (15)2 ? = 15
144 + 200 − 115 − (? )2 = 225 49. Solution (4)
(? )2 = 229 − 225 594
+ 15 × 36 +
1260
= (26)2
? 18
? =2 594
43. Solution (5) ?
+ 540 + 70 = 676
360 594 =? (676 − 610)
?
= (9)3 − 14.5 × 40 + (2)2 − 81
594
360 ?=
= 729 − 580 + 4 − 81 66
?
360 ?= 9
?= 72 50. Solution (4)
?= 5 48 x
× 250 + 100 × 500 ≈ 112 + 22
100
44. Solution (4)
120 + 5x ≈ 121 + 4
√430 + 520 + √121 = (? )2 + √36
5x ≈ 5
√430 + 520 + 11 = (? )2 + 6 x≈1
नींव Booklet
For All Banking & Insurance Exam
Ch - 3 | Missing Number Series | Maths by Arun sir
What will come in place of (?) in the following
number series problems? Q10. 1 2 4 6 9 12 ?
Q1. 5, 13, 58, 357, 2868, ? (1)15 (2) 16
(1) 25823 (2) 28695 (3) 17 (4) 18
(3) 29548 (4) 28545 (5) 20
(5) 27695
Q11. 23, 29, 17, 35, ?, 41
Q2. 495, 485, 465, 425, 345, ? (1)13 (2) 11
(1) 195 (2) 165 (3) 12 (4) 18
(3) 185 (4) 175 (5) 21
(5) 285
Q12. 2, 4, 8, 15, 26.5, ?
Q3. 32, 36, 52, 88, 152, ? (1) 47.45 (2) 43.75
(1) 266 (2) 252 (3) 44.75 (4) 45.54
(3) 422 (4) 256 (5) 42.75
(5) 225
Q13. 528, 840, 960, 1368, ?
Q4. 13, 27, 55, 97, 153, ? (1)1680 (2) 1849
(1) 243 (2) 265 (3) 1521 (4) 1296
(3) 215 (4) 223 (5) 1225
(5) 232
Q14. 3, 5, 19, 101, ?, 6445
Q5. 3, 4, 10, 33, 136, ? (1)710 (2) 712
(1) 903 (2) 822 (3) 714 (4) 715
(3) 685 (4) 548 (5) 725
(5) 658
Q15. 7, 8, 16, 12, 41, 28, ?
Q6. 1 5 14 30 55 ? (1)90 (2) 85
(1) 91 (2) 101 (3) 92 (4) 86
(3) 81 (4) 92 (5) 84
(5) None of these
Q16. 294, 298, 316 , 364, 464 , ?
Q7. 7 11 13 17 19 ? (1) 680 (2) 644
(1) 21 (2) 23 (3) 608 (4) 716
(3) 25 (4) 27 (5) 624
(5) None of these

Q8. 3 4 7 12 19 ? Q17. 7, 11, 31, 109, ?, 2341


(1) 28 (2) 39 (1) 352 (2) 570
(3) 27 (4) 26 (3) 461 (4) 453
(5) None of these (5) 471
Q9. 120 240 160 64 ?
(1) 18.3 (2) 16.04 Q18. 221 , 226 , 233, 244 , 257, 274, ?
(3) 16.40 (4) 32 (1) 293 (2) 291
(5) None of these (3) 289 (4) 295
नींव Booklet
For All Banking & Insurance Exam
Ch - 3 | Missing Number Series | Maths by Arun sir
(5) 297 (3) 170 (4) 210
(5) 240
Q19. 2, 5, 8, 22 , 64, ?
(1) 290 (2) 162 Q28. 48, 24, 72, 18, 90, ?
(3) 258 (4) 226 (1) 9 (2)12
(5) 224 (3) 21 (4)18
(5) 15
Q20. 133, 124.5, 141.5, 107.5, 175.5, ?
(1) 43.5 (2) 39.5 Q29. 9, 6, 9, 24, 108, ?
(3) 37.5 (4) 107.5 (1) 888 (2) 864
(5) 243.5 (3) 872 (4) 878
(5) 882
Q21. 64, 128, 92, 220, 148, 404, ?
(1)260 (2) 276 Q30. 15, 30, 50, 80, 130, ?
(3) 296 (4) 332 (1) 200 (2) 210
(5) 280 (3) 220 (4) 230
(5) 240
Q22. 6, 7, 9, 13, ?, 37, 69
(1) 25 (2) 21 Q31. 983, 1080, 886, 1177, 789, ?
(3) 35 (4) 37 (1)1280 (2)1278
(5) 27 (3)1276 (4)1274
(5)1272
Q23. 15, 21, 39, 77, 143, ?
(1) 243 (2) 240 Q32. 0, 6, 24, 60, ? , 210
(3) 253 (4) 245 (1) 80 (2) 90
(5) 250 (3) 120 (4) 150
(5) 180
Q24. 1, 5, 13, ?, 61, 125, 253
(1)21 (2) 49 Q33. 16, 16, 8, 24, 6, ?
(3) 29 (4) 31 (1) 30 (2) 24
(5) 45 (3) 18 (4) 27
(5) 36
Q25. 3, 5, 13, 43, ?, 891, 5353
(1)175 (2) 135 Q34. 786, 1810, 2066, 2130, 2146, ?
(3) 436 (4) 177 (1) 2152 (2) 2150
(5) None of these (3) 2156 (4) 2160
(5) 2164

Q26. 1511, 302, 75, 24, ?, 5


(1) 12 (2) 10 (3) 8
(4) 15 (5) 9 Q35. 9, 5, 6, 14, ?, 488
(1) 30 (2) 45
Q27. 7, 10, 18, 42, 90, ? (3) 52 (4) 56
(1) 150 (2) 160 (5) 60
नींव Booklet
For All Banking & Insurance Exam
Ch - 3 | Missing Number Series | Maths by Arun sir
(4) 108 (5) None of these
Q36. 2, 4, 10, 22, 42, ?
(1) 7 (2) 70 Q45. 13, 29, 50, 81, 127, 193, ?
(3) 72 (4) 75 (1)165 (2) 195
(5) 78 (3)180 (4) 190
(5) 284
Q37. 8, 4, 6, 15, ? , 236.25
(1) 46.5 (2) 48.5 Q46. 7, 30, 66, 117, 186, ? , 400
(3) 50.5 (4) 52.5 (1) 255 (2) 276
(5) 54.5 (3) 287 (4) 278
(5) 308
Q38. 15, 34, 72, 148, ? , 604
(1) 300 (2) 290 Q47. 4, 9, 17, 36, ? , 143, 283
(3) 295 (4) 280 (1) 72 (2) 70
(5) 285 (3) 68 (4)141
(5) 69
Q39. 25, 30, 20, 40, ? , 80
(1) 40 (2) 20 Q48. 123, 340, 724, ?, 2186
(3) 10 (4) 0 (1) 1320 (2) 2184
(5) 60 (3) 1324 (4) 1322
(5) 2190
Q40. 15, 8, 9, 15, 32, ?
(1) 77.5 (2) 80 Q49. 2, 4, 7, 12, 23, ?
(3) 82.5 (4) 85 (1) 56 (2) 67
(5) 87.5 (3) 43 (4)148
(5) 76
Q41. 711, 723, 747, 783, 831, 891, ?
(1) 445 (2) 963 Q50. 129, 142, 181, ?, 337, 454
(3) 1005 (4) 1275 (1) 324 (2) 298
(5) 1260 (3) 272 (4) 220
(5) 246
Q42. 13, 13, 32.5, 130, 715, ?
(1) 5420 (2)1550
(3) 5005 (4) 2055
(5) 4520

Q43. 16, 17, 15, 18, 14, ?


(1) 10 (2) 17
(3) 18 (4) 19
(5) 23

Q44. 121, 120, 124, 115, 131, ?


(1) 106 (2) 122 (3) 98
नींव Booklet
For All Banking & Insurance Exam
Ch - 3 | Missing Number Series | Maths by Arun sir

Answer Key
1. (2) 12. (3) 23. (3) 34. (2) 45. (5)
2. (3) 13. (1) 24. () 35. (5) 46. (3)
3. (2) 14. (4) 25. () 36. (3) 47. (2)
4. (4) 15. () 26. (2) 37. () 48. (3)
5. (3) 16. (2) 27. (4) 38. (4) 49. (1)
6. (1) 17. (3) 28. (5) 39. (1) 50. (5)
7. (2) 18. (1) 29. (1) 40. (3)
8. (1) 19. (4) 30. (3) 41. (2)
9. (1) 20. (2) 31. (3) 42. (3)
10. (2) 21. (1) 32. (3) 43. (4)
11. () 22. (2) 33. (1) 44. (1)
नींव Booklet
For All Banking & Insurance Exam
Ch - 3 | Missing Number Series | Maths by Arun sir

Hints & Solutions


1. Solution. (2) 1
∴ ? = 64 × 3.5 = 18.3
Series is
5 × 2 + 3 = 13
10. Solution. (2)
13 × 4 + 6 = 58 Pattern is (+1, +1, +2, +2, +3, +3, +4, +4)
58 × 6 + 9 = 357 ∴ ? = 12 + 4 = 16
357 × 8 + 12 = 2868 11. Solution.
2868 × 10 + 15 = 28695
2. Solution. (3)
Pattern is
12. Solution. (3)
– 10, −20, −40, −80, −160
2 × 1.5 + 1 = 4
∴ ? = 345 – 160
4 × 1.5 + 2 = 8
= 185
8 × 1.5 + 3 = 15
3. Solution. (2)
15 × 1.5 + 4 = 26.5
Series is /श्रृंखला है 26.5 × 1.5 + 5 = 44.75
13. Solution. (1)
23² - 1 = 528
29² - 1 = 840
31² - 1 = 960
4. Solution. (4) 37² - 1 = 1368
Pattern is 41² - 1= 1680
+14 × 1, +14 × 2, +14 × 3, +14 × 4, +14 × 5 14. Solution. (4)
∴ ? = 153 + 70 3×1+2=5
= 223 5 × 3 + 4 = 19
5. Solution. (3) 19 × 5 + 6 = 101
Pattern is 101 × 7 + 8 = 715
× 1 + 1, × 2 + 2, × 3 + 3, × 4 + 4, × 5 + 5 715 × 9 + 10 = 6445
∴ ? = 136 × 5 + 5 15. Solution.
= 685
6. Solution. (1)
Pattern is
(+2², +3², +4², +², +6²)
∴ ? = 55 + 36 = 91 16. Solution. (2)
7. Solution. (2) 294 + 1×4 = 294 + 4 = 298
Pattern fallows consecutive prime numbers 298 + 2×9 = 298 + 18 = 316
∴ ? = 23 316 + 3×16 = 316 + 48 = 364
8. Solution. (1) 364 + 4× 25 = 364 + 100 = 464
Pattern is (+1, +3, +5, +7, +9) 464 + 5× 36 = 464 + 180 = 644
∴ ? = 28 ∴ ? = 464 + 180 = 644
9. Solution. (1) 17. Solution. (3)
1 1 1 1
Pattern is (× 0.5 , × 1.5 , × 2.5 , × 3.5) Pattern is
7 × 1 + 4 = 11
नींव Booklet
For All Banking & Insurance Exam
Ch - 3 | Missing Number Series | Maths by Arun sir
11×2 + 9 = 31 24. Solution.
31×3 + 16 = 109 The pattern is +2², +2³, +24, +25…….
109×4 + 25 = 461 25. Solution.
461×5 + 36 = 2341 × 1+2, ×2+ 3, ×3 + 4, ×4 + 5……
18. Solution. (1)
Pattern is
221 + 5 = 226 26. Solution. (2)
226 + 7 = 233 (1511 – 1) ÷ 5 = 302
233 + 11 = 244 (302 – 2) ÷ 4 = 75
244 + 13 = 257 (75 – 3) ÷ 3 = 24
257 + 17 = 274 (24 – 4) ÷ 2 = 10
274 + 19 = 293 (10 – 5) ÷ 1 = 5
19. Solution. (4) 27. Solution. (4)
Pattern is
2×1.5 + 2 = 5
5×2 – 2 = 8
8×2.5 + 2 = 22
22×3 – 2 = 64
64×3.5 + 2 = 226 28. Solution. (5)
20. Solution. (2)
Pattern is
133 – 8.5 = 124.5
124.5 + 17 = 141.5 29. Solution. (1)
141.5 – 34 = 107.5
107.5 + 68 = 175.5
175.5 – 136 = 39.5
21. Solution. (1) 30. Solution. (3)

22. Solution. (2)


31. Solution. (3)

23. Solution. (3)

32. Solution. (3)

33. Solution. (1)


नींव Booklet
For All Banking & Insurance Exam
Ch - 3 | Missing Number Series | Maths by Arun sir
43. Solution. (4)
Pattern is

34. Solution. (2)

44. Solution. (1)


121 – 12 = 120
120 + 22 = 124
124 – 32 = 115
35. Solution. (5) 115 + 42 = 131
131 – 52 = 106
45. Solution. (5)
Pattern is
36. Solution. (3)

37. Solution. ∴ ? = 193 + 91= 284


46. Solution. (3)
Series is,
38. Solution. (4)

39. Solution. (1)


47. Solution. (2)
Pattern is × 2 + 1, × 2 – 1, × 2 + 2, × 2 – 2, × 2 + 3,
×2–3…
∴ ? = 36 × 2 – 2 = 70
40. Solution. (3) 48. Solution. (3)
Series is 5³ – 2 = 125 – 2 = 123
7³ – 3 = 343 – 3 = 340
9³ – 5 = 729 – 5 = 724
41. Solution. (2) 11³ – 7 = 1331 – 7 = 1324
Pattern is 13³ – 11 = 2197 – 11 = 2186
+12, +24, +36, +48, +60, +72 49. Solution. (1)
∴ ? = 891 + 72= 963 Pattern is × 4 – 4, × 4 – 9, × 4 – 16, × 4 – 25, …
42. Solution. (3) ∴ ? = 23 × 4 – 36= 56
Pattern is 50. Solution. (5)
×1, × 2.5, ×4, ×5.5, ×7, ×8.5…… Pattern is +13×1, +13×3, +13×5, +13×7, +13×9, …
∴ ? = 715×7= 5005 ∴ ? = 181 + 13 × 5 = 246
नींव Booklet
For All Banking & Insurance Exam
Ch - 3 | Missing Number Series | Maths by Arun sir
नींव Booklet
For All Banking & Insurance Exam
Ch - 4 | Wrong Number Series | Maths by Arun sir
What number is wrong according to given number
series pattern : – Q10. 48, 62, 96, 224, 992, 7136
Q1. 36, 71, 117, 175, 246, 332 (a) 48 (b) 62
(a) 71 (b) 117 (c) 224 (d) 992
(c) 246 (d) 175 (e) 7136
(e) 332
Q11. 17, 20, 25, 37, 57, 87, 129
Q2. 18, 72, 36, 150, 72, 288 (a) 17 (b) 129
(a) 72 (b) 288 (c) 25 (d) 87
(c) 150 (d) 36 (e) 20
(e) 18
Q12. 128, 64, 96, 240, 840, 3800, 20790
Q3. 25, 31, 44, 63, 93, 135 (a) 3800 (b) 128
(a) 31 (b) 44 (c) 20790 (d) 96
(c) 93 (d) 63 (e)240
(e) 135
Q13. 14, 20, 40, 82, 154, 264, 450
Q4. 15, 27, 51, 87, 135, 196 (a) 154 (b) 20
(a) 87 (b) 27 (c) 264 (d) 14
(c) 15 (d) 51 (e) 450
(e) 196
Q14. 64, 56, 65, 49, 74, 38, 87
Q5. 47, 57, 81, 130, 211, 332 (a) 87 (b) 64
(a) 57 (b) 130 (c) 38 (d) 56
(c) 211 (d) 332 (e) 49
(e) 81
Q15. 2, 5, 11, 35, 143, 719, 4319
Q6. 31, 53, 105, 182, 280, 391 (a) 11 (b) 719
(a) 391 (b) 31 (c) 5 (d) 4319
(c) 280 (d) 53 (e) 2
(e) 105
NEW PATTERN NUMBER SERIES
Q7. 1, 1, 3, 23, 367, 11745 Directions : - In each of the following questions, a number
(a) 11745 (b) None of these series is given. After the series a number is given
(c) 3 (d) 23 following by (a), (b), (c), (d) and (e). You have to
(e) 367 complete the series with the number given, following
the sequence of the original series and answer the
Q8. 125, 127, 137, 163, 213, 296 question that follow the series.
(a) 125 (b) 127
(c) 163 (d) 296 Q16. 7 5 11 49 335 3005
(e) 213 13 (a) (b) (c) (d) (e)

Q9. 675, 338, 170, 86, 44, 23 What will come in place of (b) ?
(a) 23 (b) 338 a) 31 b) 27
(c) 170 (d) 44 c) 29 d) 28
(e) 675 e) 30
नींव Booklet
For All Banking & Insurance Exam
Ch - 4 | Wrong Number Series | Maths by Arun sir

Q21. 3 13 37 87 191 401


Q17. 68 60 104 188 496 1456 1 (a) (b) (c) (d) (e)
42 (a) (b) (c) (d) (e) What will come in place of (d) ?
What will come in place of (b) ? a) 169 b) 161
a) 304 b) 286 c) 171 d) 159
c) 293 d) 281 e) None
e) 301 Q22. 8 4 6 15 52.5 236.25
12 (a) (b) (c) (d) (e)
Q18. 37 19 20 31.5 65 165 What will come in place of (c) ?
21 (a) (b) (c) (d) (e) a) 18.25 b) 19
What will come in place of (e) ? c) 22.5 d) 20.75
a) 105 b) 41 e) None
c) 110 d) 108
e) 116 Q23. 2 3 2 15 76 254 1434
3 (a) (b) (c) (d) (e) (f)
Q19. 5 9 25 91 414 2282.5 What will come in place of (c) ?
3 (a) (b) (c) (d) (e) a) 18 b) 22
What will come in place of (c) ? c) 24 d) 21
a) 63.25 b) 63.75 e) None
c) 64.75 d) 64.25
e) None Q24. 2 4.5 11 30 93 312 1136
1 (a) (b) (c) (d) (e) (f)
Q20. 3 5 9 17 33 65 What will come in place of (b) ?
7 (a) (b) (c) (d) (e) a) 6 b) 81
What will come in place of (d) ? c) 16.75 d) 18.75
a) 95 b) 51 e) None
c) 99 d) 49
e) None Q25. 2 14 18 46 82 176 338
4 (a) (b) (c) (d) (e) (f)
What will come in place of (e) ?
a) 238 b) 338
c) 218 d) 318
e) None
नींव Booklet
For All Banking & Insurance Exam
Ch - 4 | Wrong Number Series | Maths by Arun sir

Answer Key
1. (5) 8. (4) 15. 5) 22. ()
2. (3) 9. (5) 16. () 23. ()
3. (2) 10. (2) 17. () 24. ()
4. (5) 11. (5) 18. () 25. ()
5. (1) 12. (1) 19. ()
6. (3) 13. (5) 20. ()
7. (1) 14. (2) 21. ()
नींव Booklet
For All Banking & Insurance Exam
Ch - 4 | Wrong Number Series | Maths by Arun sir

Hints & Solutions


1. Solution.(5)
Patter is :

So, Wrong number is — 332


Right number should be = 246 + 85 = 331 7. Solution.(1)
2. Solution.(3)
Pattern is —
18×4=72, 8. Solution.(4)
72 ÷2=36,
36 ×4=144,
144 ÷2=72 9. Solution.(5)
72 ×4=288
So wrong number is — 150
3. Solution.(2)
Pattern is — 10. Solution.(2)
25+(2² + 2)=31
31+(3² + 3)=43
43 +(4² + 4)=63,
63 +(5² + 5)=93,
93 +(6² + 6)=135 11. Solution.(5)
So, wrong number is — 44
4. Solution.(5)
Pattern is —
15+(13×1-1)=27
27+(13×2-2)=51
51+(13×3-3)=87 12. Solution.(1)
87+(13×4-4)=135
135+(13×5-5)=195
Wrong number is 196
5. Solution.(1)
47+32 =56 13. Solution.(5)
56+52 =81
81+72 =130
130+92 =211
211+112 =332
Wrong number is 57
6. Solution.(3) 14. Solution.(2)
नींव Booklet
For All Banking & Insurance Exam
Ch - 4 | Wrong Number Series | Maths by Arun sir

15. Solution.(5)

21. Solution.()
3 13 37 87 191 401

× 2+ 7 × 2 + 11 × 2 + 13 × 2 + 17 × 2 + 19

Similarly
16. Solution.() 1 9 (a) 29 (b) 71 (c) 159 (d)
7 5 11 49 335 3005

× 1- 2 × 3–4 × 5–6 × 7–8 × 9 – 10 22. Solution.()


Similarly, 8 4 6 15 52.5 236.25

13 11 29(b)
× 0.5 × 1.5 × 2.5 × 3.5 × 4.5
×1–2 ×3–4
17. Solution.()
68 60 104 188 496 1456
2 6 (a) 9 (b) 22.5 (c)

× 1- 8 × 1.5+ 14 × 2 – 20 × 2.5+ 26 × 3 – 32
× 0.5 × 1.5 × 2.5

Similarly, 23. Solution.()


2 3 2 15 76 254
42 34 (a) 65 (b) 110 (c) 301 (d) 1434

× 1+ 1 2 × 2-22 × 3+ 32 × 4-42 × 5+ 52 × 6-62

× 1- 8 × 1.5+ 14 × 2 – 20 × 2.5+ 26 Similarly


18. Solution.() 3 4 4 21 (c)
37 19 20 31.5 65 165

× 0.5 + 0.5 × 1+ 1 × 1.5 + 1.5 × 2+ 2 × 2.5 + 2.5 × 1+ 1 2 × 2-22 × 3+ 32

Similarly 24. Solution.()


21 11 (a) 12 (b) 19.5 (c) 41 (d) 105 (e) 2 4.5 11 30 93 312
1136

19. Solution.() × 2+ 0.5 × 2+ 0.5 × 4 × 2+ 2× 4 × 2+ 8× 4 × 2 + 32 × 4 × 2 + 128 × 4

5 9 25 91 414 Similarly
1 2.5 (a) 7 (b)

× 1.5 + 1.5 × 2.5 + 2.5 × 3.5 + .5 × 4.5 + 4.5

Similarly × 2+ 0.5 × 2+ (0.5) × 4

3 6 (a) 17.5 (b) 64.75 (c) 25. Solution.()


2 14 18 46 82 176
338

20. Solution.() × 2+ 10 × 2-10 × 2+ 10 × 2-10 × 2+ 10 × 2-10

3 5 9 17 33 65 Similarly
4 18 (a) 26 (b) 62 (c) 114 (d) 238 (e) 466 (f)

× 2–1 × 2–1 × 2–1 × 2–1 ×2–1


× 2+ 10 × 2-10 × 2+ 10
Similarly
7 13(a) 25 (b) 49 (c) 97 (d)
नींव Booklet
For All Banking & Insurance Exam
Ch - 4 | Wrong Number Series | Maths by Arun sir
In each of the following questions two equations 4 6
11. I.  5 x
are given. You have to solve the equations and x x
Give answer— II. y 2  256  625
(a) if x < y
(b) if x ≤ y
12. I. 𝑥 2 + 𝑥 − 12 = 0
(c) relationship between x and y cannot be
II. 𝑦 2 − 5𝑦 + 6 = 0
determined
(d) if x ≥ y
13. I. 12x2 – 29x + 15 = 0
(e) if x > y
II. 54y 2  20 18  33  0
1. I. 𝑥 2 + 13𝑥 + 42 = 0
II. 𝑦 2 + 19𝑦 + 90 = 0 E 14. I. 4x3 + 24x2 – 64x = 0
II. 3y2 + 39y + 126 = 0
2. I. 𝑥 2 + 15𝑥 + 56 = 0
II. 𝑦 2 − 23𝑦 + 132 = 0 A 15. I. 2𝑥 2 − 15𝑥 + 27 = 0
II. 2𝑦 2 − 13𝑦 + 20 = 0
3. I. 𝑥 2 − 22𝑥 + 120 = 0 B
II. 𝑦 2 − 26𝑦 + 168 = 0 16. I. 9𝑥 2 − 21𝑥 + 10 = 0
II. 𝑦 2 − 8𝑦 + 15 = 0
4. I. 𝑥 2 + 12𝑥 + 32 = 0
D
II. 𝑦 2 + 17𝑦 + 72 = 0 17. I. 2𝑥 2 − 13𝑥 + 15 = 0
II. 2𝑦 2 − 11𝑦 + 12 = 0
5. I. x2 – 7 √3 x + 36 = 0
B
II. y2 – 11 √3 y + 84 = 0 18. I. 2𝑥 2 + 7𝑥 + 6 = 0
II. 2𝑦 2 + 17𝑦 + 30 = 0
6. I. 𝑥 = √625 A
II. y = √676 19. I. 𝑥 2 − 2𝑥 − √5𝑥 + 2√5 = 0
II. 𝑦 2 − √3𝑦 − √2𝑦 + √6 = 0
7. I. 2x2 + 5x – 3 = 0 A
II. y2 -8y + 7 = 0 20. I. 10x2 +57x +54 = 0
II. y2 +13y -30 = 0
8. I. 𝑥 2 − (16)2 = (23)2 − 56
II. 𝑦1/3 − 55 + 376 = (18)2 21. I. 10x2 +7x -45 = 0
II. y2 +5y -50 = 0
9. I. 8x2 – 18√3x + 27 = 0
II. 15y2 – 14√3y + 9 = 0 22. I. 10x2 -31x -14 = 0
II. y2 +9y -136 = 0
𝑋−√6
10. I. =0
√𝑥 23. I. 10x2 -37x -36 = 0
3 (3/2)
II. 𝑦 −6 =0 II. y2 -37y +336 = 0
24. I. 10x2 -49x +18 = 0 39. I. 100x² – 120x + 32 = 0
II. y2 +17y +60 = 0 II. 10y² – 17y + 6 = 0

25. I. 10x2 -57x +54 = 0 40. I. 4𝑥+7𝑦=12


II. y2 +2y -255 = 0 II. 5𝑥+6𝑦=16

26. I. 10x2 -7x -45 = 0 41. I. 𝑥 2 − 122𝑥 + 2632 = 0


II. y2 +4y -5 = 0 II. 𝑦 2 − 112𝑦 + 2236 = 0

27. I. 2p2+5p−12=0 42. I. 7𝑦 2 + 23𝑦 + 16 = 0


II. 2q2−q−1=0 II. 9𝑥 2 − 27𝑥 + 14 = 0

28. I. 10x2 -9x +2 = 0 43. I. 𝑥 2 + 74𝑥 − 656 = 0


II. y2 +46y +528 = 0 II. 𝑦 2 + 68𝑦 − 444 = 0

29. I. 𝑥2 − 𝑥 − 6 = 0 44. I. 55x² – 495x + 1100 = 0


II. 2𝑦 2 + 13𝑦 + 21 = 0 II. 5y² + 10y – 120 = 0

30. I. 𝑥 2 + 4𝑥 + 4 = 0 45. I. 9x² – 94.5x + 243 = 0


II. 𝑦 2 − 8𝑦 + 16 = 0 II. 4.5y² – 13.5y – 486 = 0

31. I. 𝑥 2 − (16)2 = (23)2 − 56 46. If 3 is a root of the equation ax² – 5x – 12 = 0, then


II. 𝑦1/3 − 55 + 376 = (18)2 find the sum of sum of roots and product
of roots.
7 13
32. I. y² – 186 y + 8648 = 0 (1) − 3 (2) − 3
II. x² – 172x + 7396 = 0 7 8
(3) 3 (4) 3
33. I. 7x² – 24x + 20 = 0 (5) None of these
II. 11y² – 36y + 9 = 0
47. If y² + (p + 7)y + 7p = 0 has one of the roots as -6,
34. I. 8y² – 44y + 20 = 0 then find the other root.
II. 6x² – 27x + 12 = 0 (a) -2 (b) -7
(c) -4 (d) 7
35. I. 𝑥 2 + 7𝑥 + 12 = 0 (e) 6
II. 𝑦 2 + 6𝑦 + 8 = 0
48. If x² – (p – 1)x + 210 = 0 has roots as ‘a’ and ‘b’,
36. I. 3 2
𝑥 × 13 = 𝑥 × 247 then find the value of (1/a + 1/b) where p is the 6th
II. y1/3× 14 = 294 ÷ y2/3 term of an AP whose first term is 20 and common
difference is 2.
29 49
12×4 3×4 (1) 210 (2)
37. I. 𝑥 4/7
− 𝑥 4/7
= 𝑥 10/7 200
29 29
II. y3 + 783 = 999 (3) 200 (4) − 210
(5) None of these
38. I. x² – 59x + 868 = 0
II. y² – 53y + 702 = 0
49. py² + 3py – 60 = 0 50. x² – 45x + 200 = 0
If sum of roots and product of roots of the given If roots of the given equations are α and β
equation are equal, then find discriminant (D) of respectively, then find that quadratic equation
the quadratic equation. whose roots are (1/α) and (1/β) respectively.
(1) 24 (2) 12 (1) 3x² + 87x – 270 = 0
(3) 21 (4) 18 (2) 200x² – 45x - 1 = 0
(5) 14 (3) x² + 87x – 270 = 0
(4) 200𝑥 2 − 45𝑥 + 1 = 0
(5) None of these
Hints & Solutions
1. Solution (2) Here, x y
S.I = (P × r × t) / 100 6. Solution
= (5000 × 10 × 3) / 100 = Rs. 1500
I. 𝑥 = √625 = +25
Amount payable after 3 years = 5000 + 1500 = Rs.
II. 𝑦 = √676 = +26
6500
So, 𝑦 > 𝑥
1. Solution
7. Solution
I. 𝑥 2 + 7𝑥 + 6𝑥 + 42 = 0
From I:
Or, x = – 7, – 6
2x2 + 5x - 3 = 0
II. 𝑦 2 + 10𝑦 + 9𝑦 + 90 = 0
=>2x2 – x + 6x – 3 = 0
Or, y = – 10, – 9
=>x(2x – 1) + 3(2x -1) = 0
∴ 𝑥>𝑦
=>(x + 3)(2x – 1) = 0
2. Solution
=>x = 1/2, -3
I. 𝑥 2 + 8𝑥 + 7𝑥 + 56 = 0
From II:
Or, x = – 8, – 7
y2 - y – 7y + 7 = 0
II. 𝑦 2 − 12𝑦 − 11𝑦 + 132 = 0
=> y(y – 1) – 7(y – 1) =0
Or, y = 12, 11 ∴ 𝑦 > 𝑥 =>(y – 1)(y – 7) = 0
3. Solution =>y = 1, 7
I. 𝑥 2 − 12𝑥 − 10𝑥 + 120 = 0 Therefore, x<y
Or, x = 12, 10 8. Solution
II. 𝑦 2 − 14𝑦 − 12𝑦 + 168 = 0 I. 𝑥 2 − (16)2 = (23)2 − 56
Or, y = 14, 12 or 𝑥 2 − 256 = 529 − 56
∴ 𝑦≥𝑥
∴ 𝑥 = √729 = ±27
4. Solution
II. 𝑦1/3 − 55 + 376 = (18)2
I. 𝑥 2 + 8𝑥 + 4𝑥 + 32 = 0
or 𝑦1/3 = 324 + 55 − 376
Or, x = – 8, –4
∴ 𝑦 = (3)3 = 27
II. 𝑦 2 + 9𝑦 + 8𝑦 + 72 = 0
Or, y = – 9, – 8 ∴𝑦≥𝑥
9. Solution
𝑥≥𝑦
I. 8x2 – 18√3x + 27 = 0
5. Solution
8x2 – 12√3x – 6√3x + 27 = 0
1. x 2 - 7 3x + 36 = 0 4x(2x – 3√3) – 3√3 (2x – 3√3) = 0
=x 2 -4 3x-3 3x + 36 = 0 (2x – 3√3)(4x – 3√3) = 0
=x 2 -4 3x-3 3x + 36 = 0 3 3
x 3, 3
= (x - 3 3)(x - 4 3) = 0
4 2
II. 15y2 – 14√3y + 9 = 0
x = 3 3,4 3 15y2 – 9√3y – 5√3y + 9 = 0
II. y - 1113 y + 84 = 0
3y(5y – 3√3) – √3 (5y – 3√3) = 0
= y 2 - 4 3 y - 7 3 y + 34 = 0
(3y – √3)(5y – 3√3) = 0
= y(y - 4 3 )- 783 (y - 4 3 ) = 0 1 3
y 3, 3
= (y - 7 3 ) (y - 4 3 ) = 0 3 5
y = 7 3 ,4 3 For x
x>y
10. Solution ∴ y2 + 13y + 42 = 0
x = √6, 𝑦 = √6 ∴ y2 + 7y + 6y + 42 = 0 ∴ (y + 7)(y + 6) = 0
∴x=y ∴ y = –7 or y = –6 When x = −8, x < y
11. Solution When x = 0 or 2, x > y Hence, no relation can be
4 6 established between x and y.
I. + =5 x 14. Solution
x x
46
I. 2𝑥 2 − 15𝑥 + 27 = 0
5 x ⇒ 2𝑥 2 − 6𝑥 − 9𝑥 + 27 = 0
x
9
10=5x ⇒ (𝑥 − 3)(2𝑥 − 9) = 0 ⇒ 𝑥 = 3, 2
X=2 II. 2𝑦 2 − 13𝑦 + 20 = 0
y 2  256  625 ⇒ 2𝑦 2 − 8𝑦 − 5𝑦 + 20 = 0
⇒ (𝑦 − 4)(2𝑦 − 5) = 0
y 2 +16=25 5
⇒ 𝑦 = 4,
2 2
y =25-16
No relation
y 2 =9 15. Solution
y  3 I. 9𝑥 2 − 21𝑥 + 10 = 0
While comparing the values of x and y one root ⇒ 9𝑥 2 − 6𝑥 − 15𝑥 + 10 = 0
value of y lies between the two root values of x ⇒ 3𝑥(3𝑥 − 2) − 5(3𝑥 − 2) = 0
2 5
11. Solution ⇒ 𝑥 = ,3
3
x = – 4, 3 ; y = 3, 2 II. 𝑦 2 − 8𝑦 + 15 = 0
∴ No relation ⇒ 𝑦 2 − 5𝑦 − 3𝑦 + 15 = 0
12. Solution ⇒ (𝑦 − 3)(𝑦 − 5) = 0
I. 12x2-29x+15=0 ⇒ 𝑦 = 3, 5
12x2-20x-9x+15=0 Y>x
4x(3x-5)-3(3x-5)=0 16. Solution
(3x-5)(4x-3)=0 I. 2𝑥 2 − 13𝑥 + 15 = 0
x=5/3,3/4
⇒ 2𝑥 2 − 10𝑥 − 3𝑥 + 15 = 0
II.54y 2 - 20 18y + 33 = 0 ⇒ (𝑥 − 5)(2𝑥 − 3) = 0 ⇒ 𝑥 = 5, 2
3

54 y 2  11 18 y  9 18 y  33  0 II. 2𝑦 2 − 11𝑦 + 12 = 0
18 y (3 18 y  11)  3(3 18 y  11)  0 ⇒ 2𝑦 2 − 8𝑦 − 3𝑦 + 12 = 0
3
( 18 y  3)(3 18 y  11)  0 ⇒ (𝑦 − 4)(2𝑦 − 3) = 0 ⇒ 𝑦 = 4, 2

y  3 18 y  1/ 2,11/ 3 18  11/ 9 2 No relation


17. Solution
While comparing the values of x and y one root
I. 2𝑥 2 + 7𝑥 + 6 = 0
value of y lies between the two root values of x
⇒ 2𝑥 2 + 4𝑥 + 3𝑥 + 6 = 0
13. Solution
⇒ (𝑥 + 2)(2𝑥 + 3) = 0
I. 4x3 + 24x2 – 64x = 0 3
∴ 4x(x2 + 6x – 16) = 0 ⇒ 𝑥 = −2, − 2
∴ 4x = 0 or x2 + 6x – 16 = 0 (Here 4x = 0 is II. 2𝑦 2 + 17𝑦 + 30 = 0
nothing but x = 0) ⇒ 2𝑦 2 + 12𝑦 + 5𝑦 + 30 = 0
x2 + 6x – 16 = 0 ⇒ (𝑦 + 6)(2𝑦 + 5) = 0 ⇒ 𝑦 = −6, − 2
5

∴ x2 + 8x − 2x – 16 = 0 ∴ (x − 2)(x + 8)
x>y
= 0 ∴ x = 2 or x = −8 x = −8, 0 or 2
II. 3y2 + 39y + 126 = 0
18. Solution y2 -37y +336 = (y -16)(y -21) = 0
I. 𝑥 2 − 2𝑥 − √5𝑥 + 2√5 = 0 => y = 16, 21
⇒ (𝑥 − 2) (𝑥 − √5) = 0 ⇒ 𝑥 = 2, √5 So, x < y
23. Solution
II. 𝑦 2 − √3𝑦 − √2𝑦 + √6 = 0
From equation I:
⇒ (𝑦 − √3)(𝑦 − √2) = 0 ⇒ 𝑦 = √3, √2
10x2 -49x +18 = (5x -2)(2x -9)= 0
x>y => x = 2/5, 9/2
19. Solution From equation II:
From equation I: y2 +17y +60 = (y + 5)(y + 12) = 0
10x2 +57x +54 = (5x + 6)(2x + 9)= 0 => y = -5, -12
=> x = -6/5, -9/2 So, x > y
From equation II: 24. Solution
y2 +13y -30 = (y -2)(y + 15) = 0 𝑞 = 56
=> y = 2, -15 𝑝 + 𝑞 = ±56, 𝑝 = 0
If p + q=-56 then p= -112
X = -6/5 X = -9/2 p + q=56 then p=0 so, 𝑞 > 𝑝
Y=2 x<y x<y 25. Solution
Y = -15 x > y x>y From equation I:
So, relationship cannot be established between x 10x2 -57x +54 = (2x -9)(5x -6)= 0
and y => x = 9/2, 6/5
20. Solution From equation II:
From equation I: y2 +2y -255 = (y -15)(y + 17) = 0
10x2 +7x -45 = (5x -9)(2x + 5)= 0 => y = 15, -17
=> x = 9/5, -5/2 So, relationship cannot be established between x and y
From equation II: 26. Solution
y2 +5y -50 = (y + 10)(y -5) = 0 From equation I:
=> y = -10, 5 10x2 -7x -45 = (5x + 9)(2x -5)= 0
X = 9/5 X = -5/2 => x = -9/5, 5/2
Y = -10 x > y x>y From equation II:
Y=5 x<y x<y y2 +4y -5 = (y + 5)(y -1) = 0
So, relationship cannot be established between x => y = -5, 1
and y So, relationship cannot be established between x
21. Solution and y
From equation I: 27. Solution
10x2 -31x -14 = (5x + 2)(2x -7)= 0 2𝑝2 + 8𝑝 − 3𝑝 − 12 = 0
=> x = -2/5, 7/2 2𝑝(𝑝 + 4) − 3(𝑝 + 4) = 0
From equation II: 3
𝑝 = 2 , −4
y2 +9y -136 = (y + 17)(y -8) = 0
2𝑞 2 − 2𝑞 + 𝑞 − 1 = 0
=> y = -17, 8
2𝑞(𝑞 − 1) + 1(𝑞 − 1) = 0
So, relationship cannot be established between x −1
and y 𝑞= ,1
2
22. Solution No relation can be established.
From equation I: 28. Solution
10x2 -37x -36 = (2x -9)(5x + 4)= 0 From equation I:
=> x = 9/2, -4/5 10x2 -9x +2 = (2x -1)(5x -2)= 0
From equation II: => x = 1/2, 2/5
From equation II: 10
⇒ x = 2, 7
y2 +46y +528 = (y + 22)(y + 24) = 0
II. 11y² – 36y + 9 = 0
=> y = -22, -24
⇒ 11y² – 33y – 3y + 9 = 0
So, x > y
⇒ (y – 3) (11y – 3) = 0
29. Solution 3
I. 𝑥 2 − 𝑥 − 6 = 0 ⇒ y = 3, 11
⇒ 𝑥 2 − 3𝑥 + 2𝑥 − 6 = 0 No relation.
⇒ 𝑥(𝑥 − 3) + 2(𝑥 − 3) = 0 34. Solution
⇒ (𝑥 + 2)(𝑥 − 3) = 0 I. 8y² – 44y + 20 = 0
⇒ 𝑥 = −2, 3 ⇒ 2y² – 11y + 5 = 0
II. 2𝑦 2 + 13𝑦 + 21 = 0 ⇒ 2y² – 10y – y + 5 = 0
⇒ 2𝑦 2 + 7𝑦 + 6𝑦 + 21 = 0 ⇒ (y – 5) (2y – 1) = 0
1
⇒ 𝑦(2𝑦 + 7) + 3(2𝑦 + 7) = 0 ⇒ y = 5,
2
⇒ (𝑦 + 3)(2𝑦 + 7) = 0 II. 6x² – 27x + 12 = 0
7
⇒𝑦= −3, − 2 ⇒ 2x² – 9x + 4 = 0
∴𝑥>𝑦 ⇒ 2x² – 8x – x + 4 = 0
30. Solution ⇒ (x – 4) (2x – 1) = 0
1
I. 𝑥 2 + 4𝑥 + 4 = 0 ⇒ 𝑥 = 4,
2
(𝑥 + 2)2 = 0 ⇒ 𝑥 = −2 No relation
II. 𝑦 2 − 8𝑦 + 16 = 0 35. Solution
⇒ (𝑦 − 4)2 = 0 I. 𝑥 2 + 4𝑥 + 3𝑥 + 12 = 0
⇒𝑦=4 Or, x = – 4, – 3
∴𝑦>𝑥 II. 𝑦 2 + 4𝑦 + 2𝑦 + 8 = 0
31. Solution Or, y = – 4, – 2
I. 𝑥 2 − (16)2 = (23)2 − 56 ∴ Relation can’t be established
or 𝑥 2 − 256 = 529 − 56 36. Solution
∴ 𝑥 = √729 = ±27 𝑥 3 247 1 2 294
I. 2 = II. 𝑦 3+3 =
II. 𝑦1/3 − 55 + 376 = (18)2 𝑥 13 | 14
or 𝑦1/3 = 324 + 55 − 376 𝑥 = 19 𝑦 = 21
∴ 𝑦 = (3)3 = 27 ∴ 𝑥<𝑦
∴𝑦≥𝑥 37. Solution
32. Solution 48 − 12 10
I. = 𝑥 7 II. 𝑦 3 = 999 − 783
I. y² – 186 y + 8648 = 0 4
|
𝑥7
⇒ y² – 92y – 94y + 8648 = 0 2
𝑥 = 36 𝑦 3 = 216
|
⇒ (y – 92) (y – 94) = 0
⇒ y = 92, 94 𝑥 = ±6 𝑦=6
II. x² – 172x + 7396 = 0 ∴ 𝑥≤𝑦
⇒ x² – 86x – 86x + 7396 = 0 38. Solution
⇒ (x – 86) ² = 0 I. x² – 59x + 868 = 0
⇒ x = 86, 86 x² – 28x – 31x + 868 = 0
y>x x (x – 28) – 31 (x – 28) = 0
33. Solution (x – 31) (x – 28) = 0
I. 7x² – 24x + 20 = 0 x = 28, 31
⇒ 7x² – 14x – 10x + 20 = 0 II. y² – 53y + 702 = 0
⇒ (x – 2) (7x – 10) y² – 27y – 26y + 702 = 0
y (y – 27) – 26(y – 27) = 0 ⇒ 𝑦 = −1, −
16
7
(y– 27) (y – 26) = 0
II. 9𝑥 2 − 27𝑥 + 14 = 0
y = 26, 27
⇒ 9𝑥 2 − 6𝑥 − 21𝑥 + 14 = 0
⇒x>y
⇒ 3𝑥(3𝑥 − 2) − 7(3𝑥 − 2) = 0
39. Solution
⇒ (3𝑥 − 2)(3𝑥 − 7) = 0
I. 100x² – 120x + 32 = 0 2 7
100x² – 40x – 80x + 32 = 0 ⇒𝑥= ,
3 3
20x (5x – 2) – 16(5x – 2) = 0 x>y
(20x – 16) (5x – 2) = 0 43. Solution
4 2 I. 𝑥 2 + 74𝑥 − 656 = 0
𝑥= ,
5 5 ⇒ 𝑥 2 + 82𝑥 − 8𝑥 − 656 = 0
II. 10y² – 17y + 6 = 0 ⇒ (𝑥 + 82)(𝑥 − 8) = 0
10y² – 12y – 5y + 6 = 0 ⇒ 𝑥 = −82, 8
2y (5y – 6) – 1 (5y – 6) = 0 II. 𝑦 2 + 68𝑦 − 444 = 0
(2y – 1) (5y – 6) = 0 ⇒ 𝑦 2 + 74𝑦 − 6𝑦 − 444 = 0
1 6
𝑦 = , 5 ⇒ No relation ⇒ (𝑦 + 74)(𝑦 − 6) = 0
2
40. Solution ⇒ 𝑦 = 6, – 74
I. 4x + 7y = 12 No relation
II. 5x + 6y = 16 44. Solution
Multiplying eq. (i) with 5 and eq. (ii) with 4 and I. 55x² – 495x + 1100 = 0
subtracting eq. (i) from eq (ii), we get 𝑥 2 − 9𝑥 + 20 = 0
20𝑥 + 24𝑦 = 64 ⇒ x² – 5x – 4x + 20 = 0
20𝑥 + 35𝑦 = 60 ⇒ (x – 5) (x – 4) = 0
– – –
– 11y = 4 ⇒ x = 5, 4
4 II.5y² + 10y – 120 = 0
y=–
11 ⇒ y² + 2y – 24 = 0
Put this value of y in eq. (i) ⇒ y² + 6y – 4y – 24 = 0
28
4x – 11 = 12 ⇒ (y + 6) (y – 4) = 0 ⇒ y = 4, –6
⇒x=3+
7 x≥y
11
40 45. Solution
= 11
⇒ x>y I. 9x² – 94.5x + 243 = 0
41. Solution ⇒ x² – 10.5x + 27 = 0
I. 𝑥 2 – 122𝑥 + 2632 = 0 ⇒ x² – 6x – 4.5x + 27 = 0
⇒ 𝑥 2 – 94𝑥– 28𝑥 + 2632 = 0 ⇒ (x – 6) (x – 4.5) = 0
⇒ 𝑥(𝑥 − 94) − 28(𝑥 − 94) ⇒ x = 6, 4.5
⇒ (𝑥 − 94)(𝑥 − 28) = 0 I. 4.5y² – 13.5y – 486 = 0
⇒ 𝑥 = 94, 28 ⇒ y² – 3y – 108 = 0
II. 𝑦 2 – 112𝑦 + 2236 = 0 ⇒ y² – 12y + 9y – 108 = 0
⇒ 𝑦 2 – 86𝑦 − 26𝑦 + 2236 = 0 ⇒ (y – 12) (y + 9) = 0
⇒ (𝑦 − 86)(𝑦 − 26) = 0 ⇒ y = 12, –19
y = 86, 26 No relation
no relation 46. Solution
42. Solution 𝑎𝑥 2 − 5𝑥 − 12 = 0 has one root as 3
I. 7𝑦 2 + 23𝑦 + 16 = 0 ∴ 𝑥 = 3 will satisfy the equation.
⇒ 7𝑦 2 + 7𝑦 + 16𝑦 + 16 = 0 ⇒ 𝑎(3)2 − 5(3) − 12 = 0
⇒ (y+1) (7y+16) = 0 ⇒ 9𝑎 − 15 − 12 = 0
⇒ 9𝑎 = 27 49. Solution
⇒𝑎=3 𝑝𝑦 2 + 3𝑝𝑦 − 60 = 0 𝑆𝑢𝑚 𝑜𝑓 𝑟𝑜𝑜𝑡𝑠
−(−5) 5 3𝑝
∴ 𝑆𝑢𝑚 𝑜𝑓 𝑟𝑜𝑜𝑡𝑠 = = =− = −3
3 3 𝑝
−12 Product of roots
𝑃𝑟𝑜𝑑𝑢𝑐𝑡 𝑜𝑓 𝑟𝑜𝑜𝑡𝑠 = − = −4
3 −60
5 =
⇒ 𝑟𝑒𝑞𝑢𝑖𝑟𝑒𝑑 𝑎𝑛𝑠𝑤𝑒𝑟 = − 4 𝑝
3 According to question
7
=− −60
3 −3 = , 𝑝 = 20
47. Solution 𝑝
-6 is a root of the given equation then 𝑦 = −6 will ∴ 20𝑦 2 + 60𝑦 − 60 = 0
satisfy the equation. ⇒ 𝑦 2 + 3𝑦 − 3 = 0,
∴ (−6)2 − 6(𝑝 + 7) + 7𝑝 = 0 𝐷 = 𝑏 2 − 4𝑎𝑐
⇒ 36 − 6𝑝 − 42 + 7𝑝 = 0 = (32 ) − 4 × 1 × (−3)
⇒𝑝=6 = 9 + 12 = 21
∴ Product of roots = 6 × 7 = 42 50. Solution
⇒ (−6) × 𝛽 = 42 x² – 45x + 200 = 0
⇒ 𝛽 = −7 sum of roots (𝛼 + 𝛽) = -(-45) = 45
48. Solution Product of roots
(𝛼𝛽) = 200
p = 6th term of an AP whore first term is 20 and
1 1
common difference is 2. ∴ +
α β
∴ p = 20 + (6 − 1) × 2
α+β 45 9
= 20 + 10 = 30 = = =
αβ 200 40
𝑥 2 − 29𝑥 + 210 = 0 1 1
1 1 𝑎+𝑏 and =
∴ + = αβ 200
𝑎 𝑏 𝑎𝑏
⇒ 𝑥 2 − (𝑠𝑢𝑚 𝑜𝑓 𝑟𝑜𝑜𝑡𝑠) 𝑥 + (𝑃𝑟𝑜𝑑𝑢𝑐𝑡 𝑜𝑓 𝑟𝑜𝑜𝑡𝑠) = 0
⇒ 𝑎 + 𝑏 = 29
9 1
𝑎𝑏 = 210 ⇒ 𝑥2 − 𝑥+ =0
1 1 29 40 200
∴ + = ⇒ 200𝑥 2 − 45𝑥 + 1 = 0
𝑎 𝑏 210
Short cut: for reciprocal roots interchange the
coefficients of 𝑥 2 and constant term and sign of 𝑏𝑥.
नींव Booklet
For All Banking & Insurance Exam
Ch -6 | Percentage | Maths by Arun sir
1. 75% of x is 25% more than y. if the difference 7. Twenty-five percent of Reena’s yearly income
between x and y is 192 find the value of y. equal to seventy five percent of Anunhav’s monthly
(1) 288 (2) 144 income. If Anubhav’s yearly income is 24000 what
(3) 96 (4) 192 is Reena's monthly income?
(5) None of these (1) 60000 (2) 12000
(3) 5200 (4) 8000
2. If sum of 20%, 500 and x% of 900 is 352. Find the (5) None of these
value of x?
(1) 28.8 (2) 24.4 8. Aman's expense is 30% more than Vimal's and
(3) 96 (4) 172 Vimal's expense is 10% less than Raman's. If the
(5) None of these sum of their expenses is 6447, then what would be
Aman's expense?
2 (1) 1890 (2) 2100
3. If 66 % of a no. is added with itself then result
3
(3) 2200 (4) 2457
becomes 7800. find the original number?
(5) None of these
(1) 2880 (2) 4680
(3) 7800 (4) 1920
9. In a market research project, 20% opted for Nirma
(5) None of these
detergent whereas 60% opted for Surf Blue
detergent. The remaining individuals were not
4. If the length of a rectangular field is increased by
certain. If the difference between those who opted
37(1/2)% and its breadth is decreased by 16(2/3)%
for Surf Blue and those who were uncertain was
then what will be the percentage change in its area?
720, how many respondents were covered in the
(1) 28% (2) 14%
survey?
5 7
14 % 14 % (1) 1440 (2) 1800
(3) 12 (4) 12
(3) 3600 (4) Data inadequate
(5) None of these (5) None of these

5. If a bucket is 80% full, then it contains 2 liters more 10. In a test, minimum passing percentage for girls and
2
water than it is 663 % full. What is the capacity of boys is 35% and 40% respectively. A boy scored
the bucket. 483 marks and failed by 117 marks. What is
(1) 10 litres (2) 15 litres minimum passing marks for girls?
2 (1) 425 (2) 500
(3) 16 litres
3
(4) 20 litres
(3) 520 (4) 625
(5) None of these (5) None of these

6. In a competitive examination in state A, 6%


candidates got selected from the total appeared
candidates. State B had an equal candidates got
selected with 80 more candidates got selected than 11. In a college election between two candidate got
A what as the number of candidates appeared from
55% of the total valid votes. 15 of the votes were
each state?
invalid. If the total votes were 15200, what is the
(1) 7600 (2) 8000
number of valid votes the other candidate got?
(3) 8400 (4) 9000 (1) 5814 (2) 6840
(5) None of these (3) 7106 (4) 8360
(5) None of these
नींव Booklet
For All Banking & Insurance Exam
Ch -6 | Percentage | Maths by Arun sir

passed in both the subjects. Find out the number of


12. 10% of the voters did not cast their vote in an students passed only in Quants?
election between two candidates. 10% of the votes (1) 17 (2) 23
polled were found invalid. The successful candidate (3) 33 (4) 37
got 54% of the valid votes and won by a majority of (5) None of these
1620 votes. The number of voters enrolled voters'
list was 17. The weight of an empty vessel is 20% of the weight
(1) 25000 (2) 33000 of the bucket when filled with water. Some of the
(3) 35000 (4) 40000 water has been removed then, the vessel, along with
(5) None of these the remaining water, weighed three-fifths of the
original weight. What percentage of the water has
13. A number is divided into two parts in such a way been removed?
that 80% of 1st part is 3 more than 60% of 2nd part (1) 40% (2) 62%
and 80% of 2nd part is 6 more than 90% of the 1st (3) 56% (4) 53%
part. Then the number is: (5) None of these
(1) 125 (2) 130
(3) 135 (4) 145 18. A large watermelon weighs 20 kg with 96% of its
(5) 155 weight being water. It is allowed to stand in the sun
and some of the water evaporates so that only 95%
14. In a company, 60% of the employees are men and of its weight is water. Its reduced weight will be
48% of the employees are engineers and 66.6% of (1) 16 kg (2) 16.5 kg
Engineers are men. What is the percentage of (3) 17 kg (4) 18 kg
women who are not engineers? (5) None of these
(1) 40% (2) 50%
(3) 55% (4) 60% 19. Fresh grapes contain 80 percent water while dry
(5) 70% grapes contain 10 percent water. If the weight of dry
grapes is 250 kg what was its total weight when it
15. The monthly income of Suraj and Komal together was fresh?
is Rs.28000. The income of Suraj and Komal is (1) 1000 kg (2) 1100 kg
increased by 25% and 12.5% respectively. The new (3) 1125 kg (4) 1225 kg
income of Komal becomes 120% of the new salary (5) None of these
of Suraj. What is the new income of Suraj ?
(1) Rs. 12000 (2) Rs. 14000 20. In a mixture of milk and water, the proportion of
(3) Rs. 15000 (4) Rs. 16000 water by weight was 75%. If in the 60 gms mixture
(5) Rs. 18000 15 gm of water was added, what would be the
percentage of water?
16. In a class of 60 students, 5% of the students failed (1) 75% (2) 88%
in Quants and Reasoning and 40% of the students (3) 90% (4) 100%
passed in Reasoning, and 20% of the students (5) None of these
नींव Booklet
For All Banking & Insurance Exam
Ch -6 | Percentage | Maths by Arun sir

21. The numerator and denominator of a fraction is (3) 50 % (4) 25 %


increased and decreased by 12% & and 10% (5) None of these
respectively and the fraction becomes 224/270.
What is the original fraction? 26. Mukund spends 30% of his monthly income on
(1) 3/2 (2) 1/4 food articles, 40% of the remaining on conveyance
(3) 2/3 (4) 5/3 and clothes and saves 50% of the remaining. If his
(5) None of these monthly salary is Rs. 36,800, how much money
does he save every month?
22. A vessel of 70 l is filled with milk and water. 75% (1) 7728 (2) 9806
of milk and 25% of water is taken out of the vessel. (3) 11698 (4) 13578
It is found that the vessel is vacated by 60%. Find (5) None of these
the initial quantity of water.
(1)21 litre (2)22 litre 27. Devbrat scored 30% marks and failed by 25 marks
(3)24 litre (4)25 litre Shreekar scored 40% marks and obtained 50 marks
(5) None of these more than those required to pass. The pass
percentage is:
23. In an election the votes between the winner and (1) 47.6% (2) 25%
loser candidate are in thIe ratio 5:1. If total number (3) 33.33% (4) 42.84%
of eligible voters are 1000, out of which 12% did (5) None of these
not cast their vote and among the remaining vote
10% declared invalid. What is the number of votes 28. Two successive price increases of 10% and 10% of
the winner candidate get ? an article are equivalent to a single price increase of
(1) 620 (2) 630 (1) 19% (2) 20%
(3) 640 (4) 660 (3) 21% (4) 22%
(5) None of these (5) None of these

24. In a test, Pawan scored 10% more than Anwar and 29. A number is increased by 15.38%. How much
Anwar scored 5% more than Ramesh. If Ramesh should it be decreased to get the original number?
scored 300 marks out of 400, then Pawan’s marks (1) 13.33% (2) 7.14%
are: (3) 8.33% (4) 26.33%
(1) 347 (2) 360 (5) 39.33%
(3) 305 (4) 312
(5) None of these 30. If 16.66% of the students in a school are girls and
the number of boys is 275, find the total number of
25. In an examination, (A + 6)% of the students failed students in the school.
in Mathematics and (A – 8)% failed in Science. (1) 320 (2) 380
28% of the students failed in both the subjects, If (3) 330 (4) 400
the percentage of students who passed in both the (4) None of these
subjects was 50%, then what is the value of A?
(1) 60 % (2) 40 %
नींव Booklet
For All Banking & Insurance Exam
Ch -6 | Percentage | Maths by Arun sir

31. Quantity I: Find the percentage of boys in the class (3) Quantity I ≥ Quantity II
this year. This year the percentage of girls in the (4) Quantity I ≤ Quantity II
class is 60% (5) Quantity I = Quantity II or Relation cannot be
Quantity II: Find the percentage of boys in the Established.
class this year. Last years out of the 300 students,
50% was girls and this year the number of girls are 34. Quantity I: Out of 300 students in a graduation
increased by 10% but total students remains same. college, the ratio of fit to that of non-fit is 8: 7. 25%
(1) Quantity I > Quantity II among fit are occasional exercise doers and rest are
(2) Quantity I < Quantity II daily exercise doers. 60% of the total number of
(3) Quantity I ≥ Quantity II students do gym and 20% of the total number of
(4) Quantity I ≤ Quantity II occasional exercise doers do gym. If 50% of the
(5) Quantity I = Quantity II or Relation cannot be total number of non - fit do gym, then what is the
Established. number of students who are daily exercise doers as
well as do gym?
32. Quantity I: In a school, 50% of the total number of Quantity II: 100
girls is equal to 30% of the total number of boys (1) Quantity I > Quantity II
then the number of girls is what percentage of the (2) Quantity I < Quantity II
total number of students of the school? (3) Quantity I ≥ Quantity II
Quantity II: In a school, 33.33% of the total (4) Quantity I ≤ Quantity II
number of girls is equal to 66.67% of the total (5) Quantity I = Quantity II or Relation cannot be
number of boys then the total number of girls is Established.
what percentage of the total number of students of
the school? 35. Quantity I: Income of A is 25% more than the
(1) Quantity I > Quantity II income of B. If income of A is increased in the ratio
(2) Quantity I < Quantity II 5 ∶ 6 while the income of B increased in the ratio 4
(3) Quantity I ≥ Quantity II ∶ 5. Then what percent income of A is more than the
(4) Quantity I ≤ Quantity II income of B?
(5) Quantity I = Quantity II or Relation cannot be Quantity II: Price of a commodity has increased by
Established. 25%. By what percent must a consumer reduce its
consumption to maintain the expenditure?
33. The ratio of boys to girls in a school is 4 : 5. If 45 (1) Quantity I > Quantity II
students from the same school left in the same ratio (2) Quantity I < Quantity II
and 40 new girls joined the school, then the ratio of (3) Quantity I ≥ Quantity II
boys to girls becomes 4 : 9. (4) Quantity I ≤ Quantity II
Quantity I: What percentage of total students will (5) Quantity I = Quantity II or Relation cannot be
be boys if 50% of the total number of boys leaves Established.
the school?
Quantity II: 30% 36. Quantity-I: The square of the sum of the digits of
(1) Quantity I > Quantity II a two-digit positive number is 144. When 36 is
(2) Quantity I < Quantity II
नींव Booklet
For All Banking & Insurance Exam
Ch -6 | Percentage | Maths by Arun sir

added to the original number, the digits of the


number get reversed. Find the number. (1) Quantity I > Quantity II
Quantity-II: The ratio of the two numbers is 4:3. (2) Quantity I < Quantity II
When 8 is added to both the numbers, the smaller (3) Quantity I ≥ Quantity II
number becomes 22.5% less than the larger (4) Quantity I ≤ Quantity II
number. Find the smaller number. (5) Quantity I = Quantity II or Relation cannot be
Established.
(1) Quantity I > Quantity II
(2) Quantity I < Quantity II 39. Quantity I:In a class of 80 students and 5 teachers,
(3) Quantity I ≥ Quantity II each student got sweets that are 15% of the total
(4) Quantity I ≤ Quantity II number of students and each teacher got sweets that
(5) Quantity I = Quantity II or Relation cannot be are 25% of the total number of students. How many
Established. sweets were there?
Quantity II: In an election, candidate A got 75% of
37. Quantity-I: In a college (which consist of students the total valid votes. If 15% of the total votes were
and teachers only), the ratio of the number of males declared invalid and the total numbers of votes is
to females is 8:9. If the total number of teachers is 560000, find the number of valid vote polled in
excluded which consist of 40 males and 30 females, favor of candidate?
then the ratio of the number of male students to (1) Quantity I > Quantity II
female students becomes 6:7. Find 1/10th of the (2) Quantity I < Quantity II
number of females in the college. (3) Quantity I ≥ Quantity II
Quantity-II: When 377 is subtracted from 867, (4) Quantity I ≤ Quantity II
then the resultant is 40 more than x% of 1800. Find (5) Quantity I = Quantity II or Relation cannot be
the value of ‘x’. Established.

(1) Quantity I > Quantity II


(2) Quantity I < Quantity II
(3) Quantity I ≥ Quantity II
(4) Quantity I ≤ Quantity II
(5) Quantity I = Quantity II or Relation cannot be
Established.
38. Quantity I: The ratio of income of A and B is 4: 5.
The sum of their expenditures is Rs. 40000 and the
amount of the savings of A is equal to the
expenditure of B. Find the ratio of their incomes to
the sum of their savings?
Quantity II: A and B shared a profit in the ratio of
5: 4. They invested for 7 months and 8 months
respectively. Then find the ratio of their
investments?
नींव Booklet
For All Banking & Insurance Exam
Ch -6 | Percentage | Maths by Arun sir

40. Quantity I:2/5th of the voters promise to vote for


A and the rest promised to vote for B. Of these, on
the last day 15% of the voters went back of their
promise to vote for A and 25% of voters went back
of their promise to vote for B, and A lost by 200
votes. Then, the total number of voters is?
Quantity II: The income of A is 150% of the
income of B and the income of C is 120% of the
income of A. If the total income of A, B and C
together is Rs. 86,000, what is C’s income?

(1) Quantity I > Quantity II


(2) Quantity I < Quantity II
(3) Quantity I ≥ Quantity II
(4) Quantity I ≤ Quantity II
(5) Quantity I = Quantity II or Relation cannot be
Established.

Answer Key
1. (1) 9. (4) 17. (5) 25. (2) 33. (2)
2. (2) 10. (5) 18. (5) 26. (1) 34. (1)
3. (2) 11. (1) 19. (3) 27. (3) 35. (5)
4. (4) 12. (1) 20. (5) 28. (3) 36. (2)
5. (5) 13. (3) 21. (3) 29. (1) 37. (1)
6. (2) 14. (4) 22. (1) 30. (3) 38. (1)
7. (5) 15. (3) 23. (4) 31. (2) 39. (2)
8. (4) 16. (3) 24. (2) 32. (2) 40. (2)
नींव Booklet
For All Banking & Insurance Exam
Ch -6 | Percentage | Maths by Arun sir

Hints & Solutions


1. Solution (1) 55lb
=
75 125 48
X× = ×y
100 100 55
lb  lb
3x 5 y Required% 48  100
 
4 4 lb
 x:y=5:3 =
700
%
And, x-y = 192 (because x>y) 48
 5a -3a = 192 7
= 14 %
 a = 96 12
So, y = 3a = 3 × 96 = 288
5. Solution (2)
2. Solution (2) Let the capacity of the bucket be x litres.
20 x 2
 900   500  352 Then, 80% of x - 66 % of x = 2
3
100 100 2 1
⇒ (80 − 66 ) % of x = 2 ⇒ 13 % of x = 2
 180 + 5x = 352 3 3
40 1 2×100×3
 5x = 172 ⇒ 3
× 100 × 𝑥 = 2 ⇒ 𝑥 = ( 40 ) = 15
 x = 24.4
6. Solution (2)
3. Solution (2) Let the number of candidates appeared from each
Let numbers is x state
2 be x.
x+ x = 7800
3 Then, 7% of x – 6% of x = 80
5x ⇔ 1% of x = 80
= =7800 ⇔ x = 80 × 100 = 8000.
3
X = 4680
7. Solution (5)
Let Reena's yearly income be x.
4. Solution (4) 240000
Let Length of field be l nd breadth be b. Anubhav's monthly income = ( 12
) = 240000
1 3 Then, 25% of x = 75% of 20000
37 %  25 75
2 8 ⇒ 100 𝑥 = 100 × 20000
2 1 𝑥
16 %  ⇒ 4 = 15000 ⇒ x = 60000.
3 6 60000
Reena’s monthly income = ( ) = 5000
 3 12
New length - 1   l
 8
8. Solution (4)
b 5b Let Raman’s expense be x. Then, Vimal’s expense
New breadth = b - 
6 6 = 90%
Original area = lb 90 9
Of x = (100 × 𝑥 ) = 10 𝑥`
11l 5b
New area = 
8 6
नींव Booklet
For All Banking & Insurance Exam
Ch -6 | Percentage | Maths by Arun sir
9𝑥 130 ⇔ 8% of [90% of (90% of x)] = 1620
Aman’s expense = 130% of (10) = 𝑅𝑠. (100 ×
8 90 90
9𝑥 117𝑥 ⇔ 100 × 100 × 100 × 𝑥 = 1620
) = 𝑅𝑠.
10 100 1620×100×100×100
117𝑥 9𝑥 117𝑥+0𝑥+100𝑥 ⇔x=( )= 25000.
+ +𝑥 = 6447 ⇒ = 6447 8×90×90
100 10 100
⇒ 307x = 644700
644700
⇒x= 307
= 2100 13. Solution (3)
117×2100
Hence, Aman’s expense = ( 100 ) = 2457.
14. Solution (4)
⇒ Let total employees be 100.
9. Solution (4)
Let the total number of respondents be x. ⇒ Given men are 60%, i.e. 60, anf females will be
Percentage of uncertain individuals 40.
= [100 – (20 + 60)]% = 20%. ⇒ Engineers is 48% -> 48.
60% of x – 20% of x = 720 ⇒ 40% of x = 720 ⇒ 66.6% of Engineers are men -> 2/3 of 48 -> 32.
40 720×100 ⇒ Now, Total engineers = men engineers + female
⇒ 100 𝑥 = 720 ⇔ x= ( ) = 1800
40 engineer
147. Let the maximum marks be x. ⇒ 48 = 32 + female engineer
36𝑥
Then, 36% of x = 198 + 36 ⇒ 100 = 234 ⇒ x = ⇒ Female engineer = 16, So non engineer female
234×100 = 40 - 16 => 24.
( 36
) = 650
Percentage of women not engineer = 24/40 x 100
=> 60%.
10. Solution (5)
Let the maximum marks be x. 15. Solution (3)
Then, 40% of x = 483 + 117 ⇒ Let Suraj and Komal's monthly age be x and y
40𝑥 600×100
⇒ = 600 ⇒ x = ( ) = 1500 respectively. i.e. x + y = 28000.......1
10 40
Minimum passing marks for girls = 35% of 1500 ⇒ After increase of 25% in Suraj salary and 12.5%
35 in Komal salary, we have
= (100 × 1500) = 525.
⇒ Suraj' salary = 5x/4, komal's salary = 9y/8
(using fractions)
11. Solution (1)
⇒ Given that after increase Komal' salary is 120%
Number of valid votes
of Suraj's salary,
= (100 – 15)% of 15200 = 85% of 15200
85
⇒ 9y/8 = 6/5 x 5x/4
= (100 × 15200) = 12920 ⇒ 3y = 4x .......2
Valid votes polled by other candidate ⇒ Equating in 1st and 2nd equation, we get 5x/4 =
45 Rs. 15000
= (100 – 55)% of 12920 = (100 × 12920) = 5814.

16. Solution (3)


12. Solution (1)
⇒ Total students = 60
Let the total number of voters be x. Then, votes
⇒ Passed in both = 20% of 60 = 12
polled
⇒ Failed in both= 5% of 60 = 3
= 90% of x.
Valid votes = 90% of (90% of x). ⇒ Passed in reasoning = 50% of 60=24
∴ 54% of [90% of (90% of x)] – 46% of [90% of ⇒ Those passed only in reasoning = 24-12 =12
(90% of x)] = 1620
नींव Booklet
For All Banking & Insurance Exam
Ch -6 | Percentage | Maths by Arun sir
students. Weight of water in 60 gms mixture = 75% of 60
⇒ Passed only in Quants= 60-(12+12+3) = 33 gm
75
= (100 × 60) 𝑔𝑚 = 45 𝑔𝑚
17. Solution (5)
Weight of water in 75 gms mixture = (45 + 15) gm
208. Let the weight of the vessel when it is full, be
= 60 gm.
1 kg. 60
1 Required percentage = ( × 100) % = 80%
Then, weight of empty vessel = 20% of 1 kg =5 75

kg.
1 4 21. Solution (3)
Weight of water in the vessel =(1 − 5) kg = 5 kg. Explanation: Let the fraction be a/b
On removing the water, So, 36% of the total marks=522
3
Weight of (vessel + water) =5 kg.
3 1 2
Weight of water in the vessel =(5 − 5) kg. = 5kg.
Weigh of water removed
4 2 2 2
=(5 − 5) kg = 5 kg = 5 kg.
2 5
Hence, required percentage =(5 × 4 × 100) %
22. Solution (1)
=50%
Let Milk = 𝑥 initially.
∴ water = 70 − 𝑥 initially
18. Solution (5)
Let the reduced weight be x kg. Acc. to question
Clearly, the quantity of pulp remains the same in 3 70 − 𝑥
𝑥+ = 70 × 60%
both 4 4
the cases. 2𝑥 + 70
⇒ = 42
So, (100 – 96)% of 20 kg = (100 – 95)% of x kg 4
4 ⇒ 2𝑥 = 98
⇔ 4% of 20 kg = 5% of x kg ⇒ x = (5 × 20) kg.
𝑥 = 49 ℓ
= 6kg
∴ Initial quantity of water = 21 ℓ.

19. Solution (3)


23. Solution (4)
Quantity of pulp in fresh grapes = Quantity of
pulp in Ratio b/w winner and loser 5:1
dry grapes Total no of votes casted actually =
= (100 – 10)% of 250 kg = 90% of 250 kg = 225 1000*(88/100)*(90/100) = 792
kg. 5x + x = 792, X =132
Let the total weight of fresh grapes be x kg. Votes of winner candidate = 5*132 = 660
Then, (100 – 80)% of x
𝑥
= 225 ⇒ 20% of x = 225 ⇒ 5 = 225 24. Solution (2)
⇒ x = 225 × 5 = 1125. Given, Ramesh's marks = 300
Hence, total weight of fresh grapes = 1125 kg. Ramesh's % marks = 300*100/400 = 75%
Anwar’s% marks = 80%
20. Solution (5) Pawan’s% marks = 90%
Pawan’s marks = 90% of 400 = 360
नींव Booklet
For All Banking & Insurance Exam
Ch -6 | Percentage | Maths by Arun sir
= 100x
25. Solution (2) Number of boys = 600x – 100x = 500x
n (A) = A + 6, n (B) = (A -8), n (A ∩ B) = 28. A/q,
So, n (A U B) = n (A) + n(B) - n(A∩B) = A + 6 + 500x = 275
A - 8 - 28 = 50. x = 0.55
Total number of students = 0.55 × 600
A = 40
= 330
26. Solution (1)
Saving = 50% of (100 - 40)% of (100 - 30)% of
31. Solution (2)
Rs. 36,800 Quantity I: Because percentage of girls = 60%,
= Rs. (50/100 * 60/100 * 70/100 * 36800) = Rs. So the percentage of boys = 100 – 60% = 40%
7728. Quantity II: Last year, Girls was 300 × 50% = 150,
boys = 300 × 50% = 150
27. Solution (3) girls = 150 × 110% = 165, boys = 300 – 165 = 135
Let maximum marks be 100x
According to the question,
30% of 100x + 25 = 40% of 100x – 50 32. Solution (2)
30x + 25 = 40x – 50 Quantity I: Let the number of girls = x and the
10x = 75 number of boys = y
Maximum marks = 750 then 50% of x = 30% of y
Passing marks = 30% of 750 + 25 5x = 3y
= 225 + 25 = 250 x = number of girls = 3a then y = number of boys
250 = 5a
Required percentage = (750) × 100
percentage=3/8*100=37.5%
= 33.33%
Quantity II: Let the number of girls = x and the
number of boys = y
28. Solution (3)
then 33.33% of x = 66.67% of y
x = 2y
29. Solution (1)
x:y=2:1
15.38% = 2 × 7.69% and 7.69% = 1/13
x = number of girls = 2a
∴ 15.38% = 2/13
then y = number of boys = a
Let original number be 13x
percentage=2/3*100=66.66%
After increment = 15x
Therefore, Quantity : I < Quantity : II
Let y% should be decreased to get original number
15x (100 – y)/100 = 13
33. Solution (2)
1500 – 15y = 1300
Let the number of boys = 4x and the number of
15y = 200
girls = 5x
y = 200/15
If 45 students from the same school leave the
y = 40/3% = 13(1/3)%
school
Number of boys left=45*4/9=20
30. Solution (3)
Number of girls left=45*5/9=25
16.66% = 16(2/3)% = 50/300 = 1/6
the number of boys = 4x = 4 × 15 = 60 and
Let total number of students = 600x
the number of girls = 5x = 5 × 15 = 75
So, number of girls = 600x × 1/6
नींव Booklet
For All Banking & Insurance Exam
Ch -6 | Percentage | Maths by Arun sir
When, 50% of the boys left then the remaining Quantity I:
number of boys = 50% of 60 = 30 Let the ten’s digit and unit digit of the number be x
Total students = 30 + 75 = 105 and y respectively.
Parentage=30*105/100=28.57 According to the question,
Quantity : I < Quantity : II (x + y)2 = 144
Therefore, x + y = 12........ (1)
34. Solution (1) Also, 10x + y + 36 = 10y + x
Quantity I: the number of fit = 300*8/15 = 160 Or, 9x – 9y = -36
The number of non - fit = 300 - 160 = 140 Or, x – y = -4....... (2)
The number of occasional exercise doers = 25% of On solving equation (1) and (2), we get
160 = 40 x = 4 and y = 8
The number of daily exercise doers = 160 - 40 = 120 Therefore, the number is = 10x + y = 48
The total number of students who do gym = 60% of Quantity II:
300 = 180 Let the two numbers be 4x and 3x respectively.
The number of occasional exercise doer who do According to the question,
gym = 20% of 40 = 8 (3x + 8) = 0.775(4x + 8)
The total number of non - fit who do gym = 50% of Or, 3x + 8 = 3.1x + 6.2
140 = 70 Or, 0.1x = 1.8
The number of students who are daily exercise doer Or, x = 18
as well as do gym = 180 - 70 - 8 = 102 Therefore, smaller number is 3x = 54
Quantity II: 100 37. Solution (1)
Therefore, Q1 > Q2 Quantity I:
Let the total number of males and females be 8x
35. Solution (5) and 9x respectively.
Quantity A: (8x – 40)/(9x – 30) = 6/7
Income of A = 100x × (125/100) = 125x Or, 56x – 280 = 54x – 180
After increment income of A = 125x × 6/5 = 150x Or, 2x = 100
Or, x = 50
After increment income of B = 100x × 5/4 = 125x
Total number of females in the college = 9x = 450
Income of A is more than the income of B by =
Required number = 450/10 = 45
(150x – 125x/125) × 100 = 20%
Quantity II:
Quantity B: According to the question,
Let, the expenditure on the product be Rs x. 867 – 377 – 40 = x% of 1800
Let, the earlier price of the commodity be Rs y/kg. Or, 450 = x% of 1800
Price of the commodity now = y + y × 25/100 = Or, x = 450/18 = 25
1.25y
Earlier consumption = x/y kg 38. Solution (1)
Consumption now = x/1.25y kg Quantity I:
Reduction in consumption Required ratio = (4x + 5x): (4x – y + x + y)
= x/y – x/1.25y = x/5y kg = > 9x : 5x = 9 : 5
Required percentage = > (9/5) = 1.8
= (x/5y)/(x/y) × 100% = 20% Quantity II:
Quantity A = Quantity B Investment ratio * Period ratio = Ratio of
profit
Ratio of investment = Ratio of profit / Period
36. Solution (2)
नींव Booklet
For All Banking & Insurance Exam
Ch -6 | Percentage | Maths by Arun sir
ratio
= > (5 / 7) : (4 / 8) 40. Solution (2)
= > 10 : 7 Quantity I:
= > (10/7) = 1.428 Let x be the total number of voters
Voters promised to A = 2x/5
39. Solution (2 ) Voters backed out = 15% of 2x/5
Quantity I: Voters promised to B = 3x/5
Total number of sweets Voters backed out = 25% of 3x/5
= 80 x 15 x 80/100 + 5 x 80 x 25/100
Total No. of votes for A = 2x/5 – 15% of 2x/5 +
= 960 + 100 = 1060
25% of 3x/5 = 49x/100
Quantity II:
Total number of invalid votes = 15 % of 560000 Total No. of votes for B = 3x/5 – 25% of 3x/5
= 15/100 × 560000 +15% of 2x/5 = 51x/100
= 8400000/100 51x/100 – 49x/100 = 200
= 84000 x = 10000
Total number of valid votes 560000 – 84000 = Quantity II:
476000 Let income of B is= x
Percentage of votes polled in favour of candidate Then income of A and C should be 1.5x and 1.8x
A = 75 1.5x+x+1.8x=86000
% X=20000
Therefore, the number of valid votes polled in Income of C is 1.8*20000=36000
favour of candidate A = 75 % of 476000
= 75/100 × 476000
नींव Booklet
For All Banking & Insurance Exam
Ch - 7 | Profit & Loss | Maths by Arun sir

1. Profit earned after selling a table fan for Rs. 700 is (5) 55%
2
14 7
% more than profit incurred when it is sold
6. Cost price of a locker and its key is Rs. 75.
for Rs. 650. What would be the S.P. if he wants to
Retailer sold locker at 40% profit and key at 60%
earn a profit of 25% ?
loss. Find the cost price of locker if in whole
(1) Rs. 500 (2) Rs. 476
transaction he earns Rs. 20 as profit?
(3) Rs. 750 (4) Rs. 375
(1) Rs.60 (2) Rs.65
(5) Rs. 850
(3) Rs.55 (4) Rs.50
(5) Rs.45
2. A shopkeeper sold ball at 25% profit. Cost price of
5 balls is equal to cost price of 4 balloons. On
7. A shopkeeper have 800 kg of rice, a part of which
balloons retailer earns 40% profit. If on selling one
he sells at 10% profit and remaining at 15% loss
ball and one balloon, he earns Rs. 45 profit then
thus, he incurred overall loss of 5.625%. What
find the cost price of one balloon.
would be the profit/loss percentage, if he
(1) Rs. 40 (2) Rs. 50
interchanges the quantity he sold initially.
(3) Rs.90 (4) Rs.60
(1) 1.5% (2) 0.375%
(5) Rs. 75
(3) 1.125% (4) 2.125%
(5) 0.625%
3. Aman sold a calculator in 100% profit and a torch
in 20% profit. If selling price of calculator and
8. Selling price of three items X, Y and Z is in the
torch in same then find his overall profit
ratio of 3 : 4 : 5 and profit percent earned on
percentage.
selling these items is in the ratio of 4 : 12 : 5. If
(1) 40% (2) 50%
cost price of item X and Y is equal and cost price
(3) 55% (4) 60%
of item Z is 120 Rs. Find overall gain on three
(5) 75%
items?
(1) 120 Rs. (2) 90 Rs.
4. Ambuj sells a desktop to Ravi at 20% loss. Next
(3) 160 Rs. (4) 120 Rs.
day Ambuj change his mind and purchase the
(5) 140 Rs.
same desktop from Ravi. But Ravi sells it at 20%
profit. If actual cost price of desktop is 12000 then
9. Cost price of a pencil is 60% less than cost price of
find the total actual amount paid by Ambuj.
a pen. A pencil was sold at 40% profit and a pen
(1) 23520 (2) 13920
was sold at 15% loss. If the difference between
(3) 11520 (4) 9600
their selling price is Rs. 43.5 then find the cost
(5) 4800
price of a pencil.
(1) Rs. 80 (2) Rs. 60
5. Cost price of 4 notebooks is equal to selling price
(3) Rs. 150 (4) Rs. 40
of 5 pens and cost price of 8 pens is equal to
(5) Rs. 100
selling price of 3 notebooks then find the profit
percentage on selling one notebook and one pen
together if ratio of cost price of notebook to pen is
10. A taxi seller sells a taxi for Rs.52510 and incurs a
5 : 3.
1 loss of 11%. He sells another taxi at 5% profit. If
(1) 33 3 % (2) 50% on selling both the taxi he neither earns profit nor
2
(3) 66 % (4) 75% incurs loss, then find the cost price of second taxi.
3
नींव Booklet
For All Banking & Insurance Exam
Ch - 7 | Profit & Loss | Maths by Arun sir
(1) Rs.128300 (2) Rs.129800 (5) None of these
(3) Rs.127400 (4) Rs.126800
(5) Rs.125200 16. An Article is sold at Rs. Z thus gains 37.5%. When
the same article is sold at Rs. 2Z thus gain is (Y +
11. Byomesh, a retailer mixes 10% stones in his pulse 20)%. Find the value of Y?
but he is not content with it so he again mixes 10% (1) 175 (2) 185
more stones in the previous mixture. What is the (3) 155 (4) 175
profit percentage of the Byomesh if he sells it at (5) None of these
cost price:
(1) 15% (2) 21% 2
17. A toy seller sold a toy at 16 (3) % profit on
(3) 18% (4) 16%
Selling Price. Afterwards when the cost price
(5) None of these
reduced by 10% then he also reduced the selling
price by 10%. His percentage of profit on cost
12. Jim sells a book to Carrey at a profit of 20% and
price will be?
Carrey sells this book to Sid at a profit of 25%.
(1) 20% (2) 21%
Now Sid sells this book at a loss of 10% to Simba.
(3) 19% (4) 25%
At what percentage loss should Simba sells this
(5) 26 %
book now so that his SP becomes equal to Jim’s
CP?
18. The percentage of loss when an eyeliner is sold at
(1) 26.68% (2) 25.92%
Rs 240 is same as that of profit, when it is sold at
(3) 58.66 (4) Cannot be determined
Rs 320. The above mention profit or loss on the
(5) None of these
eyeliner is?
2 2
13. Kirti sells her headphone at a profit of 20%. If she (1) 14 (7) % (2) 16 (3) %
had bought it at 20% less and sold it for Rs. 180 (3) 20% (4) 25%
less, she would have gained 25%. Find the cost (5) 30%
price of the headphone.
(1) Rs. 800 (2) Rs. 850 19. Cost price of vinegar is Rs x per litre and it is sold
(3) Rs. 900 (4) Rs. 1000 at Rs 1.5x per litre. If the retailer mixes 2 litres
(5) None of these water with every 4 litres of pure vinegar while
selling, what is his profit percentage?
14. By selling 20 sweets, a person gains cost price of 5 (1) 100% (2) 125%
sweets. Find the profit% incurred by him. (3) 80% (4) 25%
1 (5) 55%
(1) 33 ( ) % (2) 25%
3
2
(3) 20% (4) 16 (3) % 20. An article is sold for 3304.8 rupees after successive
(5) None of these discounts of 15% and x%. If the marked price of the
article is 4800 rupees. What is the value of x?
15. The percentage profit earned by selling a earbud (1) 18 (2) 18.5
for 2120 is equal to the percentage loss incurred by (3) 19 (4) 19.25
selling the same earbud for 1520. At what price (5) None of these
should the earbud be sold to make 25% profit?
(1) 2275 ₹ (2) 2100 ₹
(3) 2650 ₹ (4) 2400 ₹
नींव Booklet
For All Banking & Insurance Exam
Ch - 7 | Profit & Loss | Maths by Arun sir
21. After giving two successive discounts, each of x%, (x – 5)%. If the selling price of each horse is same
on the marked price of an article, total discount is and cost price of one laptop is 43.75% more than
Rs 518.4. If the marked price of the article is Rs the other one, then find value of x.
1440, then the value of x is: (1) 10 (2) 20
(1) 18 (2) 24 (3) 15 (4) 25
(3) 20 (4) 25 (5) None of these
(5) None of these
27. A dishonest seller sold 'w' kg sugar at the cost price
22. The marked price of a watch is Rs. 800. The of 20 kg sugar. If he used a false weight of 800
shopkeeper gives successive discount of 10% and 125/8
x% to the customer. If the customer pays Rs. 612 (1) 15 (2) 150/8
for watch find the value of x. (3) 85/4 (4) 18
(1) 10% (2) 20% (5) None of these
(3) 25% (4) 15%
(5) None of these 28. Two articles A and B, having same marked price
have been sold at 15% and 12% discounts,
23. By how much above the cost price should an article respectively. The profit earned on articles A and B
be marked up for sale so that after allowing two was 6.25% and 20%, respectively. The cost price of
successive discounts of 20% and 6.25% on it, a net article B was Rs. 160 less than the cost price of
gain of 20% is made on the cost? article A. Find the marked price of each article.
(1) Rs. 2000 (2) Rs. 2500
2 1 (3) Rs. 2400 (4) Rs. 2800
(1) 66 3% (2) 46 4%
(5) None of these
(3) 50% (4) 60%
29. Bhanumati losses 25% by selling 12 idles for Rs
(5) None of these
60. If she has to gain 25% upon selling, then how
many idles he has to sell for Rs 100?
24. A shopkeeper allows a two successive discount of
(1) 12 (2) 13
18% and 15% on a shirt. If he allows only a single
(3) 15 (4) 16
discount of 27.5% on it, he will have a more profit
(5) 20
of Rs. 35 then find the MRP of shirt ?
(1) Rs. 1250 (2) Rs. 1275
30. Sekhar sells two products A and B have equal cost
(3) Rs. 1185 (4) Rs. 1215
price. Product A is sold at 30% profit and product
(5) Rs. 1300
B is sold at an amount of 15% less than the selling
price of product A. If the total profit is Rs.162,
25. An article is marked at 12.5% above CP. It is sold
then what is cost price of B?
at by allowing a 3 discounts of 14.28%, x% and
(1) 480 (2) 400
22.22%. find the value of x if there is loss of 37.5%
(3) 525 (4) 340
on article?
(5) 260
(1) 12.5 (2) 16.66
(3) 18.18 (4) 22.22
31. Amazon listed two headphones for Rs. 476. One of
(5) None of these
the headphones was sold at a loss of 25% and the
other at a gain of 29% and the company found that
26. An engineer bought two laptops for Rs 39,000. He
each headphone was sold at the same price.
sold one at a loss of x% and another at a profit of
नींव Booklet
For All Banking & Insurance Exam
Ch - 7 | Profit & Loss | Maths by Arun sir
Quantity I: The cost price of the headphone which price. Find the profit percent or loss percent in this
was sold at 29% profit transaction?
Quantity II: The selling price of the headphone (1) Quantity I > Quantity II
which was sold at 25% loss (2) Quantity I < Quantity II
(1) Quantity I > Quantity II (3) Quantity I ≥ Quantity II
(2) Quantity I < Quantity II (4) Quantity I ≤ Quantity II
(3) Quantity I ≥ Quantity II (5) Quantity I = Quantity II or Relation cannot be
(4) Quantity I ≤ Quantity II Established.
(5) Quantity I = Quantity II or Relation cannot be 35. Quantity I: Monu purchased two articles at same
Established. price at Rs. 2550 and he sold one article at 24% of
profit and another at 42% of profit, then what is the
32. A shopkeeper gives 10% discount on the marked difference between the selling price of both
price but adds 5% tax on the discounted price. articles?
Quantity I: If the selling price of the article is Rs. Quantity II: Kajal purchased an article and marked
850.5 then what is the marked price of the article? up 10% above its cost price while selling the article
Quantity II: 900 he gave discount of 5%, then what is the cost price
(1) Quantity I > Quantity II of the article if he earned Rs. 18 as profit?
(2) Quantity I < Quantity II (1) Quantity I > Quantity II
(3) Quantity I ≥ Quantity II (2) Quantity I < Quantity II
(4) Quantity I ≤ Quantity II (3) Quantity I ≥ Quantity II
(5) Quantity I = Quantity II or Relation cannot be (4) Quantity I ≤ Quantity II
Established. (5) Quantity I = Quantity II or Relation cannot be
Established.
33. The ratio of the cost price to selling price of an
article is in the ratio of 4 : 5. If the selling price was 36. An article was sold at a profit of Rs 144 such that a
decreased by Rs. 500 then the ratio of the cost price discount of 25% was allowed while selling. The
to selling price will become 6 : 5. article was marked up at Rs 432 more than its cost
Quantity I: At what price (In Rs.) should the article price.
be sold to earn a profit of 40% on the cost price? Quantity I: The selling price of the article
Quantity II: Rs. 1700 Quantity II: If the article was marked up at Rs 140
(1) Quantity I > Quantity II more than its cost price, what would have been the
(2) Quantity I < Quantity II marked price of the article?
(3) Quantity I ≥ Quantity II (1) Quantity I > Quantity II
(4) Quantity I ≤ Quantity II (2) Quantity I < Quantity II
(5) Quantity I = Quantity II or Relation cannot be (3) Quantity I ≥ Quantity II
Established. (4) Quantity I ≤ Quantity II
(5) Quantity I = Quantity II or Relation cannot be
34. Quantity I: A shopkeeper bought some Apples at Established.
the rate of Rs. 24 for 16 and sold all of them at the
rate of Rs. 27 for 15. Find his profit percent or loss 37. Quantity I: A bangle was sold for Rs. 480 with a
percent in this transaction? profit of 20 %. If it were sold for Rs. 450, then what
Quantity II: A shopkeeper marked his product would have been the percentage of profit?
60% above cost price and sold it after two Quantity II: A person bought two books for Rs.
consecutive discounts of 10% and 15% on marked 500 each. He sold one at a profit of 25 % and the
नींव Booklet
For All Banking & Insurance Exam
Ch - 7 | Profit & Loss | Maths by Arun sir
other at a loss of 5 %. What would be his overall gets the profit of 25%, then find the cost price of
profit or loss in the transaction? the article?
(1) Quantity I > Quantity II (1) Quantity I > Quantity II
(2) Quantity I < Quantity II (2) Quantity I < Quantity II
(3) Quantity I ≥ Quantity II (3) Quantity I ≥ Quantity II
(4) Quantity I ≤ Quantity II (4) Quantity I ≤ Quantity II
(5) Quantity I = Quantity II or Relation cannot be (5) Quantity I = Quantity II or Relation cannot be
Established. Established.

38. Quantity I: The shopkeeper sold an article at 25 %


discount on marked price and he gains 10 %. If the
marked price of the article is Rs. 660, then find the
cost price?
Quantity II: The shopkeeper marks the book for
Rs. 400 and his profit percentage is 20%. Find the
cost price of the book, if he allows a discount 10 %?
(1) Quantity I > Quantity II
(2) Quantity I < Quantity II
(3) Quantity I ≥ Quantity II
(4) Quantity I ≤ Quantity II
(5) Quantity I = Quantity II or Relation cannot be
Established.

39. Quantity I: Ratio of the marked price to the cost


price of the mobile is 3:2 respectively and the
shopkeeper offers the discount of Rs.36. If the cost
price of the mobile is Rs.300, then find the profit
earned by the shopkeeper?
Quantity II: Rs.112
(1) Quantity I > Quantity II
(2) Quantity I < Quantity II
(3) Quantity I ≥ Quantity II
(4) Quantity I ≤ Quantity II
(5) Quantity I = Quantity II or Relation cannot be
Established.

40. Quantity I: A shopkeeper sold a cooker at the profit


of 15%, if he sold it for Rs.180 more he would get
a profit of 27%. What is the selling price of the
cooker, if he sold it at a loss of 15%?
Quantity II: Ratio of the cost price and marked
price of an article is 2:3 respectively. If the
shopkeeper offers the discount of Rs.300 while he
नींव Booklet
For All Banking & Insurance Exam
Ch - 7 | Profit & Loss | Maths by Arun sir

Answer Key
1. (4) 10. (2) 19. (2) 28. (3) 37. (1)
2. (5) 11. (2) 20. (5) 29. (1) 38. (1)
3. (2) 12. (2) 21. (5) 30. (2) 39. (1)
4. (2) 13. (3) 22. (4) 31. (2) 40. (1)
5. (2) 14. (2) 23. (4) 32. (5)
6. (2) 15. (1) 24. (1) 33. (2)
7. (5) 16. (3) 25. (2) 34. (2)
8. (2) 17. (1) 26. (2) 35. (1)
9. (2) 18. (1) 27. (5) 36. (1)
नींव Booklet
For All Banking & Insurance Exam
Ch - 7 | Profit & Hints
Loss | &
Maths by Arun sir
Solutions
1. Solution ((4) 9600×120
Ravi again sell to Ambuj → 100
= 11520
Let C.P. of table fan be Rs. x
Total Amount Paid by Ambuj → 12000 +
ATQ,
11520 − 9600
8
(700 – 𝑥) = (650 – 𝑥) = 13920
7
4900 – 7x = 5200 – 8x 5. Solution (2)
x = 300 Let cost price of 1 notebook = 5x
S.P to earn 25% profit Cost price of 1 pen = 3x
4×5𝑥
125 SP of 1 pen = 5
= 4𝑥
= 300 × 8×3𝑥
100 SP of 1 notebook = = 8𝑥
= Rs. 375. 3
8𝑥+4𝑥−5𝑥−3𝑥 4
2. Solution (5) Profit % = 5𝑥+3𝑥
× 100 = 8 × 100 = 50%
Let C.P. of 1 ball = 4x 6. Solution (2)
S. P. of 1 ball = 4x × 1.25 = 5x Use mixture and allegation
4x × 5 If cost price of locker = 75
C. P. of 1 balloon = = 5x
4 40
S.P. 1 balloon = 5x × 1.4 =7x Profit on locker = 75 × = 30
100
ATQ, If cost price of key = 75
5x + 7x – 4x – 5x = 45 60
Loss on key = 75 × =– 45
3x = 45 100
x = 15
cost price of one balloon = 15 × 5 = Rs. 75
3. Solution (2)
Sol.
CP SP
𝐀 1 : 2
𝐁 5 : 6

Cost price of locker = 65


S.P. is same 7. Solution (5)
∴ Multiply C.P. and S.P. ratio of calculator by ‘3’ Total rice = 800 kg
CP SP By using Allegation method
𝐂𝐚𝐥𝐜𝐮𝐥𝐚𝐭𝐨𝐫 3 : 6 Profit Loss
𝐓𝐨𝐫𝐜𝐡 5 : 6 +10% -15%
Total 8 ∶ 12
12 – 8
Required % = × 100 -5.625%
8
4 9.375 15.625
= × 100 3 : 5
8 800
= 50% rice sold at 10% profit = 8
×3 = 300 kg
4. Solution (2) 800
rice sold at 15% loss = 8
×5 = 500 kg
Sol.
Let cost price = x Rs./ kg
Cost price → 12000
12000×80 when quantity interchanged →
Ambuj sell to Ravi → = 9600
100
नींव Booklet
For All Banking & Insurance Exam
Ch - 7 | Profit & Loss | Maths by Arun sir
Selling price of that quantity which is sold at loss Amount of loss incurred on selling 1st taxi =
85 100
= 300x × 100 = 255x [52510 × − 52510]
89
Selling Price of that quantity which is sold at = 59000 – 52510 = Rs. 6490
Profit Let cost price of second taxi be Rs. 20x.
500 × 𝑥 ×110 105
= 100
= 550x Selling price of second taxi will be = 20𝑥 × 100 =
Total Cost Price = 800x 𝑅𝑠. 21𝑥
Total Selling Price = 255x + 550x = 805x ATQ,
805𝑥 –800𝑥
Profit % = ×100 = 0.625% 21𝑥 − 20𝑥 = 6490
800𝑥
8. Solution (2) 𝑥 = 6490
Let selling price of item X, Y and Z be 3y, 4y and 20𝑥 = 𝑅𝑠. 129800
5y respectively 11. Solution (2)
And, profit percentage on these items Let Initial Quantity of Pulse = 100 kg
4x, 12x and 5x respectively After mixing 10% stones, Quantity of the mixture
110
ATQ = 110 × 100 = 121 kg
3𝑦 (100+4𝑥)
= (100+12𝑥) CP of One kg of Pulse = Rs.1
4𝑦
Total CP = Rs.100
36x – 16x = 400 – 300
Total SP = Rs.121
x=5
Profit = 121 – 100 = 21
So, profit percentage on item X, Y and Z is 20%, 100
60%, 25% respectively Profit % = 21 × = 21%
100
C.P of item Z = Rs. 120 12. Solution (2)
1 Jim Carrey Sid Simba
⇒5y = 120 + 120 × 4
100 120 150 1325
5y = 150 135−100
y= 30 Rs. Required % = 135
× 100
Selling price of item X = 90 Rs. 35
= × 100
Selling price of item Y = 120 Rs. 135
Overall profit = 25.92
90 120 150 13. Solution (3)
= 6
×1+ 8
×3 + 5
×1
14. Solution (2)
= 15+ 45 + 30
Profit = Selling Price (SP) – Cost Price (CP)
= 90 Rs.
CP of 5 sweets = SP of 20 article – CP of 20
9. Solution (2)
sweets
Let the cost price of a pen be Rs. 100x.
So CP of 25 sweets = SP of 20 sweets
Then, cost price of a pencil = 40x 5
140 Profit% = 20 × 100 = 25%
S.P. of pencil = 40𝑥 × 100= 56𝑥
85 15. Solution (1)
SP of pen = 100𝑥 × = 85𝑥 ⇒ The Cost Price of the earbud be x
100
ATQ, ⇒ So as per question percentage profit earned by
85𝑥 − 56𝑥 = 29𝑥 = 43.5 selling a earbud for 2120 ₹ is equal to the
⇒ x = 1.5 percentage loss incurred by selling the same
C.P. of a pencil = 40 × 1.5 = Rs. 60 earbud for 1520 ₹
10. Solution (2) 2120 − x x − 1520
⇒ i. e. =
x x
नींव Booklet
For All Banking & Insurance Exam
Ch - 7 | Profit & Loss | Maths by Arun sir
3640 21. Solution (5)
⇒ x = = 1820 ₹
2 Each discount = 𝑥𝑦
⇒ Selling Price to make 25% profit (100 − 𝑥) (100 − 𝑥)
125 5 ∴ 1440 × ×
⇒ 1820 × = 1850 × 100 100
100 4 = (1440 − 518.4)
= 2275 ₹ 921.6 × 10000
16. Solution (3) ⇒ (100 − 𝑥)2 =
1440
Let the cost price of the article = Rs. 8a ⇒ (100 − 𝑥)2 = 6400
Selling price of article when sold at 37.5% profit = ⇒ (100 − 𝑥) = 80 ⇒ 𝑥 = 20
11
8a × = 11a 22. Solution (4)
8
2
When the article is sold at Rs. 2Z, selling price = (100 − 𝑥) 90
800 × × = 6)
2×11a = 22a 100 100
Ratio of CP and SP of Article = 8a : 22a = 4:11 6120
⇒ 100 − 𝑥 =
11 − 4 72
Y + 20% = × 100 = 175%
4 ⇒ 100 − 𝑥 = 05 ⇒ 𝑥 = 15
So, value of Y = 175 - 20 = 155 23. Solution (4)
Hence answer is option C Let cost price be Rs. 100 and CP is 𝑥% marked
17. Solution (1) ⇒ (100 + 𝑥) = 𝑀𝑃
Selling Price = SP
𝑆𝑃 = 120
Cost Price = CP 80 93.75
Profit on SP=
1 ∴ (100 + 𝑥) × × = 120
6 100 100
SP:CP=6:5 multiply by 10 for easy calculation 120 × 1000
⇒ (100 + 𝑥) =
= 60:50 8 × 93.75
54−45 ⇒ 100 + 𝑥 = 160 ⇒ 𝑥 = 60
60-6 : 50-5=54:45= 45
× 100 = 20%
24. Solution (1)
18. Solution (1) Let MRP of shirt be Rs. 100𝑥.
240 + 320
CP = = 280 SP in first case.
2 82 85
Profit = 320 − 280 = 40 = 100𝑥 × × = 69.7𝑥
100 100
40 2
Profit % = ( ) × 100 = 14 ( ) % SP in second case
280 7 (100 − 27.5)
19. Solution (2) = 100x × = 72.5𝑥
100
Let CP of 1 l pure vinegar = Re 1
⇒ 72.5𝑥 − 69.7𝑥 = 35
Then CP of 4 l of vinegar = Rs 4, and CP of 2 l of
⇒ 2.8𝑥 = 35 ⇒ 𝑥 = 12.5
water = Rs 0. So CP of 6 l mixture = Rs 4
∴ 𝑀𝑅𝑃 = 1250
And SP of 6 l of mixture = 6×1.5 = Rs 9
9−4
25. Solution (2)
So profit% = 4
× 100 = 125% 25
∴ 𝑀𝑃 = 112.5% of 𝐶𝑃 = (1 + )
20. Solution (5) 200
(100 − 15) (100 − 𝑥) 𝑀𝑃 9
4800 × × =
100 100 𝐶𝑃 8
= 3304.8 Let 𝐶𝑃 = 8, 𝑀𝑃 = 9
330480 1
⇒ (100 − 𝑥) = 14.28% =
48 × 85 7
⇒ 100 − 𝑥 = 81 ⇒ 𝑥 = 19 2
22.22% =
9
नींव Booklet
For All Banking & Insurance Exam
Ch - 7 | Profit & Loss | Maths by Arun sir
𝑥 Then, the selling price of article A = 85% of x = Rs.
𝑥% =
100 17x/20
3
37.5% = And, the selling price of article B = 88% of x = Rs.
8 22x/25
5
∴ 5𝑝 = × 8 = 5 Cost price of article A = 17x/20 × 100/106.25 = Rs.
8
6 7 𝑥 4x/5
⇒ 9 × × × (1 − )=5 Cost price of article B = 22x/25 × 100/120 = Rs.
7 9 100
𝑥 5 𝑥 1 11x/15
⇒ (1 − )= ⇒ = According to question,
100 6 100 6
50 4x/5 – 11x/15 = 160
⇒𝑥= = 16.66
3 (12x – 11x)/15 = 160
26. Solution (2) x/15 = 160
3 175 7 x = 2400
43.75% = 43 ( ) % = =
4 400 16 Therefore, the marked price of each article = Rs.
Let cost price of one laptop be Rs 16a 2400
Then, cost price of second one = 23a 29. Solution (1)
So, 16a + 23a = 3900 60
Selling price of each idly = 12 = 𝑅𝑠 5,
39a = 39000
Given that he had incurred 25% loss on this price,
a = 1000
i.e. SP = 75% of CP,
Cost price of one laptop = 16000
3
Cost price of other one = 23000 5 = × CP,
4
Selling price of first laptop 20
= 23000 ×
100 – 𝑥
= 23000 – 230x CP = Rs (Cost price of each idly)
100 3
Selling price of second laptop Now to make 25% profit on selling this idly,
= 16000 ×
100 + 𝑥 – 5
= 160x + 15200 SP = 125% of CP,
100 5 20
According to the question, SP = × ,
4 3
160x + 15200 = 23000 – 230x SP = Rs
25
(new selling price of each idly to make 25% profit)
3
390x = 7800
7800
So number of idles he has to sell for Rs 100
x= = 20 100
390 = 25 = 12 idles
27. Solution (5) 3

Let actual profit% = P% 30. Solution (2)


Net profit% = 60% Let CP of the product A = x
Profit% due to false weight = ((1000 - 800)/800) * 130 13x
SP of product A = x × =
100 = 25% 100 10
13x 17 221x
Then, 60 = P + 25 + (P * 25/100) SP of product B = × =
P% = 28% 10 20 200
13x 3x
Now, w * SP = 20 * CP profit of product A = –x =
10 10
SP = (20/w) * CP 221x 21x
Now, 28/100 = ((20/w) * CP - CP)/CP) profit of product B = =
200 200
w = 125/8 kg 3x 21x
28. Solution (3) + = 162
10 200
Let, the marked price of each article was Rs. ‘x’ 21x
60x + = 162
200
नींव Booklet
For All Banking & Insurance Exam
Ch - 7 | Profit & Loss | Maths by Arun sir
x = 400 Selling price of the product = 0.85 × 0.90 × 160 =
31. Solution Rs. 22.40
Let the cost price of Quantity II be x. Profit=22.4%
Therefore, the cost price of Quantity I will be Quantity I < Quantity II
= (476 – x) 35. Solution
Quantity II Quantity I Quantity I:
Cost price = x 476-x C.P of first article = C.P of second article = Rs.
Selling price= x*75/100 (476-x)*129/100 2550
Both sp are equal So, S.P of first article = 2550 * 124/100 = Rs.
25x = (476 – x) × 43 3162
25x + 43x = 476 × 43 And, S.P of second article = 2550 * 142/100 = Rs.
68x = 476 × 43 3621
x = 301. So, difference = 3621 - 3162 = Rs. 459
Quantity II:
Let C.P = Rs. 100x
32. Solution M.P = 100x * 110/100 = Rs. 110x
Let the marked price of the article = 10x And, S.P = 110x * 95/100 = Rs. 104.5x
then discounted price after 10% discount on the So, profit = 104.5x - 100x = 4.5x
marked price = (100 – 10)% of 10x = 90% of 10x => 18 = 4.5x
= 9x => x = 18/4.5
The selling price after adding 5% sales tax on the => x = 4
discounted price = (100 + 5)% of 9x = 105% of 9x C.P = Rs. 400
= 1.05 × 9x 36. Solution
Quantity I: 1.05 × 9x = 850.5 1.05x = 94.5 x = Let the Cost Price of the article be Rs 'x'
90 The marked price of the article = 10x = 10 × 90 So, Marked Price of the article = Rs (x + 432)
= Rs. 900 Selling Price of the article = (x + 432)*(75/100) =
Quantity II: 900 Rs ((3x/4) + 324)
Therefore, Quantity I = Quantity II So, (3x/4) + 324 - x = 144
33. Solution x/4 = 180
Let the CP = Rs. 4x then SP = Rs.5x x = 720
According to the question Quantity I:
4x/5x-500= 6/5 Selling Price of the article = (3/4)*720 + 324 = Rs
10x = 3000 864
x = 300 Quantity II:
Cost price = 4x = Rs. 1200 Marked Price of the article = 720 + 140 = Rs 860
Quantity I : SP to earn a profit of 40% = 140% of 37. Solution
1200 = Rs. 1680 Quantity I:
34. Solution According to the question,
CP of one Apple= 24/16= 1.50 (120/100) * CP = 480
SP of one Apple= 27/15=1.80 CP = 480 * (4/5) = Rs. 400
Profit=1.80-1.50=0.30 SP = 450
Profit Percentage= 0.30/1.50*100=20% Profit % = (Profit / CP) * 100
Let the cost price of the product = Rs. 100 = > (50 / 400) * 100
Marked price of the product = 1.60 × 100 = 160 = > 12.5 %
नींव Booklet
For All Banking & Insurance Exam
Ch - 7 | Profit & Loss | Maths by Arun sir
Quantity II: Profit of the mobile = 414 – 300 = Rs.114
According to the question, Quantity 2: 112
CP1= 500, Profit = 25 % Quantity I>quantity II
SP1 = 500 * (125 / 100) = 625 40. Solution
CP2 = 500, Loss = 5 % Cost price of the cooker = Rs.x
SP2= 500 × (95 / 100) = 475 selling price of the cooker = 115/100 * x
Total selling price = S.P1 + S.P2 = 625 + =115x/100
475 = Rs. 1100 selling price of the cooker after sold Rs.180 more=
Total cost price = 500 + 500 = 1000 127/100 * x =127x/100
Profit % = (100 / 1000) * 100 = 10 % (127x/100) - (115x/100) = 180
38. Solution 12x = 18000
Quantity I: x = 1500
660*(75/100) = CP*(110/100) Cost price of the cooker = Rs.1500
CP = 660*(75/100)*(100/110) = Rs. 450 Selling price of the cooker = 1500 * 85/100 =
Quantity II: Rs.1275
400*(90/100) = CP*(120/100) Ratio of the cost price and marked price of the
CP = 400*(90/100)*(100/120) = Rs. 300 article is 2x:3x
The shopkeeper offers the discount of Rs.300
while he gets the profit of 25%.
39. Solution 3x – 300 = 2x * 125/100
Cost price of the mobile = Rs.300 x = 600
Marked price of the mobile = 300 * 3/2 = Rs.450 Cost price of the article = 600 * 2 = Rs.1200
Selling price of the mobile = 450 – 36 = Rs.414 Quantity I>quantity II
नींव Booklet
For All Banking & Insurance Exam
Ch-8 | Simple Interest | Maths by Arun sir
1. Find the amount paid by a borrower after 3 years, 8. Vishwas borrowed a total amount of Rs. 30000
if he has taken a loan of Rs. 5000 at 10% per part of it on simple interest rate of 12 p.c.p.a. and
annum, simple interest. remaining on simple interest rate of 10 p.c.p.c. If at
(1) 6000 (2) 6500 the end of 2 years he paid in all Rs. 36480 to settle
(3) 5500 (4) 6750 the loan amount, what was the amount borrowed at
(5) None of these 12 p.c.p.a?
(1) 16000 (2) 18000
2. Find the amount of money that Abha should lend (3) 17500 (4) 12000
at 15% p.a If she wants to earn an interest of Rs. (5) None of these
675 in 2 (1 / 2) years?
(1) 2000 (2) 1850 9. Veena obtained an amount of Rs. 8376 as simple
(3) 1800 (4) 2010 interest on a certain amount at 8 p.c.p.(1) after 6
(5) None of these years. What is the amount invested by Veena?
(1) 17180 (2) 18110
3. A sum of money amounts to Rs. 1650 in two years (3) 16660 (4) 17450
and to Rs. 1875 in five years. Find the principal (5) None of these
and rate of simple interest?
(1) 1500 and 5% (2) 2000 and 5% 10. If the simple interest for 6 years be equal to 30%
(3) 1000 and 4% (4) CND of the principal, then it will be equal to the
(5) None of these principal after?
(1) 20 (2) 30
4. A certain sum of money amounts to Rs. 15,500 in (3) 10 (4) 22
2 years at simple rate of interest of 12% p.a. Find (5) None of these
the principal?
(1) 13000 (2) 12500 11. From a bank, Ram and Shyam together took a
(3) 14000 (4) 15000 certain amount under simple interest and they lent
(5) None of these the total amount to Mohan at 2% more simple
interest. At the end of 4 years, the total money
5. A sum of money doubles itself in 6 years. In how earned by Ram after paying the interest to the bank
many years will it becomes 6 times at the same was Rs. 400 more than that of Shyam. From the
rate of simple interest? bank, the total amount taken by Ram was how
(1) 20 (2) 25 much more than that of Shyam?
(3) 15 (4) 30 (1) 10000 (2) 20000
(5) None of these (3) 5000 (4) 25000
(5) CND
6. Abhishek lent Rs. 10500, partly to Shahid at 10%
p.a. S.I and partly to John, at 15% p.a., S.I. If at the 12. Deepika deposited Rs. 1000 in a fund in 2019
end of 5 years, total amount received by Abhishek, which provides simple interest. The interest rate on
from both, was equal to Rs. 17375, then find the the fund increases by 3% every year. If the interest
amount of money lent to Shahid? rate at the time of amount deposit was 10%, find
(1) 6000 (2) 6500 the interest earned by her after 9 years?
(3) 5500 (4) 4000 (1) 1880 (2) 1890
(5) None of these (3) 2980 (4) 1790
(5) 1980
7. Sachin would have paid Rs. 5280, at the end of 4
years, for a sum of money borrowed, at rate of 8% 13. Rayan invested a total of Rs.49000 in two different
p.a. S.I. If he wants to repay his loan an year schemes A and B The scheme A which offers
before it was due, then what is the amount paid by interest at a rate of 5% per annum and scheme B
him? offers interest at a rate of 12%. If the total interest
(1) 5060 (2) 4960 earned by Rayan after 1 year is Rs.4900 then find
(3) 4760 (4) 4660 the sum invested in scheme B?
(5) None of these (1) 34000 (2) 19000
(3) 35000 (4) 14000
नींव Booklet
For All Banking & Insurance Exam
Ch-8 | Simple Interest | Maths by Arun sir
(5) 30000 12% p.(1) for the period beyond 8 years. If the
total interest paid by him at the end of 12 years is
14. Arjun distributed 65% of the money he had Rs. 6780, how much money did he borrow?
between A and B in the ratio 6 : 7 respectively. B (1) 6000 (2) 5000
and A deposited the amount received in a scheme (3) 4500 (4) 5500
offering 8% and 7% simple interest respectively (5) 3840
for five years. Find the amount left with Arjun if
the difference in the interests earned by A and B 20. A person invested Rs. x under simple interest for
after five years is Rs. 1890. 10 years at 10% per annum. If he had invested Rs.
(1) 17800 (2) 18300 2x under simple interest for 15 years at the rate of
(3) 18900 (4) 19500 interest 20% per annum then he would have
(5) None of these received Rs. 2500 more simple interest. Find the
value of X?
15. After four years, the simple interest obtained on (1) 2500 (2) 500
Rs. 8250 at the rate of (x + 4)% is Rs. 1320 more (3) 3250 (4) 3750
than the simple interest obtained at the rate of 8% (5) None of these
per annum. Find the value of ‘x’.
(1) 12 (2) 10 21. Amount of Rs. 2 lacs was invested at a simple
(3) 8 (4) 15 Interest of 5%. After five years, the interest earned
(5) None of these in first five years is added to the principal, and
from sixth year, simple Interest is calculated on
16. A sum of Rs. 12,000 invested at 8% simple interest new Principal amount. After how many years it
becomes Rs. 15840 in x years and another sum of will become 3,50,000 ? (Interest rate remains same
Rs. 15,000 invested at 12% simple interest for all years)
becomes Rs. 18,600 in y years. Find the value of x (1) 5 (2) 7
+ y. (3) 8 (4) 10
(1) 6 (2) 7 (5) 12
(3) 5 (4) 8
(5) 10 22. A man spends 20% of his monthly income on rent.
Out of the remaining monthly income, he spends
17. The salary of a man is Rs. 60000, from which he 25% on food, Rs. ‘a’ on transportation and the
deposits x% amount at 13% simple interest. If the remaining money is deposited in the savings
accumulated amount for the sum deposited after 3 account which is 48% of the total monthly salary.
years was Rs. 29190, then find the value of ‘x’. If the amount is deposited for 5 years in the
(1) 40 (2) 35 savings account he gets a simple interest of Rs.
(3) 45 (4) 50 8294.4 at the rate of 7.2% per annum, then find the
(5) None of these value of ‘a’?
(1) 6240 (2) 7280
18. The rate of interest for the first 2 years is 3% per (3) 5760 (4) 6860
annum, for the next 3 years is 8% per annum, and (5) None of these
for the period beyond 5 years is 10% per annum. If
the man withdraws total amount of Rs. 5320 after 23. The simple interest received on a sum of money at
6 years, find the sum he deposited? 10% per annum in some years is half of the sum. If
(1) 3800 (2) 4320 the rate of simple interest is 3% more then the
(3) 2380 (4) 3380 simple interest received on the same sum for the
(5) None of these same period is Rs. 450 more. Find the sum of
money?
19. Sanjay borrowed certain amount of money at (1) 2400 (2) 2500
simple interest at the rate of 5% p.a. for the first (3) 2750 (4) 3000
three years, 10% p.a. for the next five years and (5) None of these
नींव Booklet
For All Banking & Insurance Exam
Ch-8 | Simple Interest | Maths by Arun sir
29. The interest earned when Rs. P is invested for Six
24. Ajay invests some amount of money at a certain years in a scheme offering 13% pa simple interest
rate of simple interest per annum and gets Rs. is more than the interest earned when the same
3120 as interest after 1 year. If the rate would have sum (Rs. P) is invested for Three years in another
been 4% more per annum he would have earned scheme offering 10% pa simple interest, by Rs.
Rs. 3120 more in 18months. What was the amount 3168. What is the value of P?
invested by Ajay? (1) 5500 (2) 6600
(1) 26000 (2) 24000 (3) 4400 (4) 3300
(3) 30000 (4) 32000 (5) None of these
(5) None of these
30. Manish lends x% of his monthly salary which is
25. Ashok has Rs 1,60,000 with him. He lends some Rs. 60000 at 20% simple interest to his colleague
money at 7% p.a. and rest of the money at 12% Harish. If the amount accumulated after 4 years
p.a. simple interest. He receives a total of Rs was Rs. 32400, then find the value of x?
14400 at the end of 1 year. What is the amount lent (1) 20 (2) 30
at 12% per annum? (3) 15 (4) 25
(1) 56000 (2) 64000 (5) 35
(3) 72000 (4) 96000
(5) None of these 31. Puneet invest an amount of Rs 25600 at the rate of
5% per annum for 5 years to obtain simple interest,
26. Rajat lends Rs. 20,000 to two of his friends. He later he invested the principal amount as well as
gives Rs.10,000 to the first at 15% p.(1) simple the amount obtained as simple interest for another
interest. Rajat wants to make a profit of 20% on 3 years at the same rate of interest. What amount
the whole. The simple interest rate at which he of simple interest will obtained at the end of the
should lend the remaining sum of money to the last 3 years
second friend is? (1) 6000 (2) 4800
(1) 12 (2) 16 (3) 2000 (4) 1500
(3) 20 (4) 25 (5) 7000
(5) None of these
32. Amit borrowed some money at a rate of 5% p.a.
27. The rate of interest on a sum of money is 4% per for the first 2 years, at a rate of 10% p.a. for next 4
annum for the first 2 years, 6% per annum for the years and a rate of 12% p.a. for the period beyond
next 4 years and 8% per annum for the period 6 years. If he pays total simple interest of
beyond 6 years. If the simple interest accrued on Rs.10584 at the end of 10 years. Find how much
the sum for a total period of 9 years is Rs. 1680, money did he borrowed.
what is the sum? (1) Rs. 14800 (2) Rs. 10800
(1) 3000 (2) 5000 (3) Rs. 14200 (4) Rs. 15800
(3) 4700 (4) 5500 (5) Rs. 16200
(5) None of these
33. P is borrowed a sum of RS. 6000 from T at the
28. Simple interest on a certain amount is 16/25 of the rate of 14% for 2 years. She then added some more
principal, provided that rate of interest and time money to the borrowed sum and lent it to U at the
period in years are same. What will be the simple rate of 18% of simple interest for the same time. If
interest after twelve years if the principal is Rs. P gained Rs. 650 in the whole transaction, then
25000 and the rate is same? what sum did he lend to U?
(1) 12000 (2) 24000 (1) Rs.6427.12 (2) Rs.8015.41
(3) 22000 (4) 26000 (3) Rs.6472.22 (4) Rs.7541.2
(5) None of these (5) Rs.6758.2
नींव Booklet
For All Banking & Insurance Exam
Ch-8 | Simple Interest | Maths by Arun sir
34. The rate of interest on a sum of money is 4% per 38. After four years, the simple interest obtained on
annum for the first 2 years, 6% per annum for the Rs. 8250 at the rate of (x+4)% is Rs. 1320 more
period next 4 years, 8% per annum for the period than the simple interest obtained at the rate of 8%
beyond 6 years. If the simple interest accrued by per annum. Find the value of ‘x% of 500’.
the sum for a total period of 9 years is Rs. 3360, (1) 32 (2) 10
what is the sum? (3) 30 (4) 40
(1) Rs.7000 (2) Rs.5000 (5) None of these
(3) Rs.4700 (4) Rs.5500
(5) Rs.6000 39. Sandeep invests Rs.1600 for 2 years and received
the interest of Rs.1408 at R% simple interest. If he
35. The rate of simple interest of a sum is 6% per invests the same amount at (R – 16)% for 2 years,
annum for the first 3 months, next five months the find the interest.
rate of interest is 8% per annum and 5% per (1) Rs.642 (2) Rs.734
annum for the period of beyond 8 months. If the (3) Rs.824 (4) Rs.896
total simple interest is Rs 7020 for the period of (5) None of these
one year then find what is the sum?
(1) Rs. 156100 (2) Rs. 154000 40. Two persons Rahul and Ramesh invested rs. (x +
(3) Rs. 156000 (4) Rs. 146100 3400) and (x+ 2200) at the rate of 15% and 20%
(5) Rs. 108000 respectively for two years in simple interest. After
two years total SI is rs.7220, then how much
36. A bike financier claims to be lending money at amount invested by Ramesh?
simple interest, but he includes the interest every (1) 7600 (2) 9820
six months for calculating the principal. If he is (3) 5840 (4) 6450
charging an interest of 10%, the effective rate of (5) 9240
interest becomes:
(1) 9.5% (2) 8%
(3) 10.25% (4) 10%
(5) None

37. Total sum of rs.4740 is invested by two friends


Aman and Bhuvi in a scheme at the rate interest of
12% p.a for 4 year in SI. If interest received by
Bhuvi is 67.20 more than interest received by
Aman, then find how much amount invested by
Aman?
(1) 2800 (2) 2300
(3) 1840 (4) 2350
(5) None of these
नींव Booklet
For All Banking & Insurance Exam
Ch-8 | Simple Interest | Maths by Arun sir

Answer Key
1. (2) 9. (4) 17. (2) 25. (2) 33. (1)
2. (3) 10. (1) 18. (1) 26. (4) 34. (2)
3. (1) 11. (3) 19. (1) 27. (1) 35. (4)
4. (2) 12. (5) 20. (2) 28. (2) 36. (4)
5. (4) 13. (3) 21. (3) 29. (2) 37. (2)
6. (4) 14. (3) 22. (3) 30. (2) 38. (4)
7. (2) 15. (3) 23. (4) 31.(2) 39. (1)
8. (4) 16. (1) 24. (1) 32.(3) 40. (3)
नींव Booklet
For All Banking & Insurance Exam
Ch-8 | Simple Interest | Maths by Arun sir
Hints & Solutions
1. Solution (2) Let the sum borrowed at the rate of 12 p.c.p.a be
S.I = (P × r × t) / 100 Rs. x
= (5000 × 10 × 3) / 100 = Rs. 1500 Sum borrowed at 10 p.c.p.a = Rs. (30000 – x)
Amount payable after 3 years = 5000 + 1500 = Rs. Simple interest = Rs. (36480 – 30000) = Rs. 6480
6500 According to the question,
2. Solution (3) [(x × 2 × 12) / 100] + [((30000 – x) × 2 × 10) /
S.I = 675 = P × (15 / 100) × (5 / 2) 100] = 6480
P = (675 × 100 × 2) / (15 × 5) 24x + 600000 – 20x = 648000
P = Rs. 1800 4x = 648000 – 600000 = 48000
3. Solution (1) x = 48000 / 4 = Rs. 12000
Interest for a period of 3 years = 1875 – 1650 = 9. Solution (4)
Rs. 225 S.I = (P × r × t) / 100
Interest for 1 year = 225 / 3 = Rs. 75 8736 = (P × 6 × 8) / 100 = (8736 × 100) / (6 × 8)
Interest for 2 years = 75 × 2 = Rs. 150 = Rs. 17450
Amount after 2 years = 1650 10. Solution (1)
Principal = 1650 – 150 = Rs. 1500 We are given S.I = (30 / 100) P in 6 years, so
And rate of interest = (75 / 1500) × 100 = 5% per To make S.I equal to P : multiply (30 / 100) P by
annum (100 / 30)
[ S.I for 1 year = (P × r × t) / 100 S.I = (30 / 100)P × (100 / 30) in (6 × 100) / 30
75 = (1500 × r × t) / 100 ] years
4. Solution (2) S.I = P in 20 years
Amount = P + Interest Method 2 :
15500 = P + (P × 12 × 2) / 100 30 / 100 P = (P × 6 × r) / 100
15500 = (124 / 100)P r=5%
P = 12500 finding time when S.I = P
5. Solution (4) P = P × (5 / 100) × t
When the sum doubles itself, means interest equal t = 20 years
to amount of principal is earned in 6 years. 11. Solution (3)
This means Let Ram took Rs. a and Shyam took Rs. b
SI = P in 6 years [then only Amount = P + P = 2P] Let the rate of interest was r% per annum
Now, SI should be equal to 5P such that amount They lent Rs. (a + b) to Mohan at the rate of (r +
becomes 6P 2)% per annum
So, SI is P in 6 years, it would be 5P in 6 × 5 = 30 For Ram,
years The total interest received on Rs. a = a × 4 ×r +
6. Solution (4) 2/100
Let the amount lent to Shahid =x, then The total interest paid by Ram to the bank = a × r
[ x + (x × 10 × 5) / 100 ] + [ (10500 – x) + (10500 ×4/100
– x)15 × 5 ) / 100 ] For Shyam,
= Rs. 17,375 The total interest received on Rs. b = b × 4 ×r+
(3 / 2)x + 18375 – (7 / 4)x = 17375 2/100
-(1 / 4)x + 18375 = 17375 The total interest paid by Shyam to the bank = b ×
x = Rs. 4000 r ×4/100
7. Solution (2) From the question,
P + (P × (8 / 100) × 4) = 5280 8/100a–8b/100= 400
(33 / 25)P = 5280 a – b = 50 × 100 = Rs. 5000
P = Rs. 4000 12. Solution (5)
Amount payable after 3 years Amount deposited = Rs. 1000
= 4000 + [ 4000 × (8 / 100) × 3] = Rs. 4960 Here we can use the formula of Arithmetic
8. Solution (4) Progression,
नींव Booklet
For All Banking & Insurance Exam
Ch-8 | Simple Interest | Maths by Arun sir
n/2{2a + (n – 1) d} S.I. = S.I for 1 & 2 year + S.I. for 3,4&5 year +S.I.
→ 10 × [ 9/2{2 × 10 + (9 – 1) × 3} ] for 6th year
→ 45 × 44 = 1980 ⇒ P + P ( 2 × 3 + 3 × 8 + 1 × 10 )/100 = 5320
Interest earned = Rs. 1980 ⇒ P = Rs. 3800
13. Solution (5)
Let the total investment in scheme B be Rs.x, then 19. Solution (1)
(49000 – X) ×5/100+ X ×12/100= 4900 5% for first three years,
X =5/7× 49000 = Rs. 35000 3 × 5% = 15%
14. Solution (5) 10% for next five years,
Let the amount of money Arjun had initially 5 × 10% = 50%
= Rs. x Time = 12 years,
Amount distributed between A and B = Rs. 0.65x 12% for the next 4 years,
Amount received by A = 0.65x ×6/13= Rs.0.3 x 4 × 12% = 48%
Amount received by B = 0.65x ×7/13= Rs.0.35 x Total rate interest = (15 + 50 + 48)% = 113%
Interest earned by B = 0.35x × 0.08 × 5 113% = 6780
= Rs.0.14 x 1% = 6780/113 = 60
Interest earned by A = 0.3x × 0.07 × 5 100% = 6000.
= Rs.0.105 x Principal = Rs. 6000
According to the question, 20. Solution (2)
0.14x – 0.105x = 1890 According to the question,
0.035x = 1890 2x × 15 × 20/100–x × 10 × 10/100 = 6x – x =
x = 54000 5x = 2500
So, the amount left with Arjun = 0.35 × 54000 = x = Rs. 500
Rs. 18900 21. Solution (3)
15. Solution (5) Principal = 2,00,000
According to the question, Interest per year = 5% of 2,00,000 = 10,000
8250 × (x + 4) % × 4 – 8250 × 8% × 4 = 1320 Interest after five years = 50,000
330 (x + 4) – 2640 = 1320 Now this amount is added in principal
330 (x + 4) = 3960 New Principal = 2,50,000
x + 4 = 12 ; x = 8 Interest = 3,50,000 – 2,50,000 = 1,00,000
So, the value of x = 8% Rate is 5%
16. Solution (1) Per year Interest = 12,500
Interest = A – P = 3840 Years =100000/12500= 8 years
Now, I =P × R × N/100 22. Solution (3)
3840 =12000 × 8 × x/100 Let, the total monthly income be Rs. ‘x’
x=4 Amount spent on rent = Rs. 0.2x
Interest = 18600 – 15000 = 3600 Remaining monthly income = Rs. 0.8x
3600 = 15000 × 12 × y/100 Amount spent on food = 0.25 × 0.8x = 0.2x
y=2 Amount deposited on savings account = Rs. 0.48x
x+y=4+2=6 So, 0.2x + 0.2x + a + 0.48x = x
17. Solution (2) a = 0.12x
According to the question, a = 12% of x
(x% of 60000) +(x% of 60000) × 13 × 3/100= 8294.4 =0.48x × 7.2 × 5/100= 829440 = 17.28x
29190 x = Rs. 48000
600x + 234x = 29190 So, a = 12% of x = Rs. 5760
834x = 29190 23. Solution (4)
x = 35 Let the sum of money = x
18. Solution (1) SI =P × R × T/100
Let principal be Rs P. x/2=x × t × 10
by solving, t = 5 years
नींव Booklet
For All Banking & Insurance Exam
Ch-8 | Simple Interest | Maths by Arun sir
When R = 10 + 3 = 13% then SI = x + 450 =x × 5 29. Solution (2)
× 13/100 Total interest earned in 6 years at the rate 13% =
By solving, x = 3000 13 × 6 = 78% of P
24. Solution (1) Total interest earned in 3 years at the rate 10% =
Let the amount be A and rate of interest be = 2 k 10 × 3 = 30% of P
% p.a According to the question,
A ×2k/100× 1 = 3120 ---eq (i) 78% of P – 30% of P = 3168
When invested at 4% more, rate of interest = (2k + 48% of P = 3168
4) % ⇒ P =3168 × 100/48= 6600/-
Interest = P × R × T = A × (2k + 4) ×3/2= A × (k + 30. Solution (2)
2) × 3 = 6240 ---eq (ii) Given that,
By doing eq(ii) /eq(i) (x% of 60000) +(x% of 60000) /100× 20 × 4
Solving this gives us k = 6 = 32400
The rate of interest = 12% and interest = 3120 = 600x +(600x) × 80/100= 32400
So, Amount =3120 × 100/12= Rs.26000 = 600x ( 1 +80/100 ) = 32400
25. Solution (2) = 600x {1.8} = 32400
Let the amount lent at 7% be X and hat lent at 12% = x = 30
be (1, 60,000 – X) 31. Solution (2)
But the interest at the end of one year = Rs 14400, Interest for first 5 years
which is 9% of 1, 60,000 = 25600*5*5/100
Using the allegation method, = Rs 6400
Ratio of the two amounts comes out to be 3 : 2 Amount to be received after 5 years
X/1,60, 000 – X= 3 : 2 = 25600+6400
X =3/5× 1, 60,000 = 96000 = Rs 32000
So the other amount = Rs 64,000 Interest for next 3 years
26. Solution (4) = 32000*3*5/100
Let Rajat lend the sum at x% rate to the second = Rs 4800
friend. 32. Solution (2)
According to the question, Let he borrow Rs. D
15% × 10000 + x% of 10000 = 20% of 20000 A.T.Q,
⇒ 15 × 100 + 100x = 20 × 200 𝐷×5×2 𝐷×10×4 𝐷×12×4
+ + = 10584
100 100 100
⇒ 100x = 4000 – 1500 98𝐷
⇒ 100x = 2500 ⇒ x = 25 100
= 10584
Hence, the required rate of interest is 25% p.a. D = Rs.10800
27. Solution (1) Therefore, he borrowed Rs.10800.
SI at the rate of 4% for 2 years =P × 4 × 2/100=8P/ 33. Solution (3)
100 Let the money lent to U be Rs.x
SI at the rate of 6% for 4 years =P × 6 × Therefore ,
4/100=24P/ 100 x*(18/100)*2 – 6000*(14/100)*2 = 650
SI for the next 3 years =P × 8 × 3/100=24P/ 100 => x= 6472.22
Total SI =8P/100+24P/100+24P/100 35. Solution (5)
⇒ P =1680 × 100/56= Rs. 3000 SI at the rate of 4% for 2 years ,
28. Solution (2) =( P* 4*2)/100 = 8P/100
According to the question: SI at the rate of 6% for 4 years ,
(P × r × r)/100=16/25× P [r = t] (P*6*4)/100 = 24P/100
⇒r2/100=16/25 SI for the next 3 years
⇒ r2 = 64 SI = (P*8*3)/100 = 24P/100
⇒ r = 8% Total SI = 8P/100 + 24P/100 + 24P/100
⇒ t = 8 years. =>P = (3360*100 )/56 = 6000
Now Req(4) S.I. = 25000 × 8 × 12/100= Rs.24000 35. Solution (5)
नींव Booklet
For All Banking & Insurance Exam
Ch-8 | Simple Interest | Maths by Arun sir
Let the sum be Rs. P According to the question,
𝑃×(6×3+5×8+5×4)
SI for the period of one year = 100
=
78𝑃
= 7020
1200
1200
Sum = 78 × 7020 = Rs. 108000
36. Solution (3) So, the value of x = 8%
Let the sum be Rs. 100. Then, 39. Solution (4)
S.I. for first 6 months = 100*10*1/100*2 =Rs5 We know that, formula to calculate simple
S.I. for last 6 months = 105*10*1/100* 2 interest:
=Rs5.25 SI = (p x r x t)/100
So, amount at the end of 1 year = Rs. (100 + 5 + (1600 × 2 × R) ÷ 100 = 1408
5.25) = Rs. 110.25 3200 R = 140800
Effective rate = (110.25 – 100) = 10.25% R = 44%
37. Solution (2) New rate = R – 16
Aman invested = x = 44 – 16 = 28%
Bhuvi invested = 4740 – x Interest = 1600 × 2 × 0.28
(4740 – x) 12/100 * 4 – x * 12/100 * 4 = 67.20 = Rs.896
48/100 ( 4740 – x – x) = 6720 40. Solution (1)
12/25 (4740 – 2x) = 67.20 (x + 3400) * 15 /100 * 2 + (x + 2200) * 20/100 * 2
4740 – 2x = 67.20 * 25/12 =7220
4740 – 2x = 140 (x + 3400) * 3/10 + (x+2200)*2/5 = 7220
2x = 4600 (3x + 102000/10) + (2x + 4400/5) = 7220
x = 2300 (3x + 10200 + 4x + 8800/10) = 7220
amount invests by Aman= 2300 7x + 19000 = 72200
38. Solution (5) x = 7600
1. The compound interest on a certain sum at 20%
per annum for 2 years is Rs. 196 more than the 6. Nutan borrows money at 4% p.a and lends it at 6%
simple interest on same sum at 15% per annum for per annum compound interest compounded half
2 years. What is the sum? yearly and thus gains Rs 418 in a year. What is the
(1) 1400 (2) 960 amount of money she borrows?
(3) 640 (4) 2000 (1) Rs 12000 (2) Rs 20000
(5) 1300 (3) Rs 24000 (4) Rs 30000
(5) None of these
2. A man with a sum of Rs 15612 wants to deposit in
the bank account of his two sons so that both will 7. If the difference between compound interest and
get equal money after 5yrs and 7yrs respectively at simple interest at 20% rate of interest ‘is 480, then
the rate of 4% compounded annually. Find the part find the principal amount.
of amount deposited into the account of first son? (1) 1800 (2) 1875
(1) 8112 (2) 9600 (3) 2000 (4) 2375
(3) 12000 (4) 5000 (5) None of these
(5) None of these
8. If compound Interest on certain sum for 2 years is
3. The difference between simple interest and 704 at some rate of interest and Simple Interest on
compound interest on a certain sum at 20% rate of same rate for 3 years is 960, then find the sum.
interest for 2 years is 1000. What is difference (1) Rs 1600 (2) Rs 800
between simple interest and compound interest for (3) Rs 1400 (4) Rs 1800
1st year, if interest is compounded half-yearly? (5) None of these
(1) 120 (2) 180
(3) 125 (4) 150 9. Difference between compound interest and simple
(5) 190 interest on sum of Rs. 14400 at certain rate of
interest after 2 years is Rs 32(4) What is the rate of
4. Divide Rs 20,816 between Pawan and Kedar so interest?
that Pawan’s share at the end of 7 years is equal to (1) 14% (2) 15%
Kedar’s share at the end of 9 years with compound (3) 17% (4) 12.5%
interest being 4% per annum. (5) 24%
(1) 10716, 10100 (2) 10616, 10200
(3) 10816, 10000 (4) 10800, 10016 10. Two friends Amar and Bhuvan invested same
(5) None of these amount at two different schemes for two years.
Amar invested his amount at 8% rate of compound
5. Rs 12,200 was partly invested in scheme X at 10% interest and Bhuvan invested his amount at 6%
p.a compound interest (compounded annually) for rate of simple interest. Sum of interest received by
2 year and partly in scheme Y at 10% p.a. simple them is Rs.358. Find amount invested by each of
interest for 4 years. Both the schemes give equal
them.
interests. How much was invested in scheme X?
(1) Rs 7000 (2) Rs 8000 (1) 1250 (2) 1400
(3) Rs 9000 (4) Rs 8500 (3) 1350 (4) 1200
(5) Rs 10000 (5) 1450
11. The sum of the amounts invested by Samar and then what will be possible value of (X) & (R)?
Sumit is Rs. 8500. If Samar invested his sum at (1) 2400 & 12.5 (2) 1400 & 10
30% p.a. simple interest for 4 years and Sumit (3) 4400 & 15 (4) 4400 & 12.5
invested his sum at 30% compound interest, (5) 8000 & 7.5
compounded annually for 2 years, and the sum of
the interests received by them is Rs. 8160, then 16. Akhil invested a total of Rs. ‘x’ in two schemes A
find the sum invested by Sumit. and B. Amount invested in scheme A is Rs. 750
(1) Rs. 4000 (2) Rs. 3500 more than the amount invested in scheme B.
(3) Rs. 3000 (4) Rs. 2500 Scheme A provided compound interest of 20% p.a.
(5) Rs. 4500 compounded annually, while scheme B provided
12. Rs. (x + 1500) is invested in scheme ‘A’ offering simple interest of 16% per annum. If the total
24% p.a. simple interest, for 5 years. When 75% of interest received by Akhil at the end of 2 years is
the interest received from scheme ‘A’ is invested Rs. 6220 then find the value of ‘25% of 2x’.
in scheme ‘B’ offering 20% p.a. compound (1) Rs. 8000 (2) Rs. 7750
interest, compounded annually, the amount (3) Rs. 7250 (4) Rs. 8125
received is Rs. 10368. Find the value of ‘x’. (5) None of these
(1) 5500 (2) 7200
(3) 5000 (4) 6500 17. Nachiket borrows Rs 4000 at 10% compounded
(5)- 6000 rate of interest. At the end of each year he pays
back Rs 1100. How much amount should he pay
13. Interest earned by Amit on Rs 10000 for 2 years at at the end of the third year to clear all his dues?
R% p.a. simple interest is how much more/less (1) Rs 5500 (2) Rs 5466
than interest earned by him on Rs 8000 for 2 years (3) Rs 5666 (4) Rs 5566
at (R+10)% p.a. compounding annually. If Rs (5) None of these
2000 is invested at 2R% p.a. simple interest for a
year, then it would fetch an interest of Rs 400. 18. Angad borrowed Rs.800 at rate of interest 10%.
(1) Rs 1220 (2) Rs 1000 He repaid Rs.400 at the end of first year. What is
(3) Rs 1520 (4) Rs 930 the amount required to repay at the end of second
(5) Rs 1140 year to discharge his loan which was calculated at
compound interest?
14. Dinesh invested Rs. P for 2 years in scheme – A at (1) Rs.650 (2) Rs.528
the rate of 18% p.a. on CI compounding annually (3) Rs.490 (4) Rs.780
and received a total amount of Rs.83544 from (5) Rs.472
scheme – A. He then invested Rs.(P+X) in another
scheme – B for 2 years at the rate 20% p.a. on SI. 19. Naman invested a certain amount at the rate of
If he received Rs.30000 as interest from scheme – 14% for 4 years in simple interest and after 4 years
B, then find X? he obtained a total simple interest of Rs.8400. If he
(1) Rs 24000 (2) Rs 28000 invested the same amount at the rate 6% more than
(3) Rs 12000 (4) Rs 22000 that of previous interest for 2 years in compound
(5) Rs 15000 interest, then how much interest would he has
obtained?
15. Aman invested Rs. (X) at the rate of (R)% p.a. (1) 6440 (2) 6600
on simple interest for two years and Rs. (X – (3) 5600 (4) 3300
400) at the rate of (R + 7.5)% on compound (5) 5500
interest for one year. If ratio of total simple interest
to total compound interest gets by man is 11 : 8,
20. The difference between simple interest and (1) Rs. 5720 (2) Rs. 5290
compound interest on a certain sum at 20% rate of (3) Rs. 4850 (4) Rs. 5470
interest for 2 years is 1000. What is difference (5) Rs. 4580
between simple interest and compound interest for
1st year, if interest is compounded half-yearly? 25. Rs. 12500 is invested at 40% p.a. simple interest
(1) 120 (2) 180 for 3 years. The interest received is then invested
(3) 125 (4) 150 at 20% p.a. compound interest, compounded
(5) 190 annually for 2 years. Find the interest received
from the sum invested at compound interest.
21. The compound interest on a certain sum for 2 (1) Rs. 2800 (2) Rs. 3000
years is Rs. 756 and Simple interest is Rs. 720. If (3) Rs. 4200 (4) Rs. 4000
the sum is invested such that the SI is Rs. 900 and (5) Rs. 2500
the number of years is equal to the rate percent per
annum, find the rate percent? 26. Ramesh deposited an amount of Rs. 12500 at 10%
(1) 10% (2) 12% per annum compound interest while amount of Rs.
(3) 5% (4) 15% 16900 at 7% per annum simple interest at the same
(5) None of these time for 2 years. What will be the difference in the
total interest earned in the 1st year and the total
22. Equal sums of money are deposited in two interest earned in the 2nd year?
different banks by ABC Enterprises, one at (1) Rs. 120 (2) Rs. 140
compound interest, compounded annually and the (3) Rs. 125 (4) Rs. 160
other at simple interest, both at 5% per annum. If (5) Rs. 155
after two years, the difference in the amounts
comes to Rs. 200, what are the amounts deposited 27. The simple interest on a certain sum at 12% per
with each bank? annum for 3 years is Rs. 4,140. What is the
(1) Rs. 75,000 (2) Rs. 80,000 compound interest on the same sum at 8% for two
(3) Rs. 82,000 (4) Rs. 85,000 years?
(5) None of these (1) 1820.40 (2) 1913.60
(3) 2012.60 (4) 1900
23. A certain sum is invested in scheme ‘P’ at 25% (5) None of these
p.a. for 5 years at simple interest. The same sum is
also invested in scheme ‘Q’ at 20% p.a. compound 28. A certain amount of money is lent out at
interest compounded annually for 2 years. If the compound interest at the rate of 10% per
sum of the amounts received from the two annum for two years, compounded annually. It
schemes is Rs. 9225, then find the interest received would give Rs. 881 more if the amount is
from scheme ‘Q’. compounded half yearly. Find the principle.
(1) Rs. 900 (2) Rs. 2100 (1) 120000 (2) 150000
(3) Rs. 1500 (4) Rs. 1100 (3) 160000 (4) 132000
(5) Rs. 1800 (5) None of these

24. Rs. 3500 when invested at y% p.a. simple interest 29. Compound interest on a sum for 2 yrs at 4% per
for 4 years amounts to Rs. 5600. Find the amount annum is Rs. 10(2) The SI on the same sum for the
received at compound interest, compounded same period at the same rate will be
annually when Rs. 500 more is invested than the (1) 90 (2) 80
amount invested at simple interest, for 2 years at (3) 70 (4) 60
y% p.a. (5) None of these
30. Anuj has deposited certain amount in the bank to 33. Quantity I: Find the amount on compound interest
earn compound interest at 10% per annum. The on a sum of Rs.55000 at the rate of 15% per annum
difference in the interest on the amount between after three years.
the 3rd and 2nd years is Rs 1100. What amount has Quantity II: Find the simple interest on a sum of
Anuj deposited? Rs.60000 at the rate 25% per annum after 5.5 years.
(1) 10000 (2) 110000 (1) Quantity I > Quantity II
(3) 105000 (4) 11000 (2) Quantity I < Quantity II
(5) 100000 (3) Quantity I ≥ Quantity II
(4) Quantity I ≤ Quantity II
31 Quantity I: Two equal amounts are invested for 2 (5) Quantity I = Quantity II or Relation cannot be
years at 9% per annum by Viraj, one at simple Established.
interest and the other at compound interest. If the
difference in the interests for the two years on the 34. Quantity I: A sum of money under simple interest
two amounts is 100, then what is the amount? becomes 5 times of itself in x years but it becomes
Quantity II: Two equal amounts are invested for 2 9 times of itself in x2/2 years. What is the value of
years at 11% per annum by Viraj, one at simple x?
interest and the other at compound interest. If the Quantity II: 3.5 years
difference in the interests for the two years on the (1) Quantity I > Quantity II
two amounts is 97, then what is the amount? (2) Quantity I < Quantity II
(1) Quantity I > Quantity II (3) Quantity I ≥ Quantity II
(2) Quantity I < Quantity II (4) Quantity I ≤ Quantity II
(3) Quantity I ≥ Quantity II (5) Quantity I = Quantity II or Relation cannot be
(4) Quantity I ≤ Quantity II Established.
(5) Quantity I = Quantity II or Relation cannot be
Established. 35. The compound interest received on Rs. 5000 at x%
per annum at the end of 2 years is equal to the
32 Quantity I: Rs.12700 was lent in two parts by simple interest received in four years on Rs. 2650 at
Swati. One part was lent at 4.5% simple interest per the same rate of interest.
annum and the rest was lent at 11% simple interest Quantity I: What is the total simple interest
p.a. The total interest received from both the parts received?
is Rs.1150 per year. What was the amount lent at Quantity II: Rs. 1275
11%p.a? (1) Quantity I > Quantity II
Quantity II: Rs.12000 was lent in two parts by (2) Quantity I < Quantity II
Swati. One part was lent at 5% simple interest per (3) Quantity I ≥ Quantity II
annum and the rest was lent at 10% simple interest (4) Quantity I ≤ Quantity II
p.a. The total interest received from both the parts (5) Quantity I = Quantity II or Relation cannot be
is Rs.1200 per year. What was the amount lent at Established.
10%p.a?
(1) Quantity I > Quantity II 36 Quantity I: Find the interest earned after 3 years, if
(2) Quantity I < Quantity II a person invests Rs. 52000 at C.I. at the rate of 10%
(3) Quantity I ≥ Quantity II per annum.
(4) Quantity I ≤ Quantity II Quantity II: Find the interest earned after 3 years,
(5) Quantity I = Quantity II or Relation cannot be if a person invests Rs. 28750 at S.I. at the rate of
Established. 20% per annum.
(1) Quantity I > Quantity II
(2) Quantity I < Quantity II
(3) Quantity I ≥ Quantity II 39. Rs. ‘A’ is invested in a scheme which offers 10%
(4) Quantity I ≤ Quantity II compound interest, compounded annually and the
(5) Quantity I = Quantity II or Relation cannot be interest earned after two years is Rs. 262(5)
Established. Quantity I: Find the rate of interest if Rs. 2A is
invested in a scheme at simple interest and interest
37. Quantity I: A sum of money Rs. 12000 is invested earned after two years is Rs. 8000.
into compound interest at 5% per annum for 2 years, Quantity II: Find the rate of interest if Rs. (A +
then what will be amount earned at the end of two 7500) is invested in a scheme which offers
years? compound interest compounded annually, and
Quantity II: A sum of money Rs. 15000 is invested interest received after 2 years is Rs. 6450.
into simple interest at 8% per annum for two years, (1) Quantity I > Quantity II
then what will be the amount earned at the end of (2) Quantity I < Quantity II
two years? (3) Quantity I ≥ Quantity II
(1) Quantity I > Quantity II (4) Quantity I ≤ Quantity II
(2) Quantity I < Quantity II (5) Quantity I = Quantity II or Relation cannot be
(3) Quantity I ≥ Quantity II Established.
(4) Quantity I ≤ Quantity II
(5) Quantity I = Quantity II or Relation cannot be 40. Quantity I: Sita invested Rs. 32000 on compound
Established. interest at 20% per annum for two years. What will
be interest earned after two years?
38. Quantity I: The difference between the Compound Quantity II: Ritika invested Rs. 39000 on simple
interest and simple interest earned on equal sums of interest at 18% per annum for two years. What will
money over 2 years is Rs 480, what is the Principal be the interest earned after two years?
if the rate of interest is 20% (1) Quantity I > Quantity II
Quantity II: The difference between the (2) Quantity I < Quantity II
Compound interest and simple interest earned on (3) Quantity I ≥ Quantity II
equal sums of money over 3 years is Rs 3250, what (4) Quantity I ≤ Quantity II
is the Principal if the rate of interest is 25% (5) Quantity I = Quantity II or Relation cannot be
(1) Quantity I > Quantity II Established.
(2) Quantity I < Quantity II
(3) Quantity I ≥ Quantity II
(4) Quantity I ≤ Quantity II
(5) Quantity I = Quantity II or Relation cannot be
Established.
Answer Key
1. (1) 9. (2) 17. (4) 25. (2) 33. (1)
2. (1) 10. (1) 18. (2) 26. (2) 34. (1)
3. (3) 11. (1) 19. (2) 27. (3) 35. (2)
4. (4) 12. (1) 20. (3) 28. (2) 36. (2)
5. (2) 13. (1) 21. (3) 29. (3) 37. (3)
6. (2) 14. (5) 22. (2) 30. (5) 38. (2)
7. (2) 15. (4) 23. (4) 31. (1) 39. (1)
8. (1) 16. (5) 24. (1) 32. (2) 40. (3)
Hints & Solutions
1. Solution (1) 7. Solution (2)
Let sum is Rs. x 20% =1/5……….(5)T………=(5)³= 125 =principal
compound interest for 2 years at 20 % p.a In 3yrs difference will always come
20 + 20 + 20 * 20 /100 = 44% 3A + 1 = (3*5)+1 =16
simple interest for 2 years at 15 % = 15% * 2 = 16 = 240
30% 1 = 15
x * 44/100 – x * 30/100 = 196 125 = 1875
x = 1400 8. Solution (1)
2. Solution (1) SI for 1 years= 960/3 =320 (as SI is same for every
A( 1+4/100)5 = B(1+4/100)7 year)
A/B = (1+4/100)² = 676/625 SI for 2 years= 640
676+625 = 15612 CI for 2 year=704; diff=64
1= 12 64=20% of 320
676 = 8112 hence r=20%
3. Solution (3) 20%=320
1000 = PR2/1002 100%=1600
1000 = P * 400/10000 9. Solution (2)
P = 25000 CI – SI = PR2/1002
simple interest for 1st year = 25000 * 20/100 = 324 = 14400R2/10000
5000 R^2 = 324 * 10000/14400
Rate of interest for compound half yearly P^2 = 225
= 20/2 = 10% R = 15%
by apply successive method, 10. Solution (1)
rate of CI = 10 + 10 + 10 * 10/100 = 21% Let the amount invested by Amar = the amount
compound interest = 25000 * 21/100 = 5250 invested by Bhuvan = Rs. P
Difference = 5250 – 5000 = 250 Then, interest received by Amar = P * ((1 +
4. Solution (4) 8/100)2 - 1) = 104P/625
second part + (4+4 +16/100) of second part = first And interest received by Bhuvan = P * 6 * 2/100 =
part 3P/25
second part + 8.16% of second part= first part Now, (104P/625) + (3P/25) = 358
first part/second part= 108.16/100 = 10816/10000 P = 1250
5. Solution (2) 11. Solution (1)
Let it be Rs x. in scheme X Let the sum invested by Sumit be Rs. x
So in Y=(12200-x) Therefore, sum invested by Samar = Rs. (8500 –
Therefore, [(1+10/100)2 -1] = [(12200-x) *10*4] x)
/100 According to the question,
x=Rs 8000 {(8500 – x) × 30 × 4}/100 + x(1 + 30/100)2 – x
6. Solution (2) = 8160
Let amount borrowed be Rs 100. Or, 1.2 × (8500 – x) + 0.69x = 8160
Interest to be paid = Rs 4 Or, 0.51x = 2040
Gain= 6.09-4 = Rs 2.09 Or, x = 4000
Actual amount borrowed= 418/2.09 * 100 = 20000 Therefore, sum invested by Sumit = Rs. 4000
12. Solution (1) y = Rs. 8500
Interest received from scheme ‘A’ = {(x + 1500) × x = y + y – 750 = 2y – 750 = Rs. 16250
24 × 5}/100 = Rs. (1.2x + 1800) 17. Solution (4)
According to the question, first year = 4000+400=4400-1100=3300
0.75(1.2x + 1800)(1 + 20/100)2 = 10368 second year = 3300+330 = 3630-1100=2530
Or, 0.9x + 1350 = 7200 third year=2530 + 253 = 2783
Or, x = 5850/0.9 = 6500 18. Solution (2)
13. Solution (1) Amount paid at the end of 1 year
ATQ = 800[1 + 10/100] = 880
400 =
2000×2𝑅𝑅×1 Amount left as principal for the second year
100
= 880-400 = 480
R = 10%
10000×10×2
Amount to be paid after 2nd year
Required difference = ~ 8000 ��1 + = 480 [1 + 10/100] = Rs.528
100
20 2 19. Solution (2)
� − 1�
100 Let principal be Rs. x
= 2000~3520 = Rs. 1520 (compound interest is = x * 14/100 * 4 = 8400
mor(5) x = 8400 * 100/56 = 15000
14. Solution (5) by applying successive, rate of compound interest
ATQ = 20 + 20 + 20 * 20/100 = 44%
18 2 compound interest = 15000 * 44/100 = 6600
𝑃𝑃 �1 + � = 83544
100 20. Solution (3)
𝑃𝑃 = Rs. 60000 1000 = PR^2/100^2
And, 1000 = P * 400/10000
(𝑃𝑃×𝑋𝑋)×20×2
= 30000 …(i) P = 25000
100
Put value of P in (i) simple interest for 1st year
(60000+𝑋𝑋)×20×2 = 25000 * 20/100 = 5000
= 30000
100 Rate of interest for compound half yearly
60000 + 𝑋𝑋 = 75000 = 20/2 = 10%
𝑋𝑋 = Rs. 15000 by apply successive method,
15. Solution (4) rate of CI = 10 + 10 + 10 * 10/100 = 21%
2𝑅𝑅 2𝑋𝑋𝑋𝑋
Total simple interest gets by man = 𝑋𝑋 × = compound interest = 25000 * 21/100 = 5250
100 100
And, the total compound interest gets by man = Difference = 5250 – 5000 = 250
(𝑋𝑋 − 400) ×
(𝑅𝑅+7.5) 21. Solution (3)
100 (CI − SI)for 2yrs = 36
ATQ-
2𝑋𝑋𝑋𝑋
It means SI on Ist year sum = Rs 36
11 36×100
100
(𝑅𝑅+7.𝑥𝑥) =
8
∴R= = 10%
(𝑋𝑋−400)× 360
100
2𝑋𝑋𝑋𝑋 11 360 × 100
= ∴P= = 3600
(𝑋𝑋−400)×(𝑅𝑅+7.5) 8 10 × 1
16. Solution (5) According to question
Amount invested in scheme A = Rs. y 3600ࢧ × R × R
Amount invested in scheme B = Rs. (y – 750) = 900
100
According to question, R = 5%
y × [(1.44 – 1)] + (y – 750) × 0.16 × 2 = 6220
0.44y + 0.32 × (y – 750) = 6220
0.44y + 0.32y – 240 = 6220
0.76y = 6460
22. Solution (2) Compound interest earned on it for 2 years at 8%
Difference in Amounts = Rs 200 for two yrs = 11500(1 + 8/100)2 – 11500
Or,  CI = 11500(1.08)2 – 11500
(CI − SI)2yrs = Rs 200 CI = Rs. 1,913.60
PR2 28. Solution (3)
⇒ = 200 Let the principle be P.
1002
⇒ P = Rs 80000 Interest on this amount when compounded
23. Solution (4) annually at the rate of 10% per annum
Let the sum invested be Rs. x = P [(1.1) 2 − 1]
Ratio of the amounts received from scheme ‘P’ Interest on this amount when compounded half
and ‘Q’ yearly = P [(1.05) 4 − 1]
= {(x × 25 × 5)/100 + x}:x(1 + 20/100)2 = 25:16 So ATQ,
Therefore, amount received from scheme ‘P’ = P [(1.05) 4 − 1] – P [(1.1) 2 −1] = 881
9225 × 25/41 = Rs. 5625 ∴ P [1.05 4 – 1.1 2 ] = P[1.1025 2 – 1.1 2 ]
Sum invested in scheme ‘P’ = P(0.0025×2.2025) = 881 ⇒ P = 160000
= 5625 × 4/9 = Rs. 2500 29. Solution (5)
Interest received from scheme ‘Q’ CI = 102, t = 2yrs and r = 4%
= 1.44x – x = 0.44x = Rs. 1100 CI = A – P
24. Solution (1) ⇒ A = P + 102
According to the question, ∴P+102= P(1+4100)2
(3500 ×y × 4)/100 + 3500 = 5600 ⇒ 102 = 1.0816P – P
Or, 140y = 2100 ⇒ 102 = 0.0816 × P
Or, y = 15% ⇒ P = 1250
Therefore, sum invested at compound interest Given,
= (3500 + 500) = Rs. 4000 P =1250, t = 2 years, r = 4%
Amount received at compound interest SI = (1250 × 2 × 4) / 100
= 4000(1 + 15/100)2 = Rs. 5290 ⇒ SI = 100
25. Solution (2) 30. Solution (5)
Interest received from the simple interest = (12500 Let the principal be Rs. 1000
× 40 × 3)/100 = Rs. 15000 Therefore,
Interest received from the compound interest 10
Interest for 1st year = 1000× = Rs.1100
= 15000(1 + 20/100) – 15000 = Rs. 3000 100
26. Solution (3) Therefore,
Total interest earned in 1st year = {(12500 × 10 × 10
1)/100} + {(16900 × 7 × 1)/100} = 1250 + 1183
Interest for 2 year = 1100× 100 = Rs. 110
nd
= Rs. 2433
Principal for 3rd year = 1100 + 110 = Rs. 1210
Total interest earned in 2nd year = [12500 ×
Therefore,
{(1.1)2 – 1} + (16900 × 7 × 2)/100] – 2433
10
= (2625 + 2366) – 2433 = Rs. 2558
Required difference = 2558 – 2433 = Rs. 125 Interest for 3rd year = 1210× 100 = 121
And given difference in the interest between 3rd &
27. Solution (2)
2nd year is Rs. 1100
Let the amount invested be x,
∴ (121 -110) units = 1100
Simple interest earned on it
11 unit →1100
= (x × 12 × 3) /100 = 4140
1 uit = 100
 x = (4140 × 100) / (12 × 3)
∴ Principal = 100 × 1000 = Rs. 100000
 x = 11500
31. Solution (1) Quantity I > Quantity II
Quantity I: The S.I. on amount x at the rate y (in % 34. Solution (1)
i.e. y/100) per annum for 2 years in 2 × x × y …....... Quantity I: Let the sum of money was P and rate
(i) of interest is R then we know that
And C.I. for 2 year is x × y × (2 + y) ..........…. (ii) SI=P*R*T/100
According to question, Difference = 100 ⇒ x × y × SI in x years = 5P – P = 4P
(2 + y) – 2 × x × y = 100 SI in x2/2 years = 9P – P = 8P
x = 12345.6 Hence amount = Rs. 12345.6 x = 4 years
Quantity II: The S.I. on amount x at the rate y (in Quantity II : 3.5 years
% i.e. y/100) per annum for 2 years in 2×x×y…. (i) Quantity I > Quantity II
And C.I. for 2 year is x × y × (2 + y) …. (ii) 35. Solution (2)
According to question, Difference = 97 Let the rate of interest = r% per annum then
× y × (2 + y) – 2 × x × y = 97 5000 (1 + 0.01r)2 – 5000 = 106r
⇒ x × y2 = 97 5000(1 + 0.0001r2 + 0.02r) – 5000 = 106r
x = 8016.5 Hence amount = Rs. 8016.5 5000 + 0.5r2 + 100r – 5000 = 106r
Quantity I > Quantity II 0.5r2 – 6r = 0
32. Solution (2) 0.5r = 6
Quantity I: Let the amount lent at 11% per annum r = 12
be x The amount lent at 4.5% per annum be (12700 SI= 2650*12*4/100=106*12=1272
– x) Quantity I = Rs. 1272
According to the question, Quantity II = Rs. 1275
⇒ (12700 × 4.5) – 4.5 × x + 11 × x = 1150 × 100 Quantity I < Quantity II
⇒ 51498.5 + 7.17 × x = 115000 36. Solution (2)
⇒ 7.17 × x = 63501.5 Quantity I : Compound interest = 52000 × [(1.1)3
x = 8856.6 – 1) = Rs. 17212
Quantity II: QuantityII:Simple
Let the amount lent at 10.325% per annum be x The interest=28750*3*20/100=17250
amount lent at 4.252% per annum be (12000 – x) Quantity I < Quantity II
According to the question, 37. Solution (3)
⇒ (12000 × 4.252) – 4.252 × x + 10.325 × x = 1200 Quantity I:
× 100 P = Rs. 12000
⇒ 51024 + 6.073 × x = 120000 R = 5% per annum
⇒ 6.073 × x = 68976 And T = 2 years
⇒ x = 11357.8 So, amount = P(1 + R/100)T
Hence, the amount lent at 10.325% per annum be = 12000 * (1 + 5/100)2 = Rs. 13230
Rs. 11357.8 Quantity II:
Quantity I < Quantity II P = Rs. 15000
33. Solution (1) R = 8% per annum
Quantity I: Find the amount on compound interest And, T = 2 years
on a sum of Rs.55000 at the rate of 15% per annum So, amount = P + (P * R * T)/100
after three years. = 15000 + (15000 * 8 * 2)/100 = Rs. 17400
Amount=55000*115/100*115/100*115/100 Quantity I < Quantity II
=83648 38. Solution (2)
Quantity II: Find the simple interest on a sum of Let the Principal be Rs 'P'
Rs.60000 at the rate 25% per annum after 5.5 years. Quantity I:
SI=60000*25*5.5/100=82500 Given, P(r/100)2 = 480
=> P(20/100)2 = 480 Quantity II:
=> P = 480*25 = 12,000 Principal = Rs. 20000
Quantity II: 26450 = 20000(1 + r/100)2
Given, P(r/100)2((300 + r)/100) = 3250 529/400 = (1 + r/100)2
=> P(25/100)2*(325/100) = 3250 23/20 = 1 + r/100
=> P = 3250*16*100/325 = 16,000 3/20 = r/100
39. Solution (1) r = 15%
A + 2625 = A(1 + 10/100)2 Quantity-I > Quantity-II.
A + 2625 = A × 121/100 40. Solution (3)
121A/100 – A = 2625 Quantity I:
21A/100 = 2625 Compound interest earned after 2 years = 32000 *
A = Rs. 12500 [(1 + 20/100)2 - 1] = 14080
Quantity I: Quantity II:
Principal = Rs. 25000 Simple Interest earned after 2 years = (39000 * 18
8000 = 25000 × r% × 2 * 2)/100 = 14040
r = 16% Quantity I > Quantity II
1 savings if the savings of A is one fourth of his
1. If 30% of A = 0.285 Of B = 5 of C, then find
income.
A: B:C
(1) 12 : 19 (2) 19 : 12
(1) 10 : 12 : 15 (2) 15 : 10 : 12
(3) 9 : 16 (4) 2 : 3
(3) 12 : 10 : 15 (4) 2 : 3 : 5
(5) 4 : 3
(5) None of these

7. The ratio of milk and water in 64 litres of a


2. If A, B, C, D are quantities of the same kind such
mixture is 5 : 3. What amount of water is added
that A : B = 3 : 4, B : C = 5 : 7 and C : D = 8 : 9,
to make the ratio 3 : 5 ?
Find A: B : C : D
(1) 42 (2) 42.33
(1) 15 : 20 : 28 : 63 (2) 40 : 53 : 56 : 30
(3) 42.66 (4) 14.66
(3) 30 : 46 : 56 : 63 (4) 40 : 56 : 63 : 61
(5) 20
(5) None of these

8. A bag contains one-rupee, 50 paise and 25 paise


3. A sum of Rs. 427 is to be divided among A, B and
coins in the ratio 5 : 6 : 8. If the total amount of
C is such a way that 3 times A’s share, 4 times
money in the bag is Rs. 210, find the number of
B’s share and 7 times C’s share are all equal. Find
coins of each kind.
the share of each.
(1) 126 (2) 168
(1) 196 (2) 147
(3) 184 (4) 164
(3) 84 (4) 168
(5) 200
(5) 200

9. The ratio of the number of students studying in


4. Divide Rs. 6450 among A, B, C and D such that
schools A, B and C is 6 : 8 : 7 respectively. If the
when A gets Rs. 9, B gets Rs. 8; when B gets Rs.
number of students studying in each of the
6, C gets Rs. 5 and when C gets Rs. 4, D gets Rs.
schools is increased by 20%, 15% and 20%
3
respectively, what will be the new ratio of the
(1) 1960 (2) 1125
number of students in schools A, B and C ?
(3) 1800 (4) 1680
(1) 18 : 19 : 6 (2) 13 : 14 : 19
(5) 2000
(3) 18 : 23 : 21 (4) 18 : 21 : 23
(5) None of these
5. A sum of Rs. 1290 is divided between A, B and
1
C such that A’s share is 1 times that of B and
2 10. The incomes of Sarita and Sewakram are in the
3
B’s share is 14 times that of C. what is C’s share ratio 1 : 2 and their expenditures are in the ratio
? 2: 5. If Sarita saves Rs 20,000 and Sewakram
(1) 196 (2) 140 saves Rs 35,000, what is the total income of Sarita
(3) 120 (4) 168 and Sewakram?
(5) 240 (1) Rs 90000 (2) Rs 95000
(3) Rs 100000 (4) Rs 85000
6. The ratio of incomes of A and B is 3 : 4. The ratio (5) None of these
of their expenditures is 4 : 5. Find the ratio of their
11. In the year 2021, in a college the ratio of faculties 16. The ratio of wine and cold drink in a mixture is
in three different sections Maths, Science and 4:9. If X litres of wine is added to the mixture,
Biology are 2:3:4 respectively. In the year 2022 then the ratio becomes 5:6 and then 8 litres of
in each section if ‘x’ number of faculties left then cold drink is added to the mixture, then the ratio
the ratio becomes 3:5:7. In the year 2021 in each becomes 9:14. Find the value of X?
section 5 faculties left then the ratio became (1) 17.5 (2) 10.5
1:2:3. Then what is the total number of faculties (3) 14 (4) 12.5
in the year 2022? (5) None of these
(1) 15 (2) 70
(3) 80 (4) 85 17. A shopkeeper deals in milk and 45 litre mixture
(5) None of these is to be distributed in Milk & Water in the ratio
of 4 : 1. If 4 litre milk & 3 litre water will be added
12. “x” kg of mixture contains rice and sugar in ratio in the mixture then what will be the new ratio of
4:3. If 13 kg of sugar is added then ratio becomes water and milk?
1:1. Then what is the final quantity of sugar in the (1) 5 : 6 (2) 3 : 10
mixture? (3) 4 : 5 (4) 7 : 8
(1) 48 (2) 40 (5) 10 : 3
(3) 52 (4) 42
(5) None of these 18. In a mixture of 40 liters, the ratio of milks and
water is (x + 1) : x. If 16 li water is added to this
13. In a 104 liter of mixture of milk and water the mixture, the ratio of milk and water becomes 3:4,
ratio of milk to water is 8 : 5. If ‘x’ liter of water then find the value of x.
is added in this mixture, so that the ratio of water (1) 2 (2) 4
to milk becomes 7 : 8. What is the value of ‘x’. (3) 1 (4) 3
(1) 21 liter (2) 16 liter (5) 8
(3) 14 liter (4) 15 liter
(5) 23 liter 19. A solution of ‘Lassi’ has 15% sugar. Another
solution has 5% sugar. How many liters’ of the
14. The ratio of Milk and Water in a mixture of 224 second solution must be added to the 20L of first
litres is 5:3. 12 litres of water is added to the solution to make a solution of 10 % sugar?
mixture, then find the ratio of milk and water in (1) 10 L (2) 5 L
final mixture? (3) 15 L (4) 20 L
(1) 19 : 6 (2) 14 : 19 (5) None of these
(3) 23 : 21 (4) 24 : 35
(5) 35 : 24 20. A jug contains 64 litres of pure milk.1/4of the
milk is replaced by the same amount of water.
15. A vessel contains a mixture of mango shake and Again,1/4 of the mixture is replaced by that
banana shake in a certain ratio. If Rashmi added amount of water. Find the ratio of milk and water
48 L banana shake, then the ratio becomes 5:3. in the new mixture.
Then further 48 L mango shake is added and the (1) 5 : 4
ratio becomes 9:5. Find the initial quantity of (2) 2 : 3
banana shake in the solution? (3) 3 : 4
(1) 368 L (2) 252 L (4) 5 : 6
(3) 288 L (4) 270 L (5) None of these
(5) None of these
21. L of alcohol is mixed with 75 L of water in (5) 5 : 7
vessel to form a mixture. Then, (p + 5) L of
mixture in vessel is replaced with 12.5 L of 26. In a 250 litres of mixture of two soft drinks, sprite
alcohol and 15 L of water such that the mixture in and Dew, Dew is x%. The person sold 50 litres of
vessel now contains 33(1/3)% of alcohol. Find mixture and replaced with same quantity of Dew.
the value of ‘p’. If percent of sprite in final mixture is 64%, then
(1) 15 (2) 30 what is the percentage of sprite in initial mixture?
(3) 25 (4) 20 (1) 80% (2) 75%
(5) None of these (3) 70% (4) 65%
(5) None of these
22. A jar full of whisky contains 40% alcohol. A part 27. A container contains 192 lites of mango shake. A
of this whisky is replaced by another containing seller draws out x % of mango shake and replaced
19% alcohol and now the percentage of alcohol it with same quantity of banana shake. He
was found to be 26%. The part of whisky replaced repeated the same process for three times. And
is: mango shake content in mixture in only 81 litres.
1 2 Then how much percent he withdrew every time?
(1) (2)
3 3
2 3 (1) 15% (2) 20%
(3) (4)
5 5 (3) 25% (4) 35%
4
(5) (5) None of these
5

23. 240 ml of mixture contains Lactic acid and 28. A container contains 30 l mixture of apple juice
oxalic acid in the ratio of 9:7, respectively. If ‘8x’ and orange juice in ratio p:q respectively. When
ml of mixture is taken out and 7.5 ml of Lactic 10 litres of mixture is taken out and replaced with
acid 7.5 ml of oxalic acid is added into the orange juice, then ratio becomes 2:3, then what is
remaining mixture, then ratio of Lactic acid to the initial quantity of apple juice in the container?
oxalic acid in the resultant mixture becomes 5:4. (1) 12 l (2) 20 l
Find the value of ‘x’. (3) 15 l (4) 18 l
(1) 15 (2) 25 (5) None of these
(3) 10 (4) 20
(5) None of these 29. The ratio of Milk and Water in a mixture of (x –
2) litres is 5:3. 18 litres of water and 12 litres of
24. Initially two cups of same volume are filled with milk is added to the mixture. If the ratio of milk
3 4 and water in the final mixture be 7 : 6, then find
milk upto th and th of their volumes. Water is
5 5 value of x.
then filled. Then two mixtures are mixed and (1) 40 (2) 50
poured into a jug. Find the ratio of water to milk (3) 60 (4) 30
in the mixture. (5) None of these
(1) 3 : 7 (2) 7 : 3
(3) 5 : 11 (4) 4 : 9 30. The ratio of milk and water in a mixture is 4:3. 14
(5) 5 : 7 litres of water is withdrawn and added same
quantity of milk to the mixture, then the ratio
25. Consider two alloys A and B. 50 kg of alloy A is becomes 9:5. Find the initial quantity of milk?
mixed with 70 kg of alloy B. A contains brass and (1) 88 li (2) 56 li
copper in the ratio 3 : 2, and B contains them in (3) 112 li (4) 148 li
the ratio 4 : 3 respectively. What is the ratio of (5) None of these
copper to brass in the mixture?
(1) 8 : 5 (2) 7 : 5
(3) 5 : 11 (4) 4 : 9
31. The ratio of milk and water in a mixture is 13:7. 36. 48 litre of Glycerin is mixed with 144 litre Rose
(X + 10) litres of mixture of taken out and water. X litre of total mixture is taken out and 32
replaced with same amount of water, then the litre Glycerin and 48 litre Rose water are added
ratio becomes 1:1. If the initial quantity of water in the mixture. The final mixture contains 30%
was 91 li, then find the value of 28% of X Glycerin, find the quantity of the mixture that is
(1) 7 (2) 14 taken out.
(3) 21 (4) 50 (1) 24 litre (2) 32 litre
(5) None of these (3) 40 litre (4) 20 litre
(5) None of these
32. The ratio of milk and water in a mixture is 5:3. If
24 litres of mixture taken out and replaced with 37. In a mixture, the ratio of the alchohol and water
same amount of water, then the ratio of water and is 6 : 5. When 22 litre mixture are replaced by
milk becomes 17:11. Find the initial quantity of water, the ratio becomes 9 : 13. Find the quantity
milk? of water after replacement.
(1) 525 li (2) 500 li (1) 62 litre (2) 50 litre
(3) 105 li (4) 315 li (3) 40 litre (4) 52 litre
(5) None of these (5) None of these

33. The ratio of two bewrages, coke and sprite in a 38. A jug contains 64 litres of pure milk.1/4of the
cold drink is 7:3. If 40 litres of cold drink is taken milk is replaced by the same amount of water.
out and 21 litres of sprite is added to the Again,1/4 of the mixture is replaced by that
remaining cold drink, then the ratio becomes amount of water. Find the ratio of milk and water
14:9. Find the 41.66% of the initial cold drink? in the new mixture.
(1) 50 (2) 75 (1) 5:4 (2) 2:3
(3) 20 (4) 125 (3) 3:4 (4) 5:6
(5) None of these (5) None of these

34. The ratio of milk and water in a mixture is 4 : 3 39. 30 L of alcohol is mixed with 75 L of water in
respectively. If we add (x + 12) litres of water in vessel to form a mixture. Then, (p + 5) L of
the mixture, the ratio of milk and water becomes mixture in vessel is replaced with 12.5 L of
5 : 7 respectively. If the initial quantity of the milk alcohol and 15 L of water such that the mixture in
was 80 li, then find the value of (1681 – x2). vessel now contains 33(1/3)% of alcohol. Find
(1) 80 (2) 40 the value of ‘p’.
(3) 81 (4) 61 (1) 15 (2) 30
(5) None of these (3) 25 (4) 20
(5) None of these.
35. A bottle was full of whiskey. Mr. Bansal used to
draw out 20% of the whiskey from bottle and 40. A jar full of whisky contains 40% alcohol. A part
replaced it with water. He has repeated the same of this whisky is replaced by another containing
process 4 times and thus there was only 512 ml of 19% alcohol and now the percentage of alcohol
whiskey left in bottle, the rest part of the bottle was found to be 26%. The part of whisky replaced
was filled with water. What is the initial amount is:
of whiskey in the bottle? 1 2
(1) 3
(2) 3
(1) 1.05 li (2) 1.15 li 2 3
(3) (4)
(3) 1.25 li (4) 1.30 li 5 5
4
(5) None of these (5) 5
41. 240 ml of mixture contains Lactic acid and (1) Quantity I > Quantity II
oxalic acid in the ratio of 9:7, respectively. If ‘8x’ (2) Quantity I < Quantity II
ml of mixture is taken out and 7.5 ml of Lactic (3) Quantity I ≥ Quantity II
acid 7.5 ml of oxalic acid is added into the (4) Quantity I ≤ Quantity II
remaining mixture, then ratio of Lactic acid to (5) Quantity I = Quantity II or Relation cannot be
oxalic acid in the resultant mixture becomes 5:4. Established.
Find the value of ‘x’.
(1) 15 (2) 25 45. Quantity I: A milkman has 72 litres of mixture of
(3) 10 (4) 20 water and milk in the ratio of 4:5.Then he adding
(5) None of these some amount of water and the volume of the
mixture is increased by 88 liters. The quantity of
42. Three equal jugs are filled with a mixture of milk the milk is how much percentage more or less
and water. The proportion of milk and water in than of water in the final mixture?
each glass is in the ratio 1 : 2, 2 : 3 and 3 : 4. The Quantity II: 15%
contents of the three jugs are emptied into a single (1) Quantity I > Quantity II
vessel. What is the proportion of milk and water (2) Quantity I < Quantity II
in it ? (3) Quantity I ≥ Quantity II
(1) 122 : 193 (2) 193 : 105 (4) Quantity I ≤ Quantity II
(3) 105 : 122 (4) 122 : 105 (5) Quantity I = Quantity II or Relation cannot be
(5) None of these Established.

43. In a mixture of Milk and Water, the quantity of 46. Quantity I: The ratio of the milk and water in the
water is 40% less than the quantity of milk. When mixture is 4:3. If in 35 liters of the mixture, 10
5 litres of pure milk were added then the quantity liters of water is added, then what is the
of milk becomes 80% more than the quantity of percentage of water in the final mixture?
water. Quantity II: In 2017 number of employees in the
Quantity I: What is the quantity of water in the company is 2800 and the number of employees in
mixture? 2019 is 4500. What is the approximate percentage
Quantity II: 40 litres increase in the number of employees from 2017
(1) Quantity I > Quantity II to 2019?
(2) Quantity I < Quantity II (1) Quantity I > Quantity II
(3) Quantity I ≥ Quantity II (2) Quantity I < Quantity II
(4) Quantity I ≤ Quantity II (3) Quantity I ≥ Quantity II
(5) Quantity I = Quantity II or Relation cannot be (4) Quantity I ≤ Quantity II
Established. (5) Quantity I = Quantity II or Relation cannot be
Established.
44. 10 litres of water were drawn from a cask full of
water and it was filled with 30 litres milk then the 47. When 5 litres of water were added with some
concentration of milk in the mixture become quantity of pure milk then the ratio of milk to
20%. water become 3 : 2.
Quantity I: Again, how many litres of mixture Quantity I : In the mixture, when 1.5 litres of pure
should be replaced with 10 litres milk so the milk are added then what will be the
concentration of water in the mixture will become concentration of milk in the new mixture?
60%. Quantity II : Instead of 5 litres of water, if 3.5
Quantity II: What was the original quantity of litres of water were added and the quantity of pure
water in the cask? milk remained the same then what would be the
concentration of pure milk in the mixture?
(1) Quantity I > Quantity II : 1, respectively. 200 litres of the mixture is
(2) Quantity I < Quantity II transferred from vessel X to vessel Y. If the initial
(3) Quantity I ≥ Quantity II quantity in vessel Y was 600 litres, then what
(4) Quantity I ≤ Quantity II would be the total quantity of water in vessel Y in
(5) Quantity I = Quantity II or Relation cannot be the end?
Established. (1) Quantity I > Quantity II
(2) Quantity I < Quantity II
48. Quantity I : How much water must be added to 15 (3) Quantity I ≥ Quantity II
litres of milk – water mixture of ratio 4 : 1, such (4) Quantity I ≤ Quantity II
that the ratio of milk and water in mixture (5) Quantity I = Quantity II or Relation cannot be
becomes 3 : 2? Established.
Quantity II : In what quantity of alcohol – water
mixture of ratio 2 : 1, should 2 litres of water be 51. The ratio of males to females in a town is 5: 4. If
added, such that the ratio of the final mixture is 1 5000 females got married and moved to another
: 1? town then the ratio of males to females become 7:
(1) Quantity I > Quantity II 4.
(2) Quantity I < Quantity II Quantity I: What is the total number of male
(3) Quantity I ≥ Quantity II populations in the town?
(4) Quantity I ≤ Quantity II Quantity II: 21750
(5) Quantity I = Quantity II or Relation cannot be (1) Quantity I > Quantity II
Established. (2) Quantity I < Quantity II
(3) Quantity I ≥ Quantity II
49. Quantity I : A jar contains 48 litres of milk and (4) Quantity I ≤ Quantity II
'X' litres of water. If a new mixture containing (5) Quantity I = Quantity II or Relation cannot be
'2X' litres of milk and '3X' litre water is added to Established.
the jar, then the final quantity of this mixture
becomes 60 litres. What was the quantity of milk 52. In a mixture of milk and water, the ratio of milk
in the final mixture? (in litres) to water is 2 : y. When 4 litres of milk were added
Quantity II : 51 litres in the mixture then, the concentration of water
(1) Quantity I > Quantity II becomes 50% but when 4 litres of water were
(2) Quantity I < Quantity II added in the mixture then the concentration of
(3) Quantity I ≥ Quantity II milk becomes 33.33%.
(4) Quantity I ≤ Quantity II Quantity I: Milk will be what part of the mixture
(5) Quantity I = Quantity II or Relation cannot be when, 5 litres of milk were added in the original
Established. mixture?
Quantity II: Water will be what part of the
50. Quantity I: Tank A and Tank B contain a mixture mixture when 3 litres of water were added in the
of petrol and diesel in ratio of 3 : 7 and 4 : 1, original mixture?
respectively. If 300 litres and 400 litres are drawn (1) Quantity I > Quantity II
from tank A and tank B, respectively, then what (2) Quantity I < Quantity II
would be the difference between the total (3) Quantity I ≥ Quantity II
quantity of petrol and the total quantity of diesel (4) Quantity I ≤ Quantity II
drawn from the 2 tanks? (5) Quantity I = Quantity II or Relation cannot be
Quantity II: Vessel X and vessel Y contain Established.
mixture of oil and water in the ratio of 3 : 2 and 1
Answer Key
1. (1) 10. (1) 19. (4) 28. (4) 37. (4) 46. (2)
2. (3) 11. (1) 20. (5) 29. (2) 38. (5) 47. (2)
3. (3) 12. (3) 21. (2) 30. (3) 39. (2) 48. (2)
4. (2) 13. (2) 22. (2) 31. (2) 40. (2) 49. (1)
5. (5) 14. (5) 23. (1) 32. (1) 41. (1) 50. (2)
6. (1) 15. (5) 24. (1) 33. (2) 42. (1) 51. (1)
7. (3) 16. (2) 25. (5) 34. (3) 43. (2) 52. (2)
8. (2) 17. (2) 26. (1) 35. (3) 44. (2)
9. (3) 18. (1) 27. (3) 36. (2) 45. (1)
Hints & Solutions
1. Solution (1) Sum of ratio terms = (27 + 24 + 20 + 15) = 86.
1 27
30% of A = 0.25 of B = 5 of C A’s share = Rs. (6450 × 86) = Rs. 2025;
30 25 1 3A B C 24
⇒ 100 A = 100B = 5 C ⇒ 10 = 4 = 5 = k (say) B’s share = Rs. (6450 × 86) = Rs. 1800;
10k 20
⇒A= 3
,B = 4k, C = 5k C’s share = Rs. (6450 × 86) = Rs. 1500;
10k 10 15
⇒A∶B∶C = ∶ 4 k ∶ 5k = :4 ∶ 5 D’s share = Rs. (6450 × 86) = Rs. 1125
3 3
= 10 ∶ 12 ∶ 15
5. Solution (5)
2. Solution (3) Let C’s share = Rs. x
3 7x
A:C =
A A B 3 5
= × = × = = 15 ∶ 28
15 Then, B’s share = Rs. (1 x) = Rs. ( ) ;
C B C 4 7 28 4 4
3 7x 21x
B:D=
B B C 5 8 40
= C × D = 7 × 9 = 63 = 40 ∶ 63 A’s share = Rs. (2 × 4
) = Rs. ( 8
)
D
A A B C 3 5 8 10 21x 7x 21 7
A:D= = × × = × × = A:B:C= 8
∶ 4
:x = : :1
8 4
= 21 ∶ 14 ∶ 8.
D B C D 4 7 9 21
= 10 ∶ 21 Sum of ratio terms = (21 +14 +8) = 43.
8
A : B = 3 : 4; B : C = 5 : 7 =
4
(5 × 5) ∶ (7 ×
4
) ∴ C’s share = Rs, (1290 × 43) = Rs. 240.
5
28
=4∶ 6. Solution (1)
5
7 7 28 63 Let the incomes of A and B be 3x and 4x and their
C : D = 8 : 9 = (8 × 10) : (9 × 10) = 5 ∶ 10
expenditures be 4y and 5y respectively.
28 63
∴A∶B∶C∶D=3∶4∶ : Then, A’s savings = 3x – 4y
5 10 1
= 30 ∶ 40 ∶ 56 ∶ 63. ∴ 3x − 4y = of 3x ⇒ 12x − 16y = 3x ⇒ 9x
4
9
= 16y ⇒ y = x.
3. Solution (3) 16
A′ s savings
3A = 4B = 7C = k(say) So, ratio of savings = B′ s savings
k k k k k k
⇒ A = ,B = ,C = ⇒ A ∶ B ∶ C = ∶ : 9
3 4 7 3 4 7 3x − 4y 3x − 4 × 16 x
1 1 1 = =
= 3 : 4 : 7 = 28 ∶ 21 ∶ 12 4x − 5y 4x − 5 × 9 x
16
Sum of ratio terms = (28 + 21 + 12) = 61. 9
3x − x
28
∴ A′ s Share = Rs. (427 × 61) = Rs. 196; = 4 = (3 x × 16 ) = 12 ∶ 19
45 4 19x
21 4x − 16 x
B’s share = Rs. (427 × 61) = Rs. 147;
12
C’s share = Rs. (427 × ) = Rs. 84 7. Solution (3)
61
5
Quantity of milk = (64 × 8) liltres = 40 litres
4. Solution (2) 3
4 4 Quantity of water = (64 × 8) litres = 24 litres
A : B = 9 : 8, B : C = 6 : 5 = (6 × 3) : (5 × 3)
Let x litres of water be added.
20 40 3
=8∶ , Then, 24+x = 5 ⇒ 200 = 72 + 3x
3
5 5 20
C : D 4 ∶ 3 = (4 × 3) ∶ (3 × 3) = 3 ∶ 5. ⇒ 3x = 128
128 2
20 ⇒x= 3
= 42 3
∴A∶B∶C∶D=9∶8∶ ∶5 2
3 Hence, required quantity = 42 3 litres
= 27 ∶ 24 ∶ 20 ∶ 15.
8. Solution (2) 13. Solution (2)
Let the number of one-rupee, 50 paise and 25 quantity of milk = 104 * 8/13 = 64
paise coins be 5x, 6x and 8x respectively. quantity of water = 104 * 5/13 = 40
50×6x 64/40 + x = 8/7
Then, sum of their values = Rs. (5x + 100
+
25×8x
x = 16
100
) = Rs. (5x + 3x + 2x) = Rs. 10x
∴ 10x = 210 ⇒ x = 21. ] 14. Solution (5)
So, number of one rupee coins = 5 × 21 = 105; 5
Initial quantity of milk = 224 × (8) = 140 li
Number of 50- paise coins = 6 × 21 = 126;
Initial quantity of water = 224 – 140 = 84 li
Number of 25-paise coins = 8 × 21 = 168 Required ratio = 140 : (84 + 12)
= 140 : 96 = 35 : 24
9. Solution (3)
Let the number of students studying in school A, B
15. Solution (5)
and C be 6x, 8x and 7x respectively. Let initially, there was x li mango shake and y li
Then, new strength of school A = 120% of 6x = banana shake.
36x
𝑥 5
5 So, 𝑦 + 48 = 3
46x
New strength of school B = 115% of 8x = 5
; 3x = 5y + 240
42x
New strength of school C = 120% of 7x = . 3x – 5y = 240 …(1)
5
36x 46x 42x Again,
∴ Required ratio = 5
∶ 5
∶ 5
= 18 ∶ 23 ∶ 21 𝑥 + 48 9
=
𝑦 + 48 5
10. Solution (1) 5x + 240 = 9y + 432
Income of Sarita = x, of Sewakram = 2x 5x – 9y = 192 …(2)
Expenditure of Sarita = 2y, of Sewakram = 5y Multiplying (1) by 9 and (2) by 5 and subtracting
Savings is (income – expenditure). So (2) from (1), we get
x – 2y = 20,000 (27 – 25)x = 2160 – 960
2x – 5y = 35,000 x = 600
Solve the equations, x = 30,000 Put this value of x in (1).
So total = x+2x = 3x = 3*30,000 = 90,000 1800 – 5y = 240
5y = 1560
11. Solution (1) y = 312 li
In the year 2021 = 2x: 3x: 4x
In the year 2022 = 3y:5y:7y 16. Solution (2)
3y-5/5y-5 = 1/2 Let initial quantity of wine and cold drink be 4a
y=5 and 9a li respectively.
Faculties in 2022 = 15, 25, 35 4𝑎 + 𝑋 5
Now 2x-n = 15 =
9𝑎 6
3x-n =25 24a + 6X = 45a
x =10 6𝑋 2𝑋
a = 21 = 7
Faculties in 2021 = 20, 30, 40
Again,
No of left in 2022= 3*5 = 15
4𝑎 + 𝑋 9
=
9𝑎 + 8 14
12. Solution (3) 56a + 14X = 81a + 72
4p/3p + 13 = 1 Putting the value of a in above equation
p=13 2𝑋
25 × 7
− 14𝑋 = −72
Sugar=3p+13=3*13+ 13 = 52 l
48X = 72 × 7
X = 10.5 Quantity of milk in the new mixture =[64 (1 −
16 2 3 2
17. Solution (2) ) ] litres = [64 × (4) ]litres
64
In the mixture of 45 litre, 9
45 45 (64 × 16) litres 36 litres.
Milk = × 4 = 36 litre, Water = × 1 = 9 litre
5 5 Quantity of water in the new mixture = (64 – 36)
New ratio, litres = 28litres.
Required ratio = 36 : 28 = 9 : 7.
= 9 + 3 : 36 + 4
21. Solution (2)
= 12 : 40 = 3 : 10 Ratio of initial quantity of alcohol to water in
vessel = 30:75 = 2:5
18. Solution (1) Let the quantity of alcohol and water withdrawn
Total mixture = 40 liters be 2x litres and 5x litres, respectively
milk = 40×(x + 1)/(2x + 1) Quantity of alcohol in the final mixture = 30 – 2x
water = 40×x/(2x + 1) + 12.5 = (42.5 – 2x) L
4
Final quantity of water = × (40 + 16) = 32 li Quantity of water in the final mixture = (75 – 5x +
7
15) = (90 – 5x) L
According to the question,
40 × 𝑥 Since, 33(1/3)% = 1/3, therefore
+ 16 = 32 Ratio of alcohol to water in the final mixture =
2𝑥 + 1
40 × 𝑥 (1/3):{1 – (1/3)} = (1/3):(2/3) = 1:2
= 16 According to the question,
2𝑥 + 1
40x = 32x + 16 {(42.5 – 2x)/(90 – 5x)} = 1/2
8x = 16 Or, 85 – 4x = 90 – 5x
x=2 Or, x = 5
Therefore, p + 5 = 2x + 5x
19. Solution (4) Or, p + 5 = 10 + 25
Let required amount of second solution to be Or, p = 30
added = a L
15 × 20 + 5a 22. Solution (2)
Then = 10
20 + a
⇒ 300 + 5a = 200 + 10a
⇒ 5a = 100
⇒ a = 20

20. Solution (5)


If from x units of liquid in a container, y units are 2
Part of whisky replaced is 3
taken out and replaced by water n times, then
quantity
23. Solution (1)
𝑦 𝑛
of pure liquid in the mixture =[𝑥 (1 − 𝑥 ) ]units. Quantity of Lactic acid in 240 ml of mixture = 240
1
Quantity of milk replaced each time =4of 64 litres × 9/16 = 135 ml
Quantity of oxalic acid in 240 ml of mixture = 240
= 16 litres.
– 135 = 105 ml
So, x = 64 litres, y = 16 litres, n = 2.
Quantity of Lactic acid in 16x ml of mixture =
9/16 × 8x = 4.5x ml
Quantity of oxalic acid in 16x ml of mixture = 8x
– 4.5x = 3.5x ml
According to question; 5(𝑥 − 2)
+ 12 7
(135 – 4.5x + 7.5)/(105 – 3.5x + 7.5) = 5/4 8 =
3(𝑥 − 2) 6
Multiplying by 2 in both numerator and
8 + 18
denominator 30x - 60 + 96 × 6 = 21x - 42 + 144 × 7
Or, 1140 – 36x = 1125 – 35x 30x - 21x = 966 - 516
Or, x = 15 9x = 450
x = 50
24. Solution (1)
Let the volume of each of the two cups be x. 30. Solution (3)
3
Then, volume of milk in the first cup = 5 x 112 li
2 Let initial quantity of milk and water be 4x and 3x
Volume of water in the first cup = x
5 li respectively.
4
Volume of milk in the second cup = x. According to the question,
5
x 4𝑥 + 14 9
Volume of water in the second cup = 5 =
2 x 3𝑥 – 14 5
∴ Ratio of water to milk in the jug = (5 x + 5) ∶ 20x + 70 = 27x – 126
3 4 3 7 7x = 196
(5 x + 5 x) = 5 x ∶ 5 x = 3 ∶ 7.
196
x= = 28
7
25. Solution (5) Required answer = 4 × 28 = 112 li
5:7
Brass in A = 3/5 * 50 = 30 kg, Brass in B = 4/7 * 31. Solution (2)
20
70 = 40 kg Initial quantity of mixture = ( ) × 91 = 260 li
7
Total brass = 30+40 = 70 kg Initial quantity of milk = 260 – 91 = 169 li
So copper in mixture is (50+70) – 70 = 50 kg According to the question,
So copper to brass = 50 : 70 13 7
169 – (X + 10) × 20 = 91 – (X + 10) × 20 + (X +
26. Solution (1) 10)
26
Sprite=250*(100-x/100) 169 – 91 = (X + 10) × 20
Then, 50 litres replaced it, 780
= X + 10
250*(100-x/100) – 50*(100-x/100) = 64% of 250 13
X = 60 – 10 = 50
x =20%
Required answer = 28% of 50 = 14
remaining= 80%

32. Solution (1)


27. Solution (3)
Let initial quantity of milk and water be 5x and 3x
81 = 192(1-x)^3
li respectively.
x =25 %
According to the question,
5
28. Solution (4) 5𝑥 – (8) × 24 17
p+q=30 =
3
3𝑥 − (8) × 24 + 24 11
(p-10*p/p+y) / (q-10*q/(p+q)+10) = 2/3
2p-4/3q = 20 5𝑥 – 15 17
=
3𝑥 + 15 11
29. Solution (2) 55x – 165 = 51x + 255
5 5(𝑥 − 2) 4x = 420
Initial quantity of milk = (x - 2) × 8
= 8 li
x = 105
3 3(𝑥 − 2)
Initial quantity of water = (x - 2) × 8 = 8 li Required answer = 5 × 105 = 525 li
According to the question,
33. Solution (2) 37. Solution (4)
Let initial quantity of coke and sprite be 7x and 3x Let alchohol = 6x, water = 5x
li respectively. According to the question,
According to the question, 6 5
7 6x – 22 × : 5x – 22 × + 22 = 9 : 13
7𝑥 – (10) × 40 11 11
14
= 6x – 12 : 5x – 10 + 22 = 9 : 13
3 9
3𝑥 − (10) × 40 + 21 13 (6x – 12) = 9 (5x + 12)
7𝑥 – 28 14 78x – 156 = 45x + 108
= 78x – 45x = 156 + 108
3𝑥 + 9 9
63x – 252 = 42x + 126 33x = 264
21x = 378 x=8
x = 18 Water after replacement = 5 × 8 – 10 + 22 = 40 +
Initial quantity of cold drink = 10 × 18 = 180 li 22 = 52 litre
125
Required answer = 180 × 300
38. Solution (5)
= 75
If from x units of liquid in a container, y units are
taken out and replaced by water n times, then
34. Solution (3)
7 quantity
Initial quantity of mixture = (4) × 80 = 140 li 𝑦 𝑛
of pure liquid in the mixture =[𝑥 (1 − 𝑥 ) ]units.
According to the question,
1
80 5 Quantity of milk replaced each time =4of 64 litres
=
60 + 𝑥 + 12 7 = 16 litres.
560 = 360 + 5x So, x = 64 litres, y = 16 litres, n = 2.
5x = 200
x = 40 Quantity of milk in the new mixture =[64 (1 −
Required answer = 1681 – 1600 = 81 16 2 3 2
64
) ] litres = [64 × (4) ]litres
9
35. Solution (3) (64 × ) litres 36 litres.
16
512 = x(1-1/5)^4 Quantity of water in the new mixture = (64 – 36)
x = 1.25 li litres = 28litres.
Required ratio = 36 : 28 = 9 : 7.
36. Solution (2)
Total mixture = 48 + 144 = 192 litre 39. Solution (2)
% of Glycerin = 48/ 192 × 100 = 25% = % of Rose Ratio of initial quantity of alcohol to water in
water = 75% vessel = 30:75 = 2:5
In the final mixture glycerin= 30%, Rose water = Let the quantity of alcohol and water withdrawn
70% be 2x litres and 5x litres, respectively
Ratio = 30 : 70 = 3 : 7 Quantity of alcohol in the final mixture = 30 – 2x
(48 – D × 25% + 32) : (144 – D × 75% + 48) = + 12.5 = (42.5 – 2x) L
3:7 Quantity of water in the final mixture = (75 – 5x +
7 (48 – D × 25% + 32) = 3 (144 – D × 75% + 48) 15) = (90 – 5x) L
7 (80 – 0.25 D) = 3 (192 – 0.75 D) Since, 33(1/3)% = 1/3, therefore
560 – 1.75 D = 576 – 2.25 D Ratio of alcohol to water in the final mixture =
2.25 D – 1.75 D = 576 – 560 (1/3):{1 – (1/3)} = (1/3):(2/3) = 1:2
0.5 D = 16 According to the question,
D = 32 litre {(42.5 – 2x)/(90 – 5x)} = 1/2
Or, 85 – 4x = 90 – 5x
Or, x = 5 122 193
= ∶ = 122 ∶ 193
Therefore, p + 5 = 2x + 5x 105 105
Or, p + 5 = 10 + 25
Or, p = 30 43. Solution (2)
Let the quantity of Milk = 10x litres
40. Solution (2) then the quantity of Water = (100 – 40)% of 10x
= 60% of 10x = 6x litres
When 5 litres of Milk was added then the quantity
of Milk = 10x + 5 litres
and the quantity of Water = 6x litres
According to the question, 180% of 6x = (10x + 5)
10.8x = 10x + 5 0.8x = 5 8x = 50 x = 6.25 litres
2
Quantity II : The quantity of Water = 6x = 6 × 6.25
Part of whisky replaced is 3 = 37.5 litres
Quanity : I < Quantity : II
41. Solution (1)
Quantity of Lactic acid in 240 ml of mixture = 240 44. Solution (2)
× 9/16 = 135 ml Quantity I: Let the quantity of water in the cask =
Quantity of oxalic acid in 240 ml of mixture = 240 x litres
– 135 = 105 ml then x – 10 + 30 = x + 20 litres = quantity of
Quantity of Lactic acid in 16x ml of mixture = mixture
9/16 × 8x = 4.5x ml The quantity of milk in x + 20 litres = 30 litres
Quantity of oxalic acid in 16x ml of mixture = 8x According to the question, 20% of (x + 20) = 30
– 4.5x = 3.5x ml x + 20 = 150 x = 130 litres
According to question; And the quantity of mixture = x + 20 = 150 litres
(135 – 4.5x + 7.5)/(105 – 3.5x + 7.5) = 5/4 The quantity of water = x – 10 = 120 litres
Multiplying by 2 in both numerator and Quantity II : 130 litres
denominator Therefore, Quantity : I < Quantity : II
Or, 1140 – 36x = 1125 – 35x
Or, x = 15 45. Solution (1)
Quantity I
42. Solution (1) Ratio of the red, blue and yellow balls = 4:
Let the volume of each jug be x litres. Then, 3: 5
x
Milk in 1st glass = litres; water in lst glass =
2x (5x – 18)/(3x + 10) = 3/7
3 3
9x + 30 = 35x – 126
litres;
2x 26x = 156
Milk in 2nd glass = 5
litres; water in 2nd glass = X=6
3x
litres; Total number of balls = 12x = 12 * 6 = 72
3
3x 4x Quantity II
Milk in 3rd glass = 7
litres; water in 3rd glass = 7 Income = 80000
litres. Expenditure = 32000
x 2x 3x
Total milk in final mixture = (3 + 5
+ 7
) litres Savings = 80000 – 32000 = 48000
122 Required percentage = 48000/80000 * 100
= 105 litres.
= 60%
2x 3x 4x
Total water in final mixture = ( 3 + 5
+ 7
) litres Quantity I > quantity II
193
= litres.
105
∴ Required ratio of milk and water
46. Solution (2) 12/x+3=3/2
Quantity I, ; x = 5 litres
Initial mixture = 35 liters So, Quantity I = 5 litres
Ratio of milk and water = 4:3 Quantity II : Let the required quantity of initial
Milk in the mixture =4/7 * 35=20 liters mixture be ‘x’ litres
Water in the mixture =3/7 * 35=15liters Final mixture = (x + 2) litres
After adding 10 liters of water, then quantity of Quantity of alcohol in initial mixture = Quantity of
water in the new mixture = 15 + 10=25 liters alcohol in final mixture
New mixture of milk and water = 35+10 = 45 2/3× x =1/2× (x + 2)
liters x = 6 litres
Required percentage =25/45 * 100=55.56% So, Quantity II = 6 litres
Quantity II, Quantity I < Quantity II
Number of employees in the year 2017 = 2800
Number of employees in the year 2019 = 4500 49. Solution (1)
Required percentage= [(4500 – 2800)/2800] * Quantity I :
100=60.71% Milk water
Therefore, Quantity I < Quantity II Initial mixture 48 L xL
New mixture 2x L 3x L
Final mixture 48 + 2x x + 3x
47. Solution (2) Now, 48 + 2x + x + 3x = 60
Let the quantity of pure milk = 3x litres then the ⇒ 6x = 60 – 48 = 12
quantity of water = 2x litres = 5 litres ∴x=2
X = 2.5 litres ∴ Quantity of milk = (48 + 2 × 2) = 52 litres
Therefore, the quantity of pure milk = 3x = 3 × 2.5 Quantity II : 51 litres
= 7.5 litres Here, Quantity I : > Quantity : II
Quantity I:
In the mixture, when 1.5 litres of pure milk are 50. Solution (2)
added then the quantity of pure milk = 7.5 + 1.5 = Quantity I :
9 litres and the quantity of mixture = 9 + 5 = 14 Total quantity of petrol drawn from the 2 tanks
litres = 300 ×3/10 + 400 ×4/5
The concentration of milk=9*100/40=64.28% = 90 + 320 = 410 litres
Quantity II: Total quantity of diesel drawn from the 2 tanks
Instead of 5 litres of water, if 3.5 litres of water = 300 ×7/10+ 400 ×1/5
were added and the quantity of pure milk remained = 210 + 80 = 290 litres
the same then the quantity of pure milk = 7.5 litres Required difference = 410 – 290 = 120 litres
and the quantity of water = 3.5 litres Quantity II :
The quantity of mixture = 7.5 + 3.5 = 11 litres Initial quantity of water in vessel Y =1/2× 600 =
The reqd. concentration =7.5*100/11=68.18% 300 litres
Quantity :I < Quantity :II Quantity of water in the liquid transferred from
vessel X to vessel Y
48. Solution (2) = 2/5× 200 = 80 litres
Quantity I : Quantity of milk in initial mixture = Quantity of water in the end = 300 + 80 = 380 litres
quality of milk in final mixture Therefore, Quantity I < Quantity II
4× 15/5 = 12 litres
Let x litres of water be added 51. Solution (1)
Quantity of water in final mixtures = x + 3 litres Let the population of male = 5x then the
Hence, milk – water ratio of final mixture = population of female = 4x
According to the question, From the equation (i) and (ii) 2x + 4 = 4x – 4
5x/4x – 5000=7/ 4 2x = 8
20x = 28x – 35000 x = 4 Put the value of x in the equation (i) y = 3
8x = 35000 The quantity of mil in the original mixture = 2x = 8
x = 4375 litres and the quantity of water = yx = 3 × 4 =12
The number of males = 5x = 5 × 4375 = 21875 litres
Quantity : I > Quantity : II Quantity I : when, 5 litres of milk were added in
the original mixture
52. Solution (2) Milk=13/25 part
Let the quantity of milk = 2x litres Quantity II: when 3 litres of water were added in
then the quantity of water = yx litres the original mixture
According to the question, yx = 50% of (2x + yx + 4) Water=15/23 part
2yx = 2x + yx + 4 yx = 2x + 4 ........... (i) Quantity : I < Quantity : II
2x = 33.33% of (2x + yx + 4) 6x
= 2x + yx + 4 yx = 4x – 4 ............... (ii)
1. If 9 men and 12 boys can do a piece of work in 6 6. A and B can do a piece of work in 25 days. B and
days and 10 men and 9 boys can do it in 7.5 days, C can do the same piece of work in 30 days. C and
find the ratio of efficiency of a man to a boy. A can do the same work in 37.5 days. In how
(1) 1 : 3 (2) 12 : 11 many days will A alone do the same work?
(3) 2 : 3 (4) 3 : 14 (1) 100 days (2) 60 days
(5) None of these (3) 50 days (4) 75 days
(5) None of these
2. A certain number of people were supposed to
complete a work in 20 days. The work, however, 7. A is 50% more efficient than B. If A can do a
took 28 days, since 8 people were absent
work in 30 days, then find in how many days will
throughout. How many people were supposed to
C do the work if C is 20% efficient of A and B
be working originally?
together?
(1) 32 (2) 27
(1) 15 days (2) 30 days
(3) 36 (4) 28
(5) None of these (3) 20 days (4) 10 days
(5) None of these
3. 12 men can finish a project in 12 days. 18 women
can finish the same project in 16 days and 24 8. A certain job was assigned to a group of men to do
children can finish it in 18 days. 8 women and 16 it in 28 days. But 18 men did not turn up for the
children worked for 9 days and then left. In how job and the remaining men did the job in 40 days.
many days will 10 men complete the remaining What was the original number of men in the
project? group?
(1) 10 ½ (2) 10 (1) 30 (2) 60
(3) 6 (4) 11 ½ (3) 40 (4) 100
(5) None of these (5) None of these

4. A and B together can complete a work in 24/5 9. Ramesh can do a work in 5 days. Bahu can do the
days. B and C together can complete a work in same work in 15 days. If the work on alternative
36/5 days. A and C together can complete a work days starting with Bahu then in how many days
in 72/13 days. In how many days B alone can will they complete thrice the work?
complete the work?
(1) 20 days (2) 21 days
(1) 14 days (2) 12 days
(3) 12 1/2 days (4) 19 days (3) 23 days (4) 22 days
(5) 20 days (5) None of these

5. A and B can complete a work in 12 and 20 days 10. Ramesh takes 16 minutes to type two page. If from
respectively. After 4 days, they are joined by C
3 : 00 pm to 4 : 00 pm, 1710 pages are to be typed,
who can complete the same work in 24 days, how
how many persons of same efficiency as Ramesh
much work will remain uncompleted after 2 more
can complete the work in time?
days?
(1) 207 (2) 221
(1) 53/60 (2) 41/60
(3) 249 (4) 213
(3) 13/60 (4) 11/60
(5) 7/60 (5) None of these
11. 27 workers can construct a building in 15 days, 16. Two persons X and Y can dig a pit in 16 hours
working 4 hours a day. In how many days will 20 together. They worked together 4 hours and then a
people, working 9 hours a day, complete the work? third person Z whose efficiency is half of their
(1) 10 days together’s efficiency joins them. Find in how
(2) 9 days many time the whole work will be completed?
(3) 14 days (1) 10 hrs. (2) 13 hrs.
(4) 12 days (3) 12 hrs. (4) 16 hrs.
(5) 18 hrs.
(5) None of these
17. A tyre has two punctures. The first puncture alone
12. The hourly wages of a mason have increased by
would have made the tyre flat in 9 minutes and the
25%. Since the increase, the number of hours he second alone would have done it in 12 minutes. If
works daily has reduced by 16%. If he was earning air leaks out at a constant rate, how long does it
Rs 120 per day before the increase, how much (in take both the punctures together to make it flat ?
Rs) is he earning now? (1) 18/5 min (2) 20/3 min
(1) 124.5 (3) 10/3 min (4) CND
(2) 115.5 (5) 16/5 min
(3) 126
(4) 120 18. Nikhil’s efficiency is 100% more than that of
(5) None of these Chaitanya. Vikas takes thrice as many days as
Nikhil. Chaitanya takes 12 days to finish the work
13. If 3 men and 4 boys together earn Rs 264 in 8 days alone. If they work in pairs (i.e., Vikas Chaitanya,
and 2 men and 3 boys together earn Rs 184 in the Nikhil Vikas, Chaitanya Nikhil) starting with
Vikas Chaitanya on the first day, Nikhil and Vikas
same time, then in how many days will 6 men and
on the second day and Nikhil Chaitanya on the
7 boys together earn Rs 315?
third day and so on, then how many days are
(1) 5 days (2) 6 days
required to finish the work?
(3) 9 days (4) 10 days
(1) days (2) 4.5 days
(5) None of these
(3) days (4) 8 days
14. Anu and her friend Radha undertook a piece of (5) 4 days
work for Rs 1800. Anu alone could do the work in
12 days and Radha in 18 days. With the assistance 19. Tuntun can do as much work in 2 days as
of Rama, they completed the work in 4 days. Find Jhunjhun can do in 3 days and Jhunjhun can do as
the share of Anu in the money, if the money is to much in 4 days as Munmun in 5 days. A piece of
be shared in proportion to the amount of work work takes 20 days if all work together. How long
done. Jhunjhun take to do all the work by himself?
(1) 82 days (2) 44 days
(1) Rs 550 (2) Rs 600
(3) 66 days (4) 50 days
(3) Rs 650 (4) Rs 800
(5) NOT
(5) None of these
20. A contractor predicts that of the two workers one
15. P can do of the work in 20 days while Q can do can make a wall in 12 hours and the other in 11
of the work in 20 days. How long it will take for hours. He finds from his experience that if both the
both of them to finish the work if P started the workers work together they use 125 bricks more
work and they work on alternative days. per hour, and build the wall in 4 hours. Then find
(1) 30 days (2) 64 days the number of bricks in the wall?
(1) 3300 (2) 1650
(3) 42 days (4) 24 days
(3) 2000 (4) 1850
(5) 48 days
(5) None of these
21. Sarvan and Kundan can complete a work together 25. Ankit and Vishal can complete a work in 20 days
in 7.2 days, if Sarvan and Kundan started the work and 30 days respectively. Ankit and Vishal
together but Kundan left after 6 days then total decided to work alternately with Vishal starting
work completed in 8 days. Efficiency of Sarvan to the work first. Find in how many days will they
complete the work is what percent more/less than complete 60% of the work?
efficiency of Kundan?
(1) 9 days (2) 8 days
(1) more
(2) 50% more (3) 7 days (4) 7 days

(3) less (5) 7 days

(4) 50 % less
(5) more 26. Khushbu can do a work in ( ) of time in which
Tanu alone can do and Keshav can do the same
22. Pawan and Kasim can finish a piece of work in 6 work in same time as Khushbu and Tanu together
and 8 days respectively. If Pawan and Kasim work will take. If all three working together can
on alternate days, how much time will it take to complete the work in 32 days, then find in how
complete the work if Kasim start the work? many days Khushbu and Keshav together can
(1) 6 days complete the work?
(2) 5 days (1) days
(3) 7 days
(4) 4 days (2) days
(5) 8 days
(3) days
23. David, Bekhum and Charles together can complete (4) days
a work in one day. David and Charles together can
(5) days
do the same work as Bekhum alone do while
Bekhum and Charles together can do five times as
27. Jatin and Jyoti together can do a work in 8 days
much work as David does alone. Find the time
taken by Charles to complete the work. whereas Jyoti alone can do it in 12 days. If Barkha
(1) 2 days alone can do the same work in 15 days, then find
(2) 3 days the ratio of efficiency of Barkha to efficiency of
Jatin?
(3) 4 days
(4) 8 days (1) 5 : 8 (2) 8 : 5
(5) 5 days (3) 5 : 3 (4) 2 : 3
(5) 7 : 8
24. Prateek and Babbar can complete a piece of work
28. Mukesh and Varsha together can do a work in (x –
in 6 days, Babbar and Kishore in 8 days and
Prateek and Kishore together can finish it in 12 2) days where as Varsha alone can do it in (x + 2)
days. If they work together and Prateek leaves the days. If Ankita alone can do the same work in 15
work after 4 days, then how many days Babbar days and the ratio of efficiency of Ankita to
efficiency of Mukesh is 8 : 5, then find in how
and Kishore take to complete the remaining work?
much time Varsha can complete the work working
(1) 4 days
(2) 5 days alone?
(3) 3 days (1) 5 : 8 (2) 8 : 5
(3) 5 : 3 (4) 2 : 3
(4) 6 days
(5) 7 : 8
(5) 2 days
29. (Rajkumar + Prasad) together can complete th
of a completed in 17 days. Find the number of days in
which ‘Shivendra’ alone can complete the work?
task in 24 days and (Prasad + Tushar) together can
th
(1) 12 days (2) 15 days
complete of the same task in 20 days. If for first (3) 18 days (4) 20 days
27 days Rajkumar worked alone , then for next 28 (5) 25 days
days Prasad worked alone and remaining task was
completed by Tushar alone in more days. Find 31. Pawan, Qasim and Rishabh together can do a piece
in how many days Prasad alone will complete the of work in 40 days. After working together for 16
task? days, Pawan leaves and then the remaining work
(1) 48 days (2) 36 days was completed by Qasim and Rishabh in 40 days.
(3) 30 days (4) 64 days Find out how many days Pawan alone can do the
(5) 60 days work?
(1) 100 days
30. Prashant and Shivendra are two persons working (2) 50 days
on alternative days. If ‘Prashant’ starts the work (3) 80 days
(4) 120 days
then the work is completed in days and if
(5) 75 days
‘Shivendra’ starts the work then the work is
Answer Key
1. (4) 8. (2) 15. (5) 22. (3) 29. (1)
2. (4) 9. (4) 16. (3) 23. (2) 30. (2)
3. (3) 10. (5) 17. (1) 24. (5) 31. (1)
4. (2) 11. (2) 18. (3) 25. (4)
5. (5) 12. (3) 19. (3) 26. (3)
6. (2) 13. (1) 20. (2) 27. (2)
7. (4) 14. (2) 21. (2) 28. (2)
Hints & Solutions
1. Solution (4) 6. Solution (2)
Let efficiency of 1 man and 1 boy be M and B A+B =25 ————6 (1 days work) [total=150]
respectively. B+C =30————-5 (1 days work)
(9M + 12B) × 6 = (10M + 9B) × 7.5 C+A =37.5———–4(1 days work)
⇒ 54M + 72B = 75M+ 67.5B Total = LCM of 25,30,37.5
⇒ 21M = 4.5B 1 day work=total work/days
⇒ adding the three equation
2(A+B+C) = 15
A+B+C 1 day work = 7.5
2. Solution (4)
A+B+C – (B+C) =7.5 – 5 = 2.5
Let x people were supposed to work
A =150/2.5 = 60 days
∴ (x – 8) × 28 = x × 20
⇒ 7x - 56 = 5x
7. Solution (4)
⇒ x = 28
A= 50% more efficient than B
hence A:B=150:100=3:2=30:20 (in terms of
3. Solution (3)
day)
9 days work of 8 women and 16 children
A+B = 5
( ) C = 20% = 1/5
C=1
1 = 10
∴ Remaining work = A:B:C=3:2:1
∴ No. of days taken by 10 men to complete the C = 1 = 10 days
remaining work
8. Solution (2)
Let the number of men designated for the work
4. Solution (2) be x and efficiency be 1 unit.
1/A + 1/B = 5/24 …..(1) Total work done by x men
1/B + 1/C = 5/36 ……(2) As per question;
1/A + 1/C = 13/72 ……(3)
Add equations 1 and 2 and then subtract 3 from
them.
(1/A + 1/B + 1/B + 1/C) – (1/A + 1/C) = 5/24 +
5/36 – 13/72
2/B = 1/6 9. Solution (4)
So B takes 12 days. Total units of work to be done is LCM of 5 and
15 i.e 15 units
5. Solution (5) Each day A can do 15/5=3 units of work
They work for 4 days. So complete Each day B can do 15/15=1 unit of work
(1/12 + 1/20) × 4 = 8/15 of work SO total units of work to be done=45 units
Now: In next 2 days all A, B, C completed Starting from B we have 1+3+1+3….= 45 i.e
(1/12 + 1/20 + 1/24) × 2 = 7/20 of work 4*11+1=45
So total work completed = 8/15 + 7/20 = 53/60 So total days=11*2+1/4
So remaining work = 1 – 53/60 = 7/60 Total days=22
10. Solution (5) 14. Solution (2)
M 1 T 1 W2 = M 2 T 2 W1 1 1
In four days Anu does  4  of work.
M1 = 1, T1 = 8 mins, W1 = 1 page 12 3
M2 = ?, T1 = 60 min (1 pm to 2pm), W2 = 1710 1
Her share   Rs. 1800
1 × 8 × 1710 3
∴ M2 = = 228
60 × 1
15. Solution (5)
11. Solution (2)
Let the required number of days be z. Then,
Less people, More
days (Indirect proportion)
More working hours per day, Less days (Indirect
proportion)
People 20 : 27
} :: 15 : z
Hours 9 : 4 2 day work of P and Q
z= = 9 days

days.
12. Solution (3)
So, P and Q will take 48 days if they work
Daily wages = hourly wages × work hours.
alternatively.
Let the original hourly wages and work hours be
Rs. x and y hours respectively.
16. Solution (3)
Since he used to earn Rs. 120 earlier, xy = 120
New hourly wages = Rs. (1.25x) and new 4 hour’s work of X and Y together = 4
working hours = 0.84y 16
∴ New daily wages = (1.25x)(0.84y) = 1.05xy 1

= 1.05 × 120 = Rs. 126 4
 One hour’s work of all the three persons
13. Solution (1) 1 1
 
Given, 3 men and 4 boys together earn Rs. 264 16 32
in 8 days and 2 men and 3 boys together earn
Rs. 184 in the same time = 3
32
Now, according to the Chain Rule formula,
M1D1W2 = M2D2W1  Rest work i.e. 3 th will be completed by
where, M = No. of person, D = No. of days, W 4
= Amount of work all the three in = 32  3
⇒ (3M + 4B) × 8 × 184 = (2M+ 3B) × 8 × 264 3 4
⇒ 3M = 7B ...(i) = 8 hours
Let the no. of days 6 men and 7 boys will work  Total time to complete the whole work
together to earn Rs. 315 be x = 4 + 8 = 12 hours
(6M + 7B) × x × 184 = (2M + 3B) × 8 × 315
Substituting the relation between men and boys 17. Solution (1)
from equation (i) in above equation, 1 minute’s work both the Punctures
⇒ (14B + 7B) × x × 184 =
⇒ 21B × x × 184 = 23B × 8 × 105 1 1 5
=   
⇒x=5  9 6  18
Thus, the no. of days 6 men and 7 boys will So, both the puncture will make the tyre f lat in
work together to earn Rs. 315 is 5. 18 3
=  3 min
5 5
18. Solution (3) 21. Solution (2)
Time taken by Chaitanya = 12 days Let Sarvan and Kundan can complete the work
alone in ‘x’ and ‘y’ days
ATQ,

One day’s work of pair Vikas Chaitanya And,

One day’s work of pair Nikhil Vikas On solving (i) & (ii)
x = 12, y = 18
Efficiency of Sarvan and Kundan will be 18 and
One day’s work of pair Chaitanya Nikhil
12 respectively if total work equal to ‘216’
Required %
∴ ATQ,

Next two days work (by Vikas Chaitanya and 22. Solution (3)
Nikhil Vikas together)
Remaining work after 5 days –( )

∴ days

19. Solution (3)


2 day’s work of Tuntun = 3 day’s work 23. Solution (2)
Jhunjhun Let David, Bekhum and Charles can complete a,
So, efficiency ratio of Tuntun and Jhunjhun = 3 : b and c unit of work in one day.
2 ATQ,
Similarly, efficiency ratio of Jhunjhun and a + c = b …(i)
Munmun = 5 : 4 b + c = 5a …(ii)
Tuntun : Jhunjhun = 15 : 10 & Jhunjhun : On solving (i) and (ii)
Munmun = 10 : 8
So, Tuntun : Jhunjhun : Munmun = 15 : 10 : 8
So, time taken by Jhunjhun alone = (15 + 10 + ⇒
8) × 20/10 (1 + 2 + 3) unit of work → 1 day
= 66 days (6) unit of work → 1 day

20. Solution (2)


Let the number of bricks in the wall be x.
So, number of bricks per hour used by worker 1 24. Solution (5)
= (x/12) bricks/hr Prateek, Babbar and Kishore are denoted by P, B
number of bricks per hour used by worker 2 = and K respectively.
(x/11) bricks/hr
Number of bricks used by both workers together
= (x/4) bricks/hr
According to question,
(x/4) – (x/12 + x/11) = 125
x/6 – x/11 = 125
5x = 125 × 66
x = 1650 Babbar and Kishore will complete the remaining
work in 2 days
25. Solution (4) 28. Solution (2)
Let Ankit and Vishal are denoted by A and B Ratio of efficiencies of Ankita and Mukesh
respectively. =8:5
Efficiency
So, time ratio = 5 : 8
So, time taken by Mukesh = (15/5) × 8 = 24
One day work of Mukesh = 1/24
According to the question,
1/(x – 2) – 1/(x + 2) = 1/24
24(x + 2 – x + 2) = (x2 – 4)
96 = x2 – 4
If Ankit and Vishal work alternately, starting x2 = 100
with Vishal, x = 10
Total days = 7 + time taken by Varsha = 12 days
= 7 days
29. Solution (1)
(Rajkumar + Prasad) together can complete
26. Solution (3)
Let Khushbu can do work in x days whole work in = days
Tanu can do in = 4x days 1 day work of (Rajkumar + Prasad) =
And Keshav can do work in = (Prasad + Tushar) together can complete whole
ATQ— work in = = days
All three together can complete work in=16 days
1 day work of (Prasad + Tushar) =
So,
ATQ-
+ + =
Rajkumar×27 days+Prasad×28 days + Tushar
×2 days=1 work
x = 80 days Rajkumar ×27 days + Prasad ×(27+1) days
Khushbu can complete work in x = 80 days +Tushar ×(1+1 days=1 work
Tanu can complete work in 4x =4 =320 27 days×( Rajkumar + Prasad)+1day×( Prasad +
days
Tushar)+ Tushar = 1 work
Time taken by Keshav to complete work

Tushar =
27. Solution (2) Tushar =
Jatin and Jyoti’s one day work
Prasad alone =
Jyoti’s one day work
=
Jatin’s one day work =
=
Barkha’s one day work
Ratio of Efficiency of Barkha and Jatin is
Prasad alone will complete task alone in 48 days
30. Solution (2) 31. Solution (1)
Let Prashant and Shivendra alone can do the Let work done by Pawan, Qasim and Rishabh in
work in ‘x’ and ‘y’ days respectively. 1 day = 1 unit
Work done by Pawan, Qasim and Rishabh in 40
If Prashant starts the work it completed in days = 40 unit
days ⇒ Prashant worked for 9 days & Shivendra Work done by (Pawan, Qasim and Rishabh) in
16 days = 16 unit
worked for days.
Remaining work = 40 – 16 = 24 unit
⇒ …(i) Work done by (Qasim and Rishabh) in 40 days
= 24 unit
If Shivendra starts the work it completed in 17
Efficiency of (Qasim and Rishabh)
days ⇒ Shivendra worked for 9 days and
unit/day
Prashant worked for 8 days
Efficiency of Pawan = Efficiency of (Pawan,
⇒ …(ii) Qasim and Rishabh) – Efficiency of (Qasim and
On solving (i) & (ii) Rishabh)
= 1 – 0.6
x = 20, y = 15
= 0.4 unit/day
⇒ Shivendra alone can complete the work in 15
Pawan alone will do work in days
days
1. A tank is 3/5 full. If 26 liters of water is added to 7. Pipe M fills a tank in 8 hours but due to leakage
the tank, it becomes 8/9 full. The capacity of the the tank takes 2 hours more to fill. Find in how
tank is: many hours the leakage will empty the full tank?
(1) 90 litres (2) 100 litres (1) 35 (2) 40
(3) 80 litres (4) 105 litres (3) 42 (4) 30
(5) None of these (5) None of these

2. Pipe P and pipe Q fill the tank in 15 hours and 30 8. A pipe fills ¾ part of a tank in 24 min. Then find
hours respectively. If pipe P and Q together in how many minutes ½ part of the tank will be
opened simultaneously and after 9 hours Pipe Q filled?
closed, in how many hours would pipe P take to (1) 24 (2) 15
fill the remaining tank? (3) 16 (4) 17
(1) 2 hours (2) 1.5 hours (5) None of these
(3) 5 hours (4) 6 hours
(5) None of these 9. A and B pipes of a tank fill the tank in 9 hours, B
and C pipes fill in 6 hours and C and A pipes fill
3. A pipe can fill a tank with water in 4 hours. Due to in 18 hours. In how many minutes pipe C alone
leakage in the bottom, it takes 4.5 hours to fill it. fills the tank?
In what times will the leak empty the fully filled (1) 14 (2) 12
tank? (3) 16 (4) 18
(1) 36 (2) 28 (5) None of these
(3) 30 (4) 30
(5) None of these 10. Two pipes of a tank fill the tank in 10 and 15
minutes respectively. Another pipe empties the
4. Pipe A can fill an empty tank in 3 hours and pipe full tank. When all the pipes are open the tank will
B in 7 hours. If both the pipes are opened and after fill in 10 minutes. Then find how many minutes
2 hours pipe A is closed, how much time B will the pipe empties the full tank?
take to fill the remaining tank? (1) 20 (2) 18
(1)
3
(2)
2 (3) 15 (4) 25
5
2
5
1 (5) None of these
(3) (4)
3 3
(5) None of these 11. Two pipes of a tank fill the tank in 18 minutes and
24 minutes. Both pipes are open. Find after how
5. Pipe X can fill a tank in 10 min and Pipe Y can many minutes the 1st pipe will be closed so that
empty a tank in 30 min. In how many minutes the the tank will be filled in 12 minutes?
tank will be filled when both pipes are opened? (1) 6 (2) 9
(1) 7.5 (2) 18 (3) 12 (4) 15
(3) 20 (4) 25 (5) None of these
(5) None of these
12. Pipe A is 4 times more efficient than B. Pipe A
6. A pipe P can fill a tank in 10 minutes and another fills the tank in 24 minutes. In what minute will
tank Q can fill the tank in 20 minutes. In how the tank be filled when both pipes are opened?
many minutes will the tank be filled half when (1) 30 (2) 60
both pipes are opened? (3) 20 (4) 50
(1) 4.33 (2) 3.33 (5) None of these
(3) 2 (4) 4
(5) None of these
13. Pipe A, B and C fill the tank together in 36 19. A cistern can be filled separately by two pipes P
minutes. They are open for 12 minutes after that and Q in 30 minutes and 35 minutes respectively.
Pipe C is closed and the rest is filled by pipe A and A tap R at the bottom can empty the full cistern in
B in 48 minutes. Find in how many hours C will 21 minutes. If the tap R is opened 7 minutes after
fill the tank alone? the two pipes P and Q are opened, then after what
(1) 90 min (2) 72 m time from the opening of tap R the cistern
(3) 120 min (4) 110 min becomes full (approx)?
(5) None of these (1) 40 minute (2) 36 minute
(3) 42 minute (4) 54 minute
14. Pipe A, B and C fill a tank alone in 12 ,18 and 24 (5) None of these
min respectively. All the pipe open for 3 min then
pipe C is closed then find how many min pipe A 20. Tap A fills a tank in 20 minutes while C empties it
and B can fill the rest of the part? at 1/3rd the rate at which A fills it. At 12 : 00
noon, A and C are simultaneously started and
(1) 5.3 (2) 2.1
when the tank is 50% full, tap A is turned off. At
(3) 3.3 (4) 4.5 what time will the tank be empty?
(5) None of these (1) 01 : 00 pm (2) 12: 45 pm
(3) 01 : 30 pm (4) 01 : 15 pm
15. A tank is emptied by a pipe in 3 hours. When from (5) None of these
another pipe 180 liter water per hour enters into
the tank then the tank is empty in 4 hours. Find the 21. Pipe A and pipe B fill the tank in 30 hours and 20
hours respectively. If pipe A and B together
capacity of the tank?
opened simultaneously and after 10 hours Pipe B
(1) 2113 (2) 2160 closed, if time taken by pipe A to fill the
(3) 2800 (4) 1500 remaining tank is x, find the value of x?
(5) None of these (1) 5 (2) 9
(3) 8 (4) 10
16. Pipe M fills a tank in 15 hours but due to leakage (5) None of these
the tank will fill in 20 hours. Find in how many
22. A pipe can fill a tank with water in 3 hours. Due to
hours the leakage will empty the full tank?
leakage in bottom, it takes 3.5 hour to fill it. the
(1) 60 (2) 41.75 leak will empty the fully filled tank in x hours,
(3) 80 (4) 18 find the value of x?
(5) None of these (1) 27 (2) 29
(3) 30 (4) 21
17. A pipe fills ¾ part of a tank in 24 min. Then find (5) None of these
in 12 minutes how much part of the tank will be
23. Two pipes can fill a tank in 30 min and 15 min
filled? respectively. When both pipes are open in the full
(1) 3/8 (2) 22/36 tank will be filled in y mins, find the value of y?
(3) 5/12 (4) 18/31 (1) 12 (2) 10
(5) None of these (3) 15 (4) 18
(5) None of these
18. Three Pipes P, Q and R can fill the tank in 12
24. Pipe X can fill a tank in 5 minutes and another
hours, 18 hours and 24 hours respectively. Pipe P
pipe Y can empty the tank in 10 minutes. In how
and Q started to fill the tank and after 5 hours pipe many minutes will the tank be filled ¾ parts when
R also opened. In how many hours the tank is both pipes are opened and the tank already filled ½
filled completely? part?
(1) 6(9/13) hours (2) 5(7/13) hours (1) 7.5 mins (2) 3.5 mins
(3) 4(6/11) hours (4) 5(7/11) hours (3) 2.5 mins (4) 4.5 mins
(5) None of these (5) None of these
25. Two pipes P and Q fill the tank in 12 minutes and 28. Pipe 'A' can fill 60% of a tank in 27 hours while
14 minutes respectively. Another pipe R empties pipe 'B' can fill the (11/20) part of the same tank in
the same tank in 7 minutes. Find how much part of 22 hours. If pipes 'A', 'B' and 'C' are opened
together and pipe 'B' is closed after 25 hours then
the tank will fill when all three are opened for 7
the tank can be filled by the remaining two pipes
minutes? in 20 more hours. If time taken by pipe 'C' alone to
(1) 1/12 (2) 1/18 empty x% of the tank in 14.4 hours then find the
(3) 1/8 (4) 1/10 value of x.
(5) None of these (1) 20 (2) 25
(3) 30 (4) 50
26. A and B pipe of a tank fill in 6 hour, B and C pipe (5) None of these
fill in 4 hour and C and A pipe fill in 3 hour. C
29. Pipes 'A' and 'B' can fill a tank in 12 hours and 18
alone can fill the tank in x hours, find the value of hours, respectively. If pipe 'C' is opened along
x? with pipes 'A' and 'B', then the tank gets filled in
(1) 24/5 (2) 34/5 4.5 hours. If pipe 'C' can fill (5/6)th part of the
(3) 28/5 (4) 38/5 same tank in x hours. Then find the value of x.
(5) None of these (1) 60 (2) 20
(3) 40 (4) 10
(5) None of these
27. Pipe A is 9 times more efficient than B. Pipe B
fills the tank in 60 minutes fine, then when both 30. A and B pipes of a tank fill in 9 hours, B and C
pipes are opened the tank will be filled in x hours, pipes fill in 6 hours and C and A pipes fill in 18
find x? hours. In how many minutes pipe C alone fills the
(1) 60/13 (2) 60/11 tank?
(3) 45/11 (4) 45/13 (1) 15 (2) 30
(3) 18 (4) 40
(5) None of these
(5) None of these
Answer Key
1. (1) 7. (2) 13. (2) 19. (1) 25. (1)
2. (2) 8. (3) 14. (3) 20. (2) 26. (1)
3. (1) 9. (4) 15. (2) 21. (1) 27. (2)
4. (4) 10. (3) 16. (1) 22. (4) 28. (1)
5. (1) 11. (2) 17. (1) 23. (2) 29. (4)
6. (2) 12. (3) 18. (1) 24. (1) 30. (3)
Hints & Solutions
1. Solution (1) Let time taken by B to fill the remaining tank be
Let the quantity of the tank be 5x l. x min.
3 As per question;
Tank full = × 5 x =3 xl
5 2(7 + 3) + 3𝑥𝑥 = 21
When, 26 l of water is added; => 20 + 3𝑥𝑥 = 21
8 1
3 x + 26 = × 5 x => 𝑥𝑥 = 𝑚𝑚𝑚𝑚𝑚𝑚
9 3
40 x
⇒ 3 x + 26 = 5. Solution (1)
9
1 1
13 x Time taken to fill the tank= +
⇒ 26= ⇒ x= 18l x y
9
Therefore; total quantity= 5= x 90l 1 1
= +
10 30
2. Solution (2) 4 30
= = = 7.5min
LCM (15,30) = 30 30 4
Let the quantity of the tank be 30 l.
Quantity of tank filled in 1 min by; 6. Solution (2)
P=
30
= 2 𝑙𝑙 LCM (10,20) = 20Let the quantity of the tank be
15
30 20 l.
Q= = 1 𝑙𝑙 Quantity of tank filled in 1 min by;
30
Let time taken by P to fill the remaining tank be P=
20
= 2 𝑙𝑙
10
x min. 20
As per question; Q= = 1 𝑙𝑙
20
(0.2×20)
9(2 + 1) + 2𝑥𝑥 = 30 Time taken to fill half the tank = =
2+1
=> 27 + 2𝑥𝑥 = 30 10
= 3.33 𝑚𝑚𝑚𝑚𝑚𝑚
=> 𝑥𝑥 = 1.5 ℎ𝑜𝑜𝑜𝑜𝑜𝑜𝑜𝑜 3

3. Solution (1) 7. Solution (2)


1
Tank filling rate =
1 Tank filling rate by M =
4 4

Time taken with leak = 4.5 ℎ𝑜𝑜𝑜𝑜𝑜𝑜𝑜𝑜 Time taken with leak = 8 + 2 = 10 ℎ𝑜𝑜𝑜𝑜𝑜𝑜𝑜𝑜
1
New, tank filling rate= 1/4.5 New, tank filling rate=
10
1 1 0.5 1 1 1 5−4 1
Rate of leakage = ( − )= = Rate of leakage = ( − )= =
4 4.5 18 36 8 10 40 40
Hence, it will take 36 hours to empty the tank, Hence, it will taken 40 hours to empty the tank,
when it is full. when it is full.

4. Solution (4) 8. Solution (3)


LCM (7,3) = 21 Time taken to fill 100% of tank = 24 × =
4
3
Let the quantity of the tank be 21 l.
32 𝑚𝑚𝑚𝑚𝑚𝑚
Quantity of tank filled in 1 min by;
21 Time taken to fill 1/2 part of tank =
A= = 7 𝑙𝑙 1
3 32 × ( ) = 16 𝑚𝑚𝑚𝑚𝑚𝑚
21 2
B= = 3 𝑙𝑙
7
9. Solution (4) 13. Solution (2)
LCM (9,6,18) = 18 Part of the tank (A+B+C), working together,
12 1
Let the quantity of the tank be 18 l. filled in 12 min = =
36 3
Quantity of tank filled in 1 min by; 1
18 Remaining part of tank to be filled = 1 − =
3
A+B = = 2 𝑙𝑙— (1) 2
9
18
,will be filled by A+B, in 48 minutes.
3
B+C = = 3 𝑙𝑙— (2) Therefore;
6
18 Time taken by A+B to fill the remaining tank =
C+A= = 1 𝑙𝑙— (3)
18 3
Adding all the three equation; and putting the 48 × = 72 𝑚𝑚𝑚𝑚𝑚𝑚
2
value of (1) in it; Time taken by C to fill the tank alone =
1 1
=> 2(𝐴𝐴 + 𝐵𝐵 + 𝐶𝐶) = 6 −
𝐴𝐴+𝐵𝐵+𝐶𝐶 𝐴𝐴+𝐵𝐵
=> 𝐴𝐴 + 𝐵𝐵 + 𝐶𝐶 = 3 1 1 1
= − =
=> 2 + 𝐶𝐶 = 3 36 72 72
=> 𝐶𝐶 = 1 𝑢𝑢𝑢𝑢𝑢𝑢𝑢𝑢𝑢𝑢 Hence, time taken by C will be 72 minutes.
18
Time taken by C = = 18 𝑚𝑚𝑚𝑚𝑚𝑚
1 14. Solution (3)
LCM (12,18,24) = 72
10. Solution (3) Let the quantity of the tank be 72 l.
Let another pipe take k min to empty the tank. Quantity of tank filled in 1 min by;
72
As per question; A= = 6 𝑙𝑙
12
1 1 1 1 72
+ − = B= = 4 𝑙𝑙
10 15 𝑘𝑘 10 18
72
1 1 1 1 C= = 3 𝑙𝑙
=> = − + 24
𝑘𝑘 10 10 15 Let time taken by A and B to fill the remaining
1 1 tank be x min.
=> =
𝑘𝑘 15 As per question;
=> 𝑘𝑘 = 15 𝑚𝑚𝑚𝑚𝑚𝑚 3(6 + 4 + 3) + 𝑥𝑥(6 + 4) = 72
=> 39 + 10𝑥𝑥 = 72
11. Solution (2) => 10𝑥𝑥 = 33
LCM (18,24) =72 => 𝑥𝑥 = 3.3 𝑚𝑚𝑚𝑚𝑚𝑚
Let the quantity of the tank be 72 l.
Quantity of tank filled in 1 min by; 15. Solution (2)
72 Let the capacity of the tank be C.
1st pipe = = 4 𝑙𝑙— (1) 𝐶𝐶
18
72 Rate of emptying =
2nd pipe = = 3 𝑙𝑙— (2) 3
24 When 180 liters of water per hour enter the tank
Let the 1st pipe be closed after x min.
from another pipe,
As per question;
Effective rate of emptying = Rate of emptying -
=> (4 + 3)𝑥𝑥 + 3(12 − 𝑥𝑥) = 72
Rate of filling
=> 7𝑥𝑥 + 36 − 3𝑥𝑥 = 72 𝐶𝐶
=> 4𝑥𝑥 = 36 = − 180
3
=> 𝑥𝑥 = 9 𝑚𝑚𝑚𝑚𝑚𝑚 Given that the tank empties in 4 hours with the
combined effect of both pipes. So
12. Solution (3) Effective rate of emptying =
𝐶𝐶
4
Let efficiency of pipe B be 1 unit.
𝐶𝐶 𝐶𝐶
Efficiency of A = 5 𝑢𝑢𝑢𝑢𝑢𝑢𝑢𝑢𝑢𝑢 => − 180 =
3 4
Total quantity of tank = 25 × 5 = 120 𝑢𝑢𝑢𝑢𝑢𝑢𝑢𝑢𝑢𝑢
120 => 𝐶𝐶 = 2160
Time taken by both the pipes = = 20 𝑚𝑚𝑚𝑚𝑚𝑚 Therefore, the capacity of the tank is 2160 liters.
1+5
16. Solution (1) 20. Solution (2)
1
Tank filling rate of 𝑀𝑀 = Tap A can fill the tank in 20 minutes, it means
15
Time taken with leak = 20 ℎ𝑜𝑜𝑜𝑜𝑜𝑜𝑜𝑜 that it fills 1/20th of the tank in one minute.
1 Now, Tap C empties the tank at 1/3rd the rate of
New, tank filling rate=
20 tap A. Therefore,
1 1 4−3 1
Rate of leakage = ( − )= = tap C empties 1/60th of the tank in one minute
15 20 60 60
Hence, it will taken 60 hours to empty the tank, 1 1
( × ).
3 20
when it is full 1 1 1
the net rate of filling the tank = − =
20 60 30
17. Solution (1) Now, when the tank is 50% full, it means that
Time taken to fill 100% of tank = 24 × =
4 1/2 of the tank has been filled. Therefore, 1/2 of
3
the tank is left to be emptied.
32 𝑚𝑚𝑚𝑚𝑚𝑚
12 3 The time taken to fill 1/2 of the tank
Part of tank filled in 12 min = = 1
32 8
= 2
1 = 15 minutes
30
18. Solution (1)
This means that the tank will be 50% full at
LCM (12,18,24) = 72
Let the quantity of the tank be 72 l 12:15 PM. At this point, tap A is turned off, and
Quantity of tank filled in 1 min by; only tap C is working.
72 rate of emptying the tank =
1
𝑃𝑃 = = 6 𝑙𝑙 60
12
Q=
72
= 4 𝑙𝑙 Therefore,
18
72 time taken to empty the remaining 1/2 of the
R= = 3 𝑙𝑙
24 tank
Tank filled in 5 hours by P and Q = 5(6 + 4) = 1
50 𝑙𝑙
Tank to filled = 72 − 50 = 22𝑙𝑙 = 2 = 30 𝑚𝑚𝑚𝑚𝑚𝑚𝑚𝑚𝑚𝑚𝑚𝑚𝑚𝑚
1
Time taken by all three to fill the remaining tank 60
22 22 So, the tank will be completely empty at:
= = ℎ𝑜𝑜𝑜𝑜𝑜𝑜𝑜𝑜
(6 + 4 + 3) 13 12: 15 𝑃𝑃𝑃𝑃 + 30 𝑚𝑚𝑚𝑚𝑚𝑚𝑚𝑚𝑚𝑚𝑚𝑚𝑚𝑚 = 12: 45 𝑃𝑃𝑃𝑃
22 9
Total time = 5 + = 6( ) Therefore, the tank will be empty at 12:45 PM.
13 13

19. Solution (1) 21. Solution (1)


LCM (30,35,21) = 210 LCM (30,20) = 60
Let the quantity of the tank be 210 l. Let the quantity of tank be 60 l.
Quantity of tank filled in 1 min by; Quantity of tank filled in 1 hour by;
210
P= = 7 𝑙𝑙 A=
60
= 2 𝑙𝑙
30 30
210
Q= = 6 𝑙𝑙 B=
60
= 3 𝑙𝑙
35
20
Quantity of tank emptied by R in 1 min = Let time taken by A to fill the remaining tank be
210
= 10 𝑙𝑙 x min.
21
Let time for which tap R was opened be x min. As per question;
As per question; 10(3 + 2) + 2𝑥𝑥 = 60
7(6 + 7) + 𝑥𝑥(7 + 6 − 10) = 210 => 50 + 2𝑥𝑥 = 60
=> 91 + 3𝑥𝑥 = 210 => 2𝑥𝑥 = 10
=> 3𝑥𝑥 = 119 => 𝑥𝑥 = 5 ℎ𝑜𝑜𝑜𝑜𝑜𝑜𝑜𝑜
=> 𝑥𝑥 = = 39.67 = 40 𝑚𝑚𝑚𝑚𝑚𝑚 (𝑎𝑎𝑎𝑎𝑎𝑎𝑎𝑎𝑎𝑎𝑎𝑎)
22. Solution (4)
1
Tank filling rate = 26. Solution (1)
3
Time taken with leak = 3.5 ℎ𝑜𝑜𝑜𝑜𝑜𝑜𝑜𝑜 LCM (6,4,3) = 12
1 Let the quantity of the tank be 12 l.
New, tank filling rate= Quantity of tank filled in 1 min by;
3.5
1 1 0.5 1 12
Rate of leakage = ( − ) = = A+B = = 2 𝑙𝑙— (1)
3 3.5 10.5 21 6
Hence, it will taken x = 21 hours to empty the B+C =
12
= 3 𝑙𝑙— (2)
tank, when it is full. 4
12
A+C= = 4 𝑙𝑙— (3)
3
23. Solution (2) Adding (1),(2) and (3) and putting the value of
Time taken by both the pipes = y mins (1) in it;
1 1 1 2(𝐴𝐴 + 𝐵𝐵 + 𝐶𝐶) = 9
=> + =
30 15 𝑦𝑦 9
2+1 1 => 𝐴𝐴 + 𝐵𝐵 + 𝐶𝐶 =
=> = 2
30 𝑦𝑦 9 5
=> 𝐶𝐶 = − 2 =
30 2 2
=> 𝑦𝑦 = = 10 𝑚𝑚𝑚𝑚𝑚𝑚𝑚𝑚 12 24
3 Time taken by C, x = 5 = ℎ𝑜𝑜𝑜𝑜𝑜𝑜𝑜𝑜
5
2

24. Solution (1)


If pipe X can fill the tank in 5 minutes, it means 27. Solution (2)
that it fills 1/5th of the tank in one minute. Let the rate at which B fills the tank be 1 l/ min.
Y empties the So, rate of A = 10𝑙𝑙/ 𝑚𝑚𝑚𝑚𝑚𝑚
tank in 10 minutes. Therefore, it empties 1/10th Total quantity of tank = 60 × 1 = 60 𝑙𝑙
of the tank in one minute. 60
Therefore, TIme taken by both to fill the tank = =
1+10
1 1 1 60
net rate of filling the tank = − = ℎ𝑟𝑟𝑟𝑟
5 10 10 11
1
Time taken to fill 1/2th of tank = 2
1 = 5 𝑚𝑚𝑚𝑚𝑚𝑚𝑚𝑚
10 28. Solution (1)
3
amount of the tank that needs to be filled = − Time taken by A to fill 100% of tank =
4
1 1 100
= 27 × = 45 ℎ𝑟𝑟.
2 4 60
1 Time taken by B to fill 100% of tank =
The time taken to fill 1/4th of the tank= 4
= 20
1
22 × = 40 ℎ𝑟𝑟.
10 11
2.5 𝑚𝑚𝑚𝑚𝑚𝑚𝑚𝑚𝑚𝑚𝑚𝑚𝑚𝑚 Time taken by C to empty 100% of tank =
Therefore, 𝑘𝑘 ℎ𝑟𝑟.
tank will be filled to 3/4th of its capacity = Let the quantity of the tank be 360k l.
5 𝑚𝑚𝑚𝑚𝑚𝑚𝑚𝑚 + 2.5 𝑚𝑚𝑚𝑚𝑚𝑚𝑚𝑚 = 7.5 𝑚𝑚𝑚𝑚𝑚𝑚𝑚𝑚 Efficiency A =
360𝑘𝑘
= 8𝑘𝑘
45
360𝑘𝑘
25. Solution (1) Efficiency B = = 9𝑘𝑘
40
LCM (12,14,7) = 84 360𝑘𝑘
Let the quantity of the tank be 84 l. Efficiency C = = - 360
𝑘𝑘
Quantity of tank filled in 1 min by; According to question;
84
P= = 7 𝑙𝑙 25(8𝑘𝑘 + 9𝑘𝑘 − 360) + 20(8𝑘𝑘 − 360) = 360𝑘𝑘
12
84 25(8𝑘𝑘 + 9𝑘𝑘 − 360) + 20(8𝑘𝑘 − 360) = 360𝑘𝑘
Q= = 6 𝑙𝑙
14
84 425𝑘𝑘 + 160𝑘𝑘 − 360𝑘𝑘 = 45 × 360
Quantity of tank emptied by R in 1 min = =
7 225𝑘𝑘 = 45 × 360
12 𝑙𝑙
𝑘𝑘 = 72
Tank filled in 1 mon by P, Q and R = (7 + 6 −
12) = 1 𝑙𝑙 ATP,
1×7 1 𝑥𝑥 14.4
Part of tank filled in 7 mins = = =
84 12 100 72
𝑥𝑥 = 20
29. Solution (4) 30. Solution (3)
LCM (12,18) = 36 LCM(9,6,18)=18
Let the quantity of the tank be 36 l. Let the quantity of the tank be 18 l.
Quantity of tank filled in 1 min by; Quantity of tank filled in 1min by ;
36 18
A= = 3 𝑙𝑙 A+B = = 2 𝑙𝑙— (1)
12 9
36 18
B= = 2 𝑙𝑙 B+C = = 3𝑙𝑙— (2)
18 6
Let the quantity of tank filled by C in 1 min be k C+A =
18
= 1 𝑙𝑙— (3)
18
l.
Adding (1), (2) and (3), and putting (1) in it;
As per question;
=> 2(𝐴𝐴 + 𝐵𝐵 + 𝐶𝐶) = 6
4.5(3 + 2 + 𝑘𝑘) = 36
=> 𝐴𝐴 + 𝐵𝐵 + 𝐶𝐶 = 3
=> 5 + 𝑘𝑘 = 8
=> 2 + 𝐶𝐶 = 3
=> 𝑘𝑘 = 3 𝑙𝑙
=> 𝐶𝐶 = 1 𝑙𝑙
Time taken by C to fill (5/6th) of the tank = 18
5 1 Time taken by C alone = = 18 𝑚𝑚𝑚𝑚𝑚𝑚
36 × ( ) × ( ) = 10 ℎ𝑜𝑜𝑜𝑜𝑜𝑜𝑜𝑜 1
6 3
1. A person is 30 years older than his son . After 6 7. 10 years ago P was half of Q in age . At present
years , the age of that person is four times the age the ratio of their ages is 3 : 4. Find the sum of their
of son that time. What is age of his son after 10 ages after 3 years.
years from now ? (1) 39 (2) 41
(1) 14 (2) 16 (3) 43 (4) 45
(3) 18 (4) 20
(5) none of these
(5) none of these

2. The sum of age of A and B is 38 years. After 8 8. The age of father is 40% more then his son. After
years the age of B is 2 years more than A. What is 5 years the age of father is 16 years more than the
the age of B before 5 years ? age of son. Find the present age of father.
(1) 14 (2) 15 (1) 52 (2) 54
(3) 16 (4) 18 (3) 56 (4) 58
(5) none of these (5) none of these

3. The age of B is 8 years more than A, the age of C 9. The difference between the present age of A and B
is 11 years less than B and the age of A is 5 years
is 20 years. After 3 years their ages are in the ratio
less than D. If the sum of age of A , B ,C and D is
70 years, then find the sum of age of A and B after 9 : 19. Find the present age of B.
2 years ? (1) 31 (2) 33
(1) 35 (2) 32 (3) 35 (4) 37
(3) 38 (4) 39 (5) none of these
(5) none of these
10. The present age of father is 5 times that of his son.
4. The age of son is one-third of his father. If after 5 After 4 years he will be 4 times that of his son.
years the age of father and son becomes 5 : 2, then Find the age of father 3 years ago.
find the sum of age of father and son ? (1) 57 (2) 59
(1) 40 (2) 50
(3) 55 (4) 53
(3) 60 (4) 55
(5) none of these (5) none of these

5. The average present age of A , B and C is 30 years 11. Average age of A and B is 5 years more than the
and the ratio between the present age of A and C is present age of C. The age of D is 3 years less than
4 : 5. If the age of B is 8 years less than the age of the age of C and total age of A, B, C and D is 39
C, then what is age of B after 2 years ? years. Find the sum of age of A and B.
(1) 27 (2) 28 (1) 22 (2) 24
(3) 29 (4) 30 (3) 26 (4) 28
(5) none of these (5) none of these
6. The sum of present age of A , B and C is 52 years
and the age of A is 3/5th of age of B. If after 11 12. Ratio of the present age of harsh and Samar is 5:8.
years the age of B is equal to the three times the After 4 years ratio of their ages is 2:3, then find the
present age of C, then find present of C ? age of Samar after 8 years.
(1) 21 (2) 23 (1) 20 (2) 25
(3) 25 (4) 27 (3) 30 (4) 35
(5) none of these (5) 40
13. The age of Bunty is 25% more than the age of 19. 5 years ago, average age of a family of 6 members
Vikas. The age of Renu is 6 years more than the was 30 years. At present one family member who
age of Vikas and difference between the age of is a girl got married whose age is 30 years. What is
Bunty and Renu is 2 years. What is the age of the average age of remaining 5 members of family
Bunty after 4 years ? after 4 years ?
(1) 42 (2) 44 (1) 20 (2) 25
(3) 46 (4) 48 (3) 30 (4) 15
(5) none of these (5) none of these

14. The average of the present age of mother and his 20. Ratio of present age of P & Q is 4:3 and before 2
son is 50% more than the present age of the son. If years ratio of their age is 3:2. The age of R is 12
5 years hence the sum of age of mother and his son years more than the present age of P, then find the
is 70 years, then find the present age of the mother. age of R.
(1) 30 (2) 40 (1) 20 (2) 25
(3) 50 (4) 45 (3) 30 (4) 15
(5) none of these (5) none of these

15. Average age of B and C is 20 years. The age of A 21. The present age of A and B are 30 and 33 years
is 4 years more than B and total age of A,B and C respectively, after how many years ratio of ages of
is 60 years, then find the age of A before 4 years. B and A becomes 14 : 13 ?
(1) 14 (2) 15 (1) 9 (2) 10
(3) 16 (4) 18 (3) 12 (4) 14
(5) none of these (5) none of these

16. Average of present age of P & Q is 24 years, 22. The ratio of present ages of A and B is 3 : 2 and
average of present age of P & R is 20 years and ratio of present ages of C and B is 5 : 4. If total
average of present of Q & R is 16 years. Find the present age of A, B and C is 150 years then, what
age of R after 8 years. the age of B before 6 years.
(1) 20 (2) 25 (1) 30 (2) 35
(3) 30 (4) 35 (3) 40 (4) 45
(5) none of these (5) none of these

17. Present age of son is 30% of his father age and 23. The ratio of present age of Harsh and Samayra is 4
after 5 years the ratio of age of father and son will : 5. The ratio of age of Harsh after 4 years and
become 5:2. What is the sum of the present age of Samayra before 8 years is 9 : 8, then find the age
father and son ? of Harsh after 14 years.
(1) 37 (2) 39 (1) 42 (2) 44
(3) 41 (4) 43 (3) 46 (4) 48
(5) none of these (5) none of these

18. A is 3 years older than B and after 2 years age of 24. The present age of father is two times more than
A will be 30 % more than that of B. Find the age the present age of son. If after 8 years the ratio of
of A after 5 years. age of father and son is 7 : 3, then find the age of
(1) 14 (2) 15 son after 5 years ?
(3) 16 (4) 17 (1) 20 (2) 21
(5) none of these (3) 22 (4) 23
(5) none of these
25. The age of brother is four times of his sister and 32. 5 years ago, the average age of Charu and riya is
after 18 years the ratio of age of brother and sister 23. At present, the ratio between their ages is
is 2 : 1. What is age of brother after 4 years ? 15:13. Find the Charu age after 10 years?
(1) 40 (2) 30 (1) 20 (2) 30
(3) 20 (4) 25 (3) 40 (4) 50
(5) none of these
(5) none of these
26. A is as much younger to B as he is elder to C and
the sum of the age of B and C is 48 years, then 33. The present age of a Lalit is 100% more than the
what is the age of A ? present age of his son. 10 years ago, the age of
(1) 22 (2) 24 Lalit is 200% more than as that of his son. What
(3) 26 (4) 28 will be the ratio of the ages of Lalit and his son
(5) none of these after 15 years from now?
(1) 11:7 (2) 11:5
27. The ratio of the present age of A and B is 8 : 5. If (3) 13:7 (4) 13:5
the A’s present age is 5 years more than B’s age (5) none of these
after 10 years, then what is the sum of age A and B
after 5 years ?
(1) 60 (2) 65 34. The ratio of the ages of prashant and anjali, 6 years
(3) 70 (4) 75 ago was 8:5 respectively. After 2 years, the age of
(5) 80 anjali will be 25% less than that of prashant. What
is the present age of prashant?
28. The ratio between the present ages of A and B is 5 (1) (2)
: 6. If the ratio between the one – third age of A (3) (4)
and half that of B is 5 : 9, then what is the present (5)
age of A ?
(1) 20 (2) 25 35. Average ages of Ankur, Medhavi and Mamta is 26
(3) 30 (4) can’t be determine
years. If the present age of Medhavi is two times
(5) none of these
of the age of Mamta 6 years ago and the present
29. Two years ago, the age of mother was four times age of A is 12 years more than Medhavi, then what
as old as her daughter. After four years, difference is the present age of Mamta?
between the age of mother and the age of daughter (1) 15 (2) 16
is 21 years. Find the age of mother after 8 years. (3) 17 (4) 18
(1) 35 (2) 36 (5) none of these
(3) 37 (4) 39
(5) 38 36. The sum of the present ages of mother and
daughter is 51 years. Six years ago, mother’s age
30. The ratio between the present ages of P and Q is 4
is 12 times of the age of daughter and the ratio of
: 5. If the ratio between their ages 6 years hence,
becomes 14 : 17. What is Q’s age of present ? the ages of mother to father is 14:15, then find the
(1) 40 (2) 45 difference between the age of Daughter and father.
(3) 50 (4) 55 (1) 35 (2) 36
(5) 60 (3) 42 (4) 40
(5) none of these
31. In a family, a couple has a daughter and son. The
age of the mother is three times that of his son and 37. The average ages of Charu and Riya is 28 years. If
the age of the daughter is 50% of his father. The the ratio of the ages of Charu 12 years hence and
husband is 9 years younger to her wife and the the age of Riya 8 years ago is 3:2, then find the
sister is seven years older than his brother. Find
present age of Charu?
the age of the daughter?
(1) 30 (2) 32 (1) 22 (2) 24
(3) 35 (4) 36 (3) 26 (4) 28
(5) none of these (5) none of these
38. If four years ago, the age of Punit is half of that of 40. Present age of Nitin is 10% more than the present
vikas and after 4 years, the age of Punit is 40% age of Raja. Nitin is 4 years elder than shivansh.
less than that of vikas, then what is the ratio of the Ansh will be 3 years elder than shivansh. Find the
present age of vikas and Punit ?
present age of Ansh, if the average present age of
(1) 9:5 (2) 9:7
(3) 9:11 (4) 9:13 all four person together is 31 years.
(5) none of these (1) 30
(2) 32
39. Ratio of the ages of A and B is 3:4 and the ratio of (3) 34
the ages of B and C is 8:9. If C is 6 years younger (4) 28
than D and D’s age after 10 years is 34 years, then (5) none of these
what is the age of A after 8 years?
(1) 15 (2) 20
(3) 25 (4) 30
(5) none of these
Answer Key
1. (1) 9. (3) 17. (2) 25. (1) 33. (1)
2. (2) 10. (1) 18. (3) 26. (2) 34. (?)
3. (3) 11. (3) 19. (1) 27. (4) 35. (4)
4. (3) 12. (5) 20. (1) 28. (4) 36. (2)
5. (3) 13. (2) 21. (1) 29. (5) 37. (2)
6. (3) 14. (2) 22. (2) 30. (2) 38. (1)
7. (2) 15. (3) 23. (3) 31. (1) 39. (2)
8. (3) 16. (1) 24. (2) 32. (3) 40. (2)
Hints & Solutions
1. Solution (1) 6. Solution (3)
Let the age of his son = x Let the present age of B = 5x
age of person = x + 30 the age of A = 3x
after 6 years , Let the present of C = y years
4 (x + 6) = x + 30 + 6 5x + 11 = 3y
3x = 12
y = 5x + 11/3
x=4
3x + 5x + (5x + 11)/3 = 52
age of son after 10 years = 10 + 4 = 14 years
9x + 15 x + 5x + 11 = 156
2. Solution (2) 29x = 145
Let the age of A = x years x=5
age of B = (38 – x) years the present age of B = 5 * 5 = 25 years
after 8 years,
(38 – x + 8) – (x + 8) = 2 7. Solution (2)
x = 18 10 years ago , let the age of Q = 2x
age of B before 5 years = 38 – x – 5 = 15 years and the age of P = x
at present , the age of P = x + 10
3. Solution (3)
the age of Q = 2x + 10
Let the age of D = x years
the age of A = (x – 5) years x + 10/2x + 10 = 3/4
the age of B = ( x – 5 + 8 ) = x + 3 years 6x + 30 = 4x + 40
the age of C = (x + 3 – 11) = x – 8 years 2x = 10
total sum of age of A , B ,C and D = 70 x=5
x + ( x – 5) + (x +3) + (x -8) = 70 sum of their ages after 3 years = 2x + 13 + x + 13
4x – 10 = 70 = 23 + 18 = 41 years
x = 20 years
The age of A = 20 – 5 = 15 years 8. Solution (3)
The age of B = 20 + 3 = 23 years Let age of son = 5x
sum = 15 + 23 = 38 years
and father age = 5x * 140/100 = 7x
after 5 years,
4. Solution (3)
Let the age of father = 3x (7x + 5) – (5x + 5) = 16
the age of son = x 2x = 16
after 5 years, x=8
3x + 5/x + 5 = 5/2 the age of father = 7x = 7 * 8 = 56 years
6x + 10 = 5x + 25
x = 25 9. Solution (3)
sum = 3x + x = 4x = 4 * 15 = 60 years Let present age of A = x years
present age of B = x + 20 years
5. Solution (3) after 3 years, the age of A = x + 3
Let the A and C 4x and 5x respectively
the age of B = x + 23
the age of B = 5x – 8
total age of A , B and C = 30 * 3 = 90 years x + 3 / x + 23 = 9/19
4x + 5x + 5x – 8 = 90 19x + 57 = 9x + 207
14x = 98 10x = 150
x=7 x = 15
the age of B after 2 years = (5x – 8) + 2 = 29 years the age of B = 15 + 20 = 35 years
10. Solution (1) 15. Solution (3)
Let the age of son = x years average age of B and C=20 years
the age of father = 5x years B+C/2=20
After 4 years, B+C=40
the age of son = x + 4 let the age of B=x
the age of father = 5x + 4 age of C=40-x
4 (x + 4) = 5x + 4 age of A=x+4
4x + 16 = 5x + 4 x+4+x+40-x=60
x = 12 years x+44=60
the age of father 3 years ago = 5 * 12 – 3 = 57 x=16
years age of A before 4 years=x+4-2=16+4-4=16 years

11. Solution (3) 16. Solution (1)


Let the age of C =x years average of present age of P & Q=24
ATQ, A+B/2 =x+5 P+Q/2=24
A+B =2x+10 P+Q=48
age of D =x-3 average of present age of P & R=20
Total age of A, B, C and D =39 years. P+R/2=20
2x+10+x+x-3 =39 P+R=40
4x+7= 39 average of present age of Q & R=16
x= 8years Q+R/2=16
The sum of ages of A and B =2x+10 =26 years Q+R=32
P+Q+P+R+Q+R=48+40+32
12. Solution (5) 2(P+Q+R)=120
Let the present age of harsh and Samar be 5x and P+Q+R=60
8x respectively. P+Q=48
ATQ, 5x+4/8x+4 =2/3 R=60-48=12
16x+8 =15x+12 the age of R after 8 years=12+8=20 years
x=4
age of Samar after 8 years = 8x+8 =40 years. 17. Solution (2)
let present age of father=10x
13. Solution (2) and present age of son=3x
let the age of Bunty =5x after 5 years, 10x+5/3x+5=5/2
and the age of Vikas =4x 20x+10=15x+25
the age of Renu =4x+6 5x=15
difference between the age of Bunty and Renu =2 x=3
years sum=10x+3x=13x=13*3=39 years
5x-(4x+6)=2
5x-4x-6=2 18. Solution (3)
x=8 years let age of B=x
Bunty after 4 years=5x+4=44 years and age of A=x+3
after 2 years, x+3+2=130/100(x+2)
14. Solution (2) x+5=13/10(x+2)
let the present age of mother and son be x and y 10x+50=13x+26
years respectively 3x=24
x+y/2=150/100x x=8
x+y/2=3x/2 age of A after 5 years =x+3+5=8+3+5=16 years
y=2x
also given that, x+5+2x+5=70 19. Solution (1)
3x+10=70 5 years ago, average age of 6 members=30 years
x=20 years at present total age of 6
present age of mother=y=2x=2*20=40 years members=(30*6)+(6*5)=210
one member got married who is a girl whose age is 26. Solution (2)
30 years Total age of B and C = 48
remaining age of 5 members=180 B–A=A–C
after 4 years total age of 5 members 2A = B + C
=180+(5*4)=100 2A = 48
average age of 5 members after 4 years=100/5=20 A = 24 years.
years
27. Solution (4)
20. Solution (1) Let present age of A and B are 8x and 5x
let present age of P & Q be 4x and 3x respectively respectively.
before 2 years, 4x-2/3x-2=3/2
8x – (5x + 10) = 5
8x-4=9x-6
x=2 8x – 5x – 10 = 5
present age pf P=4x=4*2=8 3x = 15
present age of R=8+12=20 years x=5
age of A after 5 years = 8x + 5 = 45
21. Solution (1) age of B after 5 years = 5x + 5 = 30
Let after x years ratio of ages of A and B becomes sum = 45 + 30 = 75 years.
13 : 14
(30 + x)/33 + x = 13/14 28. Solution (4)
420 + 14x = 429 + 13x Let the present age of A and B are 5x and 6x
x = 9 years. respectively.
(5x/3)/(6x/2) = 5/9
22. Solution (2) 5x/3 * 9 = 6x/2 * 5
Ratio of age A and B = 3 : 2 Here, we can not calculate the present age of A.
ratio of B and C = 4 : 5
A:B:C=6:4:5 29. Solution (5)
Let total ages of A, B and C = 15x Two years ago let age of daughter = x
15x = 150 age of mother = 4x
x = 10 present age of mother and daughter = (4x + 2), (x
present age of B = 10 * 4 = 40 years. + 2)
before 5 years age of B = 40 – 5 = 35 years. After 4 years, the age of mother = 4x + 2 + 4 = 4x
+6
23. Solution (3) the age of daughter = x + 2 + 4 = x + 6
let the age of harsh and samayra be 4x and 5x
respectively difference = 21
(4x+4)/(5x-8) = 9/8 4x + 6 – (x + 6) = 21
45x – 72 = 32x + 32 4x + 6 – x – 6 = 21
13x = 104 3x = 21
X=8 x=7
The age of Harsh after 14 years = 4x + 14 = 46 yrs the age of mother after 8 years = 4x + 2 + 8 = 28 +
10 = 38 years.
24. Solution (2)
Let present age of son = x 30. Solution (2)
present age of father = 3x Let present age of P and Q are 4x and 5x
3x + 8/ x + 8 = 7/3 respectively.
9x + 24 = 7x + 56 4x + 6/5x + 6 = 14/17
x = 16 70x + 84 = 68x + 102
age of son after 5 years = 16 + 5 = 21 years. 2x = 18
25. Solution (1) x=9
Let the age of sister = x Q’s present age = 5x = 45 years.
the age of brother = 4x
4x + 18/x + 18 = 2/1 31. Solution (1)
4x + 18 = 2x + 36 Let the age of the father=x years.
2x = 18 So, The age of mother is = (x + 9) years
x=9 The age of daughter is = x/2years
The age of brother after 4 years = 4x + 4 = 40 years.
The age of son is = (x/2 – 7) years
Now according to the question, 36. Solution (2)
= 3 (x/2 – 7) = (x + 9) M + D = 51-----(1)
= (3x – 42)/2 = x + 9 M – 6 = 12 * (D – 6)
M – 6 = 12D – 72
= 3x – 42 = 2x + 18
12D – M = 66-----(2)
= 3x – 2x = 42 + 18 13D = 117
= x = 60 years D=9
So, age of daughter = 60/2 = 30 years M = 51 – 9 = 42
F = 42 * 15/14 = 45
32. Solution (3) Difference = 45 – 9 = 36 years
5 years ago, their average age is 23 37. Solution (2)
Charu + Riya = 28 * 2 = 56------(1)
therefore total age = 23*2 = 46
(Charu + 12)/(Riya – 8) = 3/2
present age of Charu and riya = 46+5+5 = 56 3Charu – 24 = 2Riya + 24
Charu age = 15/28*56 = 30 3Riya – 2Charu = 48-----(2)
after 10 years = 30+10 = 40 years From (1) and (2)
Riya = 32
33. Solution (1) Charu = 56 – 32 = 24
Present age of son = x
38. Solution (1)
Present age of Lalit = 200/100 * x = 2x
P – 4 = 1/2 * (v – 4)
(x – 10)/(2x – 10) = 100/300 2P – 8 = v – 4
2x – 10 = 3x – 30 2P – v = 4 ----(1)
x = 20 (v + 4) * 60/100 = (P + 4)
Present age of Lalit = 2 * 20 = 40 3v + 12 = 5P + 20
Required ratio = (40 + 15):(20 + 15) 3v – 5P = 8 -----(2)
From (1) and (2)
= 55:35
v = 36
= 11:7 P = 20
Required ratio = 36:20
34. (?) = 9:5
6 years ago, prashant = 8x and anjali = 5x
(5x + 8) = (100 – 25)/100 * (8x + 8) 39. Solution (2)
=> x = 2 D’s present age = 34 – 10 = 24
C = 24 – 6 = 18
Present age of prashant = 8x + 6 = 22 years
B = 18 * 8/9 = 16
A = 3/4 * 16 = 12
35. Solution (4) A’s age after 8 years = 12 + 8 = 20
Ankur + Medhavi + Mamta = 26 * 3 = 78
Medhavi = 2 * (Mamta - 6) 40. Solution (2)
Ankur – Medhavi = 12 Age of Raja = x
Ankur = 12 + 2Mamta - 12 Nitin = 1.1x
Shivansh = 1.1x – 4
Mamta + 2Mamta – 12 + 2Mamta = 78
Ansh = 1.1x – 4 + 3 = 1.1x – 1
5 Mamta = 90 x + 1.1x + 1.1x – 4 + 1.1x – 1 = 31 * 4
Mamta = 18 years => x = 30
Ansh = 1.1x – 1 = 32
1. Average marks obtained in History by 45 6. Sambha travelled (1 / 3)rd of distance from
students in ‘A’ divisions are 75 and the average Delhi to Jaipur at speed of 80 kmph, next (1 /
marks obtained by 40 students in ‘B’ division 3)rd at speed of 90 kmph and the last (1 / 3)rd
are 78. What are the overall average marks distance at speed of 100 kmph, the average
obtained in History by the students in both the speed of Sambha for the entire journey is
divisions (rounded off to two digits after (1) 90 (2) 89.25
decimal) ? (3) 90.16 (4) 70.23
(1) 76.81 (2) 77.11 (5) None of these
(3) 77.41 (4) 76.41
(5) None of these 7. The average salary of Shilpa, Bipasha and Rani
is Rs. 35000 per month. Also the average salary
2. If the average marks of three batches of 55, 60 of Kajol and Raveena is Rs. 38000 per month.
and 45 students respectively are 50, 55 and 60, What is the average salary per month of all the
then the average marks of all the students is five taken together?
(1) 54.68 (2) 53.33 (1) 36200 (2) 37400
(3) 55 (4) 45 (3) 34700 (4) 39200
(5) None of these (5) 31800

8. The average daily wages of Mayawati from


3. The average age of Jai’s family consisting of 4
Monday to Friday is Rs. 450. If she earns Rs.
members, 4 years ago, was 28 years. 2 years
300 on Monday and Rs. 400 on Saturday, then
ago, a baby was born in the family. The average
find the average daily wages from Tuesday to
age of the family 2 years from now would be :
Saturday
(1) 226 (2) 24
(1) 420 (2) 465
(3) 28 (4) 31
(3) 410 (4) 470
(5) None of these (5) 490

4. The average age of a man and his son is 48 9. The average weight of 8 persons increases by
years. The ratio of their age is 11: 5 respectively. 2.5 kg when a new person comes in place of one
What will be the ratio of their age after 6 years? of them weighing 65 kg. What might be the
(1) 6 : 5 (2) 5 : 3 weight of the new person ?
(3) 4 : 3 (4) 2 : 1 (1) 65 (2) 85
(5) 6 : 1 (3) 76.5 (4) Data inadequate
(5) None of these
5. Veeru goes from A to B at 60 kmph and returns
to A at speed of 50 kmph. Find the average 10. The average age of five officers in a department
speed of Veeru for the entire journey if the is 32 years. If the age of their supervisor is
distance AB = 120 km. added, the average age increased by 1. What is
(1) 54.64 (2) 54.64 the supervisor’s age?
(3) 55 (4) 65 (1) 32 (2) 48
(5) None of these (3) 38 (4) 42
(5) None of these
11. Three different qualities of wheat flour – A, B& (1) 4 (2) 2
C are mixed together in the ratio of 7 : 5 : 4. The (3) 5 (4) 6
price of A, B and C are respectively Rs. 35, Rs. (5) None of these
40 Rs. 42 per kg. Find the average price of the
new mixture? 17. In a group of 5 friends, the sums of ages (in
(1) 38.31 (2) 42.50 years) of each group of 4 of them are 124, 128,
(3) 36.15 (4) 40.5 130, 136 and 142. The age (in years) of the
(5) None of these youngest of them is?
(1) 18 (2) 21
12. The average expenditure of P, Q,& R is Rs. (3) 23 (4) 27
5000 per month. Also, the average expenditure (5) None of these
of Q, R& S is Rs. 7000 per month. If the average
expenditure of S is thrice of that of P then the 18. The mean of 15 different natural numbers is 13.
average expenditure of Q and R is ? The maximum value of the second largest of
(1) 6000 (2) 3000 these numbers is?
(3) 5000 (4) 4000 (1) 53 (2) 52
(5) None of these (3) 51 (4) 50
(5) None of these
13. The average of four consecutive odd numbers P,
Q, R and S respectively (in increasing order) is 19. The average weight of M, N and O is equal to 65
104. What is the sum of P and S ? kg. The average weight of M and N is equal to
(1) 204 (2) 208 69.5 kg. The average weight of N and O is 66.5
(3) 206 (4) 212 kg. What is the weight of N ?
(5) None of these (1) 74 (2) 75
(3) 77 (4) 80
14. Average weight of 19 men is 74 kg, and the (5) None of these
average weight of 38 women is 63 kg. What is
the average weight (rounded off to the nearest 20. The present age of Romila is one fourth of that
integer) of all the men and the women together? of her father. After 6 years the father’s age will
(1) 59 (2) 65 be twice the age of Kapil. If Kapil celebrated
(3) 69 (4) 67 fifth birth day 8 years ago, What is Romil’s
(5) 71 present age?
(1) 7 (2) 7.5
15. The average weight of 8 persons increases by (3) 8 (4) 8.5
2.5 kg when a new person comes in place of one (5) None of these
of them weighing 65 kg. What might be the
weight of the new person ? 21. The average of 9 persons age is 63. The average
(1) 76 (2) 76.5 of 3 of them is 58, while the average of the other
(3) 85 (4) Data inadequate 3 is 60. What is the average of the remaining 3
(5) None of these numbers?
(1) 65
16. The average age of a husband and wife, who (2) 71
were married 7 years ago, was 25 years at the (3) 83
time of their marriage. Now the average age of (4) 92
the family, including husband, wife and a child (5) None of these
(born during the interval) is 22 years, what is the
present age of the child?
22. A museum has an average of 820 viewers on (1) 37 (2) 39
Saturdays and 460 on other days. The average (3) 40 (4) 41
no. of viewers per day in a month of 30 days (5) None of these
beginning with a Saturday is?
(1) 520 (2) 450 28. In a certain camp, there are 30 girls whose
(3) 640 (4) 760 average age is decreased by 6 months, when one
(5) None of these girl aged 27 yrs is replaced by a new girl. Find
the age of the new girl?
23. In Santhosh opinion, his weight is greater than (1) 10 (2) 16
54kg but less than 63 kg. His brother does not (3) 18 (4) 12
agree with Santhosh and he thinks Santhosh’s (5) None of these
weight is greater than 50kg but less than 60kg.
His father’s view is that his weight cannot be 29. The average weight of males in an office is 63
greater than 57kg. If all of them are correct in kg and that of females is 52 kg. Find the average
their estimation, what is the average of different weight of all the person in an office?
portable weights of Santhosh? (1) 55 (2) 64
(1) 55 (2) 55.5 (3) 58 (4) CND
(3) 56 (4) 54.5 (5) None of these
(5) None of these
30. A batsman has completed 15 innings and his
24. The average age of 6 consecutive even numbers average is 22 runs. How many runs must he
is 41. Find the largest of these numbers? make in his next innings so as to raise his
(1) 48 (2) 52 average to 26?
(3) 46 (4) 50 (1) 42 (2) 36
(5) None of these (3) 86 (4) 64
(5) None of these
25. If the average of 5 observations x, x+1, x+2, x+3
and x+4 is 24, then the average of last 2 31. If average weight of a family is ‘y’ kg. If a
observations is? guest weighing 30 kg arrives then the average
(1) 22 (2) 27.5 weight is increases by 1 kg. If the weight of this
(3) 24 (4) 25.5 guest had been 18 kg then the average weight of
(5) None of these the family would have decreased by 1 kg. Find
‘y’.
26. 3 years ago the average age of Rajesh and (1) 28 (2) 24
Prasanth was 21 years. Then Gokul join with (3) 30 (4) 22
them, the average age becomes 27 years. How (5) None of these
old is Gokul now?
(1) 33 (2) 25 32. The average monthly expenditure of a family for
(3) 48 (4) 54 the first four months is Rs. 2570, for the next
(5) None of these three months Rs. 2490 and for the last five
months Rs. 3030. If the family saves Rs. 5320
27. 8 candidates are to be divided into 2 groups P & during the whole year, the average monthly
Q of 5 & 3 candidates. The average percent income of the family during the years is
mark obtained by the candidate of group P is 32 (1) Rs. 3000 (2) Rs. 3185
and the average percent marks of all the 8 (3) Rs. 3200 (4) Rs. 3580
candidate is 35. What is the average percentage (5) None of these
of marks of candidate of group Q?
33. A man spends Rs. 1800 monthly on an average 37. The average age of 12 person is 32 years. If the
for the first four months and Rs. 2000 monthly age of one person is added, the average
for the next eight months and saves Rs. 5600 a decreases by one year. What is the age of the
year. His average monthly income is new person ?
(1) Rs. 2000 (2) Rs. 2200 (1) 19 years (2) 23 years
(3) Rs. 2400 (4) Rs. 2600 (3) 25 years (4) 21 years
(5) None of these (5) None of these

34. The average age of A and B is 20 years. If A is 38. The average weight of a class of 25 students is
to be replaced by C, the average would be 19 37 kg. When the weight of the teacher is also
years. The average age of C and A is 21 years. included, the average weight increases by 1 kg.
The ages of A, B and C in order (in years) What is the weight of the teacher
(1) 18, 22, 20 (2) 18, 20, 22 (1) 65 kg (2) 63 kg
(3) 22, 18, 20 (4) 22, 20, 18 (3) 54 kg (4) 58 kg
(5) None of these (5) None of these

35. The mean high temperature of the first four days 39. The average of 5 numbers is 306.4. The average
of a week is 25°C where as the mean the last of the first two numbers is 431 and the average
four days is 25.5°C. If the man of the whole of the last two numbers is 214.5. What is the
week is 25.2°C then the temperature of the 4th third number ?
days is (1) 108 (2) 52
(1) 25°C (2) 25.2°C (3) 321 (4) Can't be determined
(3) 25.5°C (4) 25.6°C (5) None of these
(5) None of these
40. Out of three numbers, the first is twice the
36. The average marks obtained by a student in 6 second and is half of the third. If the average of
subjects is 88. On subsequent verification it was the three numbers is 56, then difference of the
found that the marks obtained by him in a first and third number is
subject was wrongly copied as 86 instead of 68. (1) 12 (2) 36
The correct average of the marks obtained by (3) 24 (4) 48
him is (5) None of these
(1) 86 (2) 87
(3) 85 (4) 84
(5) None of these
Answer Key
1. (4) 9. (2) 17. (3) 25. (4) 33. (3)
2. (1) 10. (3) 18. (3) 26. (1) 34. (3)
3. (3) 11. (1) 19. (3) 27. (3) 35. (4)
4. (4) 12. (1) 20. (3) 28. (4) 36. (3)
5. (2) 13. (2) 21. (2) 29. (4) 37. (1)
6. (2) 14. (4) 22. (1) 30. (3) 38. (2)
7. (1) 15. (3) 23. (3) 31. (2) 39. (5)
8. (4) 16. (2) 24. (3) 32. (2) 40. (4)
Hints & Solutions
1. Solution (4) 7. Solution (1)
Required average marks = (45 × 75 + 40 × 78) / Average Salary of all 5 = (35000 × 3 + 38000 × 2)
(45 + 40) /5
= (3375 + 3120) / 85 =(105000 + 76000) / 5 = Rs. 36, 200
= 6495 / 85 = 76.41
8. Solution (4)
2. Solution (1) Total wages from Monday to Friday = 450 × 5 =
The required average marks = (50 × 50 + 60 × 55 2250
+ 45 × 60) / (55 + 60 + 45) Total wages from Tuesday to Saturday = 2250 –
= (2750 + 3300 + 2700) / 160 = 8750 / 160 = 300 + 400 = 2350
54.68 Average from Tuesday to Saturday = 2350 / 5 =
Rs. 470
3. Solution (3)
4 years ago, total age of 4 members = 4 × 28 = 112 9. Solution (2)
years Since, the average weight increases by 2.5 kg,
= Total Age of the family just before baby is born hence sum of the weight would increases by 2.5 ×
= 112 + 4 × 2 = 120 years 8 = 20 kg
Total Present age of the family = 120 + 5 × 2 = So, weight of new person should be 20 kg more
130 than the one who is replaced
2 years hence the total age of the family = 130 + (5 Weight of new person = 65 + 20 = 85 kg
× 2) = 140
Average age of the family two years hence = 140 / 10. Solution (3)
5 = 28 years The new average is increased by 1 year, hence
sum of the age would have increased by 6 years.
4. Solution (4) Age of supervisor = 32 + 6 = 38 years
Sum of the age of man& son = 48 × 2 = 96
Given Ratio = 11 : 5 (i.e) age of man = (11 / 16) × 11. Solution (1)
96 = 66 years& Let the Quantity of A, B& C be 7x, 5x, and 4x. So,
That of son = 6 × 5 = 30 years Average Price of Mixture = [7x(35) + 5x(40) +
Ratio of their age 6 years hence would be (66 + 6) 4x(42) ] / (7x + 5x + 4x)
: (30 + 6) = 72 : 36 = 2 : 1 = (245x + 200x + 168x) / 16x = 613 / 16 = Rs.
38.31 per kg
5. Solution (2)
Average Speed = (2xy) / (x + y) [Since distance 12. Solution (1)
travelled is same) Total expenditure of P, Q& R = 3 × 5000 = 15000
= (2 × 60 × 50) / (60 + 50) = 6000 / 110 = 54.54 Total expenditure of Q, R& S = 3 × 7000 = 21000
kmph S – P = 21000 – 15000 = 6000
Also, we are given S = 3P.
6. Solution (2) So 3P – P = 6000& P = 3000
Since distance travelled at different speeds is Total expenditure of Q& R = 15000 – 3000 =
same, we can use the shortcut 12000
Average Speed = (3xyz) / (xy + yz + zx) Average expenditure of Q& R = 12000 / 2 = Rs.
6000
13. Solution (2) 18. Solution (3)
Sum of four consecutive odd numbers = 104 × 4 = Sum of 15 numbers = 15 × 13 = 195
416 For maximum value, the other 13 numbers will be
Let P, Q, R and S be 2n – 3, 2n – 1, 2n + 1, 2n + 3 least.
respectively 1 + 2 + ……. + 13
2n – 3 + 2n – 1 + 2n + 1+ 2n + 3 = 416 = (13 × 14) / 2 = 91
8n = 416 Sum of last two numbers = 195 – 91 = 104
n = 52 Numbers = 51 and 53
Hence, P = 2 × 52 – 3 = 10 4 – 3 = 101& S = 2n +
3 19. Solution (3)
= 2 × 52 + 3 = 104 + 3 = 107. Required sum = 101 M + N + O = 3 × 65 = 195 kg
+ 107 = 208 M + N = 2 × 69.5 = 139 kg
N + O = 2 × 66.5 = 133 kg
Weight of N = (139 + 133 – 195 ) kg = 77 kg
14. Solution (4)
Required average weight = (19 × 74 + 38 × 63) / 20. Solution (3)
(19 + 38) Kapil’s present age = 5 + 8 = 13 years
=(1406 + 2394) / 57 After 6 years, Kapil’s age = 19 years
= 3800 / 57 = 66.66 = 67 kg Father’s present age = (19 × 2) – 6 = 32 years
Romila’s present age = (1 / 4) × 32 = 8 years
15. Solution (3)
Weight of new person = 65 + 8 × 2.5 = 65 + 20 = 21. Solution (2)
85 kg Sum of the remaining 3 numbers = [(9*63) –
[(3*58)+(3*60)]]
16. Solution (2) =567 – (174+180)
7 years ago, Husband + wife =567 – 354=213
2 × 25 = 50 years Required average = 213 / 3=71
Sum of present ages of husband and wife = 50 + 22. Solution (1)
14 = 64 years Since the month begins with a Saturday,
Sum of present ages of husband, wife and child = So there will be 5 Saturday in the month.
22 × 3 = 66 years Required average = ((820*5) + (460*25) / 30)
Present age of child = 66 – 64 = 2 years = (4100+11500)/30
=15600 / 30
17. Solution (3) = 520
a + b + c + d = 124 The average no. of viewers per day in a month is
b + c + d + e = 128 520
c + d + e + a = 130
d + e + a + b = 136 23. Solution (3)
e + a + b + c = 142 Let Santhosh’s weight be y kg
on adding, 4(a + b + c + d + e) = 660 According to Santhosh 54<y<63
a + b + c + d + e = 165 According to Santhosh brother 50<y<60
Age of the youngest of them = d = 165 – 142 = 23 According to Santhosh father y<57
years The value satisfying all the above conditions are
55, 56 and 57
Required average = 56 kg
24. Solution (3) = 27 – 15
Let the numbers be x, x+2, x+4, x+6, x+8 and = 12 years
x+10 Hence the age of the new girl is 12 years
Then, (x + x+2 + x+4 + x+6 + x+8 + x+10) / 6 =
41 29. Solution (4)
6x + 30 = 246 The number of males or females is not mentioned
6x = 246 – 30 in the question. The data is inadequate to answer
6x = 216 the question. So it can’t be determined
X = 216 / 6
X= 36 30. Solution (3)
Therefore largest number = x+10 = 36+10 Total runs of 15 innings = 22*15=330 runs
Largest number = 46 Suppose he needs a score of x in 16th innings,
Then average in 16th innings = (330+x) /16
25. Solution (4) (330+x) /16 = 26
We have (x+x+1+x+2+x+3+X+4) / 5 = 24 330+x = 16*26
5x +10 = 24*5 330+x = 416
5x +10 = 120 X = 416-330
5x = 110 X = 86
X = 110 / 5 He needs a score of 86 runs in 16th innings
X= 22
So the numbers are 22, 23, 24, 25 & 26 31. Solution (2)
Required average = (25+26) /2 Let the number of family members be x.
=51/2 Therefore, the total weight = xy
=25.5 Equation in the 1st scenario:
Therefore average of last 2 observations is 25.5 (x + 1) (y + 1) = xy + 30
xy + x + y + 1 = xy + 30
26. Solution (1) x + y = 29 .....(i)
Present age of Rajesh & Prasanth = (21*2 + 3*2) Equation in the 2nd scenario:
=42+6 (x + 1) (y - 1) = xy + 18
=48 yrs xy + y - x - 1 = xy + 18
Present age of Rajesh, Prasanth & Gokul = 27*3 y - x = 19 ....(ii)
=81 yrs Solving equations (i) and (ii), we get
Gokul’s age = 81 – 48 = 33 yrs 2y = 48, Therefore, y = 24.
Hence, option 2 is correct.
27. Solution (3)
Required average = [(35*8) – (32*5)]/ 3 32. Solution (2)
= [280 – 160]/ 3 Total annual income = Expenditure + Savings
= 120/ 3 = (4 × 2570 + 3 × 2490 + 5 × 3030) + 5320
= 40 = (10280 + 7470 + 15150) + 5320
The average percent marks of candidates of group = 32900 + 5320 = 38220
Q is 40. 38220
∴ Required average monthly income =
12
28. Solution (4) = Rs. 3185
Average age of new person= age of removed Hence, option 2 is correct
person – no of persons*decrease in age
Average age of new person =27 – 30*(6/12)
= 27 – 30(1/2)
33. Solution (3)  n +1
th

Total annual income = Expenditure + Savings Value of   result =


 2 
= (4 × 1800 + 8 × 2000) + 5600
 7 +1
= (7200 + 16000) + 5600   × ( 25 + 25.5 ) − 7 × 25.2
= 23200 + 5600 = Rs. 28800  2 
28800 8
∴ Required average monthly income = =   × ( 50.5 ) − 176.4 ⇒ 202 – 176.4 = 25.6
12 2
= Rs. 2400 Traditional method:
Hence, option 3 is correct. Total average of first 4 days = 4 × 25 = 100
Total average of last 4 days = 4 × 25.5 = 102
34. Solution (3) Total average of 7 days = 7 × 25.2 = 176.4
A + B = 2 × 20 = 40 yr Temperature 4th day's = 100 + 102 – 176.4 = 25.6
B + C = 2 × 19 = 38 yr Hence, option 4 is correct.

C + A = 2 × 21 = 42 yr
36. Solution (3)
On adding all three, Total number of marks = 88 × 6 = 528
2 (A + B + C) = 40 + 38 + 42 = 120 Now, 528 – 86 + 68 = 510
510
⇒ A + B + C = 60 Required average = = 85
6
∴ A = (A + B + C) – (B + C) = 60 – 38 = 22 yr Hence, option 3 is correct.
Similarly,
B = (A + B) – A = 40 – 22 = 18 yr 37. Solution (1)
Total age of 12 persons = 12 × 32 = 384 years
C = (C + A) – A = 42 – 22 = 20 yr Given that if the age of one person is added, the
Note: In this question we can save 4-5 seconds by average decreases by one year
not calculating the age of the third person as with ∴ Total age of 13 persons = 13 × 31 = 403 years
Now, age of new person = 403 – 384 = 19 years
only the respective ages of A and B we can
Hence, option (1) is correct.
confirm the correct answer out of the given
options. 38. Solution (2)
Total weight of 25 students = 37 × 25 = 925 kg
Hence, option 3 is correct.
Given that when the weight of the teacher is also
included, the average weight increases by 1 kg
35. Solution (4) ∴ Total weight along with the teacher = 38 × 26 =
To solve this question we can apply a short trick 988 kg
approach Now, the weight of the teacher = 988 – 925 = 63
th
 n +1 kg
Value of   result =
 2  Hence, option (2) is correct.
 n +1
  × ( b + c) − n × a 39. Solution (5)
 2 
Third number = Total of five number – (Total of
Where
first two numbers + Total of last two numbers)
n is the total number of term = 7 days
= 5 × 306.4 – (2 × 431 + 2 × 214.5)
b is the average of first four terms = 25
= 1532 – (862 + 429)
c is the average of last four terms = 25.5
= 1532 – 1291 = 241
a is the average of whole terms = 25.2
Hence, option (5) is correct.
By the short trick approach, we get
40. Solution (4)
Let the numbers are 2x, x, 4x, then, ⇒ 96 – 48 = 48.
Total of the numbers = 3 × 56 = 168 Hence, option 4 is correct.
⇒ 2x + x + 4x = 168
⇒ 7x = 168 ⇒ x = 24
∴ Required difference = 4x – 2x = (4 × 24) – (2
× 24)
1. A and B started the business with the investment 5. A, B and C started the business with the
of Rs.(x + 2000) and Rs.(2x + 8000) respectively. investment of Rs.1000, Rs.1500 and Rs.1200
At the end of the year, the total profit is Rs.22000 respectively. After 4 months, A added Rs.200 and
and the profit share of B is Rs.16500, then find B withdrew Rs.400. After 4 more months, C
the value of x? withdrew Rs.100. At the end of year the total
(1) 1800 (2) 2000 profit is Rs.5300, find the profit share of C?
(3) 2200 (4) 2500 (1) 1450 (2) 1550
(5) None of these (3) 1650 (4) 1750
(5) None of these
2. A and B started the business with the investment
of Rs.5000 and Rs.9000 respectively. After 5 6. A started the business with the investment of
months, A added Rs.1000 and B withdrew Rs.4500 and after 4 months B joined with the
Rs.1000 for his initial investment. At the end of business. At the end of the year the total profit is
the year, the total profit of the business is Rs.17000 and A’s profit share is Rs.9000, what is
Rs.8400, then what is the difference between the B’s investment?
profit share of A and B? (1) 4000 (2) 5000
(1) 1700 (2) 1500 (3) 6000 (4) 8000
(3) 1200 (4) 2000 (5) None of these
(5) None of these
7. A and B started the business with the investment
3. A, B and C started the business with the of Rs.4000 and Rs.6000 respectively. After 6
investment is in the ratio of 3: 8: 1. Ratio of the months B left and C joined with the investment of
investment period of A, B and C is 1: 2: 3. If A’s Rs.4500 and at the end of year the total profit of
profit share at the end of business is Rs.1800, then business is Rs.18500. What is the profit share of
find the total profit of the business? C?
(1) 14300 (2) 13200 (1) 9000 (2) 7200
(3) 12100 (4) 15400 (3) 6300 (4) 5400
(5) None of these (5) 4500

4. A started the business with the investment of 8. P started a business with a capital of Rs.80000. 1
Rs.4500. After 4 months, B and C joined with the year later, Q joined him with a capital of
investment of Rs.5000 and Rs.4800 respectively. Rs.10000. At the end of 3 years from the start of
At the end of the year, the total profit of the business, the total profit earned was Rs.39000.
business is Rs.16550, then find the profit share of The share of P in the profit exceeded the share of
C? Q by?
(1) 5000 (2) 3200 (1) 30000 (2) 32000
(3) 4800 (4) 3600 (3) 33000 (4) 36000
(5) None of these (5) None of these
9. Remo and Saran invested the same capital in a 14. A and B started a business with investment of
business. At the year end, they share the profit in Rs.x and Rs.(x+400) respectively. After 6
the ratio of 3: 4. If Remo has invested his capital months, C joined with them with an investment
for the whole year, for how many months Saran of Rs.(x+1400). If at the end of the year, the ratio
has invested his capital? of profit of A and B is 7:8, and then find the
(1) 4 (2) 8 investment of C?
(3) 12 (4) 16 (1) 2800 (2) 3200
(5) None of these (3) 5600 (4) 4200
(5) None of these
10. P, Q and R invested capital is in the ratio of 4: 5:
6. At the end of the business term, they received 15. A, B and C started a business with the investment
the profits in the ratio 7: 4: 6. Find the ratio of of Rs.3500, Rs.2100 and Rs.4000 respectively.
time for which they contributed their capital? After x months, C left the business. At the end of
(1) 35:16:20 (2) 16:35:20 the year, the profit of A is Rs.3000 out of the total
(3) 20:35:16 (4) 20:16:35 profit of Rs.6800, then find the value of x?
(5) None of these (1) 5 (2) 4
(3) 7 (4) 8
11. A, B and C started a business and invested (5) None of these
Rs.6000, Rs.3200 and Rs.4800 respectively.
After 8 months, B added Rs.1600 and C withdrew 16 A started a business with an investment of
Rs.400. At the end of the year, the total profit of Rs.3500. After x months, B joined the business
the business is Rs.2160, then find the profit share with the investment of Rs.4800. At the end of the
of C? year, the profit share of A and B is 7:4
(1) 500 (2) 900 respectively, then find the value of (12-x)?
(3) 700 (4) 800 (1) 9 (2) 3
(5) None of these (3) 7 (4) 5
(5) None of these
12. Bala and Pugal started a business with the
investment in the ratio of 8:7. If the investment 17. A, B and C started a business with the investment
period of Bala and Pugal is 15 months and x in the ratio of (4/9):(4/7):(1/5). After 4 months, A
months respectively and at the end of business, and B added 25% and 20% respectively of their
the profit share of Pugal is 40% more than the initial investment and the end of the year, the total
profit share of Bala, then find the investment time profit of the business is Rs.38730, then find the
period of Pugal? profit share of B?
(1) 18 (2) 12 (1) 15720 (2) 16840
(3) 24 (4) 20 (3) 18350 (4) 19540
(5) None of these (5) 20120

13. A, B and C started a business with the investment 18. A and B started a business with the investment of
of Rs.x, Rs.5400 and Rs.4800 respectively. At the Rs.12000 and Rs.13000 respectively. After 7
end of the year, the total profit of the business is months, A withdrew 6(2/3)% of his initial
investment and C entered the business with the
Rs.11500 and the profit share of C is Rs.4000,
investment of Rs.x. At the end of the year, the
then find the profit share of A? total profit of the business is Rs.22000 and the
(1) 4500 (2) 2700 profit share of B is Rs.7800. Find the value of x?
(3) 3000 (4) 1800 (1) 14000 (2) 24000
(5) None of these (3) 18000 (4) 22000
(5) None of these
19. A, B and C started the business with the 24. X, Y and Z started a business. Ratio of investment
investment of Rs.(x + 1000), Rs.(x + 2000) and of X and Y is 5 : a while ratio of investment of Y
Rs.x respectively. After one year, A withdrew and Z is 7 : 10. X, Y and Z invested for 8 months,
Rs.3000 and B withdrew Rs.2000 and C added 6 months and 12 months respectively. Then find
Rs.5000. At the end of two years, the total profit the value of a, if out of total profit of Rs. 3130, in
of the business is Rs.4800 and the profit share of which X shared a profit of Rs. 700?
A is Rs.1200. Find the value of x? (1) 4 (2) 6
(1) 5000 (2) 5500 (3) 8 (4) 9
(3) 6000 (4) 6500 (5) None of these
(5) 4000
25. A and B started a partnership business with the
20. A started the business with the investment of Rs.x investment of Rs.2400 and Rs.4200 respectively
and after 4 months B joined the business with the and after 6 months, C joined with the investment
investment of Rs.(x + 400). At the end of the year, of Rs.3800. At the end of the year, the total profit
the total profit of the business is Rs.1410 and the of the business is Rs.21250, what is the profit
profit share of B is Rs.600, then find the initial share of C?
investment of A? (1) 4750 (2) 5250
(1) 3200 (2) 3000 (3) 6300 (4) 7250
(3) 3600 (4) 4000 (5) None of these
(5) 4200
26. Mithun and Nirmal started a business with the
21. Pavan started a business by investing Rs.9000 investment of Rs.8000 and Rs.12000
and after 4 months, Hari joined him. If the annual respectively. After 6 months, Divya joined with
profit is divided in the ratio of 3: 4, then find the the investment of Rs.14000. At the end of the
amount invested by Hari. year, the total profit of the business is Rs.20250,
(1) 12000 (2) 14000 find the profit share of Mithun?
(3) 16000 (4) 18000 (1) 8000 (2) 10000
(5) None of these (3) 5000 (4) 6000
(5) 4000
22. P and Q entered into a partnership business. P
invested Rs.7000 and Q invested Rs.9400. After 27. A, B and C started a business with an investment
5 months, R entered with the amount of Rs.8600. is in the ratio of 2:3:6. After one year, A, B and C
If Q withdrew all his investment after 7 months, added the addition investments of Rs.4800,
Rs.3900 and Rs.5600 respectively. At the end of
find the profit of R at the end of the year, if the
2 years, the total profit of the business is
total profit received is Rs.15000. Rs.33000, find the profit share of B?
(1) 4000 (2) 4100 (1) 8000 (2) 9000
(3) 4200 (4) 4300 (3) 12000 (4) 10000
(5) None of these (5) CND

23. Nilan invested Rs.12000 in a business. Fazil 28. P and Q started a partnership business with the
investment of Rs.2000 and Rs.3800 respectively.
joined him after n months with an investment of
After 4 months, P and Q increased their invested
Rs.6000 less than Nilan. If the ratio of the profits amount by 10% and 50% respectively. What is
received by them after 3 years is 5: 2, then find the profit share of P after completion of 1 year, if
the time after which Fazil joined. the total profit earned in the year is Rs.5400.
(1) 7 month (2) 8 month (1) 1500 (2) 1600
(3) 7.2 month (4) 2500 (3) 1700 (4) 1800
(5) 8.5 month (5) None of these
29. A, B and C started a business with the investment 30. A, B and C started a business with the investment
is in the ratio of 2:3:4 and the investment period of Rs.13000, Rs.16000 and Rs.15000
of A, B and C is 8 months, 10 months and 6 respectively. After 4 months, A added Rs.3000
months respectively. At the end of business, the for his initial investment. After 4 more months, C
difference between the profits shares of A and C withdrew Rs.3000. At the end of the year, the
is Rs.4000, find the total profit of the business? total profit of the business is Rs.36000. Find the
(1) 35000 (2) 40000 profit share of C?
(3) 42000 (4) 28000 (1) 11200 (2) 11400
(5) None of these (3) 11600 (4) 11800
(5) None of these
Answer Key
1. (2) 16. (4)
2. (1) 17. (3)
3. (2) 18. (5)
4. (3) 19. (1)
5. (4) 20. (3)
6. (3) 21. (4)
7. (5) 22. (4)
8. (3) 23. (3)
9. (4) 24. (2)
10. (1) 25. (1)
11. (3) 26. (4)
12. (3) 27. (2)
13. (3) 28. (2)
14. (4) 29. (1)
15. (3) 30. (1)
Hints & Solutions
1. Solution (2) 8. Solution (3)
(x + 2000)/(2x + 8000) = 5500/16500 Capital ratio of P and Q = 80000 * 36 : 10000 * 24
2x + 8000 = 3x + 6000 = 12: 1
x = 2000 Profit gained by P = 12/13 * 39000 = Rs.36000
Profit gained by Q = 1/13 * 39000 = Rs.3000
2. Solution (1) Required difference = 36000 – 3000 = Rs.33000
Profit ratio of A and B = (5000 * 5 + 6000 *
7):(9000 * 5 + 8000 * 7) 9. Solution (4)
= 67000:101000 Remo’s profit/ Saran’s profit = 3/4
=67:101 => 3/4= 12/x
Required Difference = (101 – 67)/168 * 8400 => x = 16 months
= Rs.1700
10. Solution (1)
3. Solution (2) If investments are in the ratio a: b: c and profits in
Profit ratio of A, B and C = (3 * 1): (8 * 2): (1 * 3) the ratio p: q: r, then the ratio of time is p/a : q/b:
= 3: 16: 3 r/c.
Total profit share = 22/3 * 1800 = Rs.13200 Ratio of time = 7/4 : 4/5 : 6/6
= 35: 16: 20
4. Solution (3)
Profit ratio of A, B and C = (4500 * 12): (5000 * 8): 11. Solution (3)
(4800 * 8) Profit share of A, B and C=6000*12:
= 135: 100: 96 (3200*8+4800*4): (4800*8+4400*4)
C’s profit share = (96/331) * 16550 =180:112:140
= Rs.4800 =45:28:35
Profit share of C=2160*35/108=Rs.700
5. Solution (4)
Profit ratio of A, B and C 12. Solution (3)
= (1000 * 4 + 1200 * 8): (1500 * 4 + 1100 * 8): (8 * 15)/(7 * x) = 100/140
(1200 * 8 + 1100 * 4) 7x = 8 * 3 * 7
= 13600: 14800: 14000 x = 24 months
= 34: 37: 35
C’s profit share = 35/106 * 5300 = Rs.1750 13. Solution (3)
Profit share of B and C=5400: 4800=9:8
6. Solution (3) Profit share of B and C=4000*17/8=8500
Profit ratio of A and B = (4500 * 12): (x * 8) Profit share of A=11500-8500=3000
= 6750: x
6750/(x + 6750) = 9000/17000 14. Solution (4)
9x + 6750 * 9 = 17 * 6750 Investment ratio of A, B and C=x*12: (x+400)*12:
9x = 54000 (x+1400)*6
x = 6000 =2x: 2x+800: x+1400
2x/(2x+800)=7/8
7. Solution (5) 16x=14x+5600
Profit ratio of A, B and C = 4000 * 12:6000 * x=2800
6:4500 * 6 Investment of C=2800+1400=Rs.4200
= 16:12:9
C’s profit share = 9/37 * 18500 = Rs.4500
15. Solution (3) 21. Solution (4)
Ratio of the investment of A, B and Hari’s capital = Rs.x
C=3500*12:2100*12:4000*x (9000 * 12)/(8 * x) = ¾
=105:63:10x => x = Rs.18000
Profit share of A=Rs.3000
Profit share of B=3000*63/105=Rs.1800 22. Solution (4)
Profit share of C=6800-3000-1800=Rs.2000 Required ratio = (7000 * 12):(9400 * 7):(8600 * 7)
10x=63*2000/1800 = 60: 47: 43
x=7 Profit of R = 15000 * 43/150 = Rs.4300

16. Solution (4) 23. Solution (3)


(3500*12)/(4800*(12-x))=7/4 (12000 * 36): (6000 * (36 – n)) = 5: 2
35/(12-x)=7 => 144 = 180 -5n
(12-x)=5 => n = 7.2 months

17. Solution (3) 24. Solution (2)


Profit ratio of A, B and C = (4/9 * 4 + (4/9 * 125/100 Ratio of investment of X, Y and Z is = 35 : 7a : 10a
* 8)):(4/7 * 4 + (4/7 * 120/100 * 8)):(1/5 * 12) So, the profit sharing ratio of X, Y and Z is = 35 ×
= 56/9:54.4/7:12/5 8 : 7a × 6 : 10a × 12
= 1960:2448:756 = 140 : 21a : 60a
Profit share of B = 2448/5164 * 38730 Then, according to the question,
= Rs.18360 140/(140 + 21a + 60a) = 700/3130
140/(140 + 81a) = 70/313
18. Solution (5) 43,820 = 9800 + 5670a
Profit ratio of A, B and C = (12000 * 7 + 11200 * 5670a = 43,820 – 9800
5):13000 * 12:x * 5 5670a = 34,020
= 140000:156000:5x a=6
156000/(140000 + 156000 + 5x) = 7800/22000
x = 28800 25. Solution (1)
Profit ratio of A: B: C = (2400 * 12): (4200 * 12):
19. Solution (1) (3800 * 6)
Ratio of the Profit share of A, B and C = ((x + 1000) = 24: 42: 19
* 1 + (x + 1000 – 3000) * 1):((x + 2000) * 1 + (x + C’s profit share = 19/85 * 21250
2000 – 2000) * 1):(x * 1 + (x + 5000) * 1) = Rs.4750
= (2x – 1000):(2x + 2000):(2x + 5000)
(2x – 1000)/(6x + 6000) = 1200/4800 26. Solution (4)
6x + 6000 = 8x – 4000 Profit ratio of Mithun, Nirmal and Divya = 8000 *
2x = 10000 12:12000 * 12:14000 * 6
x = 5000 = 96:144:84
= 8:12:7
20. Solution (3) Profit share of Mithun = 8/27 * 20250
Profit ratio of A and B = (1410 – 600):600 = = Rs.6000
810:600
= 27:20
(x * 12)/((x + 400) * 8) = 27/20
9x + 3600 = 10x
x = 3600
27. Solution (2) 29. Solution (1)
Profit ratio of A, B and C = (2x * 1 + (2x + 4800) * Profit ratio of A, B and C = 2 * 8:3 * 10:6 * 4
1):(3x * 1 + (3x + 3900)):(6x * 1 + (6x + 5600) * 1) = 16:30:24
= 4x + 4800:6x + 3900:12x + 5600 = 8:15:12
Profit share of B = ((6x + 3900)/(22x + 14300)) * Total profit = 35/4 * 4000 = 35000
33000
= 3(2x + 1300)/11(2x + 1300) * 33000 30. Solution (1)
= 9000 Profit ratio of A, B and C = (13000 * 4 + 16000 *
8):(16000 * 12):(15000 * 8 + 12000 * 4)
28. Solution (2) = 180:192:168
P: Q = (2000 * 4 + 1.1 * 2000 * 8): (3800 * 4 + 1.5 = 15:16:14
* 3800 * 8) Profit share of C = 14/45 * 36000
= 8: 19 = Rs.11200
Profit share of P = 8/27 * 5400 = Rs.1600
1. Ratio of the time taken by the boat travel 150 5. A man covers a distance of 240 Km by train
km against stream to time taken to covers the with the speed of 48 Km/h, a distance of 60
same distance along with stream is 5: 1. If the Km by bus with the speed of 12 Km/h and a
distance of 20 Km by bicycle with the speed
speed of the stream is 20 kmph, then what is
of 5 km/h to reach head quarter of a bank
the speed of boat in still water? from his home. Find the average speed of the
(1) 40 (2) 25 man throughout the journey?
(3) 30 (4) 45 (1) 25.15 (2) 22.85
(5) None of these (3) 15.65 (4) 21.25
(5) 18.75
2. Arav and Ravi starts travelling in same
6. A man walks a certain distance and ride back
direction at 4 kmph and 7 kmph respectively.
in 7 ½ hours. He can walk both the ways in 9
After 3 hours, Arav doubled his speed and ¼ hours. How long will it take to ride both
Ravi reduced his speed by 2 kmph and the ways?
reached the destination together. How long (1) 7 ¼ (2) 5 1/2
the entire journey last? (3) 7 5/8 (4) 6 ½
(1) 2 (2) 4 (5) None of these
(3) 6 (4) 8
7 Rahul goes from Chennai to Bangalore and
(5) None of these
the distance between Chennai and Bangalore
is 400 km. He completes 40% distance by
3. Speed of a man in still water is 24 km/hr and bike, 30% by bus and remaining distance by
the river is running at 16 km/hr. The total train. If the speed of the bike is 60 kmph and
time taken to go to a place and come back is time taken by Rahul to cover the distance by
24 hours. Find the distance travelled by the bus is 3 hours and the ratio of the speed of
train to bus is 5: 4, what is the time taken by
man in the whole journey?
train?
(1) 240 (2) 320 (1) 2.4 (2) 2.8
(3) 125 (4) 390 (3) 3.2 (4) 3.6
(5) None of these (5) 4

4. A car can cover a distance of 1020 Km in 17 8. A man covered one-fourth of the distance by
hours. Speed of a bus is 20% less than the bus with a speed of 40 km/hr and the
remaining distance will be covered by bike
speed of the car and speed of a train is 32
with a speed of 60 km/hr. Find the total
Km/h more than the speed of the bus. Find distance covered by the man, if the total time
the respective ratio of the distance covered by taken by him is 5 hours 15 min?
the car in 20 hours and distance covered by (1) 220 (2) 340
the train in 25 hours. (3) 280 (4) 300
(1) 3:5 (2) 1:3 (5) None of these
(3) 2:3 (4) 4:7
(5) None of these
9. Ratio of the distance between A to B and B
to C is 2: 1.The car runs at the speed to cover 13. Lewis and John run a 10 km race on a circular
the distance between A to B is 40 kmph and track of length 1000 m. They complete one
the same car runs at the speed to cover the round in 200 seconds and 400 seconds
distance between B to C is 30 kmph. If the respectively. After how much time from the
time taken by the car to cover the whole start will the faster person meet the slower
distance in 5 hours, then find the distance person for the last time?
between A to C? (1) 500 sec (2) 1000 sec
(1) 120 (2) 150 (3) 2500 sec (4) 1500 sec
(3) 180 (4) 200 (5) 2000 sec
(5) None of these
14. A bus is 460m in front of the boy. The boy
10. A man goes from his house to office at the starts running towards the bus and at the same
speed of 20 kmph and reaches the office at 15 time the bus starts moving away from the
minutes late. If he increases his speed by 5 boy. If the speed of the bus is 3m/s and the
kmph and reaches the office in time, then speed of the boy is 20 km/hr. In how much
what is the distance between house and time will the boy catch the bus?
office? (1) 70 (2) 90
(1) 20 (2) 25 (3) 180 (4) 120
(3) 30 (4) 35 (5) 150
(5) None of these
15. A, B and C start running around a circular
field having circumference 150 metre at the
11. Anju starts walking from U towards V with same time from the same point. Speeds of A,
speed 5 km/hr and Rani Starts on her scooty B and C are 2 m/minute, 2.5 m/minute and 3
from V towards U with the speed 45 km/hr. m/minute. Find after how much time, they
In how much time do they meet if both starts will meet again at the same point for the first
at 6 am and distance between U and V is 320 time.
km? (1) 9 (2) 7
(1) 1:35 pm (2) 12:24 pm (3) 6 (4) 5
(3) 12:36 pm (4) 11:45 pm (5) None of these
(5) 2:24 pm
16. Devesh covers a certain distance between his
business place and home in car. With an
12. Ashwini and Alpana run a 14.4 km race on a average speed of 20 km/h, he got late by 30
circular track of circumference 960 m and the min. However with a speed of 40 km/h, he
two complete one round in 12 seconds and 16 reaches his office 15 min earlier. Find the
seconds respectively. After what time (in distance between his home and business
seconds) from the start will the faster person place?
meet the slower person for the last time? (1) 22 (2) 26
(1) 132 (2) 120 (3) 30 (4) 28
(3) 138 (4) 144 (5) 25
(5) None of these
17. Two trucks ‘X’ and ‘Z’ were moving towards 22. Ram started for Delhi from Patna at 25 km
each other which were 490 km away initially. per hour. After sometime, he realized that at
If the ratio of the speed of the trucks ‘X’ and this speed he will be late by 4 hours or could
‘Z’ was 4 : 3 and the speed of the truck ‘Z’ cover only 60% of the total distance till the
was 60 km/h, what time will it take for the scheduled time so immediately he doubled
two trucks to meet each other? his speed and reached Delhi on time. After
(1) 210 (2) 240 how many hours of starting journey did he
(3) 180 (4) 270 double his speed? (it is given that the distance
(5) 225 between Delhi and Patna is 500 km)
(1) 16 (2) 12
18. The ratio of speeds of a tiger and horse is 3 : (3) 18 (4) 20
2. Both start from place A at the same time (5) None of these
and reach place B, which is 75 km away from
place A at the same time as the tiger lost 23. The distance between a school and home is
about 12.5 minutes while hunting a deer. Find 24 km. Two persons A and B start from the
the speed of the horse? home and the school at the speed of 12 km
(1) 120 (2) 150 per hour and 10 km per hour respectively in
(3) 80 (4) 70 the same direction and meet each other at the
(5) None of these college. What is the distance between the
school and the college? (assume that the
19. In a to-and-fro journey, the speed of the bus home is before school and both are in the
is 20 kmph and 15 kmph respectively. If it same straight line)
takes 14/5 hours to complete the whole ride,
which of the following will be the total
distance traveled by the bus? (1) 144 (2) 220
(1) 40 (2) 42 (3) 120 (4) 240
(3) 24 (4) 48 (5) None of these
(5) None of these
24. A train goes 2210 km from Station A to
20. Point P and Q are 500 km apart. Car A starts station B and completes the journey in time.
from P to Q at 7 am and Car B starts from Q In return journey, due to some engine
to P at 9 am. Speed of Car A was double than problem train starts 20 hrs late and to reach
Car B. After they meet at some point, speed on time trains average speed is increased by
of both car A becomes same as Car B. A 8km/hr. What was the original speed of train?
reaches point Q at 1 pm. What is the speed of (1) 24 (2) 26
car A? (3) 27 (4) 21
(1) 50 (2) 100 (5) None of these
(3) 80 (4) 90
(5) 120 25. Ashok and Bimal runs on a 450 m long
circular track at a speed of 12 m/s and 18 m/s
21. Raju can travel from his house to school in x respectively. They start at same time but run
hours if he does not stop anywhere. One day, in opposite direction. Find the number of
he increases his speed by 4 km per hour but times, they would have met when Bimal
stops for 15 minutes on a tea shop then he covers 3240 meters?
reaches 5 minutes earlier. If the distance from (1) 12 (2) 10
his house to the school is 40 km then find the (3) 14 (4) 16
value of x? (5) 18
(1) 4 (2) 2
(3) 1 (4) 1.7
(5) None of these
26. Pankaj was travelling to point B from point A 30. In a race of 500 meters, Tom runs at the speed
with speed of 45 km/hr. After 1 hour, Pratik of 18 km per hour. Tom gives a start of 50
also started to travel from point A to point B. meters to Jerry but still beats him by 12.5
Pratik reached the point B 30 minutes before seconds. Find the speed of Jerry (in km per
Pankaj. If Pratik would have decreased his hour)?
(1) 16 (2) 15.6
speed by 6 km/hr then both would have
(3) 14.6 (4) 14.4
reached point B at the same time. Find the (5) None of these
original speed of Pratik?
(1) 55 (2) 60 31. If speed of car is reduced due to some reason
(3) 54 (4) 64 by 40 km/hr and it take 36 minutes more to
(5) 65 cover 160 km. What time it takes to cover
same distance with his normal speed?
27. Two friends A and B, with speed in the ratio (1) 1.8 hr (2) 2.2 hr
9 : 3, are running on track PQ. A starts from (3) 3.2 hr (4) 1.2 hr
P towards Q and when he reaches point (5) 1.6 hr
exactly in the middle of the track, B starts
32. The ratio between the speed of a taxi and bike
running from P towards Q. A reaches Q turns is 7 : 12 respectively. Also, a cab covers a
back and continues towards P and meets B at distance of 840 km in 8 hours. The speed of
a distance of 155 m from Q. What is the total the taxi is 60% of the speed of the cab. How
length of the track? much distance will the bike cover in 9 hours?
(1) 320 (2) 248
(3) 243 (4) 280 (1) 372 km (2) 672 km
(3) 872 km (4) 972 km
(5) None of these
(5) 924 km
28. Anurag walked 13 km to reach the bus stand 33. Akhil and Amar start cycling at the same time
from his home, then he boarded a bus whose in opposite directions from two distinct
average speed was 55 kmph and thus he points A and B of a race track. Starting from
reached his office. In this way he took a total A, Akhil reaches B in 3 hrs 40 minutes and
time of 2.5 hours. If the average speed of the Amar starting from B, reaches A in 2 hrs 15
entire journey was 36 kmph then the average minutes. What is the speed of Amar, if the
speed of walking is? speed of Akhil is 27 km/h?
(1) 11.8 (2) 11.6 (1) 44 Km/h (2) 66 Km/h
(3) 33 Km/h (4) 77 Km/h
(3) 12.1 (4) 11.4
(5) 22 km/h
(5) None of these
34. Distance between two cities Y and Z is 1260
29. The ratio between the speed of a train and a km. Amit starts from Y towards Z with usual
car is 18 : 3 respectively. Also, a bus covered speed and after covering one third of
a distance of 480 kms in 12 hours. The speed distance, speed of car increased by 20% and
of the bus is five-ninths the speed of the train. its take 1 hours 20 minutes less than what its
How much distance will the car cover in 5 take at its usual speed. Find the speed of Amit
hours? (in km/hr)?
(1) 150 (2) 180 (1) 164 km/hr (2) 148 km/hr
(3) 160 (4) 170 (3) 132 km/hr (4) 126 km/hr
(5) CND (5) 120 km/hr
35. Amar started for the station A 1 km meter
from his home walking at 1 km/h to catch the 38. Pawan covers certain distance with his own
train in time. After 6 minutes he realized that speed but when he reduces his speed by 5
he had left train ticket at home and returned kmph his time duration for the journey
with increased, but constant speed to get it increases by 15 hours while if he increases his
succeeded in catching the train. Find his latter speed by 5 kmph from his original speed he
speed? takes 10 hours less than the original time
(1) 2.75 km/hr (2) 1.25 km/hr taken. Find the distance covered by him?
(3) 3.25 km/hr (4) 2 km/hr
(5) None of these (1) 1000 km (2) 2400 km
(3) 1600 km (4) 1200 km
36. Two friends Amit and Ram start running
(5) 1500 km
simultaneously from opposite end of a
platform. But Amit takes 25 sec more than
39. The distance office and house of Sachin is
Ram to cross the platform. If speed of Ram is 240km. One day he was late by 3 hours than
twice of speed of Amit. Then find the time the normal time to leave for the office, so he
taken by Amit to cross the platform? increased his speed by 4 km/h and thus he
(1) 20 sec (2) 30 सेकं sec reached to office at the normal time. What is
the original speed of Sachin?
(3) 40 sec (4) 50 sec
(1) 28 kmph (2) 20 kmph
(5) None of these
(3) 22 kmph (4) 10 kmph
(5) 16 kmph
37. The distance of the Samar home town and
Delhi where he work is 480 km. One
40. Two friends, Raman and Samar start at the
weekend he was late by 4 hour than the
same time to ride from Kanpur to Lucknow,
normal time, so he increased his speed next
240 km away. Raman travels 16 km an hour
weekend by 10 km/hr and thus he reached to
slower than Samar. Samar reaches Lucknow
his home town at normal time. What is the
and at once turns back meeting Raman 48 km
increased speed of Samar?
from Lucknow. The speed of Raman was
(1) 30 km/hr
(1) 16 km/hr (2) 32 km/hr
(2) 40 km/hr
(3) 48 km/hr (4) 64 km/hr
(3) 25 km/hr
(5) None of these
(4) 50 km/hr
(5) None of these
Answer Key
1. (3) 9. (3) 17. (1) 25. (1) 33. (1)
2. (3) 10. (2) 18. (1) 26. (2) 34. (4)
3. (2) 11. (4) 19. (4) 27. (2) 35. (1)
4. (1) 12. (2) 20. (2) 28. (1) 36. (4)
5. (2) 13. (5) 21. (2) 29. (5) 37. (2)
6. (5) 14. (3) 22. (2) 30. (4) 38. (5)
7. (1) 15. (4) 23. (3) 31. (1) 39. (5)
8. (3) 16. (3) 24. (2) 32. (4) 40. (2)
Hints & Solutions
1. Solution (3)
Speed of the boat = x 4. Solution (1)
Speed of the car = 1020/17 = 60 Km/h
(150/(x – 20))/(150/(x + 20)) = 5/1
Speed of the bus = 60 x 80/100 = 48 Km/h
150/(x – 20) = 5 * 150/(x + 20)
Speed of the train = 48 + 32 = 80 Km/h
5x – 100 = x + 20
Required ratio = 60 x 20: 25 x 80
4x = 120
= 1200: 2000
x = 30 kmph
= 3: 5

2. Solution (3) 5. Solution (2)


Distance travelled by Arav in 3 hours = 4 * 3 = Time taken by the man to cover 240 Km by
12 hours train = 240/48 = 5 hours

Distance travelled by Ravi in 3 hours = 7 * 3 = Time taken by the man to cover 60 Km by bus
21 hours = 60/12 = 5 hours

Distance between both = 21 – 12 = 9 km Time taken by the man to cover 20 Km by


bicycle = 20/5 = 4 hours
New speed of Arav = 4 * 2 = 8 kmph
We know that
New speed of Ravi = 7 – 2 = 5 kmph
Average speed = Total distance/Total time
Relative speed = 8 – 5 = 3 kmph
= (240 + 60 + 20)/(5 + 5 + 4)
Total time = 3 + 9/3 = 6 hours
= 320/14

3. Solution (2) = 22.85 Km/h


Speed of downstream = 24 + 16 = 40 km/hr

Speed of upstream = 24 – 16 = 8 km/hr 6. Solution (5)


W + R = 7 ½ = (15/2) hours
Let the distance travelled be x,
W + W = 37/4
(x/40) + (x/8) = 24
2W = 37/4
= > x = 160 km
W = (37/8) hours
Total distance covered by a man in the whole
journey = 160*2 = 320 km R = (15/2) – (37/8) = (23/8) hours

Time taken by the man to ride both the ways,


= R + R = 2R = (23/8) * 2 = 23/4 2x/40 + x/30 = 5

= 5 (¾) hours 6x + 4x = 120 * 5

x = 60
7. Solution (1)
Distance between Chennai and Bangalore Total distance = 3 * 60 = 180 km

= 400 km 10. Solution (2)


Distance=speed * time
Speed of bike = 60 kmph
Distance=(20 * 25 * 15/60)/5
Distance covers by bus = 400 * 30/100
=25 km
= 120 km

Speed of bus = 120/3 = 40 kmph 11. Solution (4)


Speed of Anju = 5 kmph, speed of Rani
Speed of train = 5/4 * 40 = 50 kmph = 45 kmph
Distance between them = 320 km
Distance covered by train = 30/100 * 400 Relative speed = 45 + 5 = 50 kmph
Time taken =320/50= 6.4 hours
= 120 km They start at 6 am. So, they will meet at 12:24

Time taken by train = 120/50 = 2.4 hours pm.


8. Solution (3)
Total time taken by him = 5 hours 15 min
12. Solution (2)
= 5 15/60 = 5 1/4 hours Ashwini and Alpana complete one round in 12
seconds and 16 seconds respectively.
T = D/S Speeds of Ashwini and Alpana are 960/12 and
960/16 i.e. 80 m/s and 60 m/s respectively.
(x/4)/40 + (3x/4)/60 = 5 1/4 Relative speed of Ashwini with respect to
Alpana = (80 – 60) m/s = 20 m/s
x/40 + 3x/60 = (21/4)*4 Hence, Ashwini meets Alpana after every
960/20 = 48 seconds
(3x + 6x)/120 = 21 To complete 14.4 km, number of rounds
required =144/0.96= 15
9x = 21*120 Since Ashwini is faster, she will win the race
and therefore race time = 15 × 12 = 180 seconds
x = (21*120)/9
Since both of them meet each other after every
x = 280 km 48 seconds, after the start, Ashwini will meet
Alpana 3 times during the race.
9. Solution (3) Time when both of them will meet for the last
Distance = speed * time time = 48 × 3 = 144 seconds

Total distance = 2x + x = 3x 13. Solution (5)


Let us say Lewis meets John after ‘t’ seconds ⇒ y =x/30+1/4..............(ii)
he starts from the starting point. Speeds of So, equating eq. (i) and eq. (ii), we get,
Lewis and John are 5 m/s and 2.5 m/s. X/20–1/2=x/40+1/4
t =1000/5 – 2.5 = 400 seconds x = 30
Distance travelled by Lewis when meets John The distance between his home and business
for the first time = 2000m i.e., after 2 complete place = 30 km
revolutions. Lewis can make 10 complete
revolutions. As he meets John after every two 17. Solution (1)
revolutions he would meet John after the 10th Given, two trucks ‘X’ and ‘Z’ were moving
revolution for the last time i.e., after 400 × 5 = towards each other which were 490 km away
2000 seconds. initially.
The ratio of the speed of the trucks ‘X’ and ‘Z’
14. Solution (3) was respectively 4: 3 and the speed of the truck
Let the boy catch the bus after‘t’ seconds ‘Z’ was 60 km/h.
Speed of the boy = 20 km/hr So, the speed of truck ‘X’ = 60 ×4/3
→20 × 5/18m/s =50/9m/s = 80 km/h.
Speed of the bus = 3 m/s As, the both trucks were moving towards each
The time taken by the boy to catch the bus other, the relative speed will be = (80 + 60)
=460/50/9 – 3 km/h = 140km/hr.
→460 × 9/50 – 27= 180 seconds The two trucks meet each other in
= 490 /140= 3.5 hours = 210 minutes.

15. Solution (4) 18. Solution (1)


Time taken by A to complete one round of the Let speed of the horse be x km/h.
circular field Speed of the tiger =3x/2km/h
=150/2= 75 minutes Now, according to the question,
Time taken by B to complete one round of the ⇒75/x–75/3x/2=125/10 × 60
circular field =150/2.5= 60 minutes ⇒75/x–50/x=5/ 24
Time taken by C to complete one round of the ⇒ x =25 × 24/5= 120
circular field =
150/3= 50 minutes
19. Solution (4)
L. C. M of 75, 60, 50 = 300 minutes =300/60=
Average speed between 2 points
5 hours
2UV/ U + V
(U and V are speeds)
16. Solution (3)
Avg speed = (2 × 20 × 15) / 35 =120/7km/hr
Let the distance between Devesh’s business Distance travelled by bus in 2 4/5 hour at
place and home be x km and usual time
120/7km/hr
required to reach the business place be y hours.
Having an average speed of 20 km/h, he got (120/7) ×14/5= 48 km
late by 30 min.
So, we can write, 20. Solution (2)
x/20= y +30/60 Speed of Car A = 2X
⇒ y =x/20–/21..............(i) Speed of Car B = X
However with a speed of 40 km/h, he reaches Distance covered by Car A from 7 am to
his office in 15 min earlier. 9 am = 2 × 2X = 4X
We can wrote again, Relative Speed = 2X + X = 3X
x/40= y –/6015 Distance = 500 – 4X
Time =Distance/speed=500 – 4X/ 3X It means, they meet each other after 12 hours
Distance covered by A =500 – 4X/3X × 2X
We need to calculate the distance between the
= 1000 – 8X/ 3
school and the college
Now A reaches point Q at 1 pm.
He covered the next distance with X kmph. The distance travelled by B in 12 hours @ 10
He total took 6 hours. km per hour = 12 × 10 = 120 km
12X + 1000 – 8X + 12X – 500 + 4X = 1500
20X = 1000
X = 50 24. Solution (2)
Speed of Car A = 2X = 100 Distance = 2210 km
Let Original speed = k, so the return speed = k
+8
Time difference between the two journeys = 20
21. Solution (2)
hrs
Let the speed = a km per hr
When he increases his speed by 4 km per hour 2210/k–2210/ k + 8= 20
New speed = a + 4 km per hour k2 + 8k – 884 = 0
He stops for 15 minutes on a tea shop then he Solving we get k = 26 km/h
reaches 5 minutes earlier it means if he had not
stopped for 15 minutes then he would have
reached 15 + 5 = 20 minutes earlier @ a + 4 km 25. Solution (1)
per hour When Ashok and Bimal together cover
We know that, time =distance/speed 450 m distance, they meet and out of this
40/a–40/ a + 4=20/60 Ashok covers :
40 × 3(a + 4 – a) = a (a + 4) =12/12 + 18× 450 = 180
40 × 3 × 4 = 480 = a (a + 4) Bimal covers 450 – 180 = 270 m.
By solving, a = 20 km per hour So they meet, Bimal covers 270 m.
Value of x =distance/speed=40/20 When Bimal covers 3240 meters, they would
= 2 hours have meet
3240/270= 12 times
22. Solution (2)
The distance between Delhi and Patna
= 500 km 26. Solution (2)
60% of 500 = 300
Let the original time taken by Pankaj to reach
At 25 km per hour he can cover 300 km or late
point B from point A = x hours
by 4 hours
Then, original time taken by Pratik to reach
Therefore, the scheduled time = 12 + 4
point B from point A = (x – 1.5) hours
= 16 hours
Also
Let after t hours he doubled his speed then
Let original speed of Pratik = y km/h
25 × t + 50 × (16 – t) = 500
nd, y × (x – 1.5) = (y – 6) × (x – 1)
25t = 50 × 6
So, 45 × x = y × (x – 1.5)
T = 12 hours
y × (x – 1.5) = (y – 6) × (x – 1)
yx – 1.5y = yx – 6x – y + 6
23. Solution (3) 0.5y = 6x – 6
The relative speed of A and B = 12 – 10 y = 12x – 12
= 2 km per hour Therefore, 45 × x = (12x – 12) × (x – 1.5)
Time =distance/ speed=24/2= 12 hours 45x = 12x2 – 12x – 18x + 18
12x2 – 75x + 18 = 0 30. Solution (4)
4x2 – 25x + 6 = 0 The speed of Tom = 18 km per hour
4x2 – 24x – x + 6 = 0 = 18 × 5/18= 5 meters per second
4x (x – 6) – 1 (x – 6) = 0
(4x – 1) (x – 6) = 0 The time taken by Tom to run 500 meters
x =1/4 , 6 =500/5= 100 sec
So, x = 6
Therefore, speed of Pratik = 12x – 12 If Tom gives start of 50 meters then Jerry will
run for 500 – 50 = 450 meters
= 60 km/hr But still Tom beats Jerry by 12.5 meters it
means Jerry will take 100 + 12.5 = 112.5
seconds to run 450 meters
27. Solution (2)
The speed of Jerry =450/112.4
When A reaches point R which is in the middle = 4 meters per sec
of the track, B starts from point P towards Q =4 × 18/5= 14.4 km per hour
The distance covered after point that will be in
the ratio of their speeds
31. Solution (1)
Therefore,RQ + QS/ PS= 9 : 3
Let original time be ‘T’
RQ + QS + PS = 12 units
And normal speed be ‘X’ km/hr
RQ + QS + PS =3/4(2 × PQ)
Normally takes
So, PQ = 8 units and as PS = 3 units, therefore
X × T = 160 ……(i)
QS = 5 units
After reduced speed
QS = 155m = 5 units → 1 unit = 31 m 36
PQ = 8 units = 8 × 31 = 248 m (X– 40) × (T + ) = 160 ……(ii)
60
Solving (i) and (ii)
we get, T = 1.8 hours
28. Solution (1)
Total Distance = 36 × 2.5 = 90km
Distance covered by bus = 90 – 13 = 77 km 32. Solution (4)
Time taken by bus to cover the journey of 77 840
Speed of cab = = 105 km/h
km 8
=77/55 = 1.4 hr 60
Since Anurag walked 13 km in (2.5 – 1.4) Speed of taxi = 100 × 105 = 63 km/h
= 1.1 hr 12
The average walked speed by anurag =13/1.1= ∴ speed of bike = × 63 = 108 km/h
7
11.8 kmph
∴ Distance travelled by bike in 9 hours
29. Solution (5) = 108 × 9 = 972 km
Let speed of train and bus be 18x and 3x
respectively. 33. Solution (1)
Speed of bus = 480/12 = 40 km/hr
Length of race track = speed × time
As speed of bus is five ninths the speed of train 40 11
so, = 27 × (3 + 60) = 27 × 3 = 99 km
99
Speed of train =9/5×480/12= 72 km/hr Speed of Amar = 15 = 44 km/hr.
(2 + )
Speed of car = 72 ×3/18= 12 km/hr 60

Distance covered by car in 5 hrs = 12 × 5


= 60 kms. 34. Solution (4)
Let speed of Amit be x km/hr 38. Solution (5)
1260 420 840×4 4 Let distance covered be x km and original time
−( + )=3 taken be y hour
𝑥 𝑥 5𝑥
840 840×4 4
ATQ,
− =3 𝑥 𝑥
𝑥 5𝑥 − = 15 ……(i)
𝑦−5 𝑦
4200 −2730 4
=3 𝑥 𝑥
5𝑥 − 𝑦+5 = 10 …….(ii)
𝑦
20x = 840× 3 From (i) and (ii)
y = 25 hours, x = 1500 km
x = 126 km/hr
39. Solution (5)
35. Solution (1) Let his initial speed be x kmph
1000 240 240
Distance covered in 6 minutes = 6× 60 – = 3
𝑥 𝑥+4
= 100 Solving, we get
Now,
the distance which he has to cover = x = 16kmph
(1000+100) meter in (30-6) minutes 40. Solution (2)
1100
1000 Let speed of Raman was 𝑥 km/hr.
Required speed = 24 = 2.75 km/hr
60
Thus, speed of Samar = 𝑥 + 16 km/hr
So, by the time they met, Samar has travelled
36. Solution (4)
(240 + 48) km while Raman has travelled (240
Speed of Amit is half of the speed of Ram
– 48) km.
So, Amit take twice time
Time taken by slower runner = 2 × 25 Or, Samar has travelled 96 km more than
= 50 sec Raman.
Since Samar has a margin of 96 km per hour
37. Solution (2) i.e. he travelled 16 km more every hour, so it
Let the original speed be x need him 6 hours to travel 96 km more than
ATQ, Raman.
480 480 240 − 48
− 𝑥+10 = 4 ∴ Required speed of Raman =
𝑥 6
x = 30 kmph = 32 km/hr
Increased speed = 40 kmph
1. Train A crosses train B of the same length moving 6. A train crosses a 420 m long platform in 36
in the opposite direction in 20 seconds. If train A seconds at the speed of 90 kmph. If the speed of
crosses a standing man in 18 seconds with the the train is decreased by 16.67%, then find the
time taken by train to cross a 240 m long bridge?
speed of 72 kmph, then find the speed of train B?
(1) 9.6 (2) 12
(1) 24 (2) 18 (3) 10 (4) 16.5
(3) 16 (4) 12 (5) None of these
(5) None of these
7. A train crosses a car is running in the opposite
2. Train A crosses a platform of length 280 m in 38 direction at the speed of 24 kmph in 18 seconds.
seconds with the speed of 72 kmph. If the length If the speed of train is double that of car, then find
the time taken by the train to cross 140 m long
of train A is 80 m more than the length of train B
platform?
and train B crosses a 100 m long tunnel in 30 (1) 37.5 (2) 25
seconds, then find the difference between the (3) 30 (4) 28.8
speed of train A and B? (5) 14.4
(1) 18 (2) 12
(3) 15 (4) 8 8. Train A crosses train B is running in the same
(5) None of these direction in 108 seconds and also train A crosses
a tower in 21.6 seconds. If the speed of train B is
3. A train crosses 360 m long platform in 32 seconds 33(1/3)% more than the speed of train A, then
and the train also crosses a dog standing in a find the time taken by train B crosses a pole?
platform in 17.6 seconds. Find the speed of the (1) 14.4 (2) 12
(3) 18 (4) 10.8
train?
(5) CND
(1) 60 (2) 80
(3) 100 (4) 90
9. Ratio of the speed of trains A and B is 4:5. Train
(5) 72 A is 160 m long and train B is 240 m long and
train B travels at the speed of 50 kmph. If the two
4. Train A crosses a pole in 18 seconds and also trains A and B are running in opposite direction,
crosses train B is running in the same direction in then find the time taken by both trains to cross
2 minutes. If the speed of train B is 25% more each other?
than that of A and the length of train B is 240 m, (1) 26 (2) 21
then find the length of train A? (3) 12 (4) 27
(1) 260 (2) 360 (5) None of these
(3) 450 (4) 440
10. Train A crosses a standing man in 24 seconds and
(5) 560
also crosses train B running in the opposite
direction at the speed of 54 kmph in 21.6 seconds.
5. A train crosses a tower in 20.25 seconds and also If the ratio of the length of trains A and B is 4:5
crosses a 150 m long bridge in 27 seconds. Find respectively, then find the speed of train A?
the speed of the train (in kmph)? (1) 27 (2) 54
(1) 60 (2) 72 (3) 45 (4) 36
(3) 80 (4) 90 (5) None of these
(5) None of these
11. A train crosses a 400 m long platform in 42
seconds and also crosses a man running in 17. A train crosses a platform in 18 seconds and the
opposite direction at the speed of 20 kmph in 13.5 train also crosses an electric pole in 6 seconds. If
seconds. What is the speed of train? the speed of the train is 60 kmph, what is the
(1) 40 (2) 50 length of the platform?
(3) 60 (4) 80 (1) 100 (2) 200
(5) None of these (3) 300 (4) 400
(5) None of these
12. The ratio of the length of train A to B is 4: 5 and
the speed of the train A to B is 3: 4. If train A 18. Train A crosses a tunnel of length 300 m in 15
crosses Train B running in same direction in 81 seconds but the same train crosses an electronic
seconds and train A also crosses a pole in 12 pole in 6 seconds. Find the length of train?
seconds, then find the speed of train B? (1) 175 (2) 225
(1) 50 (2) 60 (3) 200 (4) 250
(3) 80 (4) CND (5) None of these
(5) None of these
19. Train A crosses a pole in 25 seconds and Train B
13. Train A crosses a platform of length 300 m in 24 crosses the same pole in 40 seconds. The length
seconds but the same train crosses a man in 8 of Train A is half the length of Train B. What is
seconds. Find the length of train? the ratio of the speed of Train A to that of Train
(1) 250 (2) 300 B?
(3) 150 (4) 200 (1) 4:5 (2) 6:7
(5) None of these (3) 3:4 (4) 2:5
(5) None of these
14. A train crosses 300 m long tunnel in 36 seconds
and the same train crosses a pole in 18 20. Two trains A and B is in the length of 350m and
seconds.What is the length of the train? 250m respectively and crosses the pole in 18
(1) 200 (2) 300 seconds and 12 seconds respectively. What is the
(3) 250 (4) CND difference between the speed of train A and B?
(5) None of these (1) 10 (2) 5
(3) 15 (4) 20
15. Train A crosses Train B running in opposite (5) None of these
direction at the speed of 60 kmph in 6 seconds
and crosses a pole in 8 seconds. If the length of 21. A train crosses a tower in 20.25 seconds and also
the train B is 100 m longer than that of train A, crosses a 150 m long bridge in 27 seconds. Find
the find the length of train A? the speed of the train?
(1) 200 (2) 400 (1) 60 (2) 72
(3) 300 (4) 250 (3) 80 (4) 90
(5) None of these (5) None of these

16. Train A crosses train B running opposite 22. The speed of train A is 25% more than the speed
direction at the speed of 60 kmph in 9.6 seconds of train B and train A crosses train B is running
and also train A crosses a electric pole in 8 in the same direction in 196 seconds. If the
seconds. If the length of train A and train B is average of the length of train A and B is 490 m,
equal, then what is the length of train B? then find the speed of train B?
(1) 100 (2) 200 (1) 60 (2) 72
(3) 300 (4) CND (3) 80 (4) 48
(5) None of these (5) None of these
23. Train A crosses train B is running in the same 28. The length of train A is 400 m long and crosses a
direction in 72 seconds. If the ratio of the speed pole in 24 seconds and the ratio of the speed of
of train A and B is 4:3 and train A crosses a pole train A and B is 5:6. If train B crosses a man
in 7.2 seconds and the length of train B is 120 m standing in a platform in 15 seconds, then find the
more than the length of train A, then find the length of train B?
length of train B? (1) 200
(1) 240 (2) 300 (2) 250
(3) 360 (4) 400 (3) 300
(5) 180 (4) 350
(5) None of these
24. Train A crosses a standing man in 24 seconds and
also crosses train B running in the opposite 29. Train A crosses an electric pole in 12 seconds and
direction at the speed of 54 kmph in 21.6 seconds. the speed of train B is 20 kmph more than the
If the ratio of the length of trains A and B is 4:5 speed of train A. If train A crosses train B is
respectively, then find the speed of train A? running in the opposite direction in 9 seconds and
(1) 27 (2) 54 also crosses 100m long tunnel in 16 seconds, then
(3) 45 (4) 36 find the length of train B?
(5) None of these (1) 150
(2) 300
25. A train crosses a car is running in the opposite (3) 250
direction at the speed of 24 kmph in 18 seconds. (4) 200
If the speed of train is double that of car, then find (5) 400
the time taken by the train to cross 140 m long
platform? 30. Train A crosses a 200 m long platform in 27
(1) 37.5 (2) 25 seconds and also crosses 300 m long tunnel in
(3) 30 (4) 28.8 31.5 seconds. Find the length of train A?
(5) 14.4 (1) 300
(2) 400
26. A train crosses a man standing on a platform in (3) 450
18 seconds and also crosses 240 m long stationery (4) 360
train in 30 seconds. Find the time taken by a train (5) 420
crossing another 200 m long train is running in
the opposite direction at the speed of 48 kmph? 31. Quantity I: A train running at a speed of 45
(1) 16.8 (2) 14.4 km/hr crosses a 90 m long platform in 20 seconds.
(3) 18 (4) 22.5 In how much time will it cross a 140 m long
(5) None of these platform?
Quantity II: A 150 m long train crosses a
27. The ratio of the length of trains A and B is 6:5 standing man in 20 seconds. In how much time
respectively and train A can crosses a platform of will it cross another man running at a speed of 9
540 m long in 15 seconds. If two trains A and B km/hr in the opposite direction?
are running at a speed of 60 m/s and 35 m/s (1) Quantity I > Quantity II
respectively, then find the time taken by train B (2) Quantity I < Quantity II
to cross the same platform? (3) Quantity I ≥ Quantity II
(1) 24 (2) 18 (4) Quantity I ≤ Quantity II
(3) 21 (4) 15 (5) Quantity I = Quantity II or Relation cannot be
(5) None of these Established.
32. Quantity I: A man covered 158 km in 3 hrs. 18 35. 250 metre long train running at the speed of 100
min. and completed the remaining journey at a kmph crosses another train running in opposite
speed of 45 km/hr. Find the average speed of his direction at the speed of 60 kmph in 9 seconds.
journey. Quantity I: The length of the other train
Quantity II: A train completes a journey of 480 Quantity II: The length of the first train shrinks
km in 10 hrs. 45 min. If the train halted for a total by 3/4th of that of the other train
time of 30 min. in the whole journey, find the (1) Quantity I > Quantity II
average speed of the train.
(2) Quantity I < Quantity II
(1) Quantity I > Quantity II
(3) Quantity I ≥ Quantity II
(2) Quantity I < Quantity II
(4) Quantity I ≤ Quantity II
(3) Quantity I ≥ Quantity II
(5) Quantity I = Quantity II or Relation cannot be
(4) Quantity I ≤ Quantity II
(5) Quantity I = Quantity II or Relation cannot be Established
Established.

33. Quantity I: 600 m length of train A crosses train 36. The speed of a 500 meters long train is 5 km per
B running in opposite direction in 36 seconds and hour more than that of a car. If the car and the
train A also crosses a man standing in a platform train travel in opposite direction then the car can
in 36 seconds. What is the speed of train B if the cross the train completely in 1.5 minutes.
length of train B is half of train A? Quantity I: What is the speed of the train?
Quantity II: A train crosses a electric pole in 18 Quantity II: What will be the speed of car when
seconds and crosses 125 m long tunnel in 36 it is increased by 50%?
seconds. What is the speed of the train? (1) Quantity I > Quantity II
(1) Quantity I > Quantity II (2) Quantity I < Quantity II
(2) Quantity I < Quantity II
(3) Quantity I ≥ Quantity II
(3) Quantity I ≥ Quantity II
(4) Quantity I ≤ Quantity II
(4) Quantity I ≤ Quantity II
(5) Quantity I = Quantity II or Relation cannot be
(5) Quantity I = Quantity II or Relation cannot be
Established
Established

34. Quantity I: Train A crosses train B running in


the opposite direction in 15 seconds and also 37. A train of length x meters travelling at the speed
crosses a man standing on a platform in 18 of 54 km per hour can cross a boy standing on a
seconds. If the length of train A is 50% more than platform in 16 seconds but at the speed of 72 km
that of train B and the speed of train B is 60 kmph, per hour it can cross a platform of y meters long
then find the length of train A? in 24 seconds
Quantity II: Train A crosses a car running in the Quantity I: What is the length of the train?
opposite direction at the speed of 45 kmph in 13.5 Quantity II: What is the length of the platform?
seconds and the speed of train A is 75 kmph. Find (1) Quantity I > Quantity II
the length of train A? (2) Quantity I < Quantity II
(1) Quantity I > Quantity II (3) Quantity I ≥ Quantity II
(2) Quantity I < Quantity II
(4) Quantity I ≤ Quantity II
(3) Quantity I ≥ Quantity II
(5) Quantity I = Quantity II or Relation cannot be
(4) Quantity I ≤ Quantity II
Established
(5) Quantity I = Quantity II or Relation cannot be
Established
38. Quantity I: A 300 m long train crosses a 150 m 40. In a race of 500 meters, A beats B by 50 meters
long tunnel at the speed of 108 km/h, then what but in a race of 1 km, B beats C by 250 meters.
is the time taken by the train to cross the tunnel? Quantity I: In a race of 800 meters, by how much
Quantity II: Train A of length 360 m crosses a distance will A beat C?
pole in 18 seconds. What is the time taken by train Quantity II: 250 meters
B of length 340 m coming from the opposite (1) Quantity I > Quantity II
direction running at the speed of 30m/s to cross (2) Quantity I < Quantity II
the running train A? (3) Quantity I ≥ Quantity II
(1) Quantity I > Quantity II (4) Quantity I ≤ Quantity II
(2) Quantity I < Quantity II (5) Quantity I = Quantity II or Relation cannot be
(3) Quantity I ≥ Quantity II Established
(4) Quantity I ≤ Quantity II
(5) Quantity I = Quantity II or Relation cannot be
Established

39. Find the distance if:


Quantity I: A man covers a distance in 15 hours.
He covers first half at 12 km/h and second half at
15 km/h.
Quantity II: Two buses moves towards each
other at a speed of 30 km/h and 40 km/h
respectively. When they meet it is found that
faster bus covers 30 km more than slower one.
(1) Quantity I > Quantity II
(2) Quantity I < Quantity II
(3) Quantity I ≥ Quantity II
(4) Quantity I ≤ Quantity II
(5) Quantity I = Quantity II or Relation cannot be
Established
Answer Key
1. (3) 9. (1) 17. (2) 25. (1) 33. (1)
2. (2) 10. (4) 18. (3) 26. (1) 34. (3)
3. (4) 11. (3) 19. (1) 27. (1) 35. (1)
4. (2) 12. (4) 20. (2) 28. (3) 36. (1)
5. (3) 13. (3) 21. (3) 29. (4) 37. (5)
6. (1) 14. (2) 22. (2) 30. (2) 38. (1)
7. (1) 15. (4) 23. (3) 31. (1) 39. (2)
8. (4) 16. (2) 24. (4) 32. (2) 40. (1)
Hints & Solutions
1. Solution (3) 6. Solution (1)
Speed of the train A=72*5/18=20 m/sec Length of the train=x
Length of the train A=20*18=360 m (x+420)=90*5/18*36
Length of the train B=360 m x=480 m
(20+x)=(360+360)/20 Speed of the train is decreased by
(20+x)=36 16.67%=90*5/6=75 m/s
x=16 m/sec Required time=(480+240)/75=720/75=9.6 seconds

2. Solution (2) 7. Solution (1)


Length of train A + 280 = 72 * 5/18 * 38 Length of train = (24 + 24 * 2) * 5/18 * 18
Length of train A = 480 m = 360 m
Length of train B = 480 – 80 = 400 m Required time = (360 + 140)/(24 * 2) * 5/18
400 + 100 = Speed of train B * 5/18 * 30 = 37.5 seconds
Speed of train B = 60 kmph
Required difference = 72 – 60 = 12 kmph 8. Solution (4)
Speed of train A = 3x
3. Solution (4) Speed of train B = 400/300 * 3x = 4x
Length of train = x Length of train A = y
Speed of train = y Length of train B = z
x + 360 = y * 5/18 * 32 y = 3x * 5/18 * 21.6
9x + 3240 = 80y y = 18x
x = y * 5/18 * 17.6 y + z = (4x – 3x) * 5/18 * 108
9x = 44y y + z = 30x
44y + 3240 = 80y z = 30x – 18x = 12x
y = 90 kmph Required time = 12x/4x * 5/18 = 10.8 seconds

4. Solution (2) 9. Solution (1)


Speed of train A = 4x Speed of train A=50*4/5=40 kmph
Speed of train B = 4x * 125/100 = 5x Relative speed=40+50=90 kmph
Length of train A = y Time taken by both trains A and B to cross each
y = 4x * 5/18 * 18 other=x
y = 20x 160+240=90*5/18*x
20x + 240 = (5x – 4x) * 5/18 * 120 400*18/5*1/90=x
60x + 240 * 3 = 100x x=16 seconds
x = 18
Length of train A = 18 * 20 = 360 m 10. Solution (4)
Length of train A=4x
5. Solution (3) Length of train B=5x
Length of train = x Let speed of train A =y
Speed of train = y 4x=y*5/18*24
x = y * 5/18 * 20.25 3x=5y
x = 5.625y 4x+5x=(y+54)*5/18*21.6
x + 150 = y * 5/18 * 27 3x=(2y+54*2)
2x + 300 = 15y 3x=2y+108
5.625y * 2 + 300 = 15y y=36 kmph
y = 80 kmph
11. Solution (3) 16. Solution (2)
Length of the train = x Length of train A = train B = x
Speed of train = y Speed of train A = y
x + 400 = y * 5/18 * 42 Distance = speed * time
3x + 1200 = 35y x = y * 5/18 * 8
x = (y + 20) * 5/18 * 13.5 9x = 20y
2x = 7.5y + 150 2x = (y + 60) * 5/18 * 9.6
2 * ((35y – 1200)/3) = 7.5y + 150 12x = 16y + 960
70y – 2400 = 22.5y + 450 12x = 16 * (9x/20) + 960
47.5y = 2850
60x = 36x + 4800
y = 60 km/hr
24x = 4800
x = 200 m
12. Solution (2)
Length of train A = 4x
Length of train B = 5x
17. Solution (2)
Speed of train A = 3y
Speed of train B = 4y Length of the train =x
9x = (4y – 3y) * 5/18 * 81 Length of platform = y
2x = y * 5, 4x = 3y * 5/18 * 12 x = 60 * 5/18 * 6
4x = 10y, 2x = 5y x = 100
We cannot find the answer. 100 + y = 60 * 5/18 * 18
y = 200 m
13. Solution (3)
The speed is equal. So,
(T.L + 300) / 24 = T.L / 8 18. Solution (3)
T.L + 300 = 3T.L Let the train length be x meter,
2T.L = 300, T.L = 150 The speed is equal while crossing the tunnel and
The length of train = 150 m electronic pole.
According to the question,
14. Solution (2) (x + 300)/15 = (x/6)
= Speed * Time 2x + 600 = 5x
Length of the train = x 5x – 2x = 600
Speed of train = y 3x = 600
x + 300 = y * 5/18 * 36
x = 200
x + 300 = 10y = y * 5/18 * 18
The length of train = 200 m
x = 5y, x + 300 = 2x
x = 300 m

19. Solution (1)


15. Solution (4)
The length of Train A = (1/2)* The length of Train
Length of train A = x
Length of train B = x + 100 B
Speed of train A = y The length of Train A: The length of Train B = 1: 2
2x + 100 = (y + 60) * 5/18 * 6 (x, 2x)
6x + 300 = 5y + 300 Speed of Train A = x/25
6x = 5y Speed of Train B = 2x/40 = x/20
x = y * 5/18 * 8 Required ratio = (x/25): (x/20) = 4: 5
We cannot be find the answer.
20. Solution (2) 25. Solution (1)
Distance = speed * time Length of train = (24 + 24 * 2) * 5/18 * 18
Speed of train A = x = 360 m
Speed of train B = y Required time = (360 + 140)/(24 * 2) * 5/18
350 = x * 5/18 * 18 = 37.5 seconds
x = 70 kmph
250 = y * 5/18 * 12 26. Solution (1)
750 = 10y Length of train = x
y = 75 kmph Speed of train = y
Difference = 75 – 70 = 5 kmph x = y * 5/18 * 18
x = 5y
21. Solution (3) x + 240 = y * 5/18 * 30
Length of train = x 3x + 240 * 3 = 25y
Speed of train = y 3x + 240 * 3 = 5x
x = y * 5/18 * 20.25 x = 360
x = 5.625y y = 360/5 = 72 kmph
x + 150 = y * 5/18 * 27 Required time = (360 + 200)/(72 + 48) * 5/18
2x + 300 = 15y = 16.8 seconds
5.625y * 2 + 300 = 15y
y = 80 kmph 27. Solution (1)
Length of train A=x
22. Solution (2) (x+540)/60=15
Speed of train B = 4x x=360 m
Speed of train A = 5x Length of train B=360*5/6=300 m
490 * 2 = (5x – 4x) * 5/18 * 196 Required time=(300+540)/35=840/35=24 seconds
x = 18
Speed of train B = 18 * 4 = 72 kmph 28. Solution (3)
Speed of train A = 5x
23. Solution (3) Speed of train B = 6x
Length of train A = 4x * 5/18 * 7.2 = 8x Speed of train A= 400/24 = 16.67 m/s
8x + 8x + 120 = x * 5/18 * 72 5x = 16.67, x= 3.33
16x + 120 = 20x Speed of train B = 6x = 20m/s
x = 30 Length of train B = 20* 15 = 300 m
Length of train B = 8 * 30 + 120 = 360 m
29. Solution (4)
Length of train A = x
24. Solution (4) Length of train B = y
Length of train A=4x Speed of train A = z
Length of train B=5x Speed of train B = z + 20
Let speed of train A =y x = z * 5/18 * 12
4x=y*5/18*24 3x = 10z
3x=5y x + 100 = z * 5/18 * 16
4x+5x=(y+54)*5/18*21.6 9x + 900 = 40z
3x=(2y+54*2) 9x + 900 = 12x, x = 300
3x=2y+108 z = 900/10 = 90 kmph
y=36 kmph 300 + y = (90 + 110) * 5/18 * 9
y = 200 meters
30. Solution (2) 33. Solution (1)
Length of train A + 200 Speed of train A = 600/36 * 18/5= 60 kmph
= x * 5/18 * 27 = 7.5x Length of train B= 600/2 = 300
Length of train A + 300 (Speed of train B + 60) * 5/18 =(600+300)/36
= x * 5/18 * 31.5 = 8.75x (Speed of train B + 60) * 5/18 = 900/36
7.5x – 200 = 8.75x – 300 Speed of train B = 30 kmph
100 = 1.25x Length of train = x
x = 80 Speed of train = y
Length of train A x = y * 5/18 * 18
= 7.5 * 80 – 200 = 400 m x = 5y
(x + 125) = y * 5/18 * 36
31. Solution (1) x + 125 = 10y
Speed of train = 45 km/hr = 45 × (5/18) 5y + 125 = 10y
= 12.5 m/sec y = 25 kmph
Now, time taken to cross the platform Quantity I > quantity II
= sum of lengths of train and platform/speed of
train 34. Solution (3)
⇒ 20 = (length of train + 90)/12.5 Let take, Length of train B = 2x
⇒ Length of train = (20 × 12.5) - 90 Length of train A = 2x * 150/100 = 3x
= 250 - 90 = 160 m Speed of train A = y
Time taken to cross 140 m long platform Speed = Length/Time
= (160 + 140)/12.5 = 300/12.5 = 24 sec. Y = 3x/18
⇒ Quantity I = 24 sec. 3x = y * 5/18 * 18
3x = 5y
Speed of train = length of train/time taken to cross
standing man = 150/20 = 7.5 m/sec. 3x + 2x = (y + 60) * 5/18 * 15
30x = 25y + 60 * 25
Speed of running man = 9 km/hr
30x = 15x + 60 * 25
= 9 × (5/18) = 2.5 m/sec.
x = 100
As the man is running in opposite direction,
Relative speed of train w.r.t. man Length of train A = 3 * 100 = 300 m
Speed of the car = 45 km/hr and speed of the
= 7.5 + 2.5 = 10 m/sec.
train = 75 km/hr
Time taken to cross the running man
Length = Speed * Time
= length of train/relative speed = 150/10 = 15 sec.
Length of train A = (75 + 45)* 5/18 * 13.5
⇒ Quantity II = 15 sec.
Length of train A= 450 m
32. Solution (2)
Quantity I < quantity II
Speed in the first part = 158/(3 + 18/60) = 47.88
km/hr.
35. Solution (1)
The average speed of two speeds ‘x’ km/hr and ‘y’
Relative speed = (100+60) km/hr
km/hr = 2xy/(x + y) km/hr
x + 250 = 400 x = 150 metres
Required average speed = (2 × 45 × 47.88)/(45 +
Quantity I: The length of other train is 150 m.
47.88) = 46.4 km/hr.
Quantity II: The length of the first train shrinks by
⇒ Quantity I = 46.4 km/hr
3/4th of that of the other train Therefore, 3/4 of
Distance travelled = 480 km 150 = 112.5 m
Time travelled = 10 + (45/60) - (30/60) = 10.25 hrs. Now, length of the first train = 250 - 112.5 = 137.5
Average speed = 480/10.25 = 46.8 km/hr m
⇒ Quantity II = 46.8 km/hr Quantity I > Quantity II
Quantity I < Quantity II
36. Solution (1) 39. Solution (2)
Let the speed of the car = x km per hr Quantity I:
The speed of the train = x + 5 km/hr Average speed=2*12*15/12+15
If they travel in opposite direction then the relative =13 1/3
speed = (x + x + 5) km per hr Distance=13 1/3*15=200 km
2x + 5 = 20 x = 7.5 km per hour Quantity II: Speed difference for 1 hour
Quantity I : The speed of the train = x + 5 = 12.5 = 40 – 30 = 10 Km/h
km per hr Therefore, In 1 hour faster bus will cover 10 km
Quantity II=150%of 7.5=11.25 more than slower one Hence to cover 30 km more
Quantity: I > Quantity : II
it will take 3 hours.
Distance= Relative Speed × Time D
37. Solution (5)
= (30 + 40) × 3 = 210 km
Quantity I: The length of the train = 15 × 16 = 240
Quantity II > Quantity I
meters
Let the length of the platform = y meters
then (240 + y) = 20 × 24 = 480 40. Solution (1)
y = 480 – 240 = 240 meters Case 1, When A goes for 500 meters B goes for 450
Quantity : I = Quantity : II meters A : B = 500 : 450 = 10 : 9
When B goes for 1000 meters, C goes for 750
38. Solution (1) meters B : C = 1000 : 750 = 4 : 3
Speed of train in m/s=108*5/18=30m/s A : B : C = 40 : 36 : 27
Time taken to cross the tunnel=300+150/30=15 Quantity I: When A travels 40x meters C travels
sec 27x meters
Quantity II : When A travels 40 × 20 = 800 meters
Speed of train A=360/18=20m/s C will travel 27 × 20 = 540 meters
Time taken to cross each other It means, A will beat C by 800 – 540 = 260 meters
360+340/20+30=14 sec Quantity I > Quantity II
Quantity I > Quantity II
नींव Booklet
For All Banking & Insurance Exam
Ch - 18 | Boat and Stream | Maths by Arun sir

Q.1 The boat can cover 120 km against the stream in 8 Q.6 Ratio of the speed of the boat A and B in still water
hours and the same boat cover 150 km along the is 3:2 and the boat A travels along the stream at the
stream in 6 hours. What is the ratio of the speed of speed of 40 kmph. If the difference between the
boat in still water to speed of stream? speed of boat A and B in still water is 12 kmph, find
A. 3:2 B. 8:7 the speed of the boat B travel against stream?
C. 4:1 D. 5:3 A. 15 B. 20
E. None of these C. 24 D. 18
E. None of these
Q.2 A boat can row 120 km downstream and comes
back to the original position in 24 hours. If the ratio Q.7 A boat takes 39 hours for travelling downstream
of the speed of stream to speed of the boat in still from point A to point B and coming back to point C
water is 2:3, then what is the speed of the boat in the midway between A and B. If the speed of the stream
still water? is 18 km/hr and the speed of the boat in still water
A. 15 B. 18 is 32 km/hr, then find the distance between A to B?
C. 21 D. 24 A. 600 B. 700
E. None of these C. 500 D. 800
E. None of these
Q.3 A boat can travel 16 km downstream in 32 minutes.
If the speed of the current is 1/4 of the speed of the Q.8 The boat travelled 80 km upstream in 16 hours and
boat in still water, what distance can the boat travel travelled 280 km downstream in 8 hours. If the boat
in 20 minutes? and stream’s speed is increased by 5 kmph and 3
A. 12 B. 16 kmph respectively, then what is the time taken by
C. 18 D. 8 the boat travelled 301 km in upstream and
E. 10 downstream?
A. 40 B. 45
Q.4 A boat takes two hours more to travel upstream than C. 50 D. 60
travel the same distance in downstream. If the E. CND
distance travelled by the boat is 120 km and the
ratio of the speed of stream to boat in still water is Q.9 Upstream speed of a boat is 4Km/h less than its
5: 1, then what is the speed of the boat in still water? downstream speed and speed of the boat in still
A. 20 B. 25 water is 12 Km/h. Time taken by the boat to go 210
C. 30 D. 35 Km downstream is what percent of the time taken
E. None of these by the boat to go 120 Km upstream?
A. 125% B. 110%
Q.5 A boat took 12 hours less to travel a certain distance C. 120% D. 105%
downstream than to travel the same distance E. None of these
upstream. If the speed of a boat in still water is 20
km/hr and speed of a stream is 12 km/hr, then find Q.10 Speed of the boat is 16(2/3)% more than the speed of
the total distance travelled by boat? the car which moves 120 km at 5 hours. The boat
A. 112 B. 140 covers a distance of 124 km downstream in 4 hrs.
C. 128 D. 156 Find the distance covered by the boat upstream in 6
E. None of these hrs.
A. 120 B. 160
नींव Booklet
For All Banking & Insurance Exam
Ch - 18 | Boat and Stream | Maths by Arun sir
C. 130 D. 100 of the original, time taken for the same journey will
E. None of these be 224 min. What is the ratio of the speed of boat in
still water and speed of current?
Q.11 Ratio between speed of boat in still water to speed A. 7:1 B. 6:1
of stream is 5 : 2. If 224 km is travelled by C. 5:3 D. 7:2
downstream in 4 hours then find the difference E. None of these
between speed of boat in still water and speed of
stream? Q.16 The speed of current is 5 km/h. What will be the
A. 24 B. 22 respective downstream speed and upstream speed
C. 28 D. 26 of a boy rowing a boat, if one third of the distance
E. 30 covered going downstream in a certain time is equal
to the distance covered going upstream in the same
Q.12 If the upstream speed of a boat is 50% less than the time?
downstream speed of the boat and if a object is A. 15,5 B. 20,10
thrown in the river it covers 100m in 50 sec, then C. 18,8 D. 24,14
how much distance boat can cover in still water in E. None of these
5 hours?
A. 900 B. 100 Q.17 There are 3 points P, Q and R in a straight line, such
C. 120 D. 108 that point Q is equidistant from points P and R. A
E. 105 man can swim from point P to R downstream in 24
hours and from Q to P upstream in 16 hours. Find
Q.13 A steamer can go 12 km in still water in 25 minutes. the ratio of speed of man in still water to speed of
One day, it went 11.25 km upstream and returned stream?
the same distance in downstream. If the difference A. 5:1 B. 6:1
between the time taken to travel upstream and C. 5:3 D. 7:1
downstream was 12.5 minutes, then what was the E. None of these
speed of stream in km per hour?
A. 7.2 B. 5.4 Q.18 Rohit can row a boat 65Km upstream and 130 Km
C. 6.3 D. 4.5 downstream in 23 hours, whereas he can swim 45
E. None of these Km upstream and 104Km downstream in 17 hours.
Find the speed of boat in still water and the speed
Q.14 The ratio of speed of A and B in still water is 3 : 2. A of stream?
and B start from the same point in the river, A goes A. 4, 9 B. 8, 5
upstream and B goes downstream. After 3 hours the C. 9, 4 D. 5, 8
stream stops flowing and A starts rowing in the E. 10, 3
opposite direction to meet B. How much time after
the stream stops flowing does A meet B? Q.19 If the ratio of the speed of a boat in upstream and
A. 16 B. 15 the speed of the stream is 8 : 1. If the boat can travel
C. 12 D. 18 500 km downstream in 20 hours then find the total
E. None of these distance travelled by the boat in still water in the
same time?
Q.15 A boat goes a certain distance upstream and comes A. 425 B. 459
back downstream to the starting point in 144 min. If C. 441 D. 450
the speed of the boat in still water becomes 66.67% E. None of these
नींव Booklet
For All Banking & Insurance Exam
Ch - 18 | Boat and Stream | Maths by Arun sir
upstream is 66.67% of the downstream speed. What
Q.20 The ratio of the time taken by a boat to go a certain is the speed of boat in still water?
distance upstream and return downstream halfway A. 21 B. 14
of the same distance is 18 : 5. If the speed of stream C. 15 D. 18
is 4km/h what is the distance covered by the boat in E. None of these
6 hours in downstream and 8 hours in upstream?
A. 192 B. 188 Q.25 A boat goes downstream and reaches a fixed point
C. 208 D. 175 in a river in 40 minutes, turns back and again sails
E. None of these upstream to reach the starting point. The speed of
stream is constant all the time and is 1/5 times the
Q.21 The time taken by a boat to go a certain distance speed of the boat. By what percent the speed of the
downstream is 33.33% less than that taken during boat should be increased in upstream so that it takes
upstream. If the still water speed of the boat same time to reach the starting point as it took in
decreases by 40%, it will take 2 hrs more to travel a downstream?
distance of 30km. How much distance will it cover A. 10% B. 20%
in 5hrs going downstream? C. 30% D. 40%
A. 36 B. 48 E. None of these
C. 60 D. 45
E. 72 Q.26 A boat starts from a point, goes upstream to some
distance and returns back downstream at its initial
Q.22 Speed of the stream is 12 km/h and speed of boat in position in 4 hours such that the time taken for the
still water is 20 km/h. A boat takes a total of 36 upstream journey was 2 hours more than the
hours for travelling downstream from Point X to downstream journey. What was the total distance it
Point Y and coming back to point Z which is travelled if speed of the boat in still water was 4
midway of point X and Y. What is the distance kmph?
between X and Y ? A. 8 B. 12
A. 384 B. 396 C. 15 D. 18
C. 352 D. 288 E. None of these
E. None of these
Q.27 A motorboat goes 48 km of upstream and comes
Q.23 Time taken by a boatman to travel (x + 40) km back to its starting point in 15 hrs. if the speed of
downstream and (x – 40) km upstream is 20 hours. the boat in still water is 66.66% more than that of
The downstream speed is 50% more than the the speed of the stream. If boat had travelled only in
upstream speed. If boatman can travel (x + 40) km upstream for 15 hours then find how much distance
in 12 hours in still water, then find the value of x? it would have travelled?
A. 140 B. 160 A. 240 B. 225
C. 180 D. 200 C. 180 D. 165
E. 240 E. None of these

Q.24 Jay rows a boat 216 km upstream and rows same Q.28 The speed of boat A is 2 km/hr less than the speed
distance downstream in a total of 30 hrs without any of boat B. The time taken by boat A to cover a
break. The speed in downstream is 20% more than distance of 20 Km downstream is 30 min more than
the speed of boat in still water and the speed in the time taken by B to cover the same distance
नींव Booklet
For All Banking & Insurance Exam
Ch - 18 | Boat and Stream | Maths by Arun sir
downstream. If the speed of the current is one-third Quantity II: The speed of the stream is how much
of the speed of boat A, what is the speed of boat B? percentage less than the speed of the motorboat in
A. 9 B. 8 still water?
C. 7 D. 6 A. Quantity I > Quantity II
E. None of these B. Quantity I < Quantity II
C. Quantity I ≥ Quantity II
Q.29 A boat goes to a place 60 km distant and comes back D. Quantity I ≤ Quantity II
in 35 hours. If it takes equal time to row 4 km with E. Quantity I = Quantity II or Relation cannot be
the stream that it takes to row 3 km against the Established.
stream. Find the average of the speed in still water
and speed in stream? Q.33 A motorboat can travel x km upstream and x + 20
A. 3 B. 1.5 km downstream in 17.5 hours. If the ratio of the
C. 2 D. 2.5 speed of the motorboat in still water to the seed of
E. None of these stream is 3: 1 and the difference between their speed
is 4 km.
Q.30 Speed of boat in still water is x km/hr and the speed
of the stream is y km/hr. It can cover 52 km Quantity I: What is the value of x?
downstream and 35 km upstream in 9 hours. Find Quantity II: How much distance the motorboat
the value of y if the boat can cover 63 km upstream will travel downstream in 5 hours 15 minutes?
and 78 km downstream in 15 hours? A. Quantity I > Quantity II
A. 13 B. 10 B. Quantity I < Quantity II
C. 7 D. 6 C. Quantity I ≥ Quantity II
E. None of these D. Quantity I ≤ Quantity II
E. Quantity I = Quantity II or Relation cannot be
Q.31 Quantity I: Speed of boat in still water, if a man Established.
can travel 54 km downstream in 9 hours and 40 km
upstream in 10 hours. Q.34 Quantity I: The ratio of speed of stream and speed
Quantity II: Speed of boat in still water, if a man of a boat is 1 : 3 respectively. If the boat can cover
can travel 45 km downstream in 9 hours and the 420 km while travelling downstream in 7 hours,
speed of stream is 1 kmph. then how long would it take to cover 270 km while
A. Quantity I > Quantity II travelling upstream?
B. Quantity I < Quantity II Quantity II: Boat M can travel 400 km upstream
C. Quantity I ≥ Quantity II and then 400 km downstream in total time of 18
D. Quantity I ≤ Quantity II hours. If the speed of the boat is 45 km/hr, then how
E. Quantity I = Quantity II or Relation cannot be long would it take boat M to only travel
Established. downstream 400 km?
A. Quantity I > Quantity II
Q.32 The speed of a motorboat in upstream is 75% less B. Quantity I < Quantity II
than that of downstream. C. Quantity I ≥ Quantity II
Quantity I: The speed of the stream is how much D. Quantity I ≤ Quantity II
percentage of the speed of the motorboat in E. Quantity I = Quantity II or Relation cannot be
downstream? Established.
नींव Booklet
For All Banking & Insurance Exam
Ch - 18 | Boat and Stream | Maths by Arun sir
Q.35 Quantity I: Boat A travels in a stream and takes 6.4 Q.38 Quantity II: Naveen can swim at 16 km/hr in still
hrs to complete a round trip, going 78 km in each water. The river flows at 4 km/hr and it takes 8
direction. If the ratio of speed in still water of the hours more upstream than downstream for the same
boat to the stream speed is 8:5, what is the speed in
distance. How far is the place?
still water of the boat?
Quantity III: 200 km
Quantity II: Amrit travels at the speed of 42 km/hr
for 3 hrs and 36 km/hr for 2 hrs, what is his average A. Quantity I > Quantity II
speed for the journey of 5 hrs? B. Quantity I < Quantity II
A. Quantity I > Quantity II C. Quantity I ≥ Quantity II
B. Quantity I < Quantity II D. Quantity I ≤ Quantity II
C. Quantity I ≥ Quantity II E. Quantity I = Quantity II or Relation cannot be
D. Quantity I ≤ Quantity II Established.
E. Quantity I = Quantity II or Relation cannot be
Established.
Q.36 Speed of boat A in still water is 10% more than the
speed of boat B in still water. Boat A can go 144
Km downstream in 9 hours and boat B can go 75
Km upstream in 15 hours in the same river.
Quantity I: Find the upstream distance travelled by
boat A in 12 hours.
Quantity II: Find the downstream distance
travelled by boat B in 6 hours.
A. Quantity I > Quantity II
B. Quantity I < Quantity II
C. Quantity I ≥ Quantity II
D. Quantity I ≤ Quantity II
E. Quantity I = Quantity II or Relation cannot be
Established.

Q.37 Quantity I: A man rows to a place 48 km apart and


back in 16 hours. He finds that he can row 6 km
downstream and 4 km upstream in the same time.
The speed of the stream is?
Quantity II: A man can row a boat at a speed of 10
km/hr in still water. He goes to a certain point
upstream and back to the starting point in a river.
The speed of the flowing water is 4 km/hr. What is
the average speed of the boat for that journey?
A. Quantity I > Quantity II
B. Quantity I < Quantity II
C. Quantity I ≥ Quantity II
D. Quantity I ≤ Quantity II
E. Quantity I = Quantity II or Relation cannot be
Established.
नींव Booklet
For All Banking & Insurance Exam
Ch - 18 | Boat and Stream | Maths by Arun sir
Q.39 Quantity I: A man takes 20 minutes to row 12 km
upstream which is a third more than the time he
takes on his way downstream. What is his speed in
still water?
Quantity II: A boat makes a return journey from
point A to point B and back in 5 hours 36 minutes.
One way it travels with the stream and on the return
it travels against the stream. If the speed of the
stream increases by 2 km/hr, the return journey
takes 9 hours 20 minutes. What is the speed of the
boat in still water? (The distance between A and B
is 16 km.)
A. Quantity I > Quantity II
B. Quantity I < Quantity II
C. Quantity I ≥ Quantity II
D. Quantity I ≤ Quantity II
E. Quantity I = Quantity II or Relation cannot be
Established.
Q.40 Quantity I: A man rows 24 km upstream in 6 hours
and a distance of 35 km downstream in 7 hours.
Then the speed of the man in still water is
Quantity II: A motorboat can travel at 5 km/hr in
still water. It travelled 90 km downstream in a river
and then returned, taking altogether 100 hours. Find
the rate of flow of the river.
A. Quantity I > Quantity II
B. Quantity I < Quantity II
C. Quantity I ≥ Quantity II
D. Quantity I ≤ Quantity II
E. Quantity I = Quantity II or Relation cannot be
Established.

Hints & Solutions


1. Solution => 5 kmph
Downstream speed = 150/6 = 25 kmph Required ratio = 20:5
Upstream speed = 120/8 = 15 kmph = 4:1
Speed of the boat in still water = 1/2(downstream 2. Solution
speed + upstream speed) Downstream speed = 2x + 3x = 5x
=> 1/2(25 + 15) = 20 kmph Upstream speed = 3x – 2x = x
Speed of stream=1/2 (downstream speed – upstream 120/5x + 120/x = 24
speed) (120 + 600)/5x = 24
=> 1/2 (25 – 15) 720 = 120x
नींव Booklet
For All Banking & Insurance Exam
Ch - 18 | Boat and Stream | Maths by Arun sir
x=6 x = (39 * 50 * 28) / 78 = 700 km
Speed of the boat in still water = 6 * 3 = 18 kmph The distance between A to B = 700 km
3. Solution 8. Solution
Let the speed of the boat in still water be x km/hr Upstream speed = 80/16 = 5 kmph
Speed of the current= x/4 Downstream speed = 280/8 = 35 kmph
Downstream speed = x + x/4= 5x/4 Speed of the boat = (35 + 5)/2 = 20 kmph
Downstream speed in km/hr = 16*60/28= 30 km/hr Speed of stream = (35 – 5)/2 = 15 kmph
So, After increasing the boat = 20 + 5 = 25 kmph
5x/4= 30 Stream speed = 15 + 3 = 18 kmph
x = 24 km/hr Required time = 301/(25 + 18) + 301/(25 – 18) = 7 +
Distance in 20 minutes= 24*20/60= 8 km 43 = 50 hours
4. Solution 9. Solution
Distance = speed * time Let, speed of the boat in still water = a Km/h
120/4x – 120/6x = 2 And speed of the stream = b Km/h
(30 – 20)/x = 2 (a + b) – (a – b) = 4
10 = 2x => a + b – a + b = 4
=>x=5 => 2b = 4
Speed of the boat in still water = 5 * 5 = 25 kmph => b = 4/2
5. Solution => b = 2 Km/h
Let the total distance be x, a = 12 Km/h
Speed of downstream = 20 + 12 = 32 km/hr Time taken by the boat to go 210 Km downstream
Speed of upstream = 20 – 12 = 8 km/hr = 210/(12 + 2) = 210/14 = 15 hours
According to the question, Time taken by the boat to go 120 Km upstream
12 = (x/8) – (x/32) = 120/(12 – 2) = 120/10
12 = (4x – x)/32 = 12 hours
(12 * 32) = 3x Required percentage = 15/12 x 100 = 125%
x = (12 * 32) / 3 = 128 km 10. Solution
The total distance travelled by boat = 128 km Speed of the boat = a km/hr = (1+ 50/300) x speed of
6. Solution the car
Speed of the boat A in still water = 3 * 12 = 36 kmph = 7/6 x 120/5
Speed of the boat B in still water = 2 * 12 = 24 kmph = 28 km/hr
Downstream Speed of the boat A = 40 kmph Speed of the stream = b km/hr
Stream speed = 40 – 36 = 4 kmph ATQ,
Upstream speed of boat B = 24 – 4 = 20 kmph 28 + b = 124/4 = 31
7. Solution b = 3 km/h
Speed of downstream = 32 + 18 = 50 km/hr 28 – 3 = D/6
Speed of upstream = 32 – 18 = 14 km/hr D = 25 x 6 = 150 km
Total time = 39 hours 11. Solution
If distance between A to B is x, then distance between Let the speed of boat in still water and speed of
B to C = x/2, stream be 5x and 2x respectively.
Time = Distance/Speed According to the question,
x / 50 + (x/2) / 14 = 39 224/4 = 5x + 2x
x / 50 + x / 28 = 39 x =32/ 8 =4
78x / (50 * 28) = 39 Required difference = 5x – 2x = 3x
नींव Booklet
For All Banking & Insurance Exam
Ch - 18 | Boat and Stream | Maths by Arun sir
3x = 24 km/hr B2 = 4C2 →B/C=2/1
12. Solution 3B/C = 6/ 1
Let the speed of boat in still water be s m/sec The ratio of speed of boat and current in 6 : 1
As the object covers 100m in 50 sec , so the distance 16. Solution
travelled by the object will be with the help of the Let the speed of boy in still water be X km/h
speed of the stream only. And the speed of current is given = 5 km/h
Speed of stream =100/50= 2 m/s Downstream speed = (X + 5) km/h
Given that, Upstream speed = (X – 5) km/h
(s – 2) = (s + 2) – 50 (s + 2)/100 Let time be ‘t’ hours.
(s – 2) = (s + 2)(1 – 1/2) ⇒(X + 5) t/3= (X – 5) t
s = 6 m/sec ⇒ X + 5 = 3X – 15
s =6 × 18/5= 108/5km/hr ⇒ 2X = 20
Distance covered by the boat in still water in 5 hours= ⇒ X = 10 km/h
108 ×5/5= 108 km Downstream speed = 10 + 5 = 15 km/h
13. Solution Upstream speed = 10 – 5 = 5 km/h
In still water, the speed of steamer =12000/25 = 480 17. Solution
meter per minute = 8 meters per second Let speed of man in still water = x km/h
Let the speed of stream = v m/sec Speed of current = y km/h
In upstream, the speed of steamer = (8 – v) m/sec Downstream speed = (x + y) km/h
In downstream, the speed of steamer = (8 + v) m/sec Upstream speed = (x – y) km/h
According to the question, Let PQ = QR = A and PR = 2A
11250/8 – v – 11250/8 + v= 12.5 × 60 = 750 seconds 2A/ x + y = 24 …..(I)
By solving, v = 2 meters per second And A/ x – y= 16….(ii)
=2 × 18/5 = 7.2 km per hour By dividing both equations-
14. olution ⇒2A (x – y)/ A (x + y) =24/16
Let the speed of current be = c km/hr ⇒ 4x – 4y = 3x + 3y
Let the speeds of A and B in still water be 3k km/hr ⇒x/ y =7/1
and 2k km/hr Required ratio = Speed of man in still water : Speed
Upstream speed of A = (3k – c) km/hr of current ⇒ 7 : 1
Downstream speed of B = (2k + c) km/hr
Distance between A and B after 3 hrs = [3 (3k – c) + 18. Solution
3 (2k + c)] km = (9k – 3c + 6k + 3c) km = 15k km Upstream, U = Speed of boat – speed of stream
After stream becomes stationary Downstream, D = Speed of boat + speed of stream
Relative speed when both go in same direction = (3k 65/ U +130/ D= 23…..i
– 2k) km/hr = k km/hr 45/ U + 104/ D = 17…..ii
Time after which they meet = 15 k/ k = 15 hrs On solving the above two equations, we will get
15. Solution U = Speed of boat – speed of stream = 5
Let the distance be D and Speed of boat in still water D = Speed of boat + speed of stream = 13
be 3B and that of current be C Thus, Speed of boat = 9 and speed of stream = 4
D/3B + C +D/3B – C = 144 ..........eq.(i) 19. Solution
When the speed of the boat becomes 66.67% → new Let the speed of boat in upstream = 8x km/hr
boat speed = 2B And the speed of the stream = x km/hr
D/2B + C+ D/2B – C = 224 ..........eq.(ii) Speed of boat in downstream =500/20= 25 km/hr
Dividing eq. (i) by eq. (ii) Let the speed of boat in still water = p km/hr
नींव Booklet
For All Banking & Insurance Exam
Ch - 18 | Boat and Stream | Maths by Arun sir
Then, p + x = 25 km/hr ..........(i) ⇒m/32+1/2×m/8= 36
P – x = 8x , p = 9x ...........(ii) ⇒3m/32 = 36
Put the value of p in the equation (i) ⇒ m =36 × 32/3
10x = 25, x = 2.5 ⇒ m = 384 km
From the equation (ii) speed of boat in still water = 23. Solution
9x = 9 × 2.5 = 22.5 km/hr Let, the speed of boatman in still water and stream
The total distance travelled by the boat in still water speed = ‘B’ km/hr and ‘s’ km/hr respectively
in 20 hours = 450 km According to the question,
20. Solution (B + s) = 1.5 (B – s)
Let the speed of boat = b and current = c 2.5s = 0.5B
Speed downstream = b + c, Speed upstream = b – c B = 5s
Let the distance boat goes upstream = 2d and returns x + 40/ B + s + x – 40/ B – s = 20
downstream is d (x + 40) (B – s) + (x – 40) (B + s) = 20 (B + s) (B –
S down = b + c, S up = b – c s)
(b + c)/ (b – c)=9/5 B = 5s
b/c= 14/ 4 (x + 40) (4s) + (x – 40) (6s) = 20 (6s) (4s)
c = 4 km/h so, b = 14 km/h Also, x + 40/B= 12
Speed down = 14 + 4 = 18 km/h, Speed up = 14 – 4 x + 40 = 60s
= 10 km/h (x + 40) (x + 40)/15+ (x – 40) (x + 40)/10 = 8 (x +
Distance covered = 18 × 6 + 10 × 8 = 188 km 40) (x + 40)/60
21. Solution 4(x + 40)2 + 6 (x – 40) (x + 40) = 8 (x + 40)2
Sb = Speed of boat in still water, 6 (x – 40) (x + 40) = 4 (x + 40)2
Sc = speed of current 3x – 120 = 2x + 80
Time downstream/ Time upstream =2/3 3x – 2x = 120 + 80
S down/ S up =3/2 x = 200
=Sb + Sc/ Sb – Sc=3/ 2 24. Solution
Sb/ Sc =5/1 Let speed of boat in still water = 100k km/hr
Let the still water speed of boat = 5k Then downstream speed = 120k km/hr and upstream
If the still water speed decreases by 40% it becomes speed = 80k km/hr
3k 216/120k + 216/80k = 30
As the speed decreases in the ratio 5 : 3, So time taken k =3/20
will increase in the ratio 3 : 5 Speed of boat in still water = 15 km/h
Let the original time be 3t and new be 5t → 5t – 3t =
2t = 2 hrs
So, original time = 3t = 3hrs for distance of 30km 25. Solution
So speed in still water 5k =30/3= 10 km/h Let the speed of the boat be ‘5b’ kmph and that of the
Sc = 1k = 2 km/h stream be ‘b’ kmph, and the distance it travels in one
Downstream speed = 12 km/h direction is ‘y’, then
Distance covered in 5 hrs = 12 × 5 = 60 km while going downstream, we have
22. Solution y/(5b + b) = 40/60=2/3hours
Let the distance between X and Y is m km. y/6b =2/3 hours ...... (i)
Speed of boat in upstream = 20 – 12 = 8 kmph Now, it increases the speed of the boat by S%, then
Speed of boat in downstream = 20 + 12 = 32 kmph the new speed while returning back would be = (1 +S/
It takes 36 hours to complete the path. 100 ) 5b kmph
नींव Booklet
For All Banking & Insurance Exam
Ch - 18 | Boat and Stream | Maths by Arun sir
We now can write 2x2 – 5x – 88 = 0
y/{(1 + S / 100) 5b – b}=40/60 =2/3 hours ...... (ii) x = 8, –11/2
Equating (i) and (ii), we get So, speed of boat B is 8 kmph
S/100= 0.4 29. Solution
S = 40% If it goes 4 km downstream in x hours
26. Solution Downstream speed =4/x
Let the total distance was 2y km and the speed of the Upstream speed =3/x
stream was v kmph. ⇒ 60 3x + 4x/12= 35
If the time to go upstream was t hours then time to go ⇒ 60 × 7x/12= 35
downstream = (4 – t) hours ⇒ X = 1 km
It is given that time to go upstream was 2 hours more Then downstream speed = 4 km/h
than time to go downstream, so Upstream speed = 3 km/h
(4 – t) + 2 = t Speed in still water = U =4 + 3/2= 3.5 km/h
t = 3 hours Speed in stream = V =4 – 3/2= 0.5 km/h
Now, time to go upstream Required average =3.5 + 0.5/2= 2 km/h
3 =y/(4 – v) 30. Solution
12 – 3v = y ---(i) Downstream speed of boat = x + y kmph
time to go downstream Upstream speed of boat = x – y kmph
1 =y/(4 + v) According to the question,
4 + v = y ---(ii) =52/ x + y +35/ x – y= 9
Solving equation (i) & (ii) we get Let 1/ x + y=a and 1/ x – y= b
y=6 So the equation becomes,
Total distance = 2y = 2 × 6 = 12 km 52a + 35b = 9
27. Solution 52a = 9 – 35b
the speed of the boat in still water is 66.66% more Also given that
than that of the speed of stream =78/ x + y+ 63/ x – y= 15
Let the speed of stream = x km/hr So, 78a + 63y = 15
Then speed of the boat will become = 166.66% of 702 – 2730b + 3276b = 780
x=5x/3km/hr 546b = 78
Speed in upstream =5x/3– x =2x/3km/hr .........(i) b =1/7
Speed in downstream =5x/3+ x = 8x/3km/hr .........(ii) So, x + y = 13 and x – y = 7
By solving this, So, x = 10 and y = 3
X=6 So, the value of y = 3 kmph
From the equation (i)
Speed of upstream = 2 ×6/3= 4km/hr
The total distance travelled by him in upstream in 15
hrs = 15 × 4 = 60 km
28. Solution 31. Solution
Let speed of Boat B = x kmph and speed of boat A = Let speed of boat = x kmph,
(x – 2) kmph speed of stream = y kmph
So, speed of current = ( x – 2 /3) kmph upstream speed = x – y=40/10=4
Now according to the question downstream speed = x + y=54/9=6
20 × 3/3x – 6 + x – 2 =20 × 3/3x + x – 2+1/2 equation 1 + equation 2
30x – 15 – 30x + 60/(x – 2) (2x + 1)=1/2 x = 5 speed of boat = 5 kmph
नींव Booklet
For All Banking & Insurance Exam
Ch - 18 | Boat and Stream | Maths by Arun sir
Quantity II: Quantity I: Let the speed of the boat and the speed
Let speed of boat = x kmph, of the stream be ‘3x’ km/hr
speed of stream = 1 kmph and ‘x’ km/hr respectively
downstream speed = x + 1 According to the question
45/9=x+1 420/3x+x=7
5 – 1 = x x = 4 Speed of boat = 4 kmph X=15
Quantity I > Quantity II So, the speed of the boat and the speed of the stream
32. Solution be ‘45’ km/hr
Let the speed of the motorboat in still water = u km and ‘15’ km/hr respectively So, the time taken by
per hour boat to cover 270 km while travelling upstream
and the speed of the stream = v km per hour 270/30=9 hrs
The speed of the motorboat in downstream = (u + v) Quantity II : Let the speed of the stream be ‘x’ km/hr
km per hour According to the question,
And the speed of the motorboat in upstream 4000/50=8 hrs
= (u – v) km per hour 35. Solution
According to the question, (u – v) = (100 – 75)% of Quantity I:
(u + v) = 25% of (u + v) 4u – 4v = u + v Let the speed in still water of the boat and the stream
3u = 5v speed be 8k km/hr and 5k km/hr.
u:v=5:3 So, 78/(8k + 5k) + 78/(8k - 5k) = 6.4
Quantity I: The speed of the motorboat in 6/k + 26/k = 6.4
downstream = 5x + 3x = 8x km per hour 32/k = 6.4
Req%=3x*100/8x=37.5% k=5
Quantity II: Speed of the boat in still water = 8*5 = 40 km/hr
Req%=(5-3)100/5=40% Quantity II:
quantity I < quantity II Average speed of Amrit = (42*3 + 36*2)/5 = 39.6
33. Solution km/hr
Let the speed of the motorboat in still water = 3a 36. Solution
km/hr Let, speed of boat B in still water = k Km/h
then the speed of the motorboat in stream = a km/hr => Speed of boat A in still water = k x 110/100 =
According to the question, 3a – a = 2a = 4 11k/10 Km/h
a = 2 km/hr And speed of the stream = y Km/h
the of the motorboat in still water = 3a km/hr = 6 144/(11k/10 + y) = 9
km/hr => 144 = 99k/10 + 9y
the speed of the motorboat in stream = a km/hr = 2 => 1440 = 99k + 90y
km per hour => 99k + 90y = 1440 --------- (i)
Upstream speed = 6 – 2 = 4 km/hr 75/(k - y) = 15
Downstream speed = 6 + 2 = 8 km per hour After => 5 = k – y
Solving speed for x , => k - y = 5 ----------- (ii)
12x = 560 – 80 = 480 Equation (i) - 99 x Equation (ii)
x = 40 99k + 90y - 99k + 99y = 1440 - 495
Quantity I: 40 => 189y = 945
Quantity II :distance=8*21/4=42 => y = 945/189
Quantity I < Quantity II => y = 5
34. Solution From (ii)
नींव Booklet
For All Banking & Insurance Exam
Ch - 18 | Boat and Stream | Maths by Arun sir
k-5=5 = > 8.4 km/hr
=> k = 10 Quantity I < Quantity II
Speed of boat B in still water = 10 Km/h 38. Solution
Speed of boat A in still water = 10 x 110/100 = 11 Quantity II:
Km/h Let the distance be x km,
Speed of the stream = 5 Km/h x / (16 - 4) – x / (16 + 4) = 8
Quantity I: x / 12 – x / 20 = 8
Upstream distance travelled by boat A in 12 hours = 8x / (12 * 20) = 8
(11 - 5) x 12 x = 240 km
= 6 x 12 Quantity III:
= 72 Km 200 km
Downstream distance travelled by boat B in 6 hours Quantity I > Quantity II
= (10 + 5) x 6 39. Solution
= 15 x 6 Let the speed in still water = x km/hr. Takes 20 min.
= 90 Km to row 12 km upstream ⇒ speed of u/s = 36 km/hr.
Quantity I < Quantity II Also, time taken for u/s is 1/3 more than for d/s.
37. Solution distance covered in d / s will be 1/3 more.
Let the time of downstream and upstream x. Hence distance covered by man for d / s in 20 min. =
Speed of downstream= 6/x km/hr 12 × (12/3) = 16km.
Speed of upstream= 4/x km/hr So speed of d / s = 48 km/hr.
48/(6/x) + 48/(4/x) = 16 x + y = 48 and x – y = 36 ⇒ x = 42 km/hr.
= > 20x=16 Let x be speed of u / s and y be the speed of d / s.
= > x= 4/5 =0.8 (16/x) + (16/y) = (28/5) and 16/(y+2) + 16/(x-2) =
Speed of downstream= 6/x = 6/0.8 =7.5 28/3
km/hr Solving these 2 equations, we get x = 4km/hr and y =
Speed of upstream= 4/x = 4/0.8 = 5 km/hr 10km/hr
The speed of the stream is, speed of boat in still water = (4+10) / 2 = 7km/hr.
= > ½ (7.5 - 5) Quantity I > Quantity II
= > 2.5/2 = 1.25 km/hr 40. Solution
Quantity II: Speed of upstream = 24 / 6 = 4 km / hr. Speed of
Speed of still water = 10 km/hr downstream = 35 / 7 = 5km / hr.
Speed of the flowing water = 4 km/hr Speed of man in still water = (4 + 5) / 2 = 4.5 km / hr.
The average speed of the boat for that Speed of boat in still water = x = 5 km/hr.
journey is, Let rate of flow of river = y km/hr. ∴ Speed of u/s =
= > Total Distance / Time 5- y and speed of d / s = 5 + y
= > 2x/ (x/(10+4) + (x/ (10-4)) 90/(5+y) + 90/(5-y) = 100 ⇒ y = 4 km/hr.
= > 2x/ (x/14 + x/6) Quantity I > Quantity II
= > 84/10
1. If the ratio of the radius and height of the cone is 6. Ratio of the height to length of the cuboids is 6:5
3:7. What is the sum of double the height and and the breadth of the cuboid is 8 cm. If the
one third of the radius of the cone, if the volume volume of the cuboid is 2160 cm3 and the
of the cone is 1782cm3? perimeter of a rectangle is 84 cm and the breadth
(1) 96 (2) 45 of the rectangle is equal to the height of the
(3) 50 (4) 52 cuboid, then find the area of the rectangle?
(5) None of these (1) 414 (2) 396
(3) 448 (4) 428
2. The circumference of the semicircle is 72 cm (5) 432
and the length of the rectangle is twice the radius
of the semicircle. If the area of the rectangle is
7. The area of the rectangle is 28.56% more than
448 cm2, then find the perimeter of the
the area of the circle in which the breadth of the
rectangle?
rectangle is 11 cm and the diameter of the circle
(1) 72 (2) 68
is 14cm. If the height of the cuboid is 83.33%
(3) 84 (4) 88
(5) 64 more than the length of the rectangle, then find
the height of the cuboid?
3. The radius of the circle is equal to the diagonal (1) 31 (2) 35
of the square whose area is 18 cm2 . find the (3) 33 (4) 37
curved surface area of the cylinder whose radius (5) None of these
is equal to the radius of the circle. The height of
the cylinder is seven-third of the radius of the 8. The breadth of the rectangle is 2 cm more than
cylinder? the side of the cube whose surface area is 1536
(1) 628 (2) 528 cm2 If the ratio of the length of the rectangle to
(3) 550 (4) 525 the side of the square is 6:5 and the perimeter of
(5) None of these the rectangle is 84 cm, then what is the
difference between the area of the rectangle and
4. The ratio of the area of the rectangle and square square?
is 2:3. If the side of the square and the length of (1) 32 (2) 36
the rectangle is equal and the perimeter of the (3) 28 (4) 24
rectangle is 60 cm, then find the perimeter of the (5) None of these
square?
(1) 68 (2) 72
(3) 80 (4) 84
9. The sum of the length, breadth of the rectangle
(5) cnd
and side of the square is 120 meters. If the
perimeter of the rectangle is 180 meters. Then
5. If the ratio of the radius of the cylinder to the
cone is 3:2 and the ratio of the height of the cone find the area of the square?
to the cylinder is 3:5, then what is the ratio of (1) 960 (2) 900
the volume of the cone to the cylinder? (3) 600 (4) 400
(1) 3:46 (2) 20:27 (5) None of these
(3) 20:31 (4) 4:45
(5) None of these
10. The length of a rectangle is 80% of the diagonal 15. Ratio of the length and breadth of a rectangle is
of a square of area 1225 cm2. Then find the area 7:9 and also the ratio of its perimeter and area is
of the rectangle, if its perimeter is 94√2 cm. 1:2. If the area of the rectangle is 25% of area of
(1) 1064 (2) 500 the square whose side is 16 cm, then find the
(3) 1604 (4) 1016 breadth of the rectangle?
(5) None of these (1) 3 (2) 14
(3) 5 (4) 17
11. The radius of Cone is equal to radius of the (5) 9
circle whose circumference is 88 cm. Then find
thecurved surface area of cone in cm², if the
16. If the circumference of the circle is 132 cm and
height of cone is 2 cm.
the radius of the circle is equal to the length of
(1) 740√2cm² (2)440√2 the rectangle whose perimeter is 80 cm, find the
(3) 517√2 (4) 540√2 area of the rectangle?
(5) 640√2 (1)369 (2) 379
(3) 399 (4) 389
12. Volume of the cylinder is 7392 cm3 and side of the (5) None of these
square is 50% more than the radius of the
cylinder. If the perimeter of the square is 84 cm 17. Ratio of the radius of cone to cylinder is 1: 2 and
and the height of the cone is equal to the height
the volume of the cylinder is 2464 cm3. If the
of the cylinder and the radius of the cone is 9
height of the cylinder is 4 cm and the slanting
cm, then what is the curved surface area of the
height of the cone is 25 cm, then find the volume
cone?
of the cone?
(1) 125 Π (2) 135 Π
(1) 2464 (2) 756
(3) 155 Π (4) 175 Π
(3) 616 (4) 1232
(5) None of these
(5) None of these
13. Ratio of the radius of cone to cylinder is 2:1.
Volume of the cylinder is 50% more than the 18. If the perimeter of the square is equal to the
volume of the cone. If the difference between perimeter of the rectangle whose area is 192 cm2
the height of cone and cylinder is 6 cm, then and the breadth of the rectangle is 4 cm less than
what is the sum of the height of cone and the length of the rectangle, then what is the area
cylinder? of the square?
(1) 15 (2) 18 (1) 144 (2) 169
(3) 20 (4) 24 (3) 225 (4) 196
(5) 10 (5) None of these

14. Ratio of the radius of the cone to height of the 19. Area of the rectangle is 216 cm2 and the ratio of
cone is 7:9 and the height of the cone is equal to the length of the rectangle to the side of the
the height of the cylinder. If volume of the cone square is 3: 4. If the perimeter of the square is 96
is 3696 cm3 and the radius of the cylinder is half cm, then find the perimeter of the rectangle?
of the radius of the cone, then find the volume of (1) 50 (2) 40
the cylinder? (3) 60 (4) 80
(1) 2826 (2)2696
(5) None of these
(3) 2456 (4)2772
(5) None of these
20. The circumference of the semi-circle is 360cm. 26. Perimeter of a rectangle is x cm and circumference
If the side of the square is 3/5th of diameter of of a circle is (x + 8) cm. The length of the
the semi-circle, then what is the perimeter of the rectangle is ______ cm. The ratio of the radius
square? of the circle and the length of the rectangle is 1 :
(1) 332 (2) 336 2 and ratio of length and breadth of the rectangle
(3) 340 (4) 344 is 7 : 3?
(5) 348 (1) 28 (2) 21
(3) 24.5 (4) 17.5
21 If the length of a rectangle is increased by 25% (5) None of these
and the breadth is reduced by 33.33% then what
will be the effect on its diagonal 27. The perimeter of a square field is 8cm more than
(approximately)? the perimeter of a rectangle. The length of the
(1) 7.6% (2) 8.33% rectangle is 51 cm which is 300% of its width. If
(3) 6% (4) 7.33% a street of width 10 cm surrounds from outside
(5) No change the square, then find the total cost of
constructing the street at the rate of Rs. 25 per
22. If the area of square is 256 cm2 and the breadth of sq. cm?
rectangle is 20% more and the length is 50% (1) 45000 (2) 45500
more than the side of the square, then find the (3) 46000 (4) 46500
ratio of area of square to the area of rectangle? (5) None of these
(1) 4:5 (2) 3:4
(3) 3:7 (4) 5:9 28. The area of a square is 28 sq. cm more than the
(5) 4:9 area of a rectangle of length 14 cm and breadth
12 cm. What will be the area of in circle of the
23. The perimeter of rectangle of length 2 (x + 3) cm square?
and breadth 2 (x + 1) cm is double the perimeter (1) 162 (2) 154
of a square of area 225 cm2. Find the area of (3) 132 (4) 160
rectangle? (5) None of these
(1) 728 (2) 896
(3) 1024 (4) 1156 29. The ratio of area of a rectangle to that of a
(5) 1596 square is 3 : 5. If the perimeter of the square is
100 cm then what can be the perimeter of the
24. The length of a rectangle, area of which is 126 cm2, rectangle if the breadth of the rectangle is
is equal to the radius of a circle of area 616 cm2. 66.67% more than that of length?
What is the perimeter of the rectangle? (1) 75 (2) 80
(1) 42 (2) 54 (3) 85 (4) 65
(3) 46 (4) 52 (5) None of these
(5) None of these
30. The area of a rectangle is one – third of the area
25. The sum of perimeter of a rectangle (R) and a of a circle and the length of the rectangle is
square (S) is 210 cm. If the length and breadth of equal to the diameter of the circle. If the breadth
rectangle (R) is 40% more and 20% less than the of the rectangle is 11 cm, then what is the
side of square(S), what is the length of diagonal perimeter of the rectangle?
of rectangle (R)? (1) 60 (2) 64
(1) 40√5 cm (2) 5√65 cm (3) 36 (4) 58
(3) 4√35 cm (4) 25√2 cm (5) None of these
(5) 16√30 cm
31 Ananya estimated the cost of painting the four
walls of her old room. 1200. If the length, 36. 1/3rd of a diagonal of a square is 3√2 units.
breadth and height of his new room is twice that What is the measure of the side of the square?
of his old room. How much will it cost to paint (1) 6 units (2) 3 units
the four walls of his new room? (3) 18 units (4) 9 units
(1) Rs 1200 (2) Rs 2400 (5) None of these
(3) Rs 3600 (4) Rs 4800
(5) None of these 37. If cost of wire which is to be fenced is rs.2 per
meter, then find cost of fencing the field?
32. If two opposite sides of a square are increased (1) 240 (2) 340
by 20 cm each, then the ratio of the sides of the (3) 130 (4) 145
previous and the new figure is 22:24. What is (5) 128
the area of the original square?
(1) 48400 cm2 (2) 46500 cm2 38. The perimeter of rectangle is 72m and ratio
(3) 45700 cm2 (4) 42550 cm2 between the length and breadth is 11 : 7. What is
(5) None of these the difference between the half of the length and
one-seventh of the breadth of rectangle?
33. From a square piece of paper who’s each side is (1) 9m (2) 6m
equal to 20 cm, the largest possible circle is to (3) 12m (4) 15m
be cut. The ratio of the area of the circle to the (5) 17m
area of the original square is:
(1) π: 5 (2) π: 4 39. There are two circle A and B and diameter of
(3) π: 6 (4) π: 7 circle A is equal to be the radius of circle B. If
(5) None of these radius of circle A is 14 cm, Then find the
difference between the perimeter of two circle?
34. The perimeter of a square is 96 cm. If the radius (1) 48cm (2) 42cm
of a circle is 3 cm less than the side of that (3) 154cm (4) 88cm
square, then what is area of the circle? (5) 28cm
(1) 1280 (2) 1345
(3) 980 (4) 1045 40. The length of the rectangle A is 4 m more than
(5) 1386 the breadth of rectangle and perimeter of
rectangle A is 56m. What is the area of rectangle
35. The area of a square is 1936 sq meter. Its side is B whose length is equal to the length of
equal to the diameter of a circle. Find the rectangle A and breadth is 8 m ?
circumference of the circle. (1) 64 m2 (2) 48 m2
2
(1) 264 m (2) 22𝜋 m (3) 24m (4) 128m2
(3) 66𝜋 m (4) 44𝜋 m (5) 120m2
(5) None of these
Answer Key
1. (2) 9. (2) 17. (4) 25. (2) 33. (2)
2. (4) 10. (1) 18. (4) 26. (1) 34. (3)
3. (2) 11. (2) 19. (3) 27. (3) 35. (2)
4. (2) 12. (2) 20. (2) 28. (2) 36. (1)
5. (4) 13. (2) 21. (1) 29. (2) 37. (3)
6. (5) 14. (4) 22. (4) 30. (5) 38. (2)
7. (3) 15. (5) 23. (2) 31. (1) 39. (2)
8. (1) 16. (3) 24. (3) 32. (4) 40. (5)
Hints & Solutions
1. Solution (2) 7. Solution (3)
Ratio of radius to height = 3:7 The breadth of the rectangle=11 cm
Volume of cone = 1/3 * 22/7 * 3x*3x * 7x = 1782 The diameter of the circle=14 cm
66x3 = 1782 r=14/2=7 cm
X3= 27 The area of the circle=22/7*r2=22/7*(7*7)=154
x=3
cm2
Radius = 3*3 = 9 cm
The area of the rectangle=154/7*9=198 cm2
Height = 7*3 = 21 cm
Required sum = (2*21)+(1/3 * 9) = 42+3 = 45 The length of the rectangle=198/11=18 cm
The height of the cuboid=18/6*11=33 cm
2. Solution (4)
Circumference of the semicircle = 22/7 * r + 2 * r 8. Solution (1)
72 = 22/7 * r + 2 * r 6 * a * a = 1536
72 = r(22/7 + 2) a = 16
Radius of the semicircle = 14 cm Breadth of the rectangle = 16 + 2 = 18 cm
Length of the rectangle = 14 * 2 = 28 cm 2 * (l + 18) = 84
Breadth of the rectangle = 448/28 = 16 cm
l = 24 cm
Perimeter of the rectangle = 2 * (28 + 16) = 88 cm
Side of the square = 5/6 * 24 = 20 cm
3. Solution (2) Difference = 24 * 18 - 20 * 20
Area of square = 18 = d2/2 = 32 cm2
Diagonal of square (d) = 6cm
Radius of Circle = 6cm 9. Solution (2)
Height of cylinder = 6*(7/3) =14cm l + b+ a = 120 meter
Curved surface area of the cylinder = 2πrh = Perimeter of the rectangle = 2(l+b) = 180
2*(22/7)*6*14 = 528 cm2 l + b = 90
a = 120 – 90 = 30 meters
4. Solution (2) Area of square = 30 * 30 = 900m2
Length of rectangle = side of square = x
Breadth of rectangle = y
(x * y)/x * x = 2/3 10. Solution (1)
y/x = 2/3 Area of square= 1225 cm2
2 * (2k + 3k) = 60 a² = 1225
k=6 a = 35 cm
Length of rectangle = 6 * 3 = 18 cm Diagonal of square = a√2 = 35√2 cm
Perimeter of the square = 18 * 4 = 72 cm Length of rectangle = 35√2 × 80/100 = 28√2 cm
Perimeter = 94√2 cm
5. Solution (4)
Required ratio = 1/3 * 22/7 * 2x * 2x * 3y:22/7 * 2(l+b) = 94√2
3x * 3x * 5y 2 × 28√2 +2b = 94√2
= 4:45 2b = 94√2 − 56√2
2b = 38√2
6. Solution (5)
2160 = 6x * 5x * 8 b = 19√2
x=3 Area= l × b = 28√2 × 19√2 = 1064 cm2
Height of cuboid = Breadth of the rectangle = 3 *
6 = 18 cm
Length of rectangle = 84/2 – 18 = 24 cm
Area of the rectangle = 24 * 18 = 432 cm2
11. Solution (2) 16. Solution (3)
According to question Circumference of the circle = 2πr = 2 * 22/7 * r
2 x 22/7 x r = 88 = 132
r = 14 cm Radius of the circle = 21 cm
Radius of cone = 14 cm Perimeter of the rectangle = 2 * (l + b)
Height of cone = 2cm Length of the rectangle = 21 cm
Slant height of cone =under root of (14² + 2²) = 80 = 2 * (21 + b)
√200 = 10√2 cm b = 19 cm
C.S.A of cone = 22/7 x 14 x 10√2 = 440√2 cm² Area of the rectangle = l * b = 19 * 21 = 399 cm2

12. Solution (2) 17. Solution (4)


Side of the square = 84/4 = 21 cm 2464 = 22/7 * r * r * 4
Radius of the cylinder = 100/150 * 21 = 14 cm Radius of the cylinder = 14 cm
7392 = 22/7 * 14 * 14 * h Radius of the cone = ½ * 14 = 7 cm
Height of the cylinder = 12 cm Height of the cone = √252 – 72
Height of the cone = 12 cm = 24 cm
Slanting height of the cone = √122 + 92 = 15 cm Volume of the cone = 1/3 * 22/7 * 7 * 7 * 24 =
CSA of the cone = 22/7 * 9 * 15 = 135Π cm2 1232 cm3

13. Solution (2) 18. Solution (4)


Volume of the cone = 1/3 * 22/7 * 2x * 2x * h1 Area of the rectangle = 192
Volume of the cylinder = 22/7 * x * x * h2 l * (l – 4) = 192
22/7 * x * x * h2 = 150/100 * (1/3 * 22/7 * 2x * l2 – 4l – 192 = 0
2x * h1) l2 – 16l + 12l – 192 = 0
h2/h1 = 2/1 l(l – 16) + 12(l – 16) = 0
2h – h = 6 (l + 12)(l – 16) = 0
h = 6 cm l = 16 cm
sum = 6 + (2 * 6) = 18 cm Breath of the rectangle = 16 – 4 = 12 cm
Perimeter of the rectangle = 2 * (12 + 16) = 56 cm
14. Solution (4) Perimeter of the square = 56 cm
3696 = 1/3 * 22/7 * 7x * 7x * 9x Side of the square = 56/4 = 14 cm
x=2 Area of the square = 14 * 14 = 196 cm2
Radius of the cone = 7 * 2 = 14 cm
Radius of the cylinder = 14/2 = 7 cm 19. Solution (3)
Height of the cone = 9 * 2 = 18 cm Side of the square = 96/4 = 24 cm
Volume of the cylinder = 22/7 * 7 * 7 * 18 Length of the rectangle = ¾ * 24 = 18 cm
= 2772 cm2 18 * breadth = 216
Breadth of the rectangle = 12 cm
15. Solution (5) Perimeter of the rectangle = 2 * (l + b)
Length: Breadth = 7:9 = 2 * (18 + 12)=60 cm
Perimeter: Area = 1:2
Area of rectangle = 1/4 x Area of square 20. Solution (2)
= 1/4 x 16 x 16 = 64 cm2 Circumference of the semi-circle = 360cm
2’s = 64 r(π+2) = 360
1’s = 32cm r(22/7+2) = 360
2(L + B) = 32 Therefore, radius of the semi-circle = 70cm
2 (7X + 9X) = 32 Diameter of the semi-circle = 140cm
X=1 Side of the square = 3/5 (140) = 84cm
Hence, breadth of the rectangle = 9X = 9 cm. Perimeter of the square = 4a = 4*84 = 336 cm
25. Solution (2)
21. Solution (1) Let the side of square be ‘s’ cm
Let the length of the rectangle = 4 units So, length of rectangle = 1.4s cm
And breadth of the rectangle = 3 units And, breadth of rectangle = 0.8s cm
Then diagonal of the rectangle = √(42 + 32) = 5 According to the question,
units 2(1.4s + 0.8s) + 4s = 210
According to the question, the length of a 4.4s + 4s = 210
rectangle is increased by 25% and the breadth is 8.4s = 210
reduced by 33.33% s =210/8.4= 25 cm
New length = 125% of 4 units = 5 units So, length of rectangle = 1.4 × 25 = 35 cm
New breadth = 66.66% of 3 units = 2 units And, breadth of rectangle = 0.8 × 25 = 20 cm
In the new rectangle, New diagonal= √(52 + 22) = diagonal of rectangle = (√35)2+202 5√65 cm
√29
= approximately 5.38 units 26. Solution (1)
Change = (5.38 – 5) ×100/5 = 7.6% Let the length and breadth of the rectangle be 7a
cm and 3a cm respectively.
22. Solution (4) Perimeter of rectangle = 2 (7a + 3a)
Area of square = 256 cm2 , Side of square = 16 cm X = 20a
Area of rectangle = l × b Radius of circle/ length of rectangle=1/ 2
Length = 16 × 1.5 = 24 cm r/7a=1/2
Breadth = 16 × 1.2 = 19.2 cm According to question
Area of rectangle = 460.8 cm2 ⇒ 2πr – 20a = 8
Reqd. ratio =256/460.8 =5/9 ⇒ 2 ×22/7×7a/2– 20a = 8
⇒ 22a – 20a = 8
23. Solution (3) ⇒ 2a = 8
Area of square = 225 cm2 ⇒a=4
Side of square = √225 = 15 cm Length of the rectangle = 7 × 4 = 28cm
Perimeter of square = 15 × 4 = 60 cm
So, perimeter of rectangle = 2 × 60 = 120 cm 27. Solution (3)
2 (2 (x + 3) + 2 (x + 1)) = 120 Let the width of the rectangle = x cm
2x + 6 + 2x + 2 = 60 Then, 300% of x = 51
4x + 8 = 60 x = 51 ×100/300 = 17 cm
4x = 52 Perimeter of the rectangle = 2(length + width)
x = 13 = 2 × (51 + 17) = 136 cm
So, length of rectangle = 2 (13 + 3) = 32 cm Perimeter of the square = (136 + 8) = 144 cm
And, breadth of rectangle = 2 (13 + 1) = 28 cm The sides of the square =144/4= 36 cm
Therefore, area of rectangle = 32 × 28 = 896 cm2 The area of the square field without street
= (36)2 sq. cm
24. Solution (3) =1296 sq. cm
Let the radius of the circle = k The area of the square field with street
Area = 616 =22/7× k × k = (36 + 20)2 sq. cm
k = 14 cm = 3136 sq. cm
Area of rectangle = 126 = L × B = 14 × B → B The area of the street = 3136 – 1296 = 1840 sq. cm
= 9 cm The total cost of constructing the street
Perimeter = 2 × (L + B) = 2 × (14 + 9) = 46 cm = 1840 × 25
= Rs. 46,000
28. Solution (2) 32. Solution (1)
The area of rectangle = l × b = 14 × 12 Let the original side of the square be X cm, then
= 168 sq. cm new sides of the new figure would be X cm and
The area of square = 168 + 28 = 196 sq. cm X+20 cm.
The side of square = square root of 196 = 14 cm Now, new ratio of the sides = X: X + 20,
Radius of incircle of a square =side/2=14/2= 7 cm ⇒ 22: 24 = X: X + 20,
The reqd. area = πr2 =22/7× 7 × 7 = 154 sq.cm Solving, X = 220 cm.
∴ Side of the original square = 220 cm.
29. Solution (2)
∴ Area of the original square = multiplication of
The side of the square =100/4= 25 cm
both sides = 220 × 220 = 48400 cm2.
The area of the square = 25 × 25 = 625 sq. m
The area of the rectangle =3 × 625/5= 375 sq. cm
33. Solution (2)
Let the length of the rectangle = 3x and breadth of
the rectangle = 166.67% of 3x = 5x then perimeter The largest possible circle cut out of square has
= 2(l + b) diameter equal to side of the square.
And area = 375 = 3x × 5x Area of circle = π × (d2/4); where d is the diameter
x = 5 cm of the circle
Perimeter = 2(3x + 5x) = 16x = 80 cm Area of circle = π × (202/4) = 100π sq.cm
Area of square = s × s; where s is the side of the
30. Solution (5) square
Let the radius of the circle = r cm the length of the Area of square = 20 × 20 = 400 sq.cm
rectangle = 2r cm ∴ Ratio of area of circle to square = 100π/400
According to the question, area of the circle = A = = π: 4
πr2= 3 × 2r × 11
22/7× r = 2 × 3 × 11 34. Solution (5)
r = 21 cm perimeter of square = 96cm
The perimeter of the rectangle = 2(l + b) = 2 (21 × side of square = 96/4 = 24cm
2 + 11) = 106 cm radius of circle = 24 – 3 = 21 cm
area of circle = πr^2 = 22/7 * 21 *21 = 1386 cm^2
31. Solution (4)
Let the length, breadth and height of old room be l.
35. Solution (4)
b and h respectively.
Area of Square = 1936 m2
Painting is done on a surface area so cost of
Side of square = 44 m
painting will be proportional to the surface area of
Radius = 22 m
the room, excluding the base area and top area.
Circumference = 2𝜋r = 2𝜋 × 22 = 44 π 𝑚
Area to be painted = 2 × (bh + lh)
∵ Cost of Painting the above area = Rs. 1200
Now, in the new room, the length, breadth and 36. Solution (4)
height are twice of those in the old room. Let the measure of side of square be a unit
∴ New length = 2l; New breadth = 2b and new
height= 2h
∴ New area to be painted = 2 × [(2b × 2h) + (2l ×
2h)]
⇒ New area to be painted = 8 × (bh + lh)
𝑂𝑙𝑑 𝑐𝑜𝑠𝑡 𝑜𝑓 𝑝𝑎𝑖𝑛𝑡𝑖𝑛𝑔
∴ New cost of painting = 𝑂𝑙𝑑 𝑎𝑟𝑒𝑎
×New
Area
1200
⇒ New cost of painting = × 8 (bh + lh) Thus, the measure of the side of the square is 9 units.
2(𝑏ℎ+𝑙ℎ)
⇒ New cost of painting = 4 × 1200 = Rs. 4800
39. Solution (4)
37. Solution (5) Radius of circle A = 14 cm
Let length and breadth are 9x and 7x respectively. Diameter of circle A = 14 * 2 = 28
9x * 7x = 252 radius of circle B = 28
x=2 perimeter of circle A = 2 * 22/7 * 14 = 88cm
length = 9 * 2 = 18 perimeter of circle B = 2 * 22/7 * 28 = 176 cm
breadth = 7 * 2 = 14 difference = 176 – 88 = 88cm
perimeter of rectangular field = 2(18 + 14) = 64m
cost of fencing wire = 64 * 2 = 128 40. Solution (4)
Let breadth of rectangle A = x
38. Solution (1) length = x + 4
Let length and breadth of rectangle 11x and 7x perimeter = 56
respectively. 2(x + x + 4) = 56
perimeter = 72m 4x + 8 = 56
2(11x + 7x) = 72 x = 12
x=2 length of rectangle A = 12 + 4 = 16
length = 11 * 2 = 22 length of rectangle B = 16
breadth = 7 * 2 = 14 breadth = 8
difference = 22/2 – 14 * 1/7 = 11 – 2 = 9m area = 16 * 8 = 128 m2
1. If the ratio of the radius of cone to cylinder is 6. If the length of the rectangle is increased by 20%
3:2 and the volume of the cylinder is 7392 cm3. and breadth of the rectangle is decreased by
If radius of the cone is equal to the radius of the 15%, then the area of the rectangle will be 510
circle whose circumference is 132 cm, what is cm2. Find the area of the original rectangle?
the height of the cylinder? (1) 480 (2) 500
(1) 12 (2) 10 (3) 540 (4) 420
(3) 14 (4) 16 (5) None of these
(5) 8
7. The volume of the cone is 1232 cm3 and the
2. If the length and breadth of the rectangle is
circumference of the circle is 44 cm. If the
16 cm and 20 cm respectively and the perimeter
radius of the circle is equal to the radius of the
of the rectangle is equal to the perimeter of the
square. What is the ratio of the area of the cone, then what is the slanting height of the
rectangle to area of the square? cone?
(1) 2:1 (2) 4:5 (1) 9 (2) 16
(3) 70:71 (4) 78:79 (3) 25 (4) 30
(5) None of these (5) 36

3. If the ratio of the height to radius of the cylinder 8. 8 The area of the circle is 1386 cm2 and the
is 3:2 and the curved surface area of the cylinder ratio of the breadth of the rectangle to radius of
is 1848 cm2. If the radius of the cylinder is equal the circle is 4:3. If the circumference of the
to the radius of the cone and the height of the circle is equal to the perimeter of the rectangle,
cone is 9cm, then what is the volume of the then what is the area of the rectangle?
cone? (1) 1032 (2) 1048
(1) 1448 (2) 1648 (3) 1056 (4) 1064
(3) 1948 (4) 1848 (5) None of these
(5) None of these
9. The ratio of the length and breadth of the
4. If the surface area of the sphere is 616 cm2 and
rectangular field is 3:2 and the area of the
the radius of the sphere is equal to the radius of
rectangular field is 216 cm2. What is the cost of
the cylinder. If the height of the cylinder is 12
cm, then what is the curved surface area of the the fencing the rectangular field at the rate of
cylinder? Rs.10 per cm?
(1) 528 (2) 548 (1) 450 (2) 560
(3) 538 (4) 518 (3) 600 (4) 640
(5) None of these (5) None of these

5. If the ratio of the radius of the circle to length of 10. If the surface area of the sphere is 5544 cm2 and
the rectangle is 1:4 and the perimeter of the the radius of the sphere is 3 cm less than the
rectangle is 80 cm. If the circumference of the height of the cone. If the radius of the cone is 7
circle is 44 cm, then what is the area of the cm, then what is the curved surface area of the
rectangle? cone?
(1) 312 (2) 328 (1) 480 (2) 520
(3) 336 (4) 345 (3) 560 (4) 420
(5) None of these (5) None of these
11. The circumference of the semi-circle is 360cm. If 17. Diameter of the circle is equal to the breadth of
the side of the square is 3/5th of diameter of the the rectangle and the ratio of the length to
semi-circle, then what is the perimeter of the breadth of the rectangle is 5:4. If the perimeter
square? of the rectangle is 126 cm, then what is the
(1) 332 (2) 336 difference between the area of the circle and
(3) 340 (4) 344 rectangle?
(5) 348 (1) 304 (2) 394
(3) 364 (4) 324
12. Ratio of the length of two rectangles is 4:5 and (5) 384
the breadth of these two rectangles is 3:2. If the
perimeter of second rectangle is 74 cm and the
18. Radius of cylinder is double the radius of cone
length of second rectangle is 13 cm more than its
and the height of the cylinder is 10 cm. If curved
breadth, then find the area of the first rectangle?
surface of the cylinder is 880 cm2 and the
(1) 270 (2) 540
slanting height of the cone is 25 cm, then find
(3) 360 (4) 480
(5) 620 the volume of the cone?
(1) 1848 (2) 3696
13. If the ratio of length, breadth and height of a (3) 616 (4) 2464
cuboid is 3:2:5 and the total surface area of a (5) 1232
cuboid is 248 cm2 and the length of the cuboid is
equal to the breadth of the rectangle whose 19. The curved surface of cone is 180Π cm2 and the
length is 4 cm more than its breadth, then find radius of the cone is 12 cm. Ratio of the radius
the area of the rectangle? of cone to cylinder is 6:7 and the height of
(1) 60 (2) 150 cylinder is 3 cm more than the height of the
(3) 120 (4) 90 cone, then find the volume of the cylinder?
(5) None of these (1) 7592 (2) 7692
(3) 7892 (4) 7392
14. What is the volume of the sphere whose radius is (5) 7492
equal to the diameter of the circle having cost of
fencing along circumference at the rate of Rs.8 20. The ratio of the length of the rectangle to side of
per cm is Rs.352? the square is 5: 4 and the ratio of the breadth of
(1) 32496/3 cm3 (2) 33496/3 cm3 the rectangle to side of the square is 1: 2. If the
3
(3) 34496/3 cm (4) 31496/3 cm3
difference between the perimeter of rectangle
(5) None of these
and square is 8 cm, then what is the sum of the
area of rectangle and square?
15. Total surface area (in cm2) of a cube is 75% of
volume (in cm3)of that cube. Find the total
(1) 424 (2) 420
surface area of the cube?
(1) 390 (2) 384 (3) 408 (4) 416
(3) 375 (4) 387 (5) None of these
(5) cnd
21. Ratio of the length and breadth of the rectangle
16. 2
The area of the circle is 616 cm and the radius is 4:3 respectively and the breadth of the
of the cone is equal to the radius of the circle. If rectangle is 60% of the side of the square. If the
the ratio of the height and radius of the cone is sum of the area of the rectangle and square is
6:7, then what is the volume of the cone? 925 cm2, then find the area of the rectangle?
(1) 3696 (2) 616 (1) 510 (2) 420
(3) 2464 (4) 1232 (3) 270 (4) 300
(5) None of these (5) None of these
22. Ratio of the volume of cone and cylinder is 2:1 27. Curved surface area of a cylinder is 1056 cm2. If
and the radius of the cone is double of the radius the radius of a cone is half of the radius of the
of the cylinder. If the height of the cone is cylinder and the volume of the cylinder is 7392
(x + 6), then find the height of the cylinder?
cm3 and the volume of the cone is 12.5% of the
(1) (x+6)/2 (2) 2x+6
(3) 3x/2+6 (4) 2x/3+4 volume of the cylinder, then find the height of
(5) None of these the cone?
(1) 30 (2) 10
23. The radius of the sphere is two times the radius (3) 15 (4) 25
of the cylinder. If the curved surface area of the (5) 20
cylinder is 1232 cm2 and the height of the
cylinder is 28 cm, then find the surface area of
28. Ratio of side of a square and height of a cylinder
the sphere?
(1) 30 (2) 10 is 7:4. If the volume of the cylinder is 1848 cm3
(3) 15 (4) 25 and the area of the square is 441 cm2, then find
(5) 20 the radius of the cylinder?
(1) 12 (2) 7
24. The cost of fencing the rectangular field is (3) 9 (4) 14
Rs.1300 at the rate of Rs.12.5 per meter. If the (5) None of these
side of the square is 4 meter less than the length
of the rectangle and the perimeter of the square
is 112 meter, then find the area of rectangular 29. The length of a rectangle is 37.5% more than the
field? breadth of the rectangle and the ratio of the
(1) 520 (2) 600 perimeter of the rectangle and square is 19:10. If
(3) 560 (4) 640 the area of the rectangle is 1408 cm2, then find
(5) None of these the area of the square?
(1) 225 (2) 576
25. Ratio of the side of the square and the height of
the cylinder is 3:1 respectively. If the area of the (3) 256 (4) 400
square is 729 cm2 and the volume of the cylinder (5) None of these
is 5544 cm3, then find the radius of the cylinder?
(1) 10 (2) 14 30. Ratio of the length and breadth of the rectangle
(3) 12 (4) 18 is 4:3. The perimeter of the rectangle is 8 cm
(5) None of these less than the perimeter of square whose area is
900 cm2. If the length of the rectangle is
26. Ratio of the diagonals of rhombus is 4:3 and the
ratio of the length and breadth of the rectangle increased by 25%, then find the area of the
is3:1. If the perimeter of the rectangle is 56 cm rectangle?
and the length of the rectangle is 40% more than (1) 880 (2) 840
the side of the rhombus, then find the area of the (3) 960 (4) 920
rhombus? (5) None of these
(1) 208 (2) 248
(3) 228 (4) 232
(5) 216
Answer Key
1. (1) 7. (3) 13. (1) 19. (4) 25. (2)
2. (5) 8. (4) 14. (3) 20. (4) 26. (5)
3. (4) 9. (3) 15. (2) 21. (4) 27. (3)
4. (1) 10. (5) 16. (3) 22. (4) 28. (2)
5. (3) 11. (2) 17. (3) 23. (2) 29. (4)
6. (2) 12. (3) 18. (5) 24. (4) 30. (3)
Hints & Solutions
1. Solution (1) 5. Solution (3)
Circumference of the circle=2 * 22/7 * r Circumference of the circle=2 * 22/7 * r
132=2 * 22/7 * r 2 * 22/7 * r=44
Radius of the circle=21 cm Radius of the circle =7 cm
Radius of the cone=21 cm Length of the rectangle = 4/1 * 7=28 cm
Radius of the cylinder=2/3 * 21=14 cm Perimeter of the rectangle = 2 * (l + b)
Volume of the cylinder=22/7 * r * r * h (28 + b) * 2= 80
7392=22/7 * 14 * 14 * h Breadth of the rectangle= 40 – 28=12 cm
Height of the cylinder=12 cm Area of the rectangle = l * b=12 * 28=336 cm2

2. Solution (5) 6. Solution (2)


Perimeter of the rectangle=2 * (l + b) Let the original area of the rectangle = x
=2 * (16 + 20) x * 120/100 * 85/100 = 510
=72 cm x = 500 cm2
Perimeter of the square=4 * a=72
Side of the square =18 cm 7. Solution (3)
Area of the square=a * a=18 * 18=324 cm Circumference of the circle=2 * 22/7 * r
Area of the rectangle=16 * 20=320 cm 44=2 * 22/7 * r
Required ratio=320:324=80:81 Radius of the circle=7 cm
Radius of the cone=7 cm
3. Solution (4) Volume of the cone=1/3 * Π * r2 * h
Curved surface area of the cylinder= 2πrh = 2 * 1232=1/3 * (22/7) * 7 * 7 * h
22/7 * r * h h=24 cm
1848 = 2 * 22/7 * 2x * 3x Slanting height of the cone=√h2 + r2
x = 7 cm =√242 + 72
Radius of the cylinder = 7 * 2 = 14 cm
Radius of the cone = 14 cm 8. Solution (4)
Volume of the cone = 1/3πr2h Area of the circle=Π * r * r
= 1/3 * 22/7 * r2 * h 1386=22/7 * r * r
=1/3 * 22/7 * 14 * 14 * 9 Radius of the circle=21 cm
=1848 cm3 Circumference of the circle=2 * Π * r
=2 * 22/7 * 21
4. Solution (1) =132 cm
Surface area of the sphere = 4πr2 Breadth of the rectangle=4/3 * 21=28 cm
= 4 * 22/7 * r * r Perimeter of the rectangle=2 * (l + b)
616 = 4 * 22/7 * r * r 132=2 * (l + 28)
Radius of the sphere = 7 cm Length of the rectangle=38 cm
Radius of the cylinder = 7 cm Area of the rectangle=l * b=38 * 28=1064 cm2
Curved surface area of the cylinder = 2πrh
= 2 * 22/7 * r * h 9. Solution (3)
= 2 * 22/7 * 7 * 12 3x * 2x=216
= 528 cm2 x=6 cm
Length=3 * 6=18 cm
Breadth=2 * 6=12 cm
Perimeter=2 * (l + b)=2 * (18 + 12)=60 cm
Cost of the fencing=60 * 10=Rs.600
14. Solution (3)
10. Solution (5) 2 * 22/7 * r = 352/8
SA of the sphere=4 * 22/7 * r * r Radius of the circle = 7 cm
5544=4 * 22/7 * r * r Diameter of circle = 7 * 2 = 14 cm
Radius of the sphere=21 cm Volume of the sphere
Height of the cone=21 + 3=24 cm = 4/3 * 22/7 * 14 * 14 * 14
Slanting height of the cone=√h2 + r2 = 34496/3 cm3
=√242 + 72
=25 cm 15. Solution (2)
CSA of the cone=22/7 * r * l Let side of the cube = ‘a’ cm
=22/7 * 7 * 25 So, total surface area of the cube = 6a2 cm2
=550 cm2 And volume of the cube = a3 cm3
From the question:
11. Solution (2) 6a2 = a3 * (75/100)
Circumference of the semi-circle = 360cm 6a2 = 3a3/4
r(π+2) = 360 a = 8 cm
r (22/7 +2) =360 So, total surface area of the cube = 6 * 8 * 8 = 384
Therefore, radius of the semi-circle = 70cm cm2
Diameter of the semi-circle = 140cm
Side of the square = 3/(5 ) (140) = 84cm 16. Solution (3)
Perimeter of the square = 4a = 4*84 22/7 * r * r = 616cm2
= 336 cm Radius of the circle = 14 cm= Radius of the cone
Height of the cone = 6/7 * 14 = 12 cm
12. Solution (3) Volume of the cone = 1/3 * 22/7 * 14 * 14 * 12
Perimeter of first rectangle = 2 * (4x + 3y) = 2464 cm3
Perimeter of second rectangle
= 2 * (5x + 2y) 17. Solution (3)
2 * (5x + 2y) = 74 2 * (5x + 4x) = 126
5x + 2y = 37 x=7
5x – 2y = 13 Breadth of rectangle = 7 * 4 = 28 cm
10x = 50 Diameter of the circle = 28 cm
x=5 Radius of the circle = 28/2 = 14 cm
y=6 Area of the circle = 22/7 * 14 * 14 = 616
Length of first rectangle = 5 * 4 = 20 Area of the rectangle = 35 * 28 = 980
Breadth of first rectangle = 6 * 3 = 18 Required Difference = 980 – 616 = 364 cm2
Area of first rectangle = 20 * 18 = 360 cm2
18. Solution (5)
13. Solution (2) 2 * 22/7 * r * 10 = 880
248 = 2 * (3x * 2x + 2x * 5x + 5x * 3x) Radius of the cylinder = 14 cm
31x2 = 124 Radius of cone = 14/2 = 7 cm
X2 = 4 Height of cone = √252 – 72 = 24 cm
x=2 Volume of the cone = 1/3 * 22/7 * 7 * 7 * 24
Breadth of the rectangle = 2 * 3 = 6 cm = 1232 cm3
Length of the rectangle = 6 + 4 = 10 cm
Area of the rectangle = 6 * 10 = 60 cm2
19. Solution (4) 23. Solution (2)
22/7 * 12 * l = 180 * 22/7 Radius of the cylinder=r
l = 15 2*22/7*28*r=1232
Height of cone = √152 – 122= 9 cm r=7 cm
Radius of cylinder = 7/6 * 12 = 14 cm Radius of sphere=7*2=14 cm
Height of cylinder = 9 + 3 = 12 cm Surface area of the sphere=4*22/7*14*14
Volume of cylinder = 22/7 * 14 * 14 * 12 =2464 cm2
= 7392 cm3
24. Solution (4)
Perimeter of the rectangular field = 1300/12.5 =
20. Solution (4)
Side of the square = 4x 104 m
Length of the rectangle = 5x Side of the square = 112/4 = 28 m
Breadth of the rectangle = 1/2 * 4x = 2x Length of the rectangular field =28 + 4 = 32 m
Perimeter of the rectangle = 2 * (5x + 2x) Breadth of the rectangle = 104/2 – 32 = 20 m
= 14x Area of the rectangle = 32 * 20 = 640 m2
Perimeter of the square = 4 * 4x = 16x
16x – 14x = 8 25. Solution (2)
x=4 Area of the square=729 cm2
Area of the rectangle = (5 * 4) * (2 * 4) Side of the square = √729=27 cm
= 160 cm2 Height of the cylinder=27*1/3=9 cm
Area of the square = (4 * 4) * (4 * 4)
Volume of the cylinder=5544 cm3
= 256 cm2
22/7*r2*9=5544
Required sum = 160 + 256 = 416 cm2
r2=196
r=14 cm
21. Solution (4)
Let the side of the square=5x 26. Solution (5)
Breadth of the rectangle=5x*60/100=3x 2 * (3x + x) = 56
Length of the rectangle=3x*4/3=4x x=7
4x*3x+(5x)2=925 Length of rectangle = 7 * 3 = 21 cm
X2(12+25)=925 Side of the rhombus = 100/140 * 21 = 15 cm
X2=25 152 = (4x/2)2 + (3x/2)2
x=5 225 = 4x2 + (9x2/4)
Side of the square=5*5=25 cm x=6
Required area of the rectangle=925-25*25=925- Area of the rhombus = 1/2 *(4 * 6) * (3 * 6)
625 = 216 cm2
=300 cm2
27. Solution (3)
22. Solution (4)
Curved surface area of a cylinder = 2 * 22/7 * r *
Height of the cone = h1
Height of the cylinder = h2 h = 1056
Radius of the cone = 2a Volume of the cylinder = 22/7 * r * r * h = 7392
Radius of the cylinder = a 528 * r = 7392
(1/3 * 22/7 * 2a * 2a * h1)/(22/7 * a * a * h2) = r = 14
2/1 Volume of a cone = 12.5/100 * 7392 = 924
4h1/3h2 = 2/1 Radius of the cone = 14/2 = 7
2h1 = 3h2 Volume of the cone = 1/3 * 22/7 * 7 * 7 * h1 =
2 * (x + 6) = 3h2 924
h2 = 2/3 * (x + 6) = (2x/3) + 4 Height of the cone = 18 cm
Perimeter of the rectangle=2*(8*4+11*4)=152 cm
28. Solution (2) Perimeter of the square=152*10/19=80 cm
Side of the square= √441=21 cm Side of the square=80/4=20 cm
Height of the cylinder =21*4/7=12 cm Area of the square =20*20=400 cm2
Radius of the cylinder =r
Volume of the cylinder = π r2 h 30. Solution (3)
22/7*r2*12=1848 Side of the square = √900 = 30 cm
R2=49 Perimeter of the square = 30 * 4 = 120 cm
r=7 Perimeter of the rectangle = 120 – 8 = 112 cm
112 = 2 * (4x + 3x)
29. Solution (4) x=8
Breadth of the rectangle=8x Length of the rectangle = 8 * 4 = 32 cm
Length of the rectangle=11x Breadth of the rectangle = 3 * 8 = 24 cm
11x*8x=1408 Area of the rectangle = 32 * 125/100 * 24 = 960
X2=16 cm2
x=4
1. How many words can be formed by using three 8. How many three digits number can be formed by
letters out of a, b, c, d, e without repetition of using the digits 0, 2, 4, 6, 7 if repetition of digits is
alphabets. not allowed.
(1) 55 (2) 60 (1) 50 (2) 46
(3) 61 (4) 69 (3) 48 (4) 40
(5) 72 (5) 36

2. How many words of three letters with or without Direction (9-10): Among the arrangements that can
meaning can be formed using all the alphabets of be made by using all the letters of the word
the word SIMPLE without repetition of alphabets. “EXAMINATION”.
(1) 100 (2) 160
(3) 95 (4) 120 9. In how many arrangements A’s come together?
(5) 130 (1) 11! (2)
11!
2!
10!
3. In how many different ways can Ram arrange the (3) (4) 11! – 10! 2!
2! 2!
letters of the word ALLAHABAD? (5) None of these
(1) 7650 (2) 7560
(3) 6750 (4) 5760 10. In how many arrangements of the above word A’s
(5) 7660 do not come together?
9 × 10!
(1) 11! – 10! 2! (2)
8
4. Find total number of the 3 digits odd numbers by 11! × 10 10!
using the digits 2, 3, 4, 5 when repetitions of digits (3) (4)
2 8
are not allowed. (5) 10!
(1) 12 (2) 22
(3) 15 (4) 18 11. All possible four-digit numbers, with distinct digits,
(5) 24 are formed using the digits {1, 3, 4, 5, 6}. How
many of them are divisible by 5?
5. In how many different ways can the letters of the (1) 8 (2) 12
word DISTINCTIONS be arranged. (3) 24 (4) 20
(1) 1001872 (2) 9979200 (5) 28
(3) 9986100 (4) 8769150
(5) None of these 12. In a word, there are six different letters. A word can
be formed by at least 2 letters. If order of letters is
6. In how many ways can 6 persons take water from 6 changed in a word, we get a different word. How
taps if no tap remains unused. many different words can be formed in this word?
(1) 1024 ways (2) 860 ways (1) 1870 (2) 1970
(3) 720 ways (4) 700 ways (3) 1950 (4) 2190
(5) 820 ways (5) 2280

7. How many three digits number can be formed by 13. How many 3 - letter words, with or without
using the digits 2, 3, 7 and 8 if meaning, can be formed out of the letters of the
(i) Repetition of digit is all allowed word, 'LOGISTICK', if repetition of letters is not
(ii) Repetition of digits is not allowed allowed?
(1) 360 (2) 720
(1) 64, 24 (2) 64, 22 (3) 5040 (4) 120
(3) 28, 24 (4) 26, 22 (5) None of these
(5) 32, 12
14. There are 2 apples, 3 guavas, 3 papayas and 2 (1) 4 (2) 6
oranges. In how many ways can a shopkeeper (3) 5 (4) 10
arrange these fruits so that all the papayas come (5) 20
together and all the oranges come together?
(1) 60,720 (2) 60,480 21. In a small Iceland, there are 87 towns, of which 52
(3) 60,540 (4) 60,690 towns have at most 2 persons. In a city development
(5) None of these programme, 20 towns are to be chosen for
assistance, of which at least 18 towns must have at
15. In how many different ways can the letters of the most 2 persons. In how many ways can the choice
be made?
word DOWNIN be arranged in such a way that the
(1) ?? (2) ??
vowels always come together?
(3) ?? (4) ??
(1) 350 (2) 250
(5) ??
(3) 720 (4) 120
(5) None of these
22. A committee of (x - 2) persons is to be constituted
from a group of 2 men and (x - 2) women. If the
16. The number of ways in which 12 different toys can total number of such ways in which this can be
be arranged on a table so that 5 particular toys shall done, be 10, then find the value of x.
not be together: (1) 4 (2) 6
(1) 12! – 5! (2) 361000 (3) 3 (4) 5
(3) 12! – 8! X 5! (4) 12! – 8! (5) None of these
(5) None of these
23. From data found in the above question, how many
17. In how many ways can the letters of the word of these committees would consist of 1 man and 2
‘DISTEMPER’ be arranged such that all the vowels women?
occur together? (1) 3 (2) 6
(1) 720 ways (2) 15120 ways (3) 5 (4) 4
(3) 5040 ways (4) 90720 ways (5) 8
(5) None of these
Directions (24 - 26): A group consists of 4 red monkeys
18. In how many ways 8 people can sit around a and 7 black monkeys. In how many ways can a team of
circular table so that Madan and Mohnish should 5 monkeys be selected if the team has
24. No red monkeys.
never sit opposite to each other.
(1) 25 (2) 21
(1) 3600 (2) 7200
(3) 28 (4) 42
(3) 1440 (4) 720
(5) None of these
(5) 1580

19. How many ways a 6-persons committee can be 25. At least one black monkey and one red monkey
formed out of 11 persons if two particular persons
(1) 221 (2) 662
must be included?
(1) 56 (2) 126 (3) 441 (4) 341
(3) 252 (4) 120 (5) (e) None of these
(5) None of these
26. At least three red monkeys.
20. There are 4 vegetables of different types viz potato, (1) 111 (2) 81
onion, lemon and brinjal. In how many ways two (3) 71 (4) 91
vegetables can be selected such that one of them is (5) 99
potato or brinjal?
27. In how many different ways can the letters of the 29. There are 2 apples, 3 guavas, 3 papayas and 2
word ‘QUESTIONS' be arranged such that the oranges. In how many ways a shopkeeper can
vowels must occupy only the odd positions? arrange these fruits so that all the papayas come
(1) 14400 (2) 3600 together and all the oranges come together?
(3) 7200 (4) 1440 (1) 60,720 (2) 60,480
(5) None of these (3) 60,540 (4) 60,690
(5) None of these
28. Find total number of the 3 digits odd numbers by
using the digits 2, 3, 4, 5 when repetitions of digits 30. In how many ways can the letters of the word
are not allowed. ‘DISTEMPER’ be arranged such that all the vowels
(1) 12 (2) 22 occur together?
(3) 15 (4) 18 (1) 720 ways (2) 15120 ways
(5) 24 (3) 5040 ways (4) 90720 ways
(5) None of these
Answer Key
1. (2) 8. (3) 15. (4) 22. (4) 28. (1)
2. (4) 9. (3) 16. (4) 23. (2) 29. (2)
3. (2) 10. (2) 17. (2) 24. (2) 30. (2)
4. (1) 11. (3) 18. (1) 25. (3)
5. (2) 12. (3) 19. (2) 26. (4)
6. (3) 13. (1) 20. (3) 27. (3)
7. (1) 14. (2) 21. (1)
Hints & Solutions
1. Solution (2) 4𝑃𝑃2 = 𝑛𝑛𝑛𝑛𝑛𝑛𝑛𝑛𝑛𝑛𝑛𝑛 𝑤𝑤ℎ𝑖𝑖𝑖𝑖ℎ 𝑎𝑎𝑎𝑎𝑎𝑎 𝑠𝑠𝑠𝑠𝑠𝑠𝑠𝑠𝑠𝑠𝑠𝑠𝑠𝑠 𝑤𝑤𝑤𝑤𝑤𝑤ℎ 𝑧𝑧𝑧𝑧𝑧𝑧𝑧𝑧
5×4×3×2×1
𝑇𝑇𝑇𝑇𝑇𝑇𝑇𝑇𝑇𝑇 𝑤𝑤𝑤𝑤𝑤𝑤𝑤𝑤𝑤𝑤 = Total ways = 5𝑃𝑃3 – 4𝑃𝑃2
2×1
= 60 –12
= 60 = 48
9. Solution (3)
EXAMINATION
2. Solution (4) There are total 11 letters, of which A, I and N
Total words formed = 6𝑝𝑝3 11!
appear twice. Total arrangements are and
6×5×4×3×2×1 2! 2!2!
= the arrangements where A’s are together are
10!
3×2×1 2!2!

= 120
10. Solution (2)
𝐻𝐻𝐻𝐻𝐻𝐻𝐻𝐻𝐻𝐻 𝑡𝑡ℎ𝑒𝑒 𝑛𝑛𝑛𝑛𝑛𝑛𝑛𝑛𝑛𝑛𝑛𝑛 𝑜𝑜𝑜𝑜 𝑤𝑤𝑤𝑤𝑤𝑤𝑤𝑤 𝑤𝑤ℎ𝑒𝑒𝑒𝑒𝑒𝑒 𝐴𝐴’𝑠𝑠 𝑎𝑎𝑎𝑎𝑎𝑎
3. Solution (2)
∴ Total required ways =
9!
(since 4 A’s and 2 11! 10
4!×2! 𝑛𝑛𝑛𝑛𝑛𝑛 𝑡𝑡𝑡𝑡𝑡𝑡𝑡𝑡𝑡𝑡ℎ𝑒𝑒𝑒𝑒 𝑖𝑖𝑖𝑖 −
2! 2! 2! 2! 2!
L’s )
=
9×8×7×6×5 10! 11 10! 9 9 × 10!
2×1 � − 1� = × =
2! 2! 2 2! 2! 2 8
= 7560

4. Solution (1) 11. Solution (3)


For a number to be odd, last digit of that number Consider four blanks
must be an odd digit. The units place is filled with 5. The remaining
∴ Required ways = 2 × 3 × 2 = 12 three blanks can be filled with 4 digits in 4𝑃𝑃3
ways.
5. Solution (2) ∴ The number of four-digit numbers required is
Total letters = 12 (3I, 2N, 2T, 2S) 24.
12!
∴ 𝑇𝑇𝑇𝑇𝑇𝑇𝑇𝑇𝑇𝑇 𝑤𝑤𝑤𝑤𝑤𝑤𝑤𝑤 = 12. Solution (3)
3! × 2! × 2! × 2!
Here, different order gives different words. So,
12 × 11 × 10 × 9 × 8 × 7 × 6 × 5 permutations are needed to make words.
=
2 ⇒ Different words that can be formed
= 99,79,200 = 6P2 + 6P3 + 6P4 + 6P5 + 6P6 = 30 + 120 + 360 +
720 + 720 = 1950
6. Solution (3) Hence, option (C) is correct.
Required ways = 6! = 720
13. Solution (1)
7. Solution (1) The word 'LOGISTICK' has 9 different alphabets
(i) when repetition of digit is allowed Hence, the number of 3-letter words (with or
Total no. formed = 4 × 4 × 4 = 64 without meaning) formed by using these letters =
(ii) when repetition of digit is not allowed then (9P3)/2!
9 𝑥𝑥 8 𝑥𝑥 7
Total no. formed = 4𝑃𝑃3 = 24 = 42
3 𝑥𝑥 2 𝑥𝑥 2
= 10 × 9 × 8/2 = 360
8. Solution (3) Therefore, option (A) is correct.
5𝑃𝑃3 = 𝑡𝑡𝑡𝑡𝑡𝑡𝑡𝑡𝑡𝑡 𝑛𝑛𝑛𝑛𝑛𝑛𝑛𝑛𝑛𝑛𝑛𝑛 𝑓𝑓𝑓𝑓𝑓𝑓𝑓𝑓𝑓𝑓𝑓𝑓
14. Solution (2) Number of ways of selecting 4 out of (11 – 2) = 9
Take all the papayas as 1 and all the oranges as 1. people = 9C4 = 9!/(4! × 5!) = 126.
Total fruits becomes = 7 ∴ there are 126 ways of forming such a committee
For total fruits = 7!
For papayas = 3!
For oranges = 2! 20. Solution (3)
Total ways = 7! × 3! × 2! = 60,480 Number of ways two vegetables can be selected
Hence, Option B is correct. out of 4 vegetables = 4C2 = 6 ways
15. Solution (4) Number of ways two vegetables can be selected
Total number of ways when all the vowels always such that neither potato nor brinjal gets selected
come together = 5! × 2!/ 2! = 120 = (4 – 2)C2 = 2C2 = 1
Number of ways two vegetables can be selected
16. Solution (4) such that either a potato or a brinjal gets selected
Number of ways in which 12 toys can be arranged = = 6 – 1 = 5 ways.
12!
Number of ways when five particular toys are 21. Solution (1)
together = 8! X 5! Given,
Therefore Number of ways when three particular Total number of towns = 87
toys are not together = 12! – 8! X 5! Number of towns with at most 2 persons = 52
Remaining towns = 87 – 52 = 35
17. Solution (2) Also, for the city development programme, 20
In the word “DISTEMPER” there are 9 letters in towns are to be chosen for assistance, of which at
which there are 3 vowels (i.e. 2 E’s and I) and 6 least 18 towns must have at most 2 persons.
consonants (i.e each of D, S, T, M, P, R) Thus, the following are the number of possible
Considering vowel as one letter, the number of choices:
letters becomes 7 which can be arranged as 52C18 × 35C2 (18 towns having at most 2
7! persons and 2 selected from other types of towns)
Vowel E appear twice , so vowels can be arranged 52C19 × 35C1 (19 towns having at most 2
as persons and 1 selected from other types of towns)
Hence, required number of ways = 7! × 3 = 15120 52C20 (All selected 20 towns having at most 2
Hence, option B is correct. persons)
Hence, the total number of possible choices
18. Solution (1) = 52C18 × 35C2 + 52C19 × 35C1 + 52C20
8 people can sit in 7! Ways around a table.
Now ATQ 22. Solution (4)
Madan and Mohnish doesn’t sit opposite Given,
Let fix their sit so other 6 can sit anywhere Men = 2
= 7! – 2 × 6! Women = (x - 2)
= 6! (7 – 2) A committee of (x - 2) persons to be constituted.
= 3600 Here, the order does not matter.
Therefore, we need to count combinations.
19. Solution (2) There will be as many committees as
Number of persons to be selected if two particular combinations of (x - 2 + 2) = x different persons
persons must be included = 6 – 2 = 4 taken (x - 2) at a time.
Hence, the required number of ways = xC(x-2)
= x!/((x - 2)! 2!) = 10 3 black monkeys and 2 red monkeys
= x(x - 1)/2 = 10 = 7C3 × 4C2 = 35 × 6 = 210
x2 - x - 20 = 0
4 black monkeys and 1 red
x = 5, -4 monkey= 7C4 × 4C1 = 35 × 4 = 140
So, x = 5
Total number of ways the team can have at least
23. Solution (2) one black monkey and one red monkey= 7 +
Committees with 1 man and 2 women: 84 + 210 + 140
1 man can be selected from 2 men in 2C1 ways.
2 women can be selected from 3 women = 441
in 3C2 ways.
Therefore, the required number of committees
= 2C1 × 3C2 26. Solution (4)
= 2 × 3C1 Given,
=2×3
=6 Number of red monkeys = 7

24. Solution (2) Number of black monkeys = 7


Given,
At least three red monkeys
Number of red monkeys = 7
Total number of ways the team can have at least
Number of black monkeys = 7 three red monkeys = 4C3 × 7C2 + 4C4 × 7C1

24. Ans. (b) = 4 × 21 + 7

No red monkeys = 84 + 7

Total number of ways the team can have no red = 91


monkeys = 4C0 × 7C5
27. Solution (3)
= 1 × 21
QUESTIONS
= 21
Vowels = U, E, I, O (4)

25. Solution (3) Consonants = Q, S, T, N, S (5) (2S, Q, T, N)


Given,
Odd places = 1,3,5,7,9
Number of red monkeys = 7
Vowels come at odd places = 5P4 = 120
Number of black monkeys = 7 120 × 5!
Required number of ways = = 7200
2!
At least one black monkey and one red monkey

1 black monkey and 4 red monkeys


= 7C1 × 4C4 = 7 × 1 = 7 28. Solution (a)

2 black monkeys and 3 red monkeys For odd number, last digit must be an odd
= 7C2 × 4C3 = 21 × 4 = 84 number.

Required answer = 3 × 2 × 2 = 12
29. Solution (2) Vowels = I, E, E (2E)

Required ways = 7! × 3! × 2! = 60480 Total letters = 9 (2E)


7! × 3!
Required number of ways = = 30240/2 =
2!
15120
30. Solution (2)

DISTEMPER
1. Two dice are rolled randomly. Find the probability 7. Two Persons attend an interview for two vacancies.
to get sum is 10. The probability of 1st person selection is (1/4) and
(1) 2/5 (2) 14 the probability of 2nd person selection is (1/6).
(3) 2/9 (4) 1/12 What is the probability that only one of them is
(5) 5/8 selected ?
(1) 1/4 (2) 2/7
2. Two dices are rolled out together, then what is the (3) 4/5 (4) 1/3
probability of getting a number of one dice greater (5) None
than the number of other dice ?
(1) 2/3 (2) 1/6 8. A committee of 4 is to be formed from among 4
(3) 3/8 (4) 1/8 girls and 5 boys. What is the probability that the
(5) 5/6 committee will have number of boys less than
number of girls?
3. If 2 cards is drawn at randomly from 52 cards, then (1) 2/3 (2) 1/6
find the probability of getting both are red cards. (3) 2/7 (4) 4/7
(1) 13/28 (2) 26/105 (5) None
(3) 28/108 (4) 25/102
(5) none of these 9. Ticket numbered from 1 to 30 are mixed up and a
ticket is drawn at random. What is the probability
4. When two coins are tossed simultaneously, then that the ticket drawn has a number which is a
find the probability of getting at least one tail. multiple of 4 or 6.
(1) 5 (2) 3 (1) 6/13 (2) 3/10
(3) 3/4 (4) 1/4 (3) 5/9 (4) 1/10
(5) 2/5 (5) None

5. A bag contains 4 red balls, 2 blue balls and 2 green 10. Out of 25 applicants 13 boys and 12 girls. Two
balls . If two balls are drawn randomly from the bag persons are to be selected for the job. Find the
, then find the probability of getting both balls of probability that at least one of the selected persons
different color. will be a boy.
(1) 2/7 (2) 5/8 (1) 32/50 (2) 42/50
(3) 5/7 (4) 4/7 (3) 39/50 (4) 28/50
(5) 8/9 (5) None

6. A problem is given to three students whose chances 11. One card is drawn at random from a pack of 52
of solving it are 1/3, 1/4 and 1/5 respectively. What cards. What is the probability that the card drawn is
is the probability that the problem will be solved? either a black card or a queen?
(1) 1/2 (2) 3/4 (1) 7/9 (2) 4/9
(3) 3/5 (4) 4/5 (3) 11/13 (4) 7/13
(5) None (5) None
12. In a school, 35% of the students play chess,45% 19. A committee of 3 members is to be made out of 6
play carom and 10% play both. If a student is boys and 5 girls. What is the probability that the
selected at random, then the probability that he committee has at least two girls?
plays chess or carom is (1) 14/33 (2) 10/32
(1) 1/10 (2) 2/5 (3) 15/31 (4) 17/35
(5) None of these
(3) 7/10 (4) 3/10
(5) None 20. 8 persons are seated at a round table. What is the
probability that 3 particular persons sit together?
13. A bag contains 8 red balls and x blue balls and the (1) 2/9 (2) 4/9
probability of choosing a blue ball is 3/5. If we (3) 1/7 (4) 2/7
randomly select two balls, find the probability that (5) None
at least one of them is red.
(1) 55/91 (2) 63/94 21. When two dice are thrown simultaneously, then
(3) 62/95 (4) 61/93 find the probability that sum of the digits on both
the faces is 8.
(5) 59/95
(1) 9/37 (2) 7/36
(3) 3/37 (4) 5/36
14. A bag contains 5 ruby balls and 7 black balls. Two (5) 11/36
balls are drawn at random without replacement, and
then find the probability of that one is ruby and 22. When two dice are thrown simultaneously, then
other is black? find the probability that sum of the digits is a
(1) 35/66 (2) 41/69 multiple of 4.
(3) 36/68 (4) 30/60 (1) 1/4 (2) 3/10
(5) None of these (3) 9/10 (4) 13/36
(5) 17/36
15. X and Y are two persons standing in a circular
23. A bag contains 6 blue balls and 8 pink balls. If a ball
arrangement with 10 more people. Find the is drawn randomly from it, then find the probability
probability that exactly 3 persons are seated of getting either pink ball or blue ball.
between X and Y? (1) 4 (2) 5
(1) 1/5 (2) 3/9 (3) 1 (4) 1/4
(3) 2/11 (4) 1/10 (5) 2/5
(5) None of these
24. In a box there are 4 red balls and 3 blue balls. If two
16. Find the probability that in a leap year, the numbers balls are drawn at random, then find the probability
of Wednesday are 53? of that none is blue.
(1) 2/5 (2) 2/7
(1) 2/7 (2) 1/7
(3) 5/8 (4) 4/15
(3) 3/7 (4) 4/7 (5) 4/18
(5) 5/7
25. A basket contains 5 white balls , 4 red balls, 2 blue
17. There are 4 ruby balls, 5 white and 3 grey balls. 3 balls and 3 yellow balls. If four balls are picked at
balls are chosen at random. What is the probability randomly. What is the probability that 2 are blue
that there is at most 1 grey ball? and 2 are red ?
(1) 48/55 (2) 40/55 (1) 5/1002 (2) 8/968
(3) 42/54 (4) 50/82 (3) 12/1005 (4) 8/950
(5) None of these (5) 6/1001

26. When two dices are thrown simultaneously, then


18. When 4 fair coins are tossed togethe what is find the probability that sum is a prime number
probability of getting at least 3 heads? which is less than 8 ?
(1) 5/16 (2) 3/4 (1) 12/38 (2) 20/36
(3) 7/13 (4) 10/14 (3) 20/35 (4) 13/36
(5) None of these (5) 15/48
27. How ,any 4-digit numbers can be formed from the 34. A bag contains 6 apples, 8 bananas and (x+2)
digits 6, 2, 5, 7, 3, which are divisible by 5 and none oranges. Two fruits are chosen at random. Find the
of the digits is repeated ? value of x if the probability that both fruits are
(1) 140 (2) 220 oranges is 2/51.
(3) 28 (4) 24 (1) 2 (2) 5
(5) 96 (3) 3 (4) 4
(5) 6
28. When three coins are tossed simultaneously, then
find the probability of getting at least one tale. 35. A box contains 20 bulbs out of which 5 are
(1) 7/8 (2) 5/8 defective. Three bulbs are randomly taken out of the
(3) 1/4 (4) 4/15 box. What is the probability that out of the three at
(5) 2 least one bulb is defective?
(1) 128/221 (2) 137/228
29. What is the probability that number selected from (3) 131/220 (4) 129/228
number 1, 2 , 3 … 100 is a prime number ? (5) 130/221
(1) 2/5 (2) 1/4
(3) 5/8 (4) 1/2 36. A bag contains ‘x’ red balls , ‘x+2’ pink balls. Two
(5) 4/7 balls are randomly drawn from the bag and the
probability that a red and a blue ball are drawn is
30. There are 250 tickets in an urn numbered 1 to 250. 4/21. Find the total number of balls in the bag.
One ticket is chosen at random. What is the (1) 33 (2) 40
probability of it being a number containing a (3) 38 (4) 49
multiple of 3 or 8? (5) 30
(1) 52/125 (2) 53/250
(3) 67/125 (4) 101/250 37. A bag contains red, blue and green balls in the ratio
(5) 13/25 of 3:5:4 resp. 10 pink balls are put in the bag and
two balls are randomly drawn from the bag. The
31. There are 2 urns. 1st urn contains 6 white and 6 blue probability that one ball is green and other is red is
balls. 2nd urn contains 5 white and 7 black balls. 20/161. Find the difference in the number of green
One ball is taken at random from first urn and put and red balls in the bag.
to second urn without noticing its color. Now a ball (1) 6
is chosen at random from 2nd urn. What is the (2) 5
probability of the second ball being a white colored (3) 7
ball? (4) 4
(1) 11/13 (2) 6/13 (5) 2
(3) 5/13 (4) 5/12
(5) 11/12 38. A committee of 5 people is formed with the help of
4 male and 6 females. Find the probability that at
32. Four persons are chosen at random from a group of most three of them are male.
3 men, 3 women and 4 children. What is the (1) ½
probability that exactly 2 of them will be men? (2) 39/41
(1) 1/9 (2) 3/10 (3) 43/44
(3) 4/15 (4) 1/10 (4) 41/42
(5) 5/12 (5) 2/3

33. A Bag contains 100 balloons, numbered from 1 to 39. The odds against an event are 1 : 3 and the odds in
100. If three balloons are selected at random and favor of another independent event are 2 : 5. Find
with replacement from the bag, what is the the probability that one of the event will occur.
probability that the sum of the three numbers on the (1) 17/28
balloons selected from the bag will be odd? (2) 5/14
(1) ½ (2) ¼ (3) 11/25
(3) 2/3 (4) 1/5 (4) 9/14
(5) None (5) 19/28
40. In a basket, there are some apples and 5 oranges. (1) 0.5
Out of which 30% of fruits are rotten. If one fruit is (2) 0.3
chosen then find the probability of that fruit being (3) 0.8
good fruit. (4) 0.7
(5) None of these
Answer Key
1. (4) 9. (2) 17. (1) 25. (5) 33. (1)
2. (5) 10. (3) 18. (1) 26. (4) 34. (1)
3. (4) 11. (4) 19. (1) 27. (4) 35. (2)
4. (3) 12. (3) 20. (3) 28. (1) 36. (4)
5. (3) 13. (3) 21. (4) 29. (2) 37. (2)
6. (3) 14. (1) 22. (1) 30. (1) 38. (4)
7. (4) 15. (3) 23. (3) 31. (1) 39. (1)
8. (2) 16. (1) 24. (2) 32. (2) 40. (4)
Hints & Solutions
1. Solution (4) 8. Solution (2)
Required = (6 , 4 ) , (4 , 6) , ( 5, 5) = 3 ways 4 girls and 0 boys
total = 6 * 6 = 36 =4C4/9C4
probability = 3/36 = 1/12
ways 3 girls and 1 boy
2. Solution (5) =4C3*5C1/9C4
Non-favorable events = (1 , 1) , (2 , 2) , (3 , 3) , Total probability = 4C4/9C4 + 4C3*5C1/9C4
(4 , 4) , (5 , 5) , (6,6) = 6 =1/126+ 20/126=21/126=1/6
total = 6 * 6 = 36
probability of non-favorable events = 6/36 = 1/6
probability of favorable events = 1 – 1/6 = 5/6 9. Solution (2)
n(s)=30
3. Solution (4) Multiple of 4 ={4,8,12,16,20,24,28}
Required = 26C2 = 26 * 25/2 = 13 * 25 Multiple of 6={6,12,18,24}
total = 52C2 = 52 * 51 / 2 = 26 * 51 Then n(e)={4,6,8,12,16,18,20,24,28}|
probability = 13 * 25 / 26 * 51 = 25 /102
P=n(e)/n(s)=9/30=3/10.
4. Solution (3)
Required = ( 1 head , 1 tail ) , ( 2 tails) = (2C1) 10. Solution (3)
+ (2C2) = 2 + 1 = 3 13C1*12C1/25C2 + 13C2 *12C0/25C2
total = 2^2 = 4 =13*12/25*12 + 13*6*1/25*12
probability = 3/4
=13/25 + 13/50==> 39/50.
5. Solution (3)
probability = 4C1 * 2C1 / 8C2 + 4C1 * 2C1 / 8C2 11. Solution (4)
+ 2C1 * 2C1 / 8C2 There are 26 black cards and 4 queen(2queen
(4 * 2 * 2)/ 8 * 7 + (4 * 2 * 2)/ 8 * 7 + (2 * 2 * 2) included in black cards)
/ 8 * 7 = 2/7 + 2/7 + 1/7 = 5/7
P=(26C1+2C1)/52C1
6. Solution (3) 28/52=7/13.
Given chances of students to solve is 1/3, 1/4,
1/5 12. Solution (3)
P(the problem will be solved) = 1 – P(none P(A or B)=P(+ P(B)-P(A and B)
solves the problem)
P(none solves the problem)=(1-1/3)*(1- =35/100+45/100-10/100
1/4)*(1-1/5) =70/100=7/10.
=2/3*3/4*4/5=24/60
P(the problem will be solved)=1-24/60 13. Solution (3)
=(60-24)/60=36/60=3/5 Probability of choosing a blue ball = 3/5
7. Solution (4) x/(x+8) = 3/5
Give chance for 1st and 2nd person get selected x = 12
is 1/4 and 1/6 Required Probability = [8C1*12C1+8C2] /
1st person not get selected chance is (1- (8+12) C2 = 62/95
1/4)=3/4
2nd person not get selected chance is (1-
1/6)=5/6 14. Solution (1)
P(only one get selected)=P(1st selected and (First ruby ball is drawn and then black ball is
2nd not selected) + P(1st not selected and 2nd drawn) + (first black ball is drawn and then ruby
selected) ball is drawn)
=(1/4*5/6) + (3/4*1/6) =8/24=1/3 (5/12)*(7/11)+(7/12)*(5/11) = 35/66
15. Solution (3) 23. Solution (3)
Fix X at one point then number of places where required probability=6/14+8/14=14/14=1
Y can be seated is 11.
Now, exactly three persons can be seated between 24. Solution (2)
X and Y, so only two places where Y can be required probability=(4C2)/(7C2)
seated. =4*3/7*6=2/7
So, X = 2/11
25. Solution (5)
16. Solution (1) required=(2C2)*(4C2)
In a leap year there are 52 complete weeks i.e. =(2*1/2*1)*(4*3/2*1)=1*6
364 days and 2 more days. total=14C4=14*13*12*11/4*3*2*1
These 2 days can be SM, MT, TW, WT, TF, FS, =7*13*11
and SS. probability=1*6/7*13*11=6/1001
So Probability of Wednesday = 2/7
26. Solution (4)
17. Solution (1) prime numbers=2,3,5,7
48/55 possibility of 2=(1,1)
possibility of 3=(1,2), (2,1)
18. Solution (1) possibility of 5=(1,4), (4,1), (3,2), (2,3)
5/16 possibility of 7=(1,6), (6,1), (2,5), (5,2), (3,4),
(4,3)
19. Solution (1) required=1+2+4+6=13
Total possible outcomes=11C3 = 165 total=6^2=36
Favourable outcomes = (1B, 2G) or (3G) probability=13/36
= 6C1 × 5C2 + 5C3
= 6 × 10 + 10 27. Solution (4)
= 70 A number to be divisible by 5, unit digit must be
Required probability = 70/165 = 14/33 0 or 5.
So, in 4-digit numbers unit place is required to be
20. Solution (3) filled by digit 5 and rest three places of required
3 particular person always sit together, then total 4-digit numbers will be filled by rest 4 digit.
person =5+1=6 numbers of ways = 4! = 4 * 3 * 2 * 1 = 24
6 persons will sit in (6-1)!=5! At round table.
No of ways in which 3 persons always sit 28. Solution (1)
together=5!*3! possibility = 1 tale or 2 tale or 3 tale
No of ways of sitting 8 persons at round table=(8- required = (3C1) + (3C2) + (3C3)
1)!=7!. =3+3+1=7
Then P= (5!*3!)/7! =1/7. total = 2^3 = 8
probability = 7/8
21. Solution (4)
total no. of outcome=6^2=36 29. Solution (2)
required no. of outcome=(3,5), (5,3), (6,2), (2,6), Probability = 25/100 = ¼
(4,4)=5
probability=5/36 30. Solution (1)
Multiples of 3 up to 250 = 250/3 = 83 (take
22. Solution (1) only whole number before the decimal part)
total no. of outcome=6^2=36 Multiples of 8 up to 250 = 250/3 = 31
required no. of outcome=(4,8,12) Multiples of 24 (3×8) up to 250 = 250/24 = 10
4=(2,2), (3,1), (1,3) So total such numbers are = 83 + 31 – 10
8=(3,5), (5,3), (6,2), (2,6), (4,4) = 104
12=(6,6) So required probability = 104/250 = 52/125
probability=(3+5+1)/36
=9/36=1/4
31. Solution (1) 2x(x+2)/(3x+7)(3x+6) = 4/21
Case 1: first was a white ball => x = 14
Now it is put in second urn, so total white balls The number of balls in the bag = 14*3+7 = 49
in second urn = 5+1 = 6, and total balls in
second urn = 12+1 = 13 37. Solution (2)
So probability of white ball from second urn = Probability that one ball is red and other is
6/13 green
Case 2: first was a blue ball = (3xC1*4xC1)/ (12x+10)C2 = 20/161
Now it is put in second urn, so total white balls => 246x^2 – 1140x – 450 = 0
in second urn remain 5, and total balls in => 41x^2 – 190x – 75 = 0
second urn = 12+1 = 13 => x = 5, -15/41
So probability of white ball from second urn = Bag contains 10 pink, 15 red , 25 blue and 20
5/13 green balls.
So required probability = 6/13 + 5/13 = 11/13 Difference in the number of green and red balls
(added the cases because we want one of these = 20 – 15 = 5
cases to happen and not both)
38. Solution (4)
32. Solution (2) 4M, 6F
2 men means other 2 woman and children Favourable cases = (0M, 5F) or (1M, 4F) or
So prob. = 3C2 × 7C2 /10C4 = 3/10 (2M, 3F) or (3M, 2F)
= 6C5 + 4C1 * 6C4 + 4C2*6C3 + 4C3*6C2
33. Solution (1) = 6 + 4 *15 + 6*20 + 4*15
P(odd) =P (even) = 1/2 (because there are 50 = 6 + 60 + 120 + 60 = 246
odd and 50 even numbers) Possible cases = 10C5
Sum of the three numbers can be odd only = 10*9*8*7*6/5*4*3*2
under the following 4 scenarios: = 42*6 = 252
Odd + Odd + Odd = 1/2*1/2*1/2 =1/8 Required prob = 246/252 = 41/42
Odd + Even + Even =1/2*1/2*1/2 =1/8
Even + Odd + Even = 1/2*1/2*1/2 =1/8 39. Solution (1)
Even + Even + Odd =1/2*1/2*1/2 =1/8 Let 2 events A and B
Adding all 1/8+1/8+1/8+1/8=4/8==>1/2. Odds against A are 1 : 3
So probability of occurrence of A = 3/(1+3)
34. Solution (1) = 3/4. And non-occurrence of A = ¼
Probability of selecting two oranges = Odds in favor of B are 2 : 5
(x+2)C2/(16+x)C2 Now, (x+2)C2/(16+x)C2 = So probability of occurrence of B = 2/(2+5)
2/51 = 2/7. And non-occurrence of B = 5/7
=> 7x^2 + 13x – 54 = 0 Case 1: A occurs and B does not occur
=> 7x^2 – 14x + 27x – 54 = 0 So probability = 3/4 * 5/7 = 15/28
=> x = 2 or -27/7 Case 2: B occurs and A does not occur
So probability = 2/7 * 1/4 = 2/28
35. Solution (2) So probability that one will occur = 15/28
Probability that atleast one bulb is defective = + 2/28 = 17/28
1 – P (All are non-defective)
= 1 – 15C3/20C3 = 137/228 40. Solution (4)
Let number of apples = x
36. Solution (4) Total fruits = (x + 5)
Total number of balls in the bag = x+x+2+x+5 Good fruits = 0.7(x + 5)
= 3x+7 Required probability = 0.7(x + 5)/(x + 5) = 0.7
Probability that a red and a blue ball are drawn
= (xC1*(x+2)C1)/(3x+7)C2 = 4/21
Direction (1-5) : Study the table given below and 5. The total no. of 10 mm size ball bearings
answer the questions that follow: manufactured in the year 2007 is approximately
Years Total number of 10 mm : what percent of total no. of 8 mm size ball
ball bearings 8 mm bearings manufactured in the year 2008?
manufactured in (1) 141% (2) 130%
5 different years (3) 110% (4) 121%
(in lakhs)
(5) 105%
2005 286 7:4
2006 290 3:2
2007 248 5:3 Direction (6-10) : The following table shows the
2008 272 9:8 total no. employees of different age group working
2009 300 13 : 12 in an MNC. Table also shows the ratio of male to
female among them. Study the table carefully to
1. What is the approximate average no. of 8 mm size answer the following questions.
ball bearings manufactured in the years 2005, 2007 Age group Total no. Ratio of
and 2009 together? (in year) of male to
(1) 132 lakhs (2) 114 lakhs employees female
(3) 150 lakhs (4) 90 lakhs 20–30 9600 5:3
(5) 84 lakhs 31–40 12800 3:1
41–50 19200 7:5
51–60 24000 2:1
2. The no. of 10 mm size ball bearings manufactured
in year 2006 is what percent more or less than the
6. Total no. of female employees of age group (20–
no. of 8 mm size ball bearings manufactured in the
30) years are how much percent more or less than
same year?
female employees of age group (31–40) years?
(1) 75% more (2) 50% less
(1) 12.5% more (2) 12.5% less
(3) 50% more (4) 75% less
(3) 10.5% more (4) 10.5% less
(5) None of these
(5) 8.5% more

3. In which year manufacturing of 8 mm size ball


7. What is the average no. of male employees of age
bearings is least?
group (31–40) years and (51–60) years together?
(1) 2009 (2) 2008
(1) 14400 (2) 10400
(3) 2006 (4) 2007
(3) 12800 (4) 11200
(5) None of these
(5) 9200

4. Find the difference between total no. of 10 mm


size ball bearings manufactured in the years 2006 8. If employees of age group (51 – 60) were
and 2008 together and total no. of 8 mm size ball retired due to endness of service bond,then
bearings manufactured in the years 2005 and 2009 employees now of this age group are what percent
together. of employees of age group (31–40) years and (41–
(1) 70 lakhs (2) 80 lakhs 50) years together?
(3) 50 lakhs (4) 90 lakhs (1) 50% (2) 60%
(5) 60 lakhs (3) 54% (4) 64%
(5) 45%
9. If 20% male employees of age group (41 –50) yrs 14. What is the difference between total males stuck in
are getting incentive due to excellent performance, traffic on Tuesday, Wednesday and Thursday
then how many male employees are performing together and total females who were stuck in
normal of the same age group? traffic on Monday, Thursday and Friday together
(1) 8560 (2) 9760 due to election rallies?
(3) 8960 (4) 9680 (1) 4,800 (2) 1,000
(5) 8160 (3) 3,600 (4) 3,200
(5) 4,200
10. What is the difference between male and female
employees of age group (20 –30) and (31 –40) yrs 15. Total number of people who were stuck due to
together? election rallies on Monday and Tuesday together is
(1) 9200 (2) 8400 what percent of total number of people who were
(3) 9600 (4) 8800 stuck due to election rallies on Tuesday and
(5) 9200 Wednesday together?
(1) 113 1 % (2) 213 1 %
Direction (11-15) : The following table shows the 3 3
total number of people who were stuck in traffic (3) 113 2 % (4) 103 1 %
3 3
due to election rallies on different days of week,
going to their office. Table also shows ratio of (5) 93 1 %
3
male to female among them. Study the table
carefully to answer the following questions. Directions (16 - 20) : The following table shows
Day Total no. Male: total no. of participants who have participated in
of people Female different season of Commonwealth Games
Monday 9600 5: 3 organized in different years and also the
Tuesday 10800 5: 4 percentage of the participants who won medals.
Wednesday 7200 2: 1 Study the table and answer the questions.
Thursday 6000 3: 2 Percentage
Friday 5400 1: 2 of
participants
11. If 44% males on Tuesday, 48% males on Total no. of who won
Wednesday reached office on regular time of Years Participants medals
office, then how many males become late on these 2000 4000 4%
two days together? 2004 6000 8(1/3)%
(1) 5677 (2) 5856 2008 5000 6%
(3) 5676 (4) 5886 2012 8000 12(1/2)%
(5) 5776 2016 9000 3(1/3)%
12. Find the average number of females who were
16. If ratio of male to female who won medals in 2004
stuck in traffic on different days given above.
(1) 2460 (2) 3660 CW games is 3 : 2, then find number of females
(3) 3360 (4) 3120 who won medals in 2004.
(5) 3480 (1) 100 (2) 200
(3) 300 (4) 250
13. Total number of males who were stuck in traffic (5) 400
due to election rallies on Monday and Thursday
together is what percent more or less than the total
17. If ratio of gold, silver and bronze medals won by
number of females who were stuck in traffic due to
election rallies on Tuesday and Friday together? participants in 2000 CW games be 4 : 5 : 7, then
(1) (2) find the difference between number of gold and
bronze medals won by participants in 2000 CW.
(3) (4) 86/7%
(1) 20 (2) 40
(5) 11% (3) 30 (4) 45
(5) 35
18. If 66(2/3)% of total medals won by participants in 22. Find the ratio between the number of girls of
2008 were Indian athletes and rest medals were colony A and boys of the same colony A.
won by UK and Japan in the ratio 1 : 3, then find (1) 24 : 11 (2) 27 : 17
medals won by India is what percent more than (3) 22 : 15 (4) 27 : 14
that by Japan in 2008? (5) 31 : 17
(1) 166(2/3)% (2) 66(2/3)%
(3) 165% (4) 133(1/3)% 23. The average number of adult female population in
(5) None of these five colonies?
(1) 2463 (2) 2412
19. What is the average number of participants who (3) 2439 (4) 2493
didn’t won any medal in 2012 and 2016? (5) 2355
(1) 7800 (2) 8700
(3) 6850 (4) 7850 24. The number of adult female population of colony
(5) None of these E is what percent more than the adult male
population of the same colony?
20. What is the percentage of number of persons who (1) 40% (2) 75%
don’t won any medal in 2012 with respect to (3) 50% (4) 33.33%
number of persons who don’t won any medal in (5) 66.67%
2008?
(1) 148(2/47)% (2) 148(41/47)% 25. The boys of colony D is approximately what
(3) 138(44/47)% (4) 148(44/47)% percent of total boys of five colony?
(5) 184(44/47)% (1) 22% (2) 17%
(3) 26% (4) 13%
Direction (21 - 25) : Read the following table (5) 8%
carefully and answer the following questions:
The table shows the total population of five Direction (26 - 30) : Study the following table and
colonies including boy, girl, male and female answer the related questions given below:
population. Table shows the data related to elephants in five
Total number of Females= adult female population zoo.
+ total girls Zoo Total no. of Ratio of male % of elephants
Total number of Males = adult male population + elephants to female who are below
total boys elephants 45 years
Total % of girls % of boys Male : P 750 4: 1 60%
Colony

population among among Female Q 1620 5: 4 75%


females males R 1350 3: 2 40%
A 24000 90 85 2:3 S 1560 8: 5 50%
B 30000 80 70 3:5 T 1080 2: 1 70%
C 36000 85 95 2:1
D 18900 95 90 4:5 26. The total no. of male elephants in Q and S together
E 33600 75 80 3:4 is how much percent more than that of male
elephants in R and T together?
21. Find the difference between the number of girls in (1) (1)
colony B and C.
(3) CND (4)
(1) 5400 (2) 7800
(3) 6600 (4) 4800 (5)
(5) 7200
27. In which zoo,elephants who are above 45 years 29. What is the total no. of elephants having age
are maximum? below 45 years in zooP, R and T together?
(1) Q (2) S (1) 2046 (2) 1446
(3) 1584 (4) 1746
(3) R (4) T
(5) 1644
(5) P
30. Total no. of female elephants in Q and T together
28. What is the average no. of male elephants in Q, R is what percent of total no. of female elephants in
and S together? P and R together?
(1) (2) 837 (1) (2)
(3) 879 (4) (3) (4)
(5) 849 (5) None of these
Answer Key
1. (2) 7. (3) 13. (1) 19. (4) 25. (4)
2. (3) 8. (1) 14. (1) 20. (4) 26. (2)
3. (4) 9. (3) 15. (1) 21. (4) 27. (3)
4. (1) 10. (4) 16. (2) 22. (2) 28. (1)
5. (4) 11. (2) 17. (3) 23. (1) 29. (4)
6. (1) 12. (3) 18. (1) 24. (5) 30. (4)
Hints & Solutions
1. Solution (2) 7. Solution (3)
Required average = × (4 × 26 + 3 × 31 + 12 × Required average
12) = 113.66≃ 114 lakhs (approx.)

2. Solution (3)
Required percentage = 12800
= × 100 = 50%
8. Solution (1)
Remaining employees
3. Solution (4)
No. of 8 mm size ball bearings manufactured in ( )
year 2005 = 16000
∴Required percentage

= 50%

9. Solution (3)
Male employees who are not getting incentive
∴ No. of 8 mm size ball bearings manufactured
in year 2007 is min.

4. Solution (1)
Required difference = (3 × 58 + 9 × 16) – (26 × = 8960
4 + 12 × 12) = 318 – 248 = 70 lakhs
10. Solution (4)
Required difference
5. Solution (4)
( ) ( )
Required percentage = × 100
= 6000 + 9600 – 3600 – 3200
≃ 121% (approximately) = 8800

6. Solution (1) 11. Solution (2)


Female employees of age group (20 – 30) yrs Required no. of males
=
= 3360 + 2496 = 5856
Female employees of age group (31 – 40) yrs
12. Solution (3)
= 3200 Required average
Required percentage = (
)
= 12.5% more = =
= 3,360
13. Solution (1) 18. Solution (1)
Total males stuck in traffic on Monday and 66(2/3)% = 2/3
Thursday together Medals won by India = 2/3 × 300 = 200
Medals won by UK and Japan = 1/3 × 300 = 100
= Medals won by Japan = ¾ × 100 = 75
= 6000 + 3600= 9600 Required % =
Total females stuck in traffic on Tuesday and = 166(2/3)%
Friday together
= 19. Solution (4)
= 4800 + 3600 = 8400 Required average =
– = 15700/2 = 7850
Required% =
20. Solution (4)
14. Solution (1) Required percentage = × 100
Required difference = 148(44/47)%
= ( )–(
21. Solution (4)
)= 6000 + 4800 + Total number of girls in colony B
= 30000 × 5/8 × 80/100
3600 – 3600 – 2400 – 3600= 4800
= 15000
Total number of girls in colony C
15. Solution (1) = 36000 × 1/3 × 85/100
Required percentage = = 10200
Required difference = 15000 – 10200= 4800

22. Solution (2)


Number of girls in colony A
Solution (16-20) : Number of participants who = 24000 × 3/5 × 90/100
won medals = 12960
In 2000 = 0.04 × 4000 = 160 Number of boys in colony A
In 2004 = 6000 × 1/12 = 500 = 24000 × 2/5 × 85/100
= 8160
In 2008 = 5000 × 0.06 = 300
Required ratio =
In 2012 = 8000 × 1/8 = 1000 = 12960 : 8160= 27: 17
In 2016 = 9000 × 1/30 = 300
Years Total no. of Number of 23. Solution (1)
Participants participants Total adultfemale population in colony A
who won = 24000 × 3/5 × 10/100 = 1440
medals Total adultfemale population in colony B
2000 4000 160 = 30000 × 5/8 × 20/100 = 3750
2004 6000 500 Total adultfemale population in colony C
2008 5000 300 = 36000 × 1/3 × 15/100 = 1800
2012 8000 1000 Total adultfemale population in colony D
2016 9000 300 = 18900 × 5/9 × 5/100 = 525
Total adultfemale population in colony E
16. Solution (2) = 33600 × 4/7 × 25/100 = 4800
Total adultfemale population in five colonies
Required answer = = 200 = 1440 + 3750 + 1800 + 525 + 4800
= 12315
17. Solution (3) Required average = 12315/5 = 2463
Required answer = = 30
24. Solution (5) 27. Solution (3)
Total adultfemale population in colony E From observation of table, there are only two
= 33600 × 4/7 × 25/100 = 4800 zoo in which the elephants having age above 45
Total adultmale population in colony E
years seems maximum
= 33600 × 3/7 × 20/100 = 2880
These are R and S
⇒Required percentage =
So, elephants (having age > 45 years) in R
= 66.67%
= = 810
25. Solution (4) And, elephants (age>45 year) in S
Total boys in colony A
=
= 24000 × 2/5 × 85/100
= 8160 = 780
Total boys in colony B Clearly, the required answer is R.
= 30000 × 3/8 × 70/100
= 7825 28. Solution (1)
Total boys in colony C
Required average no.
= 36000 × 2/3 × 95/100
= 22800 ( )
Total boys in colony D
= 18900 × 4/9 × 90/100 = = 890
= 7560
Total boys in colony E
29. Solution (4)
= 33600 × 3/7 × 80/100
= 11520 Total no. of elephants having age below 45 years
Total boys in five colonys in P, R and T
= 8160 + 7825 + 22800+ 7560 + 11520 =
= 57915
Required percentage = 450 + 540 + 756 = 1746
= (7560/57915) × 100
= 13.05% ≈ 13% 30. Solution (4)
Total no. of female elephants in Q and T
26. Solution (2) together =
Total no. of male elephants in Q and S together
= = 720 + 360
= 1080
= 900 + 960
= 1860 Total no. of female elephants in P and R
Total no. of male elephants in R and T together together =
= 150 + 540 = 690
= 1530 ∴Required percentage
∴ Required percentage =
= =
=
Direction (1-5) : Given below is the bar graph 5. Chairs sold by store B is what percent more or less
which shows the number of Tables and Chairs sold than Chairs sold by store C.
by four different shops A, B, C and D. (1) (2) 18%
(3) 15% (4) 22%
(5) 20%

Direction (6-10) : Study the bar graph carefully


and answer the given questions.
In the bar graph data is given about obtained
percentage (%) of marks of three students in the
different subjects.

1. What is the average number of chairs sold by store


B, C and D?
(1) 3750 (2) 3400
(3) 3850 (4) 3800
(5) 3600
6. If average marks obtained by all three students in
2. Tables sold by store A is what percent more than Sanskrit is 170 then what is full marks of this
Chairs sold by store C (approximately)? subject?
(1) 66% (2) 120% (1) 200 (2) 300
(3) 175% (4) 133% (3) 150 (4) 100
(5) 166% (5) None of these

3. Total Chairs sold by store B and C together are 7. If full marks is 200 in Hindi and 150 in English
how much more or less than total Tables sold by then what is respective ratio of total obtained
store C and D together? marks of all three in these subjects?
(1) 4200 (2) 4000 (1) 141: 172 (2) 172: 141
(3) 4250 (4) 3000 (3) 152: 133 (4) 133: 152
(5) 3600 (5) None of these

4. What is the ratio of tables sold by store A and C 8. If respective ratio of full marks of History and
together to total Chairs sold by stores B and D Geography is 3: 4 then what is ratio between
together? marks obtained by A in History and Geography?
(1) 32: 23 (2) 33: 19 (1) 31: 60 (2) 33: 61
(3) 32: 21 (4) 33: 20 (3) 33: 64 (4) 64: 33
(5) 32: 19 (5) None of these
9. Marks obtained by C in English is what percent of 14. If university A students in year 2015 decreases by
average obtained marks of all three in this subject? 20% as compared to 2004, Then what was the total
(a) 141.8% (2) 121.8% number of students in university A in 2004?
(3) 112.7% (4) 114.8% (1) 3875 (2) 2650
(5) None of these (3) 3700 (4) 3750
(5) 3675
10. If full marks in Geography is 200 then what is
15. Find the ratio of students in university A in the
average obtained marks of all three in this subject?
year 2016 and 2017 to the students in university B
(1) 142.7 (2) 145.7 in the year 2018 and 2019 together.
(3) 129.7 (4) 138.7 (1) 12: 11 (2) 8: 7
(5) None of these (3) 1: 6 (4) 3: 2
(5) 11: 12
Direction (11-15) : Given below are the number
of students (in hundreds) in two universities A and Direction (16-20) : Study the following graph and
B in different years. Answer the following answer the given questions:
questions based on given bar graph. The bar graph shows quantity of Item A and Item
B produced by a country A over the given years.

11. What is the average number of students of 16. What is the average quantity of Item A produced
university A in the year 2016, 2017 and 2019 and by country A in the years 2010, 2012, 2014 and
number of students of university B in the year 2015 (in thousand tons)?
2015 and 2018 together? (1) 3780 (2) 3660
(1) 6600 (2) 6000 (3) 3870 (4) 3320
(3) 6200 (4) 4500 (5) 3540
(5) 4250
17. The quantity of Item B produced by country A in
the year 2008 was 20% less than that in the year
12. If in year 2020, number of students of university
2010. If the Item B produced by country A in the
A decreases by 25% and that of university B
year 2009 is 30% more than that in the year 2008.
increases by 10% as compared to year 2019. Find What is the respective ratio between the quantity
total number of students of university A and B of Item B produced in the year 2009 and that in the
together in 2020 year 2015?
(1) 4500 (2) 10400 (a) 26 : 27 (2) 26 : 35
(3) 6500 (4) 4800 (3) 13 : 15 (4) 13 : 16
(5) 5000 (5) None of these

13. Total number of students of university B in year 18. By what percent the quantity of Item A produced
2018 is approximately what percent more than that by country A in the year 2015 is less than that in
of total number of students of university Aof year the year 2013?
2015? (1) 8 (2)
(1) 164% (2) 162% (3) % (4) 8
(3) 170% (4) 172%
(5) 167% (5) %
19. What is the difference between (In thousand tons) 23. What is the average number of students from the
the total quantity of Item B and Item A produced faculty of Arts from all the colleges together?
by country A in the year 2012 and the total (1) 5500 (2) 4500
(3) 4000 (4) 6500
quantity of Item B and Item A produced by
(5) None of these
country A in the year 2014?
(1) 890 (2) 940 24. The students from the faculty of commerce
(3) 660 (4) 825 studying in college Q is approximately what
(5) 980 percent of the total number of students in the same
college?
20. The quantity of excess Item A produced by (1) 30% (2) 48%
(3) 53% (4) 35%
country A in the year 2014 over the previous
(5) 43%
year is what percent more than the quantity of
excess Item B produced by country A in the year 25. What is the difference between the total number of
2012 over the previous year? students from the faculty of Commerce from all
(1) 144% (2) 156% the colleges together and the total number of
(3) 88% (4) 182% students from the faculty of Science from all the
(5) None of these colleges together?
(1) 5000 (2) 6000
(3) 6500 (4) 7000
Direction (21-25) : Study the following bar (5) None of these
carefully to answer these questions.
Number of students (In hundred) from various Direction (26-30) : Answer the questions based on
faculties studying in various colleges. the information given below.
There are different number of professors in three
departments i.e. Maths, English and Science of
five different Universities. The bar graph given
below shows the percentage of professors in Maths
department, percentage of professors in English
department and number of professors in Science
department, out of the total number of professors
of the respective university. Total number of
professors in Science department of all the given
universities together is 4712.

21. What is the ratio of the number of students from


the faculty of Arts studying in college R to the
number of students from the faculty of Arts
studying in college S?
(1) 9:16 (2) 16: 13
(3) 10: 9 (4) 11: 16
(5) None of these
26 What is the average number of professors in
22. What is the average number of students studying science department of universities A, B, D and E
in college P from all the faculties together? together?
(1) 6000 (2) 5500 (1) 962 (2) 924
(3) 4550 (4) 5000 (3) 741 (4) 841
(5) None of these (5) None of these
What is the difference between number of
professors in Maths department of university A
27 Total number of professors in university E is how 29 and E together, and the number of professors in
much percent more/less than total number of English department of university of A and B
professors in university A? together?
(1) 45.8% (2) 41.5%
(3) 46.67% (4) 43.3%
(5) 54.8% 1. prime number 2. multiple of 23
3. multiple of 3 4. odd number
(1) both 2 and 3 (2) only 1
28 What is the ratio of total number of professors in
(3) both 2 and 4 (4) all 2,3 and 4
Maths department of university C and D together, (5) None of these
to number of professors in English department of
university C and D together? 30 What is the total number of professors in English
(1) 34:51 (2) 51:34 department of university C, D and E together?
(3) 53:32 (4) 54:31 (1) 2846 (2) 2026
(5) None of these (3) 2244 (4) 2124
(5) None of these
Answer Key
1. (4) 7. (2) 13. (5) 19. (4) 25. (3)
2. (5) 8. (3) 14. (4) 20. (2) 26. (1)
3. (1) 9. (4) 15. (5) 21. (2) 27. (1)
4. (3) 10. (5) 16. (3) 22. (1) 28. (4)
5. (5) 11. (3) 17. (4) 23. (5) 29. (1)
6. (1) 12. (2) 18. (3) 24. (5) 30. (4)
Hints & Solutions
1. Solution (4) 10. Solution (5)
Required average Required average
= 1200 + 1000 + 1600= 3800 =

2. Solution (5)
Required percentage
11. Solution (3)

3. Solution (1)
Required number
= (4800 + 6000) – (3600 + 3000)
= 4200
12. Solution (2)
4. Solution (3) Total number of students in school A and B
Required ratio together in 2010

= 32: 21 = 6000 + 4400


= 10400
5. Solution (5)
Required % 13. Solution (5)

6. Solution (1)
Sanskrit = 170
Sanskrit =
14. Solution (4)
7. Solution (2) Let no. of students in year 2004 of school A is
Required ratio x
= 200 ATQ,
= 4 X 215 : 3 X 235 = 172 : 141

8. Solution (3)
Required ratio
=
15. Solution (5)
9. Solution (4)
Required % =

= = 114.8%
22. Solution (1)
16. Solution (3) ⇒Required Average
3 Required averages = ⇒( ) hundred
= 3870 thousand tons. ⇒6000

17. Solution (4) 23. Solution (5)


The quantity of Item B produced by country A ⇒Required average number of Arts students:
in the year 2008 = ( )
⇒( ) hundred
thousand tons.
⇒6250
The quantity of Item B produced by country A
in the year 2009 = ( ) 24. Solution (5)
thousand tons. ⇒Total number of students in college Q:
∴Required ratio = 3380: 4160 = 13: 16. ⇒ (80 + 60 + 45) hundred = 185 hundred
⇒Number of commerce students in college Q:
18. Solution (3) ⇒80 hundred
Required percentage ⇒Required percentage

= = 43%

19. Solution (4) 25. Solution (3)


Required difference ⇒Total number of Commerce students
= (30 + 80 + 50 + 20) hundred
= (4570 + 2715) - (3620 + 2840)
= 180 hundred
= 825 thousand tons.
⇒ Total number of Science students
= (90 + 60 + 25 + 70) hundred
20. Solution (2)
= 245 hundred
Quantity of excess Item B produced in the year
⇒Required difference
2012 with respect to previous year
= (245 – 180) hundred
= 2840 - 2715
= 6500
= 125 thousand tons.
Quantity of excess Item A produced in the year Solution (26-30) :
2014 with respect to previous year According to question:
= 4570 - 4250 3x – 54 + 4x + 206 + 3.2x + 64 + 3.6x – 144 +
= 320 thousand tons. 4.8x – 10 = 4712
∴ Hence, the required percent 18.6x + 62 = 4712
= 18.6x = 4650
x = 250
So, number of professors in science department
of university:
21. Solution (2) A = 3 × 250 – 54 = 696
⇒Number of students from the faculty of Arts: B = 4 × 250 + 206 = 1206
⇒College R = 8000 C = 3.2 × 250 + 64 = 864
⇒College S = 6500 D = 3.6 × 250 – 144 = 756
⇒Required ratio = 8000: 6500 = 16: 13 E = 4.8 × 250 – 10 = 1190
27. Solution (1)
Universities

of professors professors in professors in Desired percentage


Math English
= [(3500 – 2400)/2400] × 100
department department
= 45.8%
A 696/0.26=2400 0.325 2400 0.385
=780 28. Solution (4)
B 1206/0.402=30 0.35 3000 0.248 Desired Ratio
00 =1050
= (910 +1134) :(486 + 630)
C 864/0.40=2160 0.375 2160 0.225
=810 = 54:31
D 756/0.30=2520 0.45 0.25
29. Solution (1)
E 1190/0.34=350 0.372 0.288 Desired difference
0
= 780 + 1302 – 924 – 744 = 414

26. Solution (1) 30. Solution (4)


Desired Average Desired answer
= (696 + 1206 + 756 + 1190)/4 = 962 = 486 + 630 + 1008 = 2124
Q.1-5. The following line graph shows the production (in 5 What is the difference between total production of
lakhs) of shoes of two companies CAMPUS and shoes in two companies over all the years? (In
PUMA in five different years. lakhs)
Study the graph and answer the following questions. (1) 0 (2) 2
(3) 4 (4) 6
(5) 1

Q. 6-10.Study the following information carefully to


answer the questions given below it:
Number of persons in three different Villages over
the years.

1. Find the average number of CAMPUS shoes over


all the years:
(1) 50 lakhs (2) 40 lakhs
(3) 30 lakhs (4) 54 lakhs
(5) None of these
6. What was the average number of persons in all the
2. In which year production of CAMPUS shoes Villages together in the year 2012?
minimum than the shoes of PUMA? (1) 1800 (2) 1600
(1) 2007 (2) 2008 (3) 2600 (4) 1300
(3) 2009 (4) 2005 (5) 1500
(5) 2006
7. Total number of persons in village Y and village Z
3. By how percent production of CAMPUS shoes is together in the year 2010 was what percentage of
more than the production of PUMA shoes in year the total number of persons in village Y and village
2009? Z together in the year 2013?
(1) 65% (2) 80% (1) 97.5% (2) 86%
(3) 75% (4) 70% (3) 87.5% (4) 92.5%
(5) None of these (5) 98%

4. What is the ratio of total production of PUMA


8. How many times the total number of persons in all
shoes in years 2005, 2006 and 2008 together to the
the three villages X, Y and Z together was exactly
total production of CAMPUS shoes in the same
equal among the given years?
years together?
(1) Two (2) Three
(1) 23 : 22 (2) 25 : 24
(3) 23 : 24 (4) 24 : 23 (3) Four (4) One
(5) None of these (5) None
9. What was the average number of persons in village 14. Number of employees working in D and C together
X over all the years together? in Year ‘2012’is approximately what per cent of the
(1) 2100 (2) 1800 total number of employees working in these two
(3) 1300 (4) 2400 companies together in all three years?
(5) 1900 (1) 36% (2) 24%
(3) 34% (4) 29%
10. What was the difference between the total number (5) 26%
of persons in all the villages together in year 2009
and number of persons in village Y in the year 15. What is the respective ratio between number of
2011? employees working in A and E together in year
(1) 4500 (2) 4200 ‘2013’ and the number of employees working in E
(3) 3300 (4) 4800 in year ‘2014’?
(5) 4100 (1) 7: 5 (2) 12: 7
(3) 11: 8 (4) 11: 7
Q.11-15.Study the following graph carefully to (5) 11: 6
answer the given questions.
Number of employees working in different 16-20. Line graph given below shows percentage of
companies in three different years. labour (male labour and female labour) out of total
labour working in a firm ‘A’ in five different years.
Read the data carefully answer the questions.
Note – Total labour = (male labour + female labour
+ child labour)

11. What is the ratio between total number of


employees working in A in all three years together
and the total number of employees working in D in
all three years taken together?
(1) 13: 22 (2) 13: 23
Note: Male and female include only adult male
(3) 5: 9 (4) 13: 24
and adult female.
(5) 12: 23
16. If ratio of girls’ child labour to boy’s child labour
12. Number of employees working in B from year 2012
working in the year 2017 is 3 : 4 and their difference
is what percent of the total number of employees
is 96, then the difference between adult male labour
working in all six companies together in the same
and adult female labour working in that year is X?
year?
(1) X >= 200 (2) 190> X > 168
(1) 33.66% (2) 24%
(3) X >= 195 (4) 200 > X > 196
(3) 21(2/3)% (4) 33.33%
(5) None of these
(5) 25%
17. Total labour working in 2019 are 20% more than
13. What is the difference between total number of
that of total labour working in 2019, then find total
employees working in all six companies together
adult female labour working in 2019 is what percent
from years ‘2012’ and ‘2014’?
more than total child labours working in 2019?
(1) 110 (2) 80
(1) 42% (2) 48%
(3) 120 (4) 105
(3) 40% (4) 36%
(5) 90
(5) 44%
18. If ratio of adult male labours working in 2015 to together to the total number of girls who play Chess
2016 is 5: 4 and total labours working in these two and Hockey together.
years is 2200, then find total child labours working (1) None of the given options
in these two years? (2) 12: 53
(1) 420 (2) 480 (3) 53: 12
(3) 440 (4) 400 (4) 23: 8
(5) 520 (5) 51: 14

19. A child NGO inspection team in 2018 in the city 22. Total number of boys who play Cricket and Hockey
inspected firm X and imposed fine on firm of Rs. together is how much less than total no. of students
25 for each child labour. If inspection team imposed (boy + girl) who play Hockey and Football
total Rs. 2000 on the firm and total child labour together?
working in 2019 are 220 more than that of in 2018, (1) 90
then find ratio of adult male labours working in (2) 380
2018 and 2019 respectively? (3) 180
(1) 3: 7 (2) 4: 9 (4) None of the given options
(3) 4: 7 (4) 4: 5 (5) 290
(5) 4: 3
23. Total number of boys who play Chess and Football
20. If ratio of total labours working in 2015, 2016 and together is what percent of the total girl who play
2017 is 8:10: 5 and total adult female workers same games?
working in these three years is 720, then find total 3
(1) 169 %
number of child labours working in these three 13
years? (2) None of the given options
(1) 550 (2) 450 2550
(3) 400 (4) 300 (3) %
13
(5) 500 (4) 168%
(5) 196%
Q.21-25. Line graph given below shows total
students (boy + girls) who play four different games 24. Out of total boys and girl who play Hockey, 50%
and number of boys who play these games 4
and 28 7 % respectively were awarded for their
respectively. Study the data carefully and answer the
following questions. performances. Find total students (boys + girl) who
Note: One boy plays only one game not more than were not awarded for their performances for the
one. same game is what percent of total students (boy +
girl) who play this game?
(1) 40% (2) 49.75%
(3) 69.375% (4) 59.375%
(5) 79.5%

25. Find the average number of girls who play all the
games together.
(1) 160 (2) 150
(3) 130 (4) 170
(5) 180

21. Find the ratio between total number of students


(boy + girl) who play Cricket, Chess and Football
26-30. Line graph shows the quantity of 6 different
electronic items purchased by a person from 28. If price of a Desktop and a TV is Rs. 12600 and Rs.
flipkart. 8400 respectively then find the ratio of total price
of Desktop to total price of TV?
(1) 13:25 (2) 1:2
(3) 3:5 (4) 18:25
(5) 12:13
29. Total number of Tablet and Smart watch purchased
is what percent of total number of Mobile and TV
purchased?
1 1
(1) 87 % (2) 83 %
3 3
26. If sum of price of a Smart watch and a tablet is (3) 74% (4) 92%
Rs.16800 and the ratio of a Jeans to a Smart watch 7
is 11 : 10. Then, find the difference of total price of (5) 63 %
11
Smart watch and Jeans?
(1) multiple of 7 (2) multiple of 3 30. f price of a Tablet, Mobile and TV is Rs. 11200, Rs.
(3) even number (4) both a and b 6400 and Rs. 8000 respectively then find the sum
(5) both a and c of difference of total price of TV and that of Mobile
and difference of total price of Mobile and that of
27. If total price of Pen drive is Rs. 180000 and that of Tablet?
mobile is Rs. 300000. Then price of a Pen drive is (1) Rs. 40000 (2) Rs. 36000
‘X’ percent more or less than price of a Mobile? (3) Rs. 24000 (4) Rs.16000
(1) X > - 2 (2) 0 < X < 6 (5) Rs. 32000
(3) X < 2 (4) 5 > X > 10
(5) None of these
Answer Key
1. (2) 7. (3) 13. (4) 19. (3) 25. (2)
2. (1) 8. (2) 14. (4) 20. (5) 26. (5)
3. (3) 9. (2) 15. (1) 21. (3) 27. (1)
4. (4) 10. (2) 16. (5) 22. (2) 28. (4)
5. (1) 11. (3) 17. (5) 23. (1) 29. (5)
6. (3) 12. (5) 18. (3) 24. (4) 30. (5)
Hints & Solutions
1. Solution (2) = 2300 + 1500 + 2500 = 6300
Required average Total persons in 2010
1 = 2100 + 1800 + 2400 = 6300
= × (30 + 40 + 50 + 45 + 35)
5 Total persons in 2013
= 40 lakhs = 1500 + 3000 + 1800 = 6300

2. Solution (1) 9. Solution (2)


From graph it is clear that the require year is 2007. Total persons in Village X in 2008 = 1100
Total persons in Village X in 2009 = 2500
3. Solution (3) Total persons in Village X in 2010 = 2100
35 − 20 Total persons in Village X in 2011 = 1600
Required percentage = × 100
20 Total persons in Village X in 2012 = 2000
= 75% Total persons in Village X in 2013 = 1500
Required average
4. Solution (4) = (1100 + 2500 + 2100 + 1600 + 2000 + 1500)/6 =
Required ratio 10800/6 = 1800
= (35 + 45 + 40): (30 + 40 + 45)
= 120: 115 = 24: 23 10. Solution (2)
Total persons in Village X in 2009 = 2500
5. Solution (1) Total persons in Village Y in 2009 = 1500
Required difference Total persons in Village Z in 2009 = 2300
= (35 + 45 + 60 + 40 + 20) – (30 + 40 + 50 + 45 + Total number of persons in village Y in the year
35) = 0 2011 = 2100
Required difference
6. Solution (3) = (2500 + 1500 + 2300) – 2100 = 4200
Total persons in Village X in 2012 = 2000
Total persons in Village Y in 2012 = 3200 11. Solution (3)
Total persons in Village Z in 2012 = 2600 120+125+130 375
= 200+225+250 = 675
=5∶9
Required average = 7800/3 = 2600

7. Solution (3) 12. Solution (5)


Total number of persons in village Y in 2010 Required percentage
225
= 1800 =
120+225+130+200+125+100
× 100
Total number of persons in village Z in 2010 = 25%
= 2400
Total number of persons in village Y in 2013 13. Solution (4)
= 3000 Required difference = (125 + 150 + 175 + 225 +
Total number of persons in village Z in 2013 200 + 130) - (120 + 225+ 130 + 200 + 125 + 100)
= 1800 = 105
Required percentage= (1800 + 2400)/(3000 + 1800)
× 100 = 87.5% 14. Solution (4)
Required percentage
8. Solution (2) 200 + 130
= 455 + 675 × 100 ≈ 29%
Total persons in 2009
19. Solution (3)
Total child labour working in 2018
15. Solution (1) 2000
130+150
= = 80
Required ratio = = 7: 5 25
200 Total male labours working in 2018
60
16. Solution (5) = 80 × = 480
10
Let total labour working in 2017 = 100x Total child labour working in 2019
Total child labour working in 2017 = 28x = 80 + 220 = 300
ATQ- Total male labours working in 2019
4 3 56
28x × (7 − 7) = 96 = 300× 20 = 840
4x = 96 480
Required ratio = 840 = 4 ∶ 7
x = 24
Required difference
40 32 20. Solution (5)
= 2400 × (100 − 100) Let total labours working in 2015, 2016 and 2017 be
= 192 80a, 1000a and 50a respectively
Total female labours working in 2015
17. Solution (5) 30
= 80𝑎 × = 24𝑎
Let total labours working in 2019 = 500y 100
So, total labours working in 2019 = 600y Total female labours working in 2016
32
Total female labour working in 2019 = 100a × 100 = 32𝑎
24
= 600y × 100 = 144𝑦 Total female labours working in 2017
32
Total child labours working in 2019 = 50a× = 16𝑎
20 100
= 500y × 100
= 100𝑦 ATQ
Required percentage 24a + 32a + 16a = 720
144𝑦 − 100𝑦 72a = 720
= × 100 = 44%
100𝑦 a = 10
Total number of child labours working in 2015, 2016
18. Solution (3) and 2017
Let total labours working in 2015 = x 20 20 28
= 800 × + 1000 × + 500 ×
And, total labours working in 2016 = y 100 100 100
Total male labours working in 2015 = 0.5x = 160 + 200 + 140 = 500
And, Total male labours working in 2016 = 0.48y
ATQ- 21. Solution (3)
0.5𝑥 5 Required ratio
= (360+300+400) 1060
0.48𝑦 4 = = = 53: 12
(100+140) 240
x:y=6:5
5𝑥
xo. Y = 22. Solution (2)
6
5𝑥 Required difference
ATQ, 𝑥+ 6
= 2200
= (320 + 400) – (160 + 180) = 380
x = 1200 y = 1000
Total child labours working in 2019 and 2016
23. Solution (1)
together (200+240)
20 20 Required percentage = (100+160) × 100
= 1200 × + 1000 × = 440 4400 3
100 100 = % = 169 13 %
26
= 160000 – 132000
24. Solution (4) = 28000
Total boys who were not awarded for Hockey
50 27. Solution (1)
= 180 - 100 × 180= 90
180000
Total girl who were not awarded for Hockey Price of a Pen drive= 18
= 1000
200 300000
= 140 - 700 × 140= 100 Price of a Mobile = 30 = 1000
∴ Required percentage 1000−1000
Required % = 1000 × 100 = 0%
190
= 320 × 100 = 59.375%
28. Solution (4)
12600×12 18
25. Solution (2) Required ratio = = 25
8400×25
Required average
1
= 4 (200 + 100 + 140 + 160) 29. Solution (5)
= 600/4 = 150 15 +20 7
Required%= 30+25
× 100 = 63 11 %

26. Solution (5)


11
30. Solution (5)
Price of a Tablet = 16800 × 21 = 8800 Required sum
10
Price of a smart watch = 16800 × 21 = 8000 = (8000 × 25) – (11200 × 15)
= 2,00,000 – 168000
Required difference
= 32000
= (20 × 8000 – 15 × 8800)
Direction (1-5) : Given pie-chart shows the Q5. Find the difference between runs scored by D and
percentage distribution of runs scored by six F together to the runs scored by B and A together
Indian batsmen against Pakistan in a test match if India scored 1200 runs.
and the total runs scored by India is the total runs (1) 15 (2) 3
scored by only these six Indian batsmen. (3) 9 (4) 6
(5) 12

Direction (6-10) : The following pie-chart shows


the angular distribution (in degrees) of tables
manufactured in six different companies. Study the
graph carefully to answer the following question.

Q1. If Pakistan scored 744 runs and lost by 56 runs


then find the runs scored by C is how much less
than the runs scored by E.
(a) 40 (2) 20
(3) 44 (4) 52
(5) 46
Q6. Find the total number of tables manufactured by
Q2. Find the ratio between runs scored by B to that of companies M, N and P together.
D. (1) 13600 (2) 12580
(1) 16 : 13 (2) 15 : 16 (3) 14200 (4) 13400
(3) 16 : 17 (4) 16 : 15 (5) 14400
(5) 14 : 13
Q7. What is the ratio of the number of tables
Q3. If India makes 650 runs and loss by 26 runs then manufactured by O and R together to that of N and
runs scored by A is what percent of the runs scored Q together?
by Pakistan? (1) 7: 11 (2) 11: 7
(1) (2) (3) 5: 11 (4) 11: 5
(5) 11: 12
(3) (4)
(5) Q8. If 33 1 % tables of M and 16 2 % tables of P
3 3
are defective, then find the good tables
Q4. If India wins by same runs which are scored by E manufactured by these two companies.
then find minimum runs scored by Pakistan. (1) 13,000 (2) 10,000
(1) 166 (2) 83 (3) 7000 (4) 11000
(3) 243 (4) CND (5) 22,400
(5) None of these
Q9. What is the difference between tables produced by Q13. If in year 2020 expenditure on Cricket and
Q and R? basketball increases by 20% and 12% than the
(1) 2100 (2) 1800 previous year respectively and the ratio of
(3) 2400 (4) 1600 expenditure between these two sports in 2019 is 2 :
(5) 2600 1 (cricket : basketball) then find the total
expenditure of these two sports in 2020. (Assume
Q10. Another company X manufactures tables 20% less total expenses remained constant in both year)
than that by M and defective tables manufactured (1) 480 L (2) 660 L
by company X are 12 ½% of its total manufactured (3) 550 L (4) 440 L
tables. What is the total number of good tables (5) 470 L
manufactured by X ?
(1) 1800 (2) 1200 Q14. If expenditure on football and Hockey increases
(3) 4200 (4) 16,00 20% and 25% in 2020 than that in 2019
(5) 1600 respectively then what is the total expenditure for
Direction (11-15) : Given below is the pie chart these two sports in 2020? (Assume total expenses
which shows the percentage expenditure issued by remained constant in both year)
the government on different sports in a state in the (1) 620 .65 L (2) 570.50
year 2019. (3) 551.25 (4) 590.00
Total Expenses = 1500 Lakhs (5) 601.5

Q15. Total expenditure of Tennis and football together


in 2019 is what percent of total expenditure on all
sports in 2020 if in 2020 expenditure on all sports
increases by 20% than that in 2019?

(1) (2)
(3) (4)
(5)

Q11. What is the ratio of expenditure on Football and Direction (16-20) : The following pie chart shows
Golf together to the expenditure on Hockey and the angular distribution of weapons bought by
Tennis together? India from Russia. Study the graph carefully to
(1) 11 : 10 (2) 9 : 10 answer the following questions
(3) 10 : 11 (4) 11 : 12 Total weapons bought by India from Russia =
(5) 5 : 6 72000

Q12. What is the difference between average


expenditure on sports Golf, Football together to
the average of expenditure on sport tennis and
Hockey together?
(1) 14.25L (2) 18.75L
(3) 16.25L (4) 25.75L
(5) 27.25L
Q16. Total number of A and C together bought by India Note : - Vacancies in officer Scale I and officer
from Russia is what percent of total number of D Scale II are same in C city
and E together?
(1)110% (2)120% Q21. Vacancies in Scale I in A is 12,000 then find the
(3)140% (4)130% difference between vacancies in Scale I and Scale
(5)125% II in D.
(1) 4,800 (2) 5,400
(3) 5,800 (4) 6,200
Q17. What is the average number of B, C and E together (5) 6,600
bought by India from Russia?
(a)5400 (b)5600 Q22. Find the ratio between Total vacancies in E to
(c)7800 (d)4800 total vacancies in C.
(e)6400 (1) 9 : 8 (2) 23 : 20
(3) 69 : 80 (4) 15 : 16
(5) CND
Q18. What is the ratio of the number of B and D
together to the total number of A and C? Q23. Vacancies in Scale I in B are further divided into
(1) 3: 4 (2) 3: 2 three parts i.e. A, B and C in the ratio 2 : 3 : 4. If
(2) 2: 3 (4) 4: 3 vacancies in ‘C’ is 6,000 then find total vacancies
(5) 4: 5 in B in scale I and scale II together.
(1) 19,700 (2) 20,200
Q19. If the total cost of E and D to India was Rs. 360 (3) 20,700 (4) 21,200
(5) 21,700
crores, then what was the price of one D. It is
given that the ratio of price of one E to one D is 3 : Q24. Vacancies in scale I in A are what percent
1. more/less than vacancies in scale II in D?
(1) 1.33 lacs (2) 0.66 lacs (1) CND (2) 100%
(3) 2.4 lacs (4) 1 lac (3) 50% (4) 250%
(5) 2.2 lacs (5) 150%

Q25. Find the average vacancies in scale I in A, C and


Q20. What is the difference between the total number of E all together if average vacancies in scale II in B
A and D weapons together and total number of rest and C is 11,200
weapons together? (1) 15,000 (2) 12,500
(1) 7200 (2) 4800 (3) 17,500 (4) 20,000
(3) 6400 (4) 7600 (5) 22,500
(5) 5800
Direction (26-30) : Given below is a pie-graph
that shows (percentage +degree) of HR managers
Direction (21-25) : Pie chart given below shows out of total HR managers in five different
vacancies in officer scale I and in officer scale II in companies i.e. A, B, C, D and E in March, 2016.
a bank in five different cities. Study the chart
carefully and answer the questions.
Note: Any HR manager leaves or enters a Q.28 The ratio of male to female HR managers in
company only in April 2017. company C in 2016 was 5 : 4 while in 2017 the
ratio was 11 : 9. If 13 female HR managers joined
Q.26 The ratio of male HR managers to female HR company C in 2017, then find the total number of
managers in company D in 2016 was 3 : 2 and 24 HR managers in company C in 2017.
female HR managers left company D in 2017. (1) 200 (2) 150
Find the percentage increase in the number of (3) 100 (4) 250
female HR managers in company D in 2017 if (5) None of these
there were 42 male managers out of newly
recruited 72 managers. Q.29 In 2017, 25% of HR managers from company D
(1) 8 left the job and company B recruited 40% more
HR managers more than the number of HR
(2) 8%
managers it had in 2016. If the ratio of male to
(3) 9%
female HR managers in company D and B
(4) 6 becomes 8 : 7 and 7 : 5 respectively, then find the
(5) None of these ratio of number of male HR managers in company
D in 2017 to number of female HR managers in
Q.27 In 2017 33 1 % HR managers from company A company B in 2017.
3
(1) 71:32 (2) 45:34
shifted to company B. If ratio of male to female
(3) 65:36 (4) 72:35
HR managers in company A and B in 2016 was 5 :
(5) None of these
7 and 3 : 1 respectively and 25 female manager left
company A, then find remaining male managers in
Q.30 If in 2017 company A and C fired 20% and 25%
company A is what percent of number of male
of their HR managers respectively, then find the
managers in company B in 2017.
remaining number of HR managers in these two
(1) companies in 2017 is what percent of number of
(2) total HR managers in company E in 2016.
(approximate value)
(3)
(a) 100% (2) 200%
(4) 38 (3) 300% (4) 250%
(5) None of these (5) None of these
Answer Key
1. (1) 7. (1) 13. (2) 19. (2) 25. (3)
2. (4) 8. (3) 14. (3) 20. (1) 26. (1)
3. (5) 9. (4) 15. (1) 21. (4) 27. (2)
4. (2) 10. (3) 16. (2) 22. (1) 28. (3)
5. (5) 11. (1) 17. (1) 23. (3) 29. (4)
6. (1) 12. (2) 18. (3) 24. (5) 30. (3)
Hints & Solutions
1. Solution (1) 8. Solution (3)
Pakistan’s score = 744
So, India’s score = 744 + 56 = 800
Required no. of good tables
( – )
Required difference = = 40 =
= 4000 + 3000 = 7000
2. Solution (4)
Let total runs scored by India = 100x 9. Solution (4)
Runs scored by B = = 16x ( – )
Required difference =
Runs scored by D = = 15x = 1600
Required Ratio = 16 : 15
10. Solution (3)
3. Solution (5) 12
Pakistan’s score = 650 + 26 = 676 ∴ Required answer
A’s score = = 130 =( – ) = 4200
Required % =
11. Solution (1)

4. Solution (2)
If India’s score = 100
So E scored = 12. Solution (2)
Average expenditure on football and Golf
For minimum runs = should be 1
together
So Pakistan’s score = 100 – 17 = 83

5. Solution (5)
Total score = 1200 Average of expenditure Tennis and Hockey
Runs scored by D and F together
( )
=
= 420 Required difference = 18.75 L
Runs scored by B and A together
( )
= 13. Solution (2)
= 432 Expenditure on Cricket in 2019
Required Difference = 432 – 420 = 12 ( )
{ }

6. Solution (1)
Required no. of tables
( )
= Expenditure on Basketball in 2019
= 204 × 200/3= 13600

7. Solution (1) Required expenditure in 2020


( )
Required ratio = ( )
= 84: 132 = 7: 11
14. Solution (3) 22. Solution (1)
Required expenditure Let total vacancies in Scale I = 2500a
And total vacancies in Scale II = 1600a
Required ratio

15. Solution (1)


Required percentage
( )
23. Solution (3)
Let total vacancies in Scale I = 2500a
And total vacancies in Scale II = 1600a
16. Solution (2) ATQ,
Total no. of A and Cs together
( )
= = 36000
Total no. of D and E together
( )
= = Total vacancies in B =
30,000
∴ Required percentage
= = 120%

17. Solution (1) 24. Solution (5)


Required average Let total vacancies in Scale I = 2500a
( ) And total vacancies in Scale II = 1600a
= = 10800
Vacancies in scale I in A
18. Solution (3) =
Required ratio = Vacancies in scale II in D

19. Solution (2) =


Let price of one D and one E be x and 3x Required % =
respectively.
∴ = 360 crores
54x =
x = 0.66 lacs
25. Solution (3)
20. Solution (1) Let total vacancies in Scale I = 2500a
Required difference
And total vacancies in Scale II = 1600a
(( )–( ))
= = 7200 ATQ,

[ ]
21. Solution (4)
Vacancies in Scale I in D
=
Vacancies in Scale II in D
[ ]
=
Required difference
=
26. Solution (1) 28. Solution (3)
360 = 10% and 900 = 25% No. of male HR managers in company C in
In 2016, 2016
No of HR managers in company D

No. of Female HR managers in company C in


No. of male HR managers in company D
2016
Total number of female HR managers in 2017
No. of female HR managers in company D in company C
Total no. of HR managers in company C in
Newly recruited HR female managers 2017
No. of female HR managers who left = 24 ( )
Total no. of female HR mangers in 2017

( )
Req. percentage% 29. Solution (4)
No of male HR managers in company D in
2017
( )
27. Solution (2)
No. of male HR managers in company A in
2016 No. of female HR managers in company B in
2017
No. of Male HR managers in company A in ( )
2017
Required ratio
( )
( )
30. Solution (3)
No. of male HR managers in company B ( )
Required%
( )

Required%
Q1-5. Study the following pie-chart and bar (1) 1820 (2) 1860
graph and answer the following questions. (3) 1680 (4) 1640
The given pie chart shows the percentage-wise (5) 1260
distribution of total students in six schools. Bar
graph shows no. of boy students in each school. 4. In which of the following schools is the total
number of students equal to the number of girls
in School E?
(1) A (2) B
(3) C (4) D
(5) F

5. The number of girls in School A is


approximately what percentage of the total
number of students in School B?
(1) 55 (2) 50
(3) 35 (4) 45
(5) 40

Q(6-11). The following first bar graph shows the


percentage of females out of the total customers
on each day and the second bar graph shows the
total number of customers visiting V-mart on six
different days of a week. Read the data carefully
1. What is the sum of the number of girls in School and answer the following questions.
C, in School E, and the number of boys in
School D together?
(1) 2700 (2) 1900
(3) 3600 (4) 2800
(5) 2300

2. What is the ratio of the number of boys in


School C and the number of girls in School B
together to the total number of students in
School E?
(1) 45: 97 (2) 43: 95
(3) 52 : 87 (4) 65 : 87
(5) 73 : 43

3. What is the difference between the total number


of students in School F and the number of boys
in School E?
Read the data carefully and answer the
6. The number of male customers on Thursday is questions.
what percent of the number of female customers
on Tuesday?
1 4
(1) 148 % (2) 148 %
9 27
4 1
(3) 146 % (4) 146 %
27 9
2
(5) 147 %
27

7. What is average of the number of male


customers on Monday, Wednesday, Friday and Days Number of females
Saturday? visited restaurant
(1) 3340 (2) 3660 Monday 104
(3) 3480 (4) 3460 Tuesday 132
(5) 3690
Wednesday 128
8. Total number of female customers on Friday and Friday 92
Saturday is approximately what percent more or
less than the total number of male customers on
Monday and Tuesday? 11. Find the ratio of the total male who visited
(1) 11% (2) 7% restaurant on Tuesday and Wednesday together
(3) 6% (4) 10% and total female visited restaurant on
(5) 8% Wednesday and Friday together?
(1) 16: 11 (2) 14 : 11
9. Find the difference between the average number (3) 18 : 11 (4) 12 : 11
of male customers on Tuesday and Friday and (5) 11 : 16
the average number of female customers on
Saturday and Wednesday. 12. Total male visited on Sunday in restaurant are
(1) 20 (2) 15 44 more than that of on Monday and total
(3) 10 (4) 17 number of males visited on Sunday in restaurant
2
(5) 18 are 66 % of total people visited restaurant on
3
that day. Find total male visited restaurant on
10. Find the ratio of the total number of customers Friday is what percent less than total people
on Monday and Saturday together to the total visited restaurant on Sunday?
number of male customers on Friday and (1) 40% (2) 30%
Wednesday together? (3) 20% (4) 25%
(1) 49 : 24 (2) 49 : 30 (5) 15%
(3) 49 : 32 (4) 49 : 34
(5) 49 : 36 13. Find difference between average number of
males visited restaurant on Monday and Friday
Q.11-15. Line graph given below shows number and average number of females visited
of people visited a restaurant ‘ABC’ on four restaurant on Tuesday and Wednesday?
different days and table shows number of females (1) 15 (2) 18
visited restaurant on these days. (3) 6 (4) 12
(5) 24
(3) 45% (4) 40%
14. Total males visited Wednesday is what percent (5) 42%
less than total male visited on Tuesday
(approximate)? 17. If total male unsubscribed viewers in D is 66
(1) 14.5% (2) 12.5% 2
3
% more than that of female unsubscribed
(3) 9.5% (4) 6.5%
viewers, then find ratio of total male
(5) 3.5%
unsubscribed viewers in D to total unsubscribed
viewers in A and C together?
15. Total people visited on Saturday are 9
1
(1) 5:12 (2) 25:54
11
% more than that of on Monday, then find (3) 54:7 (4) 25:36
total people visited on Tuesday is what percent (5) 54:25
more than that of on Saturday?
(1) 15% (2) 25% 18. Find the sum of total unsubscribed viewers in B
(3) 12.5% (4) 20% and C and total subscribed viewers in E
(5) 10% together?
(1) 2125 (2) 2475
Q.16-20. Given below the pie chart shows (3) 2655 (4) 2745
percentage distribution of viewers of a OTT (5) 2340
platform in five different villages (A, B, C, D and
E) and table shows number of viewers who 3
subscribed the platform. Read the data carefully 19. Out of total viewers in village C, 46( )% are
7
and answer the questions. 7
Note – Total viewers = Total subscriber + Total female and th of total female are
13
un-subscribers
unsubscribed viewers, then find the total
Total viewers = 9000 unsubscribed male viewers from village C?
(1) 370 (2) 480
20%
12% (3) 410 (4) 570
15%
(5) 550

25%
28% 20. Find the difference between the total number of
subscribed viewers from C and D together and
the total number of subscribed viewers from A
A B C D E and E together.
(1) 1180 (2) 1170
(3) 1070 (4) 1050
Villages People who subscribed
(5) 1150
A 660
B 750 Q. 21-25. Study the bar-graph and table given
C 1320 below carefully and answer the question
D 1050 accordingly. Bar-graph shows the number of
E 540 people in five different cities and table shows
the percentage of male in five cities and the ratio
of literate and illiterate people in five different
16. Total unsubscribed viewers from B and E
cities.
together is what percent more than total
unsubscribed viewers from C?
(1) 50% (2) 55%
(2) 7:5
(3) 5:7
(4) Cannot be determined
(5) None of these

Q.26-30. Study the charts given below carefully


and answer the following questions.
Pie chart shows the percentage distribution of
total shoes manufactured by 5 different shoe
manufacturers (P, Q, R, S and T) and table
Percentage Literate: shows the defective shoes manufactured by
City of Males Illiterate these 5 companies.
K 40% 1: 3
Total shoes manufactured = 10000
L 60% 7: 3
M 20% 1: 4
N 75% 3: 2
30% 25%
O 68% 9: 11

15%
21. If 40 percent of male from city K went to city L
then what is the total number of males in city L? 10% 20%
(1) 34400 (2) 38400
(3) 32400 (4) 36400 P Q R S T
(5) None of these

22. What is the percentage of literate people from


city L to the illiterate people from city M?
(1) 52.5% (2) 72.5% Defective Shoes
(3) 62.5% (4) 42.5%
500
(5) None of these
400
23. If 30 percent of male from city M are illiterate. Then 300
find the ratio of illiterate male to the illiterate female
from city M? 200
(1) 21 : 259 (2) 21 : 269 100
(3) 22 : 259 (4) 23 : 259
0
(5) None of these
P Q R S T

24. What is the average of illiterate people in five Note – Total shoes manufactured by any
cities? company = Total (defective + non-defective)
(1) 50000 (2) 40800 shoes manufactured by that company.
(3) 42000 (4) 46000
(5) None of these 26. non-defective shoes manufactured by P are what
percent more or less than total shoes
25. What is the ratio of total females from city M manufactured by R?
and O to the total illiterate males from city K (1) 24.5% (2) 12.5%
and city L? (3) 19.5% (4) 27.5%
(1) 2 : 5 (5) 32.5%
27. Non – defective shoes manufactured by T are 29. Find the central angle (in degrees) of total shoes
how much more than defective shoes manufactured by P and R together.
manufactured by P, Q and R together? (1) 144 (2) 150
(1) 1900 (2) 1800 (3) 180 (4) 162
(3) 1500 (4) 1700 (5) 200
(5) 1600
30. If T also manufactures shirts and ratio of shoes
28. If cost of manufacturing a shoe for S is Rs.40 and to shirts manufactured by T is 5 : 7, then find
S wants to earn 20% profit on the total cost of shirts manufactured by T are how much more
manufacturing and S does not sell defective than non-defective shoes manufactured by R?
shoes, then find at what price S should sell all the (1) 5200 (2) 3600
non defective shoes. (3) 4800 (4) 5400
(1) Rs.128 (2) Rs.156 (5) 5000
(3) Rs.145 (4) Rs.148
(5) Rs.154
Answer Key
1. (3) 7. (1) 13. (4) 19. (4) 25. (4)
2. (3) 8. (4) 14. (3) 20. (2) 26. (2)
3. (3) 9. (1) 15. (2) 21. (4) 27. (5)
4. (2) 10. (5) 16. (2) 22. (3) 28. (1)
5. (5) 11. (1) 17. (2) 23. (1) 29. (4)
6. (2) 12. (2) 18. (5) 24. (2) 30. (2)
Hints & Solutions
70 7200×55 55 5000×44
1. Solution (3) (4800×
100
+
100
)–(8000× +
100 100
)
Required No. of students (boys and girls) = 70 55 × 100
(4800× +7200× )
100 100
26 ≈ 10%
=( × 12000 − 1800) + 1200
100
29
+( × 12000 − 2400) 9. Solution (1)
100 1 60
= 7800 – 4200 = 3600 Required difference = 2
(9000 × 100 + 5000 ×
44 1 55 8000×45
100
) − 2 (7200 × 100 + 100 )
2. Solution (3)
9 = 3800–3780 = 20
1800+ ×12000−800
100
Required Ratio = 12000
29×
100 10. Solution (5)
= 52: 87 Required ratio
4800+5000
= 40×9000 45
3. Solution (3) (
100
+8000× )
100
Required difference 9800
= 7200= 49 : 36
= 2400 – 720 = 1680

11. Solution (1)


4. Solution (2)
Total male visited restaurant on Tuesday and
No. of girls in School E
Wednesday
= 3480 – 2400 = 1080
= (300 – 132) + (280 – 128)
Now
= 168 + 152 = 320
1080 = Total no. of students in school B
Total female visited restaurant on Wednesday and
Friday
5. Solution (5)
1440−1000 = 128 + 92 = 220
Required % = 1080
× 100 ≈ 40% Required ratio = 320 : 220 = 16 : 11

6. Solution (2) 12. Solution (b)


Required % Total male visited Sunday
75
6400×
100 = (220 – 104) + 44 = 160
= 45 × 100
7200× Total people visited restaurant on Sunday
100
4 3
= 148 % = 160 × 2 = 240
27
Total male visited on Friday
= (260 – 92) = 168
7. Solution (1) 240 −168
Required percentage = 240
× 100
Required average 72
1 70 40 45 = × 100 = 30%
= (4800 × + 9000 × + 8000 × + 240
4 100 100 100
56
5000 × 100) 13. Solution (4)
1
= 4 (3360 + 3600 + 3600 + 2800) Average number of males visited restaurant on
Monday and Friday
= 3340 (220−104)+(260−92)
= 2
116+168
8. Solution (4) = =142
2
Required percentage Average number of females visited restaurant on
Tuesday and Wednesday
132+128 Total unsubscribed viewers from B
= =130
2 15
Required difference = 142 – 130 = 12 = 9000 × 100 − 750 = 600
Total unsubscribed viewers from C
14. Solution (3) 28
= 9000 × 100 − 1320 = 1200
Total male visited on Tuesday
Total unsubscribed viewers in B and C and
= 300 – 132 = 168
subscribed viewers in E
Total male visited on Wednesday
= 600 + 1200 + 540
= 280 – 128 = 152
= 2340
Required percentage
168−152
= 168
× 100 ≈ 9.5% 19. Solution (4)
Total female viewers in village C
15. Solution (2) 28 325 1
= 9000 × 100 × 7
× 100 = 1170
1
Total people visited on Saturday = 220 + 220 × 11= Total female unsubscribed viewers in village C
240 7
= 1170 × = 630
300−240 13
Required percentage = 240
× 100 Total unsubscribed male viewers in village C
60 28
= 240
× 100 = 25% = (9000 × 100 − 1320)– 630 = 570

16. Solution (2) 20. Solution (2)


Total unsubscribed viewers from B Total subscribed viewers from village C and D
15 together = 1320+1050 = 2370
= 9000 × 100 − 750 = 600
Total subscribed viewers from village A and E
Total unsubscribed viewers from E
20 together = 660 + 540 = 1200
= 9000 × 100 − 540 = 1260 Required difference
Total unsubscribed viewers from C = 2370 – 1200 = 1170
28
= 9000 × 100 − 1320 = 1200
Required percentage 21. Solution (4)
(600+1260)−1200 Number of males in city K
= × 100 40
1200
660 = 100 × 40,000= 16000
= × 100 Number of males who left city K
1200
= 55% 40
= × 16000
100
17. Solution (2) = 6400
Let total female unsubscribed viewers in D be 3x Number of males in city L
60
So, total male unsubscribed viewers in D will be 5x = 100 × 50,000 = 30000
Total male unsubscribed viewers in D Total number of males in city L after Males who
25 5𝑥
= (9000 × 100
− 1050) × 8𝑥
= 750 joined city L
Total unsubscribed viewers in A and C = 30000 + 6400 = 36400
12 28
= (9000× 100 − 660) + (9000 × 100 − 1320)
22. Solution (3)
= 420 + 1200 = 1620 Literate people from city L
750
Required ratio = 1620 7
= × 50,000 = 35000
= 25 : 54 10
Illiterate people from city M
18. Solution (5) 8
= × 70,000 = 56000
10
∴ Percentage =
35000
× 100 = 62.5% 30
56000 = (10000 × ) − 450 = 2550
100
Defective shoes manufactured by P, Q and R
23. Solution (1)
together
30 percent of male from city M
= 250 + 300 +400 =950
20 30
= × 70,000 × Required difference = 2550 – 950 = 1600
100 100
= 4200
28. Solution (1)
∴ 4200 males from city M are illiterate
Total cost of manufacturing shoes for S
Female from city M who are illiterate 10
4 = (10000 × 100) × 40 = Rs. 40000
= × 70,000 − 4200
5 Total revenue that S wants
= 56000 − 4200 = 51800 120
4200
= 40000 × = Rs.48000
100
∴ Ratio = = 21 ∶ 259
51800 Non-defective shoes of S
10
= (40000 × 100) − 250 = 3750
24. Solution (2)
480000
Required average Required selling price = 3750
30000+15000+56000+48000+55000
= = 40800 = Rs.128
5

25. Solution (4) 29. Solution (4)


Since the illiterate males from city K and city L Required central angle
25+20
cannot be determined. =( 100
)× 360 = 162°

26. Solution (2) 30. Solution (2)


Non-defective shoes manufactured by P Shoes manufactured by T
25 30 7
= (10000 × ) − 250 = 2250 = (10000 × × )
100 100 5
Total shoes manufactured by R = 4200
20 Non-defective shoes manufactured by R
= (10000 × ) = 2000
100 20
2250−2000 = (10000 × ) − 400 = 600
Required % = 2000 × 100 = 12.5% 200
Required difference = 4200 – 600 = 3600
27. Solution (5)
Non-defective shoes manufactured by T
1-5. Study the following information carefully and (3) 13.34% (4) 17.43%
answer the questions given beside. (5) 11.43%
In a science class there are a total of 538 students
who take admission to learn Physics and Chemistry. 6-10. Answer the questions based on the information
Each student learns single science form only. Boys given below.
who learn Physics is 70% of girls who learn The given data is about number of animals (cows
Physics. Girls who learn Chemistry is 20 more than and horses) in four different farms (P, Q, R and S).
boys who learn Chemistry. Girls who learn Physics The number of cows in farm ‘P’ is 200. The
is same as Boys who learn Chemistry. number of horses in farm ‘R’ is 200 more than
number of cows in farm ‘P’. Total number of
1. What is the difference between number of boys animals in farm ‘R’ is 1000. Number of cows in
and girls who took admission to learn Physics? farm ‘S’ is 25% more than that in ‘R’. The ratio of
(1) 34 (2) 42 number of horses in farm ‘P’ and ‘R’ is 6:5,
(3) 56 (4) 66 respectively. Total number of cows in all the farms
(5) 28 together is 1800. Number of horses in farm ‘Q’ is
350 which is 150 less than that in farm ‘S’.
2. How many students took admission in Chemistry? 6. Find the ratio of the number of horses in farm ‘P’
(1) 240 (2) 320 to the total number of animals in farm ‘Q’.
(3) 300 (4) 480 (1) 2:5 (2) 3:7
(5) 170 (3) 4:5 (4) 1:2
(5) 5:8
3. The number of boys who took admission in
Physics is what percent of the total number of 7. Find the difference between the total number of
students who took admission in Chemistry? animals in farm ‘S’, and number of cows in farm
(1) 32.66% (2) 36.66% ‘Q’.
(3) 34.66% (4) 33.66% (1) 1000 (2) 1200
(5) 35.66% (3) 750 (4) 900
(5) 1060
4. If 100 new students took admission in Physics,
then how many new boys took admission in 8. The number of cows in farm ‘S’ is how much
Physics, so that the ratio of boys to girls in Physics percent more/less than total number of animals in
becomes 89 : 80? ‘R’?
(1) 89 (2) 70 (1) 20% (2) 25%
(3) 98 (4) 80 (3) 35% (4) 30%
(5) 90 (5) 18%

5. If the number of girls in the Chemistry is increased 9. Find the average of the number of horses in farm
by 10% then by what percent the number of girls ‘R’ and ‘S’ together.
in Physics should be decreased so that the total (1) 500 (2) 400
number of students i.e., 538 remains same? (3) 480 (4) 420
(1) 15.24% (2) 18.33% (5) 450
10. If 20% of horses in farm ‘Q’ do not give milk, then 15. 40% and 60% of the females in city P and city Y,
find the number of milk giving horses in farm ‘Q’. respectively are married. Find the number of
(1) 320 (2) 280 married females in city P and city Y together.
(3) 260 (4) 240 (1) 394 (2) 364
(5) None of these (3) 354 (4) 384
(5) None of these
11-15. Answer the questions based on the information
given below. 16-20. Read the following information carefully and
Number of females in city X is 20% more than answer the questions given below it.
number of males in city Y. Total population of city Five sports hockey, Cricket, Tennis, Badminton and
X is 1080. Ratio of number of males in city X and Baseball are included in a sports Competition. The
city Z is 4:5 respectively. Number of females in total number of players in this sports competition is
city Z is (7/6)th of number of males in city Y. 800. The ratio between the total woman and total
Number of males in city Z is 630. Number of man players is 1 : 3.
females in city X is 60% of total population of city 25% players are in cricket out of total players, 110
P and ratio of number of males to number of player play Badminton, 10% of total players play
females in city P is 9:7. Total population of city Y tennis. Hockey players are two times of Badminton
is 100% more than number of females in city P. players, while remaining players play Baseball.
30% of cricket players are woman.
11. What is the ratio of number of males to number of Half of woman cricketers are equal to woman
females in city X? badminton players. 10% of total Hockey players are
(1) 7:8 (2) 5:4 equal to woman tennis players. Hockey and
(3) 7:9 (4) 7:5 Baseball have equal woman players.
(5) None of these
16. What is the ratio between the woman hockey
12. What is the average of number of males in city P players and man badminton players?
and number of males in city Y? (1) 20 : 13 (2) 11 : 20
(1) 500 (2) 510 (3) 13 : 20 (4) 11 : 23
(3) 520 (4) 530 (5) None of these
(5) None of these 17. What is the total number of man players in hockey,
cricket and baseball?
13. Total population of city Y is how much percent (a) 464 (b) 454
more/less than the total population of city P? (c) 462 (d) 432
(1) 15% (2) 15.5% (e) None of these
(3) 12.5% (4) 25%
(5) None of these 18. Woman baseball players are what percent of man
hockey players?
14. Number of females in city K is 75% more than (1) 25% (2) 34%
number of females in city Z. Find total number of (3) 24% (4) 15%
females in city K. (5) None of these
(1) 920 (2) 980
(3) 960 (4) 1080 19. What is the difference between the man baseball
(5) None of these players and woman tennis players?
(1) 134 (2) 136
(3) 122 (4) 126
(5) None of these
20. In which sports, women are maximum, and men percent of the total number of non-veg burger sold
are minimum? by Y and Z together?
(1) Cricket and badminton (1) 112% (2) 115%
(2) Cricket and hockey (3) 108% (4) 104%
(3) Baseball and cricket (5) 118%
(4) Cricket and Tennis
(5) Tennis and Hockey 25. What is the ratio of the number of non-veg burgers
sold by X and Z together to the number of veg
21-25. There are three different burger shops – X, Y and Z burger sold by Y and Z together?
which sells Veg burger and non-veg burger. The (1) 101:123 (2) 103:124
ratio of the number of veg burgers and non-veg (3) 103:123 (4) 101:124
burgers sold by X is 9:7. The ratio of the number of (5) None of these
veg burger and non-veg burgers sold by Y is 3:4.
The total number of veg and non-veg burgers sold 26-30. Total income of five persons Aari, Bala, Chitra,
by Z is 324 and the ratio of the number of veg and Divya and Ezhil together is Rs.70000. Ratio of the
non-veg burgers sold by Z is 7:5. The total number total savings of all the persons together to the
of burgers (veg and non-veg) sold by all the shops savings of Chitra is 11:2. Expenditure of Bala is half
together is 1128. The number of veg burgers sold of the savings of Chitra. The expenditure of Bala is
by X is 20% more than the veg burgers sold by Y. 25% of the expenditure of Divya and the
expenditure of Ezhil is Rs.2000 more than the
21. What is the average number of veg burgers sold by expenditure of Chitra. Income of Bala is Rs.12000
all shops? which is Rs.5000 more than the savings of Ezhil.
(1) 192 (2) 195 Savings of Aari is 20% more than the savings of
(3) 198 (4) 201 Divya and the ratio of the expenditure of Aari to the
(5) None of these savings of Ezhil is 4:7. Total Expenditure of all the
persons together is Rs.4000 more than the total
22. If the number of veg burgers sold by Y is savings of all the persons together.
increased by 33(1/3)% and the number of non-veg
burgers sold by X is increased by 75%, then what 26. What is the ratio of the income of Divya to Ezhil?
is the sum of the number of veg burgers sold by Y (1) 2:1 (2) 3:1
and non-veg burger sold by X? (3) 1:2 (4) 1:3
(1) 486 (2) 495 (5) None of these
(3) 528 (4) 534
(5) 546 27. What is the average income of Divya, Chitra and
Ezhil?
23. If the selling price of each veg burger and each (1) 12000 (2) 14000
non-veg burger sold by Y is Rs.60 and Rs.90 (3) 16000 (4) 10000
respectively, then the total amount obtained by Y? (5) 18000
(1) 30000 (2) 32400
(3) 33000 (4) 36000 28. What is the difference between the Savings and
(5) None of these Expenditure of Bala, Chitra and Divya together?
(1) 2000 (2) 2500
(3) 3000 (4) 3500
(5) 4000
29. The Savings of Aari is what percent of the income (3) 165 (4) 155
of Aari? (5) 125
(1) 60% (2) 40%
(3) 80% (4) 70% 34. The number of lion sanctuary in MP is
(5) 50% approximately what percent of the total number of
tiger sanctuary in Tamilnadu?
30. What is the ratio of the Savings to Expenditure of (1) 78.12% (2) 70.37%
Ezhil? (3) 82.34% (4) 75.09%
(1) 7:8 (2) 7:10 (5) 68.23%
(3) 3:5 (4) 14:19
(5) None of these 35. What is the difference between the total number of
lion sanctuary and the total number of tiger
31-35. Total number of wildlife sanctuary (both tiger and sanctuary in all the states together?
lion sanctuary) in India is 600. Number of tiger (1) 34 (2) 36
sanctuary in HP is 40 more than the number of lion (3) 28 (4) 32
sanctuary in Kerala. Total number of wildlife (5) 30
sanctuary in UP is 135. The number of lion 36-40. A company manufactures three types of product i.e.
sanctuary in UP is equal to the number of tiger Product A, B, C. In the year 2018, the company
sanctuary in Tamilnadu. Number of tiger sanctuary manufactured a total of 16000 products. Out of the
in UP is equal to the number of lion sanctuary in total products some are plastic material and some
HP. The number of tiger sanctuary in MP is 4 more are metallic material.
than the number of tiger sanctuary in Kerala. Ratio 35% of the total products are product B. Ratio of
of the number of wildlife sanctuary in HP and plastic and metallic products in product B is 4:3.
Tamilnadu is 2:3. The number of lion sanctuary in Number of Plastic materials of product A is 10%
MP is 57% of the total number of wildlife sanctuary more than the number of plastic materials of
in MP. Number of wildlife sanctuary in MP is 20 product B. Ratio of metallic materials of product C
less than the total number of wildlife sanctuary in and the number of plastic materials of product A is
HP. The number of wildlife sanctuary in Kerala is 3 :5 respectively. Number of Metallic materials of
70 less than the number of wildlife sanctuary in UP. product A is 20% less than the number of metallic
materials of product B.
31. What is the ratio of the number of lion sanctuary in
Tamilnadu to the number of tiger sanctuary in UP? 36. Find the difference between the number of plastic
(1) 4:3 (2) 5:7 materials and the number of metallic materials of
(3) 11:6 (4) 10:9 product C?
(5) 12:7 (1) 694 (2) 784
(3) 796 (4) 736
32. Total number of lion sanctuary in all the states (5) 682
together is what percent of the total number of
wildlife sanctuary in the India? 37. Find the respective ratio of the number of metallic
(1) 50.91% (2) 57.32% materials of product A to the number of metallic
(3) 44.89% (4) 52.33% materials of product B.
(5) 41.93% (1) 1:2 (2) 3:2
(3) 4:5 (4) 2:5
33. What is the sum of the tiger sanctuary in HP and the (5) 5:4
number of lion sanctuary in Tamilnadu?
(1) 175 (2) 185
38. Find the total number of product A and product B? 41. The number of female employees in IT department
(1) 11000 (2) 11020 from B is approximately what percent of total
(3) 11080 (4) 11010 number of employees in sales department from A?
(5) 11040 (1) 31% (2) 33%
39. Number of plastic materials of product A is (3) 35% (4) 37%
approximately what percentage of total number of (5) 39%
materials of product B?
(1) 63% (2) 72% 42. What is the total number of male employees from A
(3) 56% (4) 44% in all the departments together?
(5) 85% (1) 104 (2) 109
(3) 113 (4) 118
40. Find the difference between the number of Plastic (5) None of these
materials of product B and the number of metallic
materials of product A? 43. What is the ratio of the total number of employees
(1) 1400 (2) 1380 in HR department from B to the total number
(3) 1320 (4) 1200 employees in Sales department from A?
(5) 1280 (1) 4:3 (2) 5:3
(3) 3:2 (4) 2:1
40-45. There are two different company A and B and have (5) None of these
three different departments – IT, HR and Sales.
A: Total number of employees from A is 180. 30% 44. What is the difference between the number of
of the employees are working in IT department and female employees from A and B in all the
the ratio of the number of employees are working in departments together?
HR department to sales department is 4:3. Ratio of (1) 2(2) 4 (3) 6
the number of male to female employees from HR (4) 8 (5) None of these
department is 5:4. Number of female employees in
IT department is half of the number of male 45. What is the ratio of the number of male employees
employees in HR department. Number of female in IT department from B to number of male
employees from sales department is 10 less than employees in Sales department from A?
that of the number of male employees in IT (1) 1:1 (2) 2:1
department. (3) 3:2 (4) 4:3
B: 25% of the total number of employees is (5) None of these
working in IT department. Number of female
employees in HR department is equal to the number 46-50. There are three boxes A, B and C has three different
of male employees in Sales department. Number of color balls-Red, Blue and Yellow. The total number
male employees in HR department is one – fourth of red balls in all the boxes together is 30.The
of the total number of employees from B. Ratio of number of Red balls in B is 5 which is equal to the
the number of male to female employees in IT number of yellow balls in C. Ratio of the number of
department is 3:2. Number of female employees in red balls in A to C is 2:3. Total number of balls in
sales department is 20. Total number of employees B is 27.The number of blue balls in B is 10. The
in sales department from B is 6 more than that of number of Red balls in C is sum of the number of
the total number of IT department employees from blue balls in A and C. The number of Yellow balls
A. in B is 5 more than the number of blue balls in A.
The total number of balls in all the boxes together
is 80.
46. What is the ratio of the number of Blue balls in
box A to the number of Yellow balls in box B?
(1) 2:3 (2) 3:5
(3) 8:15 (4) 7:12
(5) None of these

47. The number of Yellow balls in box A and C


together is approximately what percent of the total
number of Red balls in all the boxes together?
(1) 40% (2) 43%
(3) 47% (4) 45%
(5) 49%

48. What is the total number of Blue balls in all the


boxes together?
(1) 30 (2) 20
(3) 25 (4) 15
(5) 35

49. What is the difference between the total number of


balls in box A and C?
(1) 2 (2) 5
(3) 3 (4) 4
(5) None of these

50. What is the ratio of the sum of the number of Red


balls in box C and the number of Blue balls in box
B together to the total number of Yellow balls in
all the boxes together?
(1) 1:1 (2) 2:1
(3) 2:3 (4) 3:4
(5) None of these
Answer Key
1. (2) 11. (3) 21. (5) 31. (4) 41. ()
2. (4) 12. (4) 22. (5) 32. (1) 42. ()
3. (2) 13. (3) 23. (2) 33. (2) 43. ()
4. (3) 14. (2) 24. (3) 34. (2) 44. ()
5. (1) 15. (1) 25. (4) 35. (1) 45. ()
6. (5) 16. (4) 26. (2) 36. (4) 46. ()
7. (3) 17. (3) 27. (2) 37. (2) 47. ()
8. (3) 18. (5) 28. (3) 38. (3) 48. ()
9. (1) 19. (1) 29. (5) 39. (3) 49. ()
10. (2) 20. (5) 30. (3) 40. (4) 50. ()
Hints & Solutions
1-5. Common explanation : 100-y are girls.
In Physics: let the number of girls be 10x, number
of boys will be 7x. 98 + y 89
=
In Chemistry: number of boys is 10x, number of 140 + 100 – y 80
girls will be 10x + 20.
So, 98 × 80 + 80y = 89 × 240 – 89y
37x + 20 = 538 169 y = 13520
518 y = 80
x=
37 Hence, option D is correct.
x = 14 5. Solution
1. Solution Following the common explanation, we get
Following the common explanation, we get Number of girls in Chemistry is increased by 10%
Number of girls who took admission in Physics = i.e., 160 (1.1) = 176
10x = 140 So, extra 176 – 160 = 16 girls are increased.
Since, the total 538 remains same, so 16 girls
Number of boys who took admission in Physics = should be removed from girls of Physics science.
7x = 98 So, 16 girls should be removed from 140 girls
Required difference = 140 – 98 = 42 Reqd % = (16/140)* 100 = 11.43%
Hence, option B is correct. Hence, option E is correct.
2. Solution 6-10. Common Explanation
Following the common explanation, we get Number of cows in farm ‘P’ = 200
Number of boys who took admission in Chemistry Number of horses in farm ‘R’ = 200 + 200 = 400
= 10x = 140 Number of cows in farm ‘R’ = 1000 – 400 = 600
Number of girls who took admission in Chemistry Number of cows in farm ‘S’ = 1.25 × 600 = 750
= 10x + 20 = 160 Number of cows in farm ‘Q’ = 1800 – (750 + 600
Total number of students who took admission in + 200) = 250
Chemistry = 140 + 160 = 300 Number of horses in farm ‘P’ = 400 × (6/5) = 480
Hence, option C is correct.. Number of horses in farm ‘Q’ = 350
Number of horses in farm ‘S’ = 350 + 150 = 500
3. Solution Therefore,
Following the common explanation, we get Number of Number Total
Number of boys who admission part in Physics cows of animals
= 98 horses
Total number of students who took admission in P 200 480 680
Chemistry science = 140 + 160 = 300 Q 250 350 600
98 × 100 R 600 400 1000
Reqd. % = = 32.66%
300 S 750 500 1250
Hence, option A is correct. Total 1800 1730
4. Solution 6. Solution.
Following the common explanation, we get Required ratio = 480:600 = 4:5
100 new students took admission in Physics Hence, option c.
science, let y students took admission are boys and 7. Solution.
Required difference = 1250 – 250 = 1000
Hence, option a. 14. Solution.
8. Solution. Desired number of females = 1.75 × 560 = 980
Required percentage = {(1000 – 750)/1000} × 100 Hence, option b.
= 25% 15. Solution.
Hence, option b. Desired females = 0.40 × 420 + 0.60 × 360 = 168
9. Solution. + 216 = 384
Required average = (400 + 500)/2 = 450 Hence, option d.
Hence, option e. 16. Solution.
10. Solution. Total number of players = 800
Required number of horses = 0.8 × 350 = 280 1
Number of woman players = × 800 = 200
4
Hence, option b. 3
Number of man players = × 800 = 600
11-15. Common Solution. 4
Let number of males in city Y is x. Number of cricket players = 25% of 800 = 200
Number of females in city X = 1.20 × x = 1.2x Number of badminton players = 110
Number of males in city X = 1080 – 1.2x Number of tennis players = 10% of 800 = 80
Number of males in city Z = 5/4 × (1080 – 1.2x) Number of baseball players = 800 – (200 + 110 +
According to question; 80 + 220) = 800 – 610 = 190
5/4 × (1080 – 1.2x) = 630 Number of woman cricket players
Or, 1080 – 1.2x = 504 Or, 1.2x = 576 Or, x = 30% of 200 = 60
= 480 ∴ Number of man cricket players = 200 – 60 = 140
Number of males in city Y = x = 480 Number of woman badminton players
1
Number of females in city X = 1.2x = 576 = × 60 = 30
2
Number of males in city X = 1080 – 1.2x = 504 ∴ Number of man badminton players
Number of females in city Z = 7/6 × 480 = 560 = 110 – 30 = 80
Total population of city Z = 630 + 560 = 1190 Number of woman tennis players
Total population of city P = 576/0.60 = 960 = 10% of 220 = 22
Number of males in city P = 9/16 × 960 = 540 ∴ Number of man tennis players = 80 – 22 = 58
Number of females in city P = 7/16 × 960 = 420 Number of woman hockey players = Number of
Total population of city Y = 420 × 2 = 840 woman baseball players
Number of females in city Y = 840 – 480 = 360 1 1
= �200 − (60 + 30 + 22) = [200 − 112] =
Cities Number Number of Total 2 2
88
of males females population 2
� = 44
city X 504 576 1080 ∴ Number of man hockey players = 220 –44 = 176
city Z 630 560 1190 And number of man baseball players
city P 540 420 960 = 190 – 44= 146
city Y 480 360 840 Tabular form of above information is as follows
11. Solution. Games Number of Number of
Desired ratio = 504:576 = 7:8 Man players woman players
Hence, option a. Cricket 140 60
12. Solution. Badminton 80 30
Desired Average = (540 + 480)/2 = 510
Tennis 58 22
Hence, option b.
Hockey 176 44
13. Solution.
Desired Percentage = [(960 – 840)/960] × 100 = Baseball 146 44
12.5% Total 600 200
Hence, option c.
From the table, number of woman hockey players 22. Solution.
= 44 Required total = 400/300 * 180 + 175/100 * 168
Number of man badminton players = 80 = 240 + 294
∴ Required ratio = 44 : 80 = 11 : 20 = 534
17. Solution. 23. Solution.
From the table, it is clear that the total number of Required amount = 60 * 180 + 240 * 90
man players in hockey, cricket and baseball = Rs.32400
= 176 + 140 + 146 = 462 24. Solution.
18. Solution. Required percentage = (189 + 216)/(240 + 135) *
Number of woman baseball players = 44 100
Number of man hockey players = 176 = 405*100/375
∴ Required percentage =
44
× 100% = 25% = 108%
176
25. Solution.
19. Solution.
Required ratio = (168 + 135):(180 + 189)
Number of man baseball players = 146
= 303:369
Number of woman tennis players = 22
= 101:123
∴ Required difference = 146 – 22 = 124
26-30. Solution
20. Solution.
Total savings = (70000- 4000)/2 = Rs.33000
From the table, it is clear that women are
Total expenditure = 33000 + 4000 = 37000
maximum in cricket and men are minimum in
Savings of Chitra = 2/11 * 33000 = 6000
tennis.
Income of Bala = 12000
21-25. Solution.
Expenditure of Bala = 6000/2 = 3000
Number of veg burger sold by X = 9a
Savings of Bala = 12000 – 3000 = 9000
Number of Non-veg burger sold by X = 7a
Expenditure of Divya = 100/25 * 3000 = 12000
Number of veg burger sold by Y = 3b
Savings of Ezhil = 12000 – 5000 = 7000
Number of non-veg burger sold by Y = 4b
Savings of Aari and Divya = 33000 – 6000 – 9000
Total number of burger sold by Z = 324
– 7000 = 11000
Number of veg burger sold by Z = 7/12 * 324 =
Ratio of the savings of Aari and Divya = 120:100
189
= 6:5
Number of non-veg burger sold by Z = 5/12 * 324
Savings of Aari = 6/11 * 11000 = 6000
= 135
Savings of Divya = 5/11 * 11000 = 5000
Number of veg burger sold by X = 120/100 * 3b =
Expenditure of Aari = 4/7 * 7000 = 4000
18b/5
Expenditure of Chitra and Ezhil = 37000 – 4000 –
18b/5 = 9a
3000 – 12000 = 18000
b/a = 5/2
Expenditure of Ezhil is Rs.2000 more than the
18k + 14k + 15k + 20k + 324 = 1128
expenditure of Chitra.
k = 12
Expenditure of Ezhil = 10000
Number of veg burger sold by X = 18 * 12 = 216
Expenditure of Chitra = 8000
Number of Non-veg burger sold by X = 14 * 12 =
26. Solution
168
Required ratio = (5000 + 12000):(7000 + 10000)
Number of veg burger sold by Y = 15 * 12 = 180
= 1:1
Number of non-veg burger sold by Y = 20 * 12 =
27. Solution
240
Required average = ((8000 + 6000) + (10000 +
21. Solution.
7000) + (5000 + 12000))/3
Required average = (216 + 180 + 189)/3 =585/3=
= (14000 + 17000 + 17000)/3
195
= 48000/3= 16000
28. Solution 34. Solution
Savings of Bala, Chitra and Divya = 9000 + 6000 Required percentage = 57/81 * 100 = 70.37%
+ 5000 = 20000 35. Solution
Expenditure of Bala, Chitra and Divya = 3000 + Number of Lion sanctuary in all the states = 57 +
8000 + 12000 = 23000 26 + 81 + 54 + 99 = 317
Required Difference = 23000 – 20000 = 3000 Number of tiger sanctuary in all the states = 39 +
29. Solution 43 + 66 + 81 + 54 = 283
Required percentage = 6000/10000 * 100 = 60% Required Difference = 317 – 283 = 34
30. Solution 36-40. Solution
Required ratio = 7000:10000 = 7:10 Total = 16000
31-35. Solution 35% of the total products are product B
Total number of wildlife sanctuary in UP = 135 Total products of B type = 35% 16000 = 5600
Total number of wildlife sanctuary in Tamilnadu = Ratio of plastic and metallic products in product B
3x is 4:3
Total number of wildlife sanctuary in HP = 2x Plastic products of product B = 4/7 * 5600 = 3200
Total number of wildlife sanctuary in MP Metallic products of product B = 3/7 * 5600 = 2400
= 2x – 20 Plastic products of product A = 110% (number of
Total number of wildlife sanctuary in Kerala plastic products of product B)
= 135 – 70 = 65 = > 110 % 3200 = 3520
3x + 2x + 2x – 20 + 135 + 65 = 600 Ratio of metallic products of product C and the
7x = 420 number of plastic product of product A is 3:5
x = 60 respectively
Total number of wildlife sanctuary in Tamilnadu Metallic products of product C = 3/5 * 3520 = 2112
= 180 Number of Metallic products of product A = 80%
Total number of wildlife sanctuary in HP = 120 of 2400 =1920
Total number of wildlife sanctuary in MP = 100 Total products of B = 5600
Number of lion sanctuary in MP Total products of A = 3520 + 1920 = 5440
= 100 * 57/100 = 57 Total Products of C = 16000 – (5600 + 5440)
Number of tiger sanctuary in MP = 100 – 57 = 43 = 4960
Number of tiger sanctuary in Kerala = 43 –4= 39 Total Plastic products of Product C = 4960 – 2112
Number of lion sanctuary in Kerala = 65 – 39 = 26 = 2848
Number of tiger sanctuary in HP = 26 + 40 = 66 Type Plastic Metallic Total
Number of lion sanctuary in HP = 120 – 66 = 54 A 3520 1920 5440
Number of tiger sanctuary in UP = 54 B 3299 2400 5600
Number of lion sanctuary in UP = 135 – 54 = 81 C 2848 2112 4960
Number of tiger sanctuary in Tamilnadu = 81 36. Solution
Number of lion sanctuary in Tamilnadu Difference between the number of plastic materials
= 180 – 81 = 99 and the number of metallic materials of product C
31. Solution = 2848 – 2112 =736
Required ratio = 99:54= 11:6 37. Solution
32. Solution Required Ratio = 1920: 2400 = 4:5
Number of Lion sanctuary in all the states = 57 + 38. Solution
26 + 81 + 54 + 99 = 317 Total = 5440 + 5600 = 11040
Required percentage = 317/600 * 100 = 52.83% 39. Solution
33. Solution Number of plastic materials of product A = 3520
Required sum = 66 + 99 = 165 Total product of B = 5600
Required percentage = (3520 / 5600) * 100= 62.85 Number of male employees in IT department
% ≈ 63% = 3/5 * 50 = 30
40. Solution Number of female employees in IT department
Difference = 3200 – 1920= 1280 = 2/5 * 50 = 20
41-45. Solution 41. Solution
A: Required percentage = (20/54) * 100 = 37%
Total number of employees = 180 42. Solution
Total number of IT employees Male employees from A = 34 + 40 + 30 = 104
= 180 * 30/100 = 54 43. Solution
Number of employees from sales and HR Required ratio = 90:54
department = 180 – 54 = 126 =5:3
Total number of HR employees = 126 * 4/7 = 72 44. Solution
Total number of Sales employees = 126 * 3/7 = 54 Number of female employees from A
Number of Male employees in HR department = = 20 + 32 + 24 = 76
72 * 5/9 = 40 Number of female employees from B
Number of female employees in HR department = 20 + 40 + 20 = 80
= 72 * 4/9 = 32 Difference = 80 – 76 = 4
Number of female employees in IT department 45. Solution
= 40/2 = 20 Required ratio = 30:30 = 1:1
Number of male employees in IT department = 54 46-50. Solution
– 20 = 34 Total number of balls = 80
Number of female employees in sales department Total number of Red balls = 30
= 34 – 10 = 24 Number of Red balls in box B = 5
Number of male employees in sales department Number of Red balls in A and C = 30 – 5 = 25
= 54 – 24 = 30 Number of Red balls in box A = 2/5 * 25 = 10
B: Number of Red balls in box C = 3/5 * 25 = 15
Number of female employees in sales department Number of Blue balls in box B = 10
= 20 Number of Yellow balls in box B
Total number of sales department employees = 27 – 10 – 5 = 12
= 54 + 6 = 60 Number of Yellow balls in box C = 5
Number of male employees in sales department Number of Blue balls in A = 12 – 5 = 7
= 60 – 20 = 40 Number of Blue balls in C = 15 – 7 = 8
Number of female employees in HR department Total number of Blue balls = 10 + 7 + 8 = 25
= 40 Number of Yellow balls in box A
Total number of employees from B= x = 80 – 30 – 25 – 5 – 12 = 8
Total number of employees in IT department Total number of Yellow balls = 8 + 12 + 5 = 25
= x * 25/100 = x/4 Total number of balls in box A = 10 + 7 + 8 = 25
Number of male employees in HR department Total number of balls in box B = 27
= x/4 Total number of balls in box C = 15 + 5 + 8 = 28
x/4 + x/4 + 40 + 60 = x 46. Solution
x/2 = 100 Required ratio = 7:12
x = 200 47. Solution
Number of male employees in HR department Required % = (8 + 5)/30 * 100 = 43.33%~ 43%
= 200/4 = 50 48. Solution
Total number of employees in IT department Blue balls = (7 + 10 + 8) = 25
= 200/4 = 50 49. Solution
Difference = 28 – 25 = 3 = 25:25
50. Solution = 1:1
Required ratio = (15 + 10):(8 + 12 + 5)
न�व Booklet
For All Banking & Insurance Exam
Ch -29 | Caselet DI - Venn Diagram| Maths by Arun sir

Directions (1-5): There are 960 students in a college. 5. What is the ratio of the number of students who
All of them appeared for half yearly exam that had 3 cleared both subject A and B but not C to the number of
subjects: A, B and C. Every student cleared at least 1 students who cleared both subject A and C but not B?
subject. (a) 5 : 4
The number of students who cleared only subject C was (b) 7 : 3
double the number of students who cleared only subject (c) 7 : 4
B. 15% of the students cleared both subjects B and C (d) 8 : 5
and 288 students cleared both subjects A and B. 168 (e) 8 : 3
students cleared only subject A which is 40 greater than
the number of students who cleared only subject B. 176 Directions: Study the information given below carefully
students cleared subject B but not subject A. The and answer the questions related to it.
number of students who cleared subject A was 64 more In sector 125, Noida, there are 2400 people who play
than the number of students who cleared subject B. three different type of sports Tennis, Volleyball and
1. How many students cleared subject C? Badminton. 56% of total population play Badminton.
(a) 392 44% of total population play Volleyball. 240 people
(b) 416 play both Badminton and Volleyball.
(c) 468 8% of total population play Tennis and Volleyball both.
(d) 472 The total population who play Tennis is 24%. The
(e) 478 people who play all the three types of games viz Tennis,
Volleyball and Badminton are 4% of total population.
2. How much percentage of students did not clear
subject A? 6. The population who play Tennis and Badminton both
(a) 35% is approximately what percent of people who play
(b) 40% Tennis?
(c) 45% (a) 40%
(d) 50% (b) 42%
(e) 55% (c) 48%
(d) 49%
3. How many students cleared at least 2 subjects? (e) 45%

7. What is the average number of people who play


(a) 408 Badminton only, Volleyball only and Tennis only?
(b) 392 (a) 648
(c) 464 (b) 662
(d) 440 (c) 640
(e) 480 (d) 650
(e) 658
4. What is the difference between the number of
students who cleared only subject A and the number of 8. What is the total number of people who play only
students who cleared only subject C? Tennis and Volleyball both and only Tennis and
(a) 64 Badminton both
(b) 84 (a) 240
(c) 72 (b) 246
(d) 48 (c) 236
(e) 88 (d) 256
(e) 242
न�व Booklet
For All Banking & Insurance Exam
Ch -29 | Caselet DI - Venn Diagram| Maths by Arun sir

9. If the ratio of male to female is 13 : 9 in that 13. What is the difference between the number of people
population who play Volleyball then the total number of who watch only ESPN and number of people who watch
males who play Volleyball are what percent of total only TEN SPORTS?
population of given sector? के वल ESPN चैनल देखने वाले लोगों क� संख्या और के वल TEN SPORTS
(a) 24% चैनल देखने वाले लोगों क� संख्या में अंतर िकतना है?
(b) 26% (a) 80
(c) 28% (b) 110
(d) 30% (c) 160
(e) 32% (d) 100
10. What is the difference between people who play (e) 120
Badminton only and Volleyball only?
(a) 244 14. What percentage of people who watch STAR
(b) 238 SPORTS also watch TEN SPORTS?
(c) 235 (a) 44%
(d) 240 (b) 52%
(e) 245 (c) 62%
Directions (11-15): The following is the data about the (d) 56%
number of people in a society that watch none or more (e) 48%
of 3 sports channels: ESPN, TEN SPORTS and STAR
SPORTS. 15. The number of people who watch both ESPN and
There are 4000 people in the society, 12.5% of those TEN SPORTS but not TEN SPORTS is what percent of
watch both ESPN and TEN SPORTS but not STAR the number of people who watch both ESPN and TEN
SPORTS. 45% of the people watch exactly 1 channel. SPORTS but not TEN SPORTS?
1160 people watch both ESPN and STAR SPORTS. (a) 55%
1860 people watch at least 2 channels. The number of (b) 50%
people who watch only STAR SPORTS is 520 greater (c) 45%
than the number of people who watch both TEN (d) 35%
SPORTS and STAR SPORTS but not ESPN. The ratio (e) 40%
of the number of people who watch all 3 channels to the
number of people who watch only TEN SPORTS is 8:5. Directions (16-20): Read the data carefully and answer
2260 people watch TEN SPORTS. the following questions.
A survey is carried out on 480 persons to check whether
11. What percentage of people don't watch any of the 3 they like three games Hockey, Baseball and Basketball
channels? or not. A person may like more than one game and it is
(a) 6% also possible that a person may not like any of these
(b) 8.5% games.
(c) 12% Ratio of persons who like only Hockey to those who like
(d) 15.5% only Basketball is 1: 3 and out of total persons 20% like
(e) 18% both Hockey and Baseball but not Basketball. Total
persons who like both Hockey and Basketball but not
12. How many people watch ESPN? Baseball is 80. Total persons who like both Baseball and
(a) 1980 Basketball is 56 and total persons who like only
(b) 2140 Baseball is 32. Total persons who like both Hockey and
(c) 2220 Baseball is 112 while 50% of total persons who
(d) 2300 involved in survey like Hockey.
(e) 1880
न�व Booklet
For All Banking & Insurance Exam
Ch -29 | Caselet DI - Venn Diagram| Maths by Arun sir

16. How many persons are there who like Basketball of people who like both Horse racing and Bike racing
with at least one more game? are 25% more than number of people who like only car
(a) 30 racing. Number of people who like both car racing and
(b) 28 Horse racing but not Bike racing are 72 more than
(c) 36 number of people who like both car racing and Bike
(d) 32 racing but not Horse racing, and total number of people
(e) 34 who like exactly two racing is 718. Total number of
people who like Bike racing is 1338. Ratio of number
17. How many persons are there who did not like any of of people who like only car racing to number of people
the three games? who like Horse racing is 4:13. Total number of people
(a) 24 who like only one of all the three racing is 1400.
(b) 32
(c) 16 21. What is the value of X?
(d) 48 A - 1194
(e) 40 B - 1200
C - 1209
18. How many persons are there who like Baseball but D - 1220
not Hockey? E - None of these
(a) 96
(b) 72 22. What is the ratio of number of people who like only
(c) 112 Bike racing to number of people who like only Horse
(d) 64 racing?
(e) 84 A - 7:5
B - 9:16
19. What is the total number of persons who like less C - 4:3
than three games (Do not include those who do not like D - 8:5
any game)? E - None of these
(a) 440
(b) 480 23. Number of people who like both car racing and Bike
(c) 520 racing but not Horse racing is how much percent
(d) 400 more/less than number of people who like both car
(e) 360 racing and Horse racing but not Bike racing?
A - 37.5%
20. What percent of total persons involved in survey like B - 22.5%
only Basketball? C - 20%
(a) 10% D - 35%
(b) 20% E - None of these
(c) 30%
(d) 40% 24. What is the difference between the number of people
(e) 50% who like only Bike racing and number of people who
Directions: Answer the questions based on the like only car racing?
information given below. A - 320
A survey was conducted between ‘2X’ numbers of B - 240
people, who like at least one of the three racing i.e. car C - 280
racing, bike racing and horse racing. Total number of D - 360
people who like only car racing are 20% more than E - None of these
number of people who like all the three racing. Number
न�व Booklet
For All Banking & Insurance Exam
Ch -29 | Caselet DI - Venn Diagram| Maths by Arun sir

(e) 260
25. What is the number of people who like all the three Q28. How many athelets participated in at most two
racing? athelatics?
A - 240 (a) 240
B - 248 (b) 260
C - 280 (c) 300
D - 300 (d) 500
E - None of these (e) 960

Directions (26-30): Q29. How many athelets not participated in athelatics


Study the following paragraph carefully & Q?
answer the question given below. (a) 440
There are 1000 athletes in a state. Out of 1000 (b) 360
athletes some participated in athletics ‘P’, ‘Q’ (c) 540
and ‘R’ while some not. Number of athletes not (d) 640
participated in any athletics is equal to number (e) None of these
of athletes participated in athletics ‘R’ only.
Number of athletes participated in athletics ‘Q’ Q30. How many athelets participated in athelatics P or
is 360. Ratio of number of athletes participated in athelatics R?
in athletics ‘P’ and ‘Q’ only to number of (a) 240
athletes participated in athletics ‘Q’ and ‘R’ (b) 360
only is 2 : 3. Number of athletes participated in (c) 500
athletics ‘P’ and ‘R’ both is half of number of (d) 680
athletes participated in only athletics ‘R’. (e) 760
Number of athletes participated in athletics ‘P’
only is 50% more than number of athletes Directions: Answer the questions based on the
participated in ‘Q’ only. Number of athletes information given below.
participated in all the three athletics is 4% of 1920 students went to a library to study only four books
the total number of athletes in the state. Number (P, Q, R and S). As per the rules of the library, when a
of athletes participated in ‘Q’ athletics only is student studies book P, he should study either book Q
same as number of athletes participated in ‘Q’ or R, and it is not necessary for the student to study book
and ‘R’ only. Q or book R. When a student studies book R, then he
should study either book P or S, and it is not necessary
Q26. How many athelets participated in at least two for the student to study P or S. No one studies book R
athelatics? with Q, and S with P. The student can study any of the
(a) 240 four books individually. The number of students who
(b) 260 studied only P is 60% more than number of students
(c) 300 who studied S with R. Number of students who studied
(d) 360 only book R is 210 more than number of students who
(e) 500 studied only book P. The ratio of number of students
who studied only book R to number of students who
Q27. How many athelets participated in two athelatics studied only book Q is 5:4. Number of students who
only? studied book R with P is half the number of students
(a) 280 who studied only book Q. Number of students who
(b) 220 studied book P with Q is 60 more than number of
(c) 340 students who studied P with R. The number of students
(d) 300 who studied only book S is 300.
न�व Booklet
For All Banking & Insurance Exam
Ch -29 | Caselet DI - Venn Diagram| Maths by Arun sir

in dessert. Each of them like at least one of the three


31. What is the ratio of number of students who studied type of sweet. Numbers of people who like both A and
only book P to number of students who studied book P B but not C are 20% more than the number of people
with Q? who like all three types of sweet. Ratio of number of
A - 3:2 people who like both A and C to number of people who
B - 2:1 like both A and B is 12:11. Number of people who like
C - 4:3 only B are 300% more than the number of people who
D - 1:1 like both A and C but not B. Number of people who like
E - None of these exactly two types of sweet are 220. Ratio of number of
people who like B to number of people who like C is
32. Which books is studied by maximum number of 4:3. Number of people who like only A is 40% more
students? than number of people who like only C and total number
A-Q of people who like A is 390.
B-R
C-P 36. What is the value of N?
D-S A) 920
E - Both (b) and (c) B) 910
C) 900
33. Number of students who studied only book Q are D) 890
how much percent more/less than the number of E) None of these
students who studied only book S?
A - 25% 37. What is the number of people who like exactly one
B - 30% type of sweet?
C - 40% A) 640
D - 20% B) 6620
E - None of these C) 10
D) 600
34. What is the number of students who studied book P E) None of these
with book R?
A - 240 38. Number of people who like B is how much percent
B - 150 more/less than number of people who like only C?
C - 180 A) 110%
D - 360 B) 180%
E - 270 C) 220%
D) 200%
35. What is the total number of students who studied E) None of these
book Q?
A - 600 39. What is the ratio of number of people who like both
B - 720 A and C to number of people who like both C and B?
C - 660 A) 2:3
D - 780 B) 3:4
E - None of these C) 4:5
D) 6:7
Directions: Answer the questions based on the E) None of these
information given below.
In a survey held between ‘N’ numbers of people
regarding the type sweet (A, B or C) they prefer to eat
न�व Booklet
For All Banking & Insurance Exam
Ch -29 | Caselet DI - Venn Diagram| Maths by Arun sir

40. What is the number of people who like all three C. even number
types of sweet? D. both (a) and (c)
A. multiple of 5 E. both (b) and (c)
B. multiple of 3
Solution
2. Sol.
1. Sol. Number of students who did not cleared subject
Number of students who cleared only subject B = A
168 - 40 = 128 = 128 + 48 + 256 = 432
Number of students who cleared only subject C = Required percentage = (432/960)*100% = 45%
2*128 = 256 3. Sol.
Number of students who cleared both subject B Number of students who cleared at least 2
and C but not A subjects
= 176 - 128 = 48 = 192 + 96 + 72 + 48 = 408
Number of students who cleared both subjects B 4. Sol.
and C = (15/100)*960 = 144 Required difference = 256 - 168 = 88
Number of students who cleared all three subjects 5. Sol.
= 144 - 48 = 96 Required ratio = 192 : 72 = 8 : 3
Number of students who cleared both subjects A 6. Sol.
and B but not subject C Total population = 2400
= 288 - 96 = 192 n(A) = People who play Badminton
Number of students who cleared subject B = 288 n(B) = people who play Volleyball
+ 176 = 464 n(C) = People who play Tennis
Number of students who cleared subject A= 464 n(A∩B) = People who play Badminton and
+ 64 = 528 Volleyball both
Number of students who cleared both subject A n(B∩C) = People who play Volleyball and
and C but not subject B Tennis both
= 528 - 168 - 288 = 72 n(A∩C) = People who play Badminton and
Completing the Venn diagram: Tennis both
n(A∩B∩C) =People who play all the three types
of games
240
240 → converting into percentage = × 100
2400
= 10%
∴ 100 = 56 + 44 + 24 – (10 + 8 + x) + 4
⇒ x = 128 – 118
= 10% = n(A∩C)

Number of students who cleared subject C


= 72 + 96 + 48 + 256
= 472
न�व Booklet
For All Banking & Insurance Exam
Ch -29 | Caselet DI - Venn Diagram| Maths by Arun sir

Drawing the Venn-diagram and marking the


regions:

Sol.
10
Required percentage = × 100 ≃ 42%
24 So, 500 + 5k + 200 + 8k = 2260
7. Sol. 13k = 1560
Required average (From vain diagram) k = 120
1 (40+30+10)
= ×
3 100
× 2400 Number of people who watch exactly 1 channel
= 640 = (45/100)*4000 = 1800
8. Sol. Number of people who watch only ESPN = 1800
Required answer = (6 + 4) × 24 - 5k - 720 = 480
= 240 Number of people who don't watch any of the 3
9. Sol. channels = 4000 - 1800 - 1860 = 340
Required no. of males in that population who Completing the Venn-diagram:
play Volleyball
13
= × 44 × 24
22
= 624
624
∴ Required percentage = × 100
2400
= 26%

10. Sol.
Required difference
(40−30)
= 100
× 2400
= 240

11. Sol.
Number of people who watch both ESPN and
TEN SPORTS but not STAR SPORTS =
Required percentage = (340/4000)*100% = 8.5%
(12.5/100)*4000 = 500
12. Sol.
Number of people who watch both TEN SPORTS
Number of people who watch ESPN = 480 + 500
and STAR SPORTS but not ESPN = 1860 - 1160
+ 200 + 960 = 2140
- 500 = 200
13. Sol.
Number of people who watch only STAR
Required difference = 600 - 480 = 120
SPORTS = 200 + 520 = 720
14. Sol.
न�व Booklet
For All Banking & Insurance Exam
Ch -29 | Caselet DI - Venn Diagram| Maths by Arun sir

Number of people who watch TEN SPORTS = Total persons who like at least one of these games
200 + 960 + 200 + 720 = 2080 = 48 + 96 + 32 + 80 + 16 + 40 + 144 = 456
Required percentage = ((960 + 200)/208)*100% Total persons who did not like any of these games
≈ 56% = 480 - 456
15. Sol. = 24
Required percentage = (200/500)*100% = 40% 18. Sol.
16. Sol. Total persons who like Baseball app but not
Total persons involved in survey = 480 Hockey = 32 + 40
Let persons who like only Hockey to those who = 72
like only Basketball is 'x' and '3x' respectively. 19. Sol.
Total persons who like both Hockey and Baseball Total persons who like exactly one game = 48 +
but not Basketball = 20% of 480 = 96 32 + 144 = 224
Total persons who like both Hockey and Total persons who like exactly two games = 96 +
Basketball but not Baseball = 80 80 + 40 = 216
Total persons who like both Baseball and Total number of persons who like less than three
Basketball = 56 games = 224 + 216 = 440
Let total persons who like all the three games = 20. Sol.
'y' Total persons involved in survey = 480
Total persons who like both Baseball and Total persons who like only Basketball = 144
Basketball but not Hockey = (56 - y) Required percent = (144/480) * 100
Total persons who like only Baseball = 32 = 30%
Total persons who like both Hockey and Baseball 21. Solution
= 112 Let total people who like all three racing be x.
96 + y = 112 Number of people who like only car racing =
y = 16 1.2x
Total persons who like Hockey app = 50% of 480 Number of people who like both Horse racing
= 240 and Bike racing = 1.25 × 1.2x = 1.5x
x + 96 + y + 80 = 240 Number of people who like both Horse racing
x = 48 and Bike racing but not car racing = 1.5x – x =
0.5x
Let number of people who like both car racing
and Bike racing but not Horse racing be y.
Number of people who like both car racing and
Horse racing but not Bike racing = y + 72
Number of people who like only Bike racing =
1338 – (y + x + 0.5x) = 1338 – 1.5x – y
Number of people who like Horse racing = 13/4
× 1.2x = 3.9x
Number of people who like only Horse racing =
3.9x – x – 0.5x – y – 72 = 2.4x – y – 72
According to question;
y + y + 72 + 0.5x = 718
Total persons who like Basketball along with at 0.5x + 2y = 646
least one more game = 80 + 16 + 40 x + 4y = 1292………………………(1)
= 136 And, 1.2x + 2.4x – y – 36 + 1338 – 1.5x – y =
17. Sol. 1400
2.1x – 2y = 134…………………….(2)
न�व Booklet
For All Banking & Insurance Exam
Ch -29 | Caselet DI - Venn Diagram| Maths by Arun sir

From equation (1) and (2), we get; = No. of athelets participated in Q and R only =
x = 300 and y = 248 3x

Number of athelets participated in athelatics P


2
and Q only = × 3𝑥𝑥 = 2𝑥𝑥
3

N = 360 + 640 + 400 + 320 + 248 + 300 + 150 =


2418
Hence, option c.
22. Solution
Desired Ratio = 640:400 = 8:5 Now, 2x + 3x + 3x + 40 = 360
Hence, option d. ⇒ x = 40
23. Solution 𝑎𝑎
𝑎𝑎𝑎𝑎𝑎𝑎, 12.5𝑥𝑥 + 𝑎𝑎 + + 𝑎𝑎 = 1000
2
Desired Percentage = [(320 – 248)/320] × 100 = 5𝑎𝑎
22.5% = 500
2
Hence, option b. ⇒ a = 200
24. Solution
Desired Difference = 640 – 360 = 280
Hence, option c.
25. Solution
Number of people who like all three racing =
300
Hence, option d.
Solutions (26-30):
Total athelets = 1000
Let, athelets participated in athelatics R only = a
Total athelets participated in athelatics Q = 360 26Sol.
Ratio of number of athelets participated in Athelets participated in at least two athelatics
athelatics P and Q = 80 + 60 + 40 + 120 = 300
only to athelets participated in athelatics Q and 27. Sol.
R only = 2 : 3 Athelets participated in two athelatics only = 80
Athelets participated in athelatics P and R both + 60 + 120 = 260
= a/2 28Sol.
Number of athelets participated in all three Athelets participated in at most two athelatics =
4 180 + 120 + 200 + 60 + 80 + 120 + 200= 960
athelatics = × 1000 = 40
100 29Sol.
Number of athelets participated in Q athelatics Athelete not participated in athelatics Q = 1000
only – 360 = 640
30Sol.
न�व Booklet
For All Banking & Insurance Exam
Ch -29 | Caselet DI - Venn Diagram| Maths by Arun sir

Athelets participated in athelatics P or in Number of students who studied book S = 300 +


athelatics R = 180 + 60 + 40 + 80 + 200 + 120 = 150 = 450
680 Number of students who studied book R = 150 +
31Solution 450 + 180 = 780
Number of students who studied book P = 180 +
240 + 240 = 660
Number of students who studied book Q = 240
+ 360 = 600
Hence, option b.
33. Solution

Required % = [(120 – 100)/100] × 100 = 20%


Let the number of students who studied book S Hence, option d.
with R be ‘x’ 34. Solution
The number of students who studied only P = Number of students who studied book P with R
1.6x = 180
Number of students who studied only R = (1.6x
+ 210) 35. Solution
Number of students who studied only Q = 4/5 × Total number of students who studied Q = 600
(1.6x + 210) Hence, option a.
Number of students who studied R with P = 0.5 36. Let number of people who like all three
× [4/5 × (1.6x + 210)] kinds of sweet be ‘x’
Number of students who studied P with Q = 0.5 Number of people who like A and B but not C =
× [4/5 × (1.6x + 210)] + 60 1.2x
According to question, Number of people who like both A and B = 1.2x
x + 1.6x + (1.6x + 210) + 4/5 × (1.6x + 210) + + x = 2.2x
0.5 × [4/5 × (1.6x + 210)] + 0.5 × [4/5 × (1.6x + Number of people who like both A and C =
210)] + 60 = 1920 – 300 12/11 × 2.2x = 2.4x
2.6x + (1.6x + 210)(1 + 4/5 + 4/10 + 4/10) + 60 Number of people who like both A and C but
= 1620 not B = 2.4x – x = 1.4x
2.6x + (1.6x + 210)(13/5) = 1560 Number of people who like both C and B but
2.6x + 2.6(1.6x + 210) = 1560 not A = 220 – 1.4x – 1.2x = 220 – 2.6x
2.6x + 4.16x + 546 = 1560 Number of people who like only B = 4 × 1.4x =
6.76x = 1014 5.6x
x = 150 Number of people who like C = 3/4 × (220 –
2.6x + x + 1.2x + 5.6x) = 0.75 × (220 + 5.2x) =
165 + 3.9x
Number of people who like only C = 165 + 3.9x
– x – 1.4x – 220 + 2.6x = 4.1x – 55
Number of people who like only A = 1.4 × (4.1x
– 55)
According to question;
1.4 × (4.1x – 55) + x + 1.4x + 1.2x = 390
Desired ratio = 240 : 240 = 1:1 5.74x – 77 + 3.6x = 390
Hence, option d. 9.34x = 467
32. Solution So, x = 50
न�व Booklet
For All Banking & Insurance Exam
Ch -29 | Caselet DI - Venn Diagram| Maths by Arun sir

N = 280 + 150 + 210 + 90 + 70 + 60 + 50 = 910


Hence, option b.
37. Sol.
Number of people who like exactly one type of
sweet = 210 + 280 + 150 = 640
Hence, option a.
38. sol
Desired Percentage = [(480 – 150)/150] × 100 =
220%
Hence, option c.
39. Sol.
Desired Ratio = 120:140 = 6:7
Hence, option d.
40. Sol.
Number of people who like all three types of
sweet = 50
Hence, option a.
Directions (01 - 05): Study the following information ICSE board schools is 5:3:2. Find the total
carefully and answer the questions given below. number of CBSE and ICSE boards in state Z?
The graph shows the number of schools in five states [P, (1) 462 (2) 464
Q, R, S and T] of three different boards [CBSE, ICSE, (3) 498 (4) 460
and State]. (5) None of these
P
260 4. Average student in each CBSE, ICSE and State
board school of state S is 250, 200, and 400
220
respectively. Find the total number of students in
180 all schools together?
T 140 Q (1) 177000 (2) 358000
(3) 352000 (4) 356000
100
(5) None of these

5. Find the ratio between the total Number of


schools in state P of all boards together and total
S R school in state T of all boards together?
CBSE ICSE State (1) 67:42 (2) 65:47
(3) 61:41 (4) 69:43
(5) None of these
1. In another state A, number of CBSE board
school is 5 more than the number of CBSE Direction [06 - 10]: Read the following information
board in state P, number of State board schools carefully and answer the questions based on it.
is 10% more than the number state board In the graph given below percentage of students in six
schools in state T. If total 600 school in the state different section A, B, C, D, E and F (including both
A, then find the number of ICSE board schools males and females) out of total students and percentage
in state A? of females out of total females is mentioned. Read the
(1) 210 (2) 202 information and answer the questions.
(3) 209 (4) 221
(5) 230

2. Find the difference between total CBSE board


school in state P, Q, and R together and total
ICSE board school in state R, S and T together?
(1) 80 (2) 45
(3) 75 (4) 30
(5) 88

3. In another state Z, the total Number of schools is


the same as the total number of schools in state
Q. Ratio of the number of State, CBSE and
NOTES: Values in the radar graph are in percentage
terms. 1/3rd of the total students in in sections E and F is
65. Ratio of males and females in section F is 2:7.

6. Females from section A and males from section


F buy movie tickets that cost Rs. 10800. Find
the cost of each ticket.
(1) Rs. 26 (2) Rs. 17
(3) Rs. 32 (4) Rs. 27
(5) None of these

7. Females in section B are how much percent less


than total students in that section?
Note:
(1) 48% (2) 55%
(1) Total number of elders in all five companies together
(3) 53% (4) 57%
is 2494, while number of elders in company C is same as
(5) None of these
total number of elders in company A and B together.
(2) Number of male elders < Number of female elders
(companies B and D only).
8. Find the ratio of average count of females in
(3) Number of elders in company D is 350, which is 34
sections A, B, C and D with respect to average
less than that of company E, while number of elders in
males in sections A, C and E.
company B is 1/5 more than that of company A.
(1) 5:8 (2) 5:7
(3) 3:8 (4) 7:8
11. In company B, number of male employees is
(5) None of these
twice as that of females, then find number of
youngster females?
9. Find females in sections D and E together are
(1) 220 (2) 100
what percent of the males in section E?
(3) 160 (4) 360
(1) 90 % (2) 93(1/3) %
(5) None of these
(3) 73(1/3) % (4) 33(1/3) %
(5) None of these
12. In company C, number of female employees is
160 more than those male employees, then find
10. In section P females are 10% more than that in
total number of youngster males are what
section B. Males in section P are 80% of the
percent of total number male employees?
total students in section P. Find the sums of
(1) 40% (2) 25%
males in sections P and C together.
(3) 60% (4) 50%
(1) 215 (2) 265
(5) None of these
(3) 315 (4) 365
(5) None of these
13. Find N is how much percent more or less than
that of M?
Directions [11 – 15]: Read the following information
I) M = difference between number of male
carefully and answer the questions based on it.
elders in company A and female elders in
The given below radar graph shows the percentage by
company E.
which number of male elders in a company is more or
II) N = Difference between female elders in
less than that of female elders, and percentage of
company B and male elders in company D.
youngsters in a company out of total employees in that
(1) 46.33% (2) 62.5%
company.
(3) 87.5% (4) 50%
(5) None of these
14. Difference between the total number of 17. Number of people who read Novels and
employees in companies D and E is what Magazine together in City B is what percentage
percent of difference between total number of more/less than the number of people who read
employees in companies A and B? Novels in City C and City D together?
(1) 430% (2) 230% (1) 55% less (2) 70% more
(3) 630% (4) 530% (3) 45% less (4) 60% more
(5) None of these (5) None of these

15. Find the difference between male and female 18. If the number of people who read Novels in City
elders in all the given companies together? D and City P together is 20% more than the
(1) 34 (2) 94 number of people who read Magazine in City A
(3) 74 (4) 54 and out of the number of people who read
(5) None of these Novels in City P, 33(1/3) % are females, then
find the number of males who read Novels in
Directions (16 – 20): Read the following information City P?
carefully and answer the questions. (1) 420 (2) 520
The given radar graph shows the average number of (3) 480 (4) 440
people who read Novels and Magazine in five different (5) None of these
cities i.e., City A, City B, City C, City D and City E on
Saturday and also given the number of people reading 19. Find the average number of people who read
Novels in these five cities Magazine in City B, City C and City E together?
(1) 300 (2) 800
(3) 700 (4) 400
(5) None of these

20. Find the difference between the number of


people who read Novels and Magazine together
in City E and the number of people who read
Magazine in City C and City D together?
(1) 200 (2) 1000
(3) 400 (4) 600
(5) None of these

16. Find the ratio of the number of people who read


Magazine in City A and City C together and the
number of people who read Novels in City B
and City D together?
(1) 14:9 (2) 7:12
(3) 8:5 (4) 11:10
(5) None of these
Answer Key
1. (3) 5. (5) 9. (2) 13. (3) 17. (4)
2. (4) 6. (4) 10. (2) 14. (5) 18. (2)
3. (2) 7. (5) 11. (2) 15. (4) 19. (3)
4. (1) 8. (1) 12. (1) 16. (1) 20. (1)
Hints & Solutions
1. Solution (3) Females in section F = 70
Total number of ICSE board school in state A = Total males = 600 – 250 = 350
600 - [210 + 5] - [160 * 110/100] = 209 Males in section A = 60
Males in section B = 70
2. Solution (4) Males in section C = 45
Required difference = [210 + 190 + 170] – [210 + Males in section D = 80
210 + 120] = 30 Males in section E = 75
Males in section F = 20
3. Solution (2) According to question,
Total number of schools in Z = 190 + 180 + 210 = Females from section A buy movie tickets
580 Males from section F buy movie tickets
Total number of CBSE and ICSE board school in Tickets cost them Rs. 2700
state Z = 580 * 8/10 = 464 60x + 40x = 2700
So, x = 27
4. Solution (1) Cost of each ticket = Rs. 27
Total number of students in state S = 220 * 250 + Hence, answer is option 4
200 * 210 + 200 * 400 = 177000
7. Solution (5)
5. Solution (5) According to question,
Required ratio = [210 + 200 + 240]: [140 + 120 + Females in section B = 50
160] = 650:420 = 65:42 Total students in section B = 120
Required percentage = 70/120 x 100 = 58.33%
6. Solution (4) Hence, answer is option 5
Common Explanation [06 - 10]:
As per the radar graph,
8. Solution (1)
Let the total students in all sections = 100m According to question,
Let the total females in all sections = 100n Average count of females in sections A, B, C and
1/3rd x students in section E and F = 64 D = 150/4
32.5m = 195 Average males in sections A, C and E = 180/3
Thus, m = 6 Required ratio = 5:8
Total students in all the sections = 600 Hence, answer is option 1
Students in section A = 90
Students in section B = 120
9. Solution (2)
Students in section C = 75 According to question,
Students in section D = 120 Females in sections D and E together = 70
Students in section E = 105 Males in section E = 75
Students in section F = 90 Required percentage = 70/75 x 100 = 93(1/3) %
Males in section F = 20 Females in sections D and E are 93(1/3) % of the
Females in section F = 70 males in section E
Males in section F: females in section F = 2:7 Hence, answer is option 2
Now total females = 250
Females in section A = 30
10. Solution (2)
Females in section B = 50 According to question,
Females in section C = 30 Females in section P = 110% x females in section
Females in section D = 40 B
Females in section E = 30
Males in section P = 80% x total students in Total number of employees = 1200
section A Number of male employees = 2 x number of
In section P, females: males = 1:4 female employees
Thus, females = 55 So, number of female employees = 1/3 x 1200 =
Males = 220 400
Total males in sections A and C = 220 + 45 = 265 Number of female elders = 300
Hence, answer is option 2 So, number of youngsters females = 400 – 300 =
100
11. Solution (2) Hence answer is option 2
Common Explanation [11 – 15]:
Number of elders in company D = 350 12. Solution (1)
So, number of elders in company E = 350 + 34 = In company C,
384 Difference between male and female employees =
So, number of elders in companies (A + B + C) = 160
2494 – 350 – 384 = 1760 Total number of employees = 1760
Number of elders in companies (A + B) = Number So, number of male employees = (1760 – 160)/2 =
of elders in company C = 1760/2 = 880 800
Number of elders in company B is 1/5 more than Number of male elders = 480
that of elders in company A. So, number of youngsters male employees = 800 –
So, number of elders in company A = 5/11 x 880 = 480 = 320
400 Required percentage = 320/800 x 100 = 40%
So, number of elders in company B = 880 – 400 = Hence answer is option 1
480
In company A, 13. Solution (3)
Number of elders = 400 Male elders in company A = 240
Number of youngsters = 75% of total employees Female elders in company E = 160
So, number of elders = 25% of total employees Value of M = 240 – 160 = 80
So, total number of employees = 400/25 x 100 = Male elders in company D = 150
1600 Female elders in company B = 300
Number of youngsters = 75% x 1600 = 1200 Value of N = 300 – 150 = 150
Percentage by which male elders is more than that Required percentage change = (150 – 80)/80 x 100
of female elders = 50% = 87.5%
So, number of male elders = 3/5 x 400 = 240 Hence answer is option 3
Number of female elders = 400 – 240 = 160
Similarly, we can find the data for all companies. 14. Solution (5)
Compa Total Youngst Elde Mal Fema Difference between total employees in company D
ny employ ers rs e le and E = 1920 – 600 = 1320
ees elde elder Difference between total employees in company A
rs s and B = 1600 – 1200 = 400
A 1600 1200 400 240 160 Required percentage = 1320/400 x 100 = 330%
B 1200 720 480 180 300 Hence answer is option 5
C 1760 880 880 480 400
D 600 250 350 150 200 15. Solution (4)
Total number of male elders in all given
E 1920 1536 384 224 160
companies together = 240 + 180 + 480 + 150 +
224 = 1274
According to question,
In company B,
Total number of female elders in all given Number of people who reading Novels in City P =
companies together = 160 + 300 + 400 + 200 + 1080 – 300 = 780
160 = 1220 Required answer = 780 * 2/3 = 520
Required difference = 1274 – 1220 = 54
Hence answer is option 4 19. Solution (3)
Number of people who read Magazine in City C =
16. Solution (1) 600 * 2 – 700 = 500
Number of people who read Novels in City B and Number of people who read Magazine in City B =
City D together = 600 + 300 = 900 800 * 2 – 600 = 1600 – 600 = 1000
Number of people who read Magazine in City A = Number of people who read Magazine in City E =
700 * 2 – 500 = 900 700 * 2 – 800 = 1400 – 800 = 600
Number of people who read Magazine in City C = Required average = (500 + 1000 + 600)/3 = 2100/3
600 * 2 – 700 = 500 = 700
Number of people who read Magazine in City A
and City C together = 900 + 500 = 1400 20. Solution (1)
Required ratio = 1400:900 = 14:9 Number of people who read Novels and Magazine
together in City E = 700 * 2 = 1400
17. Solution (4) Number of people who read Magazine in City C =
Number of people who read Novels in City C and 600 * 2 – 700 = 500
City D together = 700 + 300 = 1000 Number of people who read Magazine in City D =
Number of people who read Novels and Magazine 500 * 2 – 300 = 700
together in City B = 800 * 2 = 1600 Required difference = 1400 – (500 + 700) = 1400
Required percentage = (1600 - 1000)/1000 * 100 = – 1200 = 200
600/10 = 60% more

18. Solution (2)


Number of people who reading Magazine in City
A = 700 * 2 – 500 = 900
Number of people who reading Novels in City D
and City P = 900 * 6/5 = 1080

You might also like